Peds Test 2

Ace your homework & exams now with Quizwiz!

A 1-month-old infant is admitted to the hospital. The infants mother is 17 years old and single and lives with her parents. Who signs the informed consent for the 1-month-old infant? a. The infants mother b. The maternal grandparents of the infant c. The paternal grandparents of the infant d. Both the infants mother and the maternal grandparents

ANS: A An emancipated minor is one who is legally under the age of majority but is recognized as having the legal capacity of an adult under circumstances prescribed by state law, such as pregnancy, marriage, high school graduation, independent living, or military service.

1The nurse is teaching a preschool child with a cognitive impairment how to throw a ball overhand. What teaching strategy should the nurse use for this child? a. Demonstrate how to throw a ball overhand. b. Explain the reason for throwing a ball overhand. c. Show pictures of children throwing balls overhand. d. Explain to the child how to throw the ball overhand.

ANS: A Children with cognitive impairment have a deficit in discrimination, which means that concrete ideas are much easier to learn effectively than abstract ideas. Therefore, demonstration is preferable to verbal explanation, and the nurse should direct learning toward mastering a skill rather than understanding the scientific principles underlying a procedure. Demonstrating how to throw the ball is the best teaching strategy.

29. What is an effective strategy to reduce the stress of burn dressing procedures? a. Involve the child and give choices as feasible. b. Explain to the child why analgesics cannot be used. c. Reassure the child that dressing changes are not painful. d. Encourage the child to master stress with controlled passivity.

ANS: A Childrenwhohaveanunderstandingoftheprocedureandsomeperceivedcontroldemonstratelessmaladaptivebehavior.Theyrespondwelltoparticipatingindecisionsandshouldbegivenas many choices as possible. Analgesia and sedation can and should be used. The dressingchangeprocedureisverypainfulandstressful.Misinformationshouldnotbegiventothechild.Encouragingthechildtomasterstresswithcontrolledpassivityisnotapositivecopingstrategy

1. The parents of a child born with ambiguous genitalia tell the nurse that family and friends are asking what caused the baby to be this way. Tests are being done to assist in gender assignment. What should the nurses intervention include? a. Explain the disorder so they can explain it to others. b. Help parents understand that this is a minor problem. c. Suggest that parents avoid family and friends until the gender is assigned. d. Encourage parents not to worry while the tests are being done.

ANS: A Explaining the disorder to parents so they can explain it to others is the most therapeutic approach while the parents await the gender assignment of their child. Ambiguous genitalia is a serious issue for the family. Careful testing and evaluation are necessary to aid in gender assignment to avoid lifelong problems for the child. Suggesting that parents avoid family and friends until the gender is assigned is impractical and would isolate the family from their support system while awaiting test results. The parents will be concerned. Telling them not to worry without giving them specific alternative actions would not be effective.

1. Nurses should be alert for increased fluid requirements in which circumstance? a. Fever b. Mechanical ventilation c. Congestive heart failure d. Increased intracranial pressure

ANS: A Fever leads to great insensible fluid loss in young children because of increased body surface area relative to fluid volume. The mechanically ventilated child has decreased fluid requirements. Congestive heart failure is a case of fluid overload in children. Increased intracranial pressure does not lead to increased fluid requirements in children.

1. The nurse is selecting a site to begin an intravenous infusion on a 2-year-old child. The superficial veins on his hand and arm are not readily visible. What intervention should increase the visibility of these veins? a. Gently tap over the site. b. Apply a cold compress to the site. c. Raise the extremity above the level of the body. d. Use a rubber band as a tourniquet for 5minutes

ANS: A Gently tapping the site can sometimes cause the veins to be more visible. This is done before the skin is prepared. Warm compresses (not cold) may be useful. The extremity is held in a dependent position. A tourniquet may be helpful, but if too tight, it could cause the vein to burst when punctured. Five minutes is too long.

What technique facilitates lip reading by a hearing-impaired child? a. Speak at an even rate. b. Avoid using facial expressions. c. Exaggerate pronunciation of words. d. Repeat in exactly the same way if child does not understand.

ANS: A Help the child learn and understand how to read lips by speaking at an even rate. Avoiding using facial expressions, exaggerating pronunciation of words, and repeating in exactly the same way if the child does not understand interfere with the childs understanding of the spoken word.

What observation in a child should indicate the need for a referral to a specialist regarding a communication impairment? a. At 2 years of age, the child fails to respond consistently to sounds. b. At 3 years of age, the child fails to use sentences of more than five words. c. At 4 years of age, the child has impaired sentence structure. d. At 5 years of age, the child has poor voice quality.

ANS: A If a 2-year-old child fails to respond consistently to sounds, it is an indication for referral to a specialist regarding communication impairment. At age3 years, the child failing to use sentences of three words would be an indication for referral; impaired sentence structure would be seen in a 5-year-old child and poor voice quality in an older child who has a communication impairment

1. Surgery is performed on a child to correct cryptorchidism. The parents understand the reason for the surgery if they tell the nurse this was done to do what? a. Prevent damage to the undescended testicle. b. Prevent urinary tract infections. c. Prevent prostate cancer. d. Prevent an inguinal hernia.

ANS: A If the testes do not descend spontaneously, orchiopexy is performed before the childs second birthday, preferably between 1 and 2 years of age. Surgical repair is done to (1) prevent damage to the undescended testicle by exposure to the higher degree of body heat in the undescended location, thus maintaining future fertility; (2) decrease the incidence of malignancy formation, which is higher in undescended testicles; (3) avoid trauma and torsion; (4) close the processus vaginalis; and (5) prevent the cosmetic and psychologic disability of an empty scrotum. Parents understand the teaching if they respond the surgery is done to prevent damage.

1. The nurse is providing support to parents adapting to the hospitalization of their child to the pediatric intensive care unit. The nurse notices that the parents keep asking the same questions. What should the nurse do? a. Patiently continue to answer questions, trying different approaches. b. Kindly refer them to someone else for answering their questions. c. Recognize that some parents cannot understand explanations. d. Suggest that they ask their questions when they are not upset.

ANS: A In addition to a general pediatric unit, children may be admitted to special facilities such as an ambulatory or outpatient setting, an isolation room, or intensive care. Wherever the location, the core principles of patient and family-centered care provide a foundation for all communication and interventions with the patient, family, and health care team. The nurse should do the therapeutic action and patiently continue to answer questions, trying different approaches.

A 4-year-old girl is admitted to outpatient surgery for removal of a cyst on her back. Her mother puts the hospital gown on her, but the child is crying because she wants to leave on her underpants. What is the most appropriate nursing action at this time?? a. Allow her to wear her underpants. b. Discuss with her mother why this is important to the child. c. Ask her mother to explain to her why she cannot wear them. d. Explain in a kind, matter-of-fact manner that this is hospital policy.

ANS: A It is appropriate for the child to leave her underpants on. If necessary, the underpants can be removed after she has received the initial medications for anesthesia. This allows her some measure of control in this procedure. The mother should not be required to make the child more upset. The child is too young to understand what hospital policy means.

29. A 7-year-old child with acute diarrhea has been rehydrated with oral rehydration solution (ORS). The nurse should recommend that the childs diet be advanced to what kind of diet? a. Regular diet b. Clear liquids c. High carbohydrate diet d. BRAT (bananas, rice, applesauce, and toast or tea) diet

ANS: A It is appropriate to advance to a regular diet after ORS has been used to rehydrate the child. Clear liquids are not appropriate for hydration or afterward. A high carbohydrate diet may contribute to loose stools because of the low electrolyte content and high osmolality. The BRAT diet has little nutritional value and is high in carbohydrates.

A 6-year-old boy is hospitalized for intravenous antibiotic therapy. He eats very little on his regular diet trays. He tells the nurse that all he wants to eat is pizza, tacos, and ice cream. What nursing action is the most appropriate? a. Request these favorite foods for him. b. Identify healthier food choices that he likes. c. Explain that he needs fruits and vegetables. d. Reward him with ice cream at the end of every meal that he eats.

ANS: A Loss of appetite is a symptom common to most childhood illnesses. To encourage adequate nutrition, the nurse should request favorite foods for the child. The foods he likes provide nutrition and can be supplemented with additional fruits and vegetables. Ice cream and other desserts should not be used as rewards or punishment.

A 10-year-old child requires daily medications for a chronic illness. Her mother tells the nurse that the child continually forgets to take the medicine unless reminded. What nursing action is most appropriate to promote adherence to the medication regimen? a. Establish a contract with her, including rewards. b. Suggest time-outs when she forgets her medicine. c. Discuss with her mother the damaging effects of her rescuing the child. d. Ask the child to bring her medicine containers to each appointment so they can be counted.

ANS: A Many factors can contribute to the childs not taking the medication. The nurse should resolve those issues such as unpleasant side effects, difficulty taking medicine, and time constraints before school. If these factors do not contribute to the issue, then behavioral contracting is usually an effective method to shape behaviors in children. Time-outs provide negative reinforcement. If part of a contract, negative consequences can work, but they need to be structured. Discussing with her mother the damaging effects of her rescuing the child is not the most appropriate action to encourage compliance. For a school-age child, parents should refrain from nagging and rescuing the child. This child is old enough to partially assume responsibility for her own care. If the child brings her medicine containers to each appointment so they can be counted, this will help determine if the medications are being taken, but it will not provide information about whether the child is taking them by herself.

23. What condition is often associated with severe diarrhea? a. Metabolic acidosis b. Metabolic alkalosis c. Respiratory acidosis d. Respiratory alkalosis

ANS: A Metabolic acidosis results from the increased absorption of short-chain fatty acids produced in the colon. There is an increase in lactic acid from tissue hypoxia secondary to hypovolemia. Bicarbonate is lost through the stool. Ketosis results from fat metabolism when glycogen stores are depleted. Metabolic alkalosis and respiratory alkalosis do not occur from severe diarrhea.

The community nurse is planning prevention measures designed to avoid conditions that can cause cognitive impairment. Taking folic acid supplements during pregnancy to prevent neural tube defects is which type of prevention strategy? a. Primary b. Secondary c. Tertiary d. Rehabilitative

ANS: A Primary prevention strategies are those designed to avoid conditions that cause cognitive impairment. Use of folic acid supplements during pregnancy to prevent neural tube defects is a primary prevention strategy. Secondary prevention activities are those designed to identify the condition early and initiate treatment to avert cerebral damage. Tertiary prevention strategies are those concerned with treatment to minimize long-term consequences. Rehabilitation services is an example of tertiary prevention.

1. Parents of a hospitalized child often question the skill of staff. The nurse interprets this behavior by the parents as what? a. Normal b. Paranoid c. Indifferent d. Wanting attention

ANS: A Recent research has identified common themes among parents whose children were hospitalized, including feeling an overall sense of helplessness, questioning the skills of staff, accepting the reality of hospitalization, needing to have information explained in simple language, dealing with fear, coping with uncertainty, and seeking reassurance from the health care team. The behavior does not indicate the parents are paranoid, indifferent, or wanting attention.

An 8-year-old girl is being admitted to the hospital from the emergency department with an injury from falling off her bicycle. What intervention will help her most in her adjustment to the hospital? a. Explain hospital schedules to her, such as mealtimes. b. Use terms such as honey and dear to show a caring attitude. c. Explain when parents can visit and why siblings cannot come to see her. d. Orient her parents, because she is too young, to her room and hospital facility.

ANS: A School-age children need to have control of their environment. The nurse should offer explanations or prepare the child for what to expect. The nurse should refer to the child by the preferredname.Explainingwhenparentscanvisitandwhysiblingscannotcomefocusesonthelimitationsratherthanhelpingheradjusttothehospital.Attheageof8years,thechildshouldbeorientedtotheenvironmentalongwiththeparents

1. Ongoing fluid losses can overwhelm the childs ability to compensate, resulting in shock. What early clinical sign precedes shock? a. Tachycardia b. Slow respirations c. Warm, flushed skin d. Decreased blood pressure

ANS: A Shock is preceded by tachycardia and signs of poor tissue perfusion and decreased pulse oximetry values. Respirations are increased as the child attempts to compensate. As a result of the poor peripheral circulation, the child has skin that is cool and mottled with decreased capillary refilling after blanching. In children, lowered blood pressure is a late sign and may accompany the onset of cardiovascular collapse.

The mother of a young child with cognitive impairment asks for suggestions about how to teach her child to use a spoon for eating. The nurse should make which recommendation? a. Do a task analysis first. b. Do not expect this task to be learned. c. Continue to spoon feed the child until the child tries to do it alone. d. Offer only finger foods so spoon feeding is unnecessary.

ANS: A Successful teaching begins with a task analysis. The endpoint (self-feeding, toilet training, and so on) is broken down into the component steps. The child is then guided to master the individual steps in sequence. Depending on the childs functional level, using a spoon for eating should be an achievable goal. The child requires demonstration and then guided training for each component of the self-feeding. Feeding finger foods so spoon feeding is unnecessary eliminates some of the intermediate steps that are necessary to using a fork and spoon. For socialization purposes, it is desirable that a child use feeding implements.

The nurse should suspect a hearing impairment in an infant who fails to demonstrate which behavior? a. Babbling by age 12 months b. Eye contact when being spoken to c. Startle or blink reflex to sound d. Gesturing to indicate wants after age 15 months

ANS: A The absence of babbling or inflections in voice by at least age 7 months is an indication of hearing difficulties. Lack of eye contact is not indicative of a hearing loss. An infant with a hearing impairment might react to a loud noise but not respond to the spoken word. The child with hearing impairment uses gestures rather than vocalizations to express desires at this age.

1. What urine test result is considered abnormal? a. pH 4.0 b. WBC 1 or 2 cells/ml c. Protein level absent d. Specific gravity 1.020

ANS: A The expected pH ranges from 4.8 to 7.8. A pH of 4.0 can be indicative of urinary tract infection or metabolic alkalosis or acidosis. Less than 1 or 2 white blood cells per milliliter is the expected range. The absence of protein is expected. The presence of protein can be indicative of glomerular disease. A specific gravity of 1.020 is within the anticipated range of 1.001 to 1.030. Specific gravity reflects level of hydration in addition to renal disorders and hormonal control such as antidiuretic hormone.

29. What is the most immediate threat to life in children with thermal injuries? a. Shock b. Anemia c. Local infection d. Systemic sepsis

ANS: A The immediate threat to life in children with thermal injuries is airway compromise and profound shock. Anemia is not of immediate concern. During the healing phase, local infection or sepsis is the primary complication.

32. The nurse is administering an IM injection into a vastus lateralis muscle of a 6-month-old infant. What should the length of the needle and amount to be given be? a. 5/8 to 1 inch; 0.5 to 1.0 ml b. 1 inch to 1 1/2 inch; 1.0 to 2.0 ml c. 1 inch to 1 1/2 inch; 0.5 to 1.0 ml d. 5/8 to 1 inch; 0.75 to 2 ml

ANS: A The length of a needle for an infant should be 5/8 to 1 inch, and the amount of solution should not exceed 1 ml.

The mother of a 7-month-old infant newly diagnosed with cystic fibrosis is rooming in with her infant. She is breastfeeding and provides all the care except for the medication administration. What should the nurse include in the plan of care? a. Ensuring that the mother has time away from the infant b. Making sure the mother is providing all of the infants care c. Determining whether other family members can provide the necessary care so the mother can rest d. Contacting the social worker because of the mothers interference with the nursing care

ANS: A The mother needs sufficient rest and nutrition so she can be effective as a caregiver. While the infant is hospitalized, the care is the responsibility of the nursing staff. The mother should be made comfortable with the care the staff provides in her absence. The mother has a right to provide care for the infant. The nursing staff and the mother should agree on the care division.

A child, age 7 years, has a fever associated with a viral illness. She is being cared for at home. What is the principal reason for treating fever in this child? a. Relief of discomfort b. Reassurance that illness is temporary c. Prevention of secondary bacterial infection d. Avoidance of life-threatening complications

ANS: A The principal reason for treating fever is the relief of discomfort. Relief measures include pharmacologic and environmental intervention. The most effective is the use of pharmacologic agents to lower the set point. Although the nurse can reassure the child that the illness is temporary, the child is often uncomfortable and irritable. Intervention helps the child and family minimize the discomfort. Most fevers result from viral, not bacterial, infections. Few life- threatening events are associated with fever. The use of antipyretics does not seem to reduce the incidence of febrile seizures.

What flush solution is recommended for intravenous catheters larger than 24 gauge? a. Saline b. Heparin c. Alteplase d. Heparin and saline combination

ANS: A The recommended solution for flushing venous access devices is saline. The turbulent flow flush with saline is effective for catheters larger than 24 gauge. The use of heparin does not increase the longevity of the venous access device. In 24-gauge catheters, heparin may offer an advantage. Alteplase is used for treating catheter-related occlusions in children. The heparin and saline combination does not offer any advantage over saline or heparin individually.

1. What diagnostic test allows visualization of renal parenchyma and renal pelvis without exposure to external-beam radiation or radioactive isotopes? a. Renal ultrasonography b. Computed tomography c. Intravenous pyelography d. Voiding cystourethrography

ANS: A The transmission of ultrasonic waves through the renal parenchyma allows visualization of the renal parenchyma and renal pelvis without exposure to external-beam radiation or radioactive isotopes. Computed tomography uses external radiation, and sometimes contrast media are used. Intravenous pyelography uses contrast medium and external radiation for radiography. Contrast medium is injected into the bladder through the urethral opening. External radiation for radiography is used before, during, and after voiding in voiding cystourethrography.

1. The nurse is preparing to admit a child to the hospital with a diagnosis of minimal change nephrotic syndrome. The nurse understands that the peak age at onset for this disease is what? a. 2 to 3 years b. 4 to 5 years c. 6 to 7 years d. 8 to 9 years

ANS: A Thepeakageatonsetforminimalchangenephroticsyndromeis2to3yearsofage.

When a preschool-age child is hospitalized without adequate preparation, the nurse should recognize that the child may likely see hospitalization as what? a. Punishment b. Loss of parental love c. Threat to the childs self-image d. Loss of companionship with friends

ANS: A Therationaleforpreparingchildrenforthehospitalexperienceandrelatedproceduresisbasedontheprinciplethatafearoftheunknown(fantasy)exceedsfearoftheknown.Preschool-agechildrenseehospitalizationasapunishment.Lossofparentallovewouldbeatoddlersreaction.Threat to the childs self-image would be a school-age childs reaction. Loss of companionshipwithfriendswouldbeanadolescentsreaction

Girls experience an increase in weight and fat deposition during puberty. What do nursing considerations related to this include? a. Give reassurance that these changes are normal. b. Suggest dietary measures to control weight gain. c. Encourage a low-fat diet to prevent fat deposition. d. Recommend increased exercise to control weight gain.

ANS: A A certain amount of fat is increased along with lean body mass to fill the characteristic contours of the adolescents gender. A healthy balance must be achieved between expected healthy weight gain and obesity. Suggesting dietary measures or increased exercise to control weight gain would not be recommended unless weight gain was excessive because eating disorders can develop in this group. Some fat deposition is essential for normal hormonal regulation. Menarche is delayed in girls with body fat contents that are too low.

The school nurse needs to obtain authorization for a child who requires medications while at school. From whom does the nurse obtain the authorization? a. The parents b. The pharmacist c. The school administrator d. The prescribing practitioner

ANS: A A child who requires medication during the school day requires written authorization from the parent or guardian. Most schools also require that the medication be in the original container appropriately labeled by the pharmacist or physician. Some schools allow children to receive over-the-counter medications with parental permission. The pharmacist may be asked to appropriately label the medication for use at the school, but authorization is not required. The school administration should have a policy in place that facilitates the administration of medications for children who need them. The prescribing practitioner is responsible for ensuring that the medication is appropriate for the child. Because the child is a minor, parental consent is required

Deficiency of which vitamin or mineral results in an inadequate inflammatory response? a. A b. B1 c. C d. Zinc

ANS: A A deficiency of vitamin A results in an inadequate inflammatory response. Deficiencies of vitamins B1 and C result in decreased collagen formation. A deficiency of zinc leads to impaired epithelialization.

The nurse is admitting a child with frostbite. What health care prescription should the nurse question and verify? a. Massage the injured tissue. b. Apply a loose dressing after rewarming. c. Avoid any application of dry heat to the area. d. Administer acetaminophen (Tylenol) for discomfort.

ANS: A A frostbite victim should not have injured tissue rubbed. It is contraindicated because it can cause damage by rupture of crystallized cells. After rewarming, a loose dressing is applied to the affected skin, and analgesia is administered if indicated. Dry heat is not applied.

A 9-year-old child has just been diagnosed with recurrent abdominal pain (RAP). In preparing for discharge, the nurse should include what in the home care instructions to the parents? a. Following a high-fiber diet b. Using stimulant laxatives c. Using ice packs on the abdomen when pain occurs d. Sitting on the toilet for 30 minutes after each meal

ANS: A A high-fiber diet with possible addition of bulk laxatives is beneficial for children with RAP. Bulk-forming laxatives such as psyllium are recommended. Stimulant laxatives may produce painful cramping for the child. Warm packs, such as a heating pad, may help ease the discomfort. Bowel training is recommended to assist the child in establishing regular bowel habits. Thirty minutes is too long for the child to sit on the toilet. The time should be limited to 15 minutes.

The nurse is planning care for a 3-year-old boy who has Down syndrome and is on continuous oxygen. He recently began walking around furniture. He is spoon fed by his parents and eats some finger foods. What goal is the most appropriate to promote normal development? a. Encourage mobility b. Encourage assistance in self-care. c. Promote oral-motor development. d. Provide opportunities for socialization.

ANS: A A major principle for developmental support in children with complex medical issues is that it should be flexible and tailored to the individual childs abilities, interests, and needs. This child is exhibiting readiness for ambulation. It is an appropriate time to provide activities that encourage mobility, for example, longer oxygen tubing. Parents should provide decreasing amounts of assistance with self-care as he is able to develop these skills. The boy is receiving oral foods and is eating finger foods. He has acquired this skill. Mobility is a new developmental task. Opportunities for socialization should be ongoing.

The home health nurse is planning care for a 3-year-old boy who has Down syndrome and is receiving continuous oxygen. He recently began walking around furniture. He is spoon-fed by his parents and eats some finger foods. Which is the most appropriate goal to promote normal development? a. Encourage mobility. b. Encourage assistance in self-care. c. Promote oral-motor development. d. Provide opportunities for socialization.

ANS: A A major principle for developmental support in children with complex medical issues is that it should be flexible and tailored to the individual child's abilities, interests, and needs. This child is exhibiting readiness for ambulation. It is an appropriate time to provide activities that encourage mobility, for example, longer oxygen tubing. Parents should provide decreasing amounts of assistance with self-care as he is able to develop these skills. He is receiving oral foods and is eating finger foods. He has acquired oral-motor development. Mobility is a new developmental task. Opportunities for socialization should be ongoing.

The nurse is conducting a staff in-service on renal ultrasounds. Which statement describes this diagnostic test? a. Computed tomography uses external radiation to visualize the renal system. b. Visualization of the renal system is accomplished without exposure to radiation or radioactive isotopes. c. Contrast medium and x-rays allow for visualization of the renal system. d. External radiation for x-ray films is used to visualize the renal system, before, during, and after voiding.

ANS: A A renal ultrasound transmits ultrasonic waves through the renal parenchyma, allowing for visualization of the renal system without exposure to external beam radiation or radioactive isotopes. Computed tomography uses external radiation and sometimes contrast media to visualize the renal system. An intravenous pyelogram uses contrast medium and external radiation for x-ray films. The voiding cystourethrogram visualizes the renal system with injection of a contrast media into the bladder through the urethral opening and use of x-ray before, during, and after voiding.

Preschoolers' fears can best be dealt with by which intervention? a. Actively involving them in finding practical methods to deal with the frightening experience b. Forcing them to confront the frightening object or experience in the presence of their parents c. Using logical persuasion to explain away their fears and help them recognize how unrealistic the fears are d. Ridiculing their fears so that they understand that there is no need to be afraid

ANS: A Actively involving them in finding practical methods to deal with the frightening experience is the best way to deal with fears. Forcing a child to confront fears may make the child more afraid. Preconceptual thought prevents logical understanding. Ridiculing fears does not make them go away.

When liquid medication is given to a crying 10-month-old infant, which approach minimizes the possibility of aspiration? a. Administer the medication with a syringe (without needle) placed along the side of the infant's tongue. b. Administer the medication as rapidly as possible with the infant securely restrained. c. Mix the medication with the infant's regular formula or juice and administer by bottle. d. Keep the child upright with the nasal passages blocked for a minute after administration.

ANS: A Administer the medication with a syringe without needle placed along the side of the infant's tongue. The contents are administered slowly in small amounts, allowing the child to swallow between deposits. Medications should be given slowly to avoid aspiration. The medication should be mixed with only a small amount of food or liquid. If the child does not finish drinking or eating, it is difficult to determine how much medication was consumed. Essential foods also should not be used. Holding the child's nasal passages will increase the risk of aspiration.

Which aspect of cognition develops during adolescence? a. Capability to use a future time perspective b. Ability to place things in a sensible and logical order c. Ability to see things from the point of view of another d. Progress from making judgments based on what they see to making judgments based on what they reason

ANS: A Adolescents are no longer restricted to the real and actual. They also are concerned with the possible; they think beyond the present. During concrete operations (between ages 7 and 11 years), children exhibit these characteristic thought processes.

The nurse gives an injection in a patient's room. The nurse should perform which intervention with the needle for disposal? a. Dispose of syringe and needle in a rigid, puncture-resistant container in the patient's room. b. Dispose of syringe and needle in a rigid, puncture-resistant container in an area outside of the patient's room. c. Cap needle immediately after giving injection and dispose of in a proper container. d. Cap needle, break from syringe, and dispose of in a proper container.

ANS: A All needles (uncapped and unbroken) are disposed of in a rigid, puncture-resistant container located near the site of use. Consequently, these containers should be installed in the patient's room. The uncapped needle should not be transported to an area distant from use. Needles are disposed of uncapped and unbroken.

The nurse is teaching parents of preschoolers about plants that are poisonous. What plant should the nurse include in the teaching session? a. Azalea b. Begonia c. Boston fern d. Asparagus fern

ANS: A All parts of the azalea are poisonous. Begonias, Boston ferns, and asparagus ferns are nonpoisonous plants.

A child has an evulsed (knocked-out) tooth. Which medium should the nurse instruct the parents to place the tooth in for transport to the dentist? a. In cold milk b. In cold water c. In warm salt water d. In a dry, clean jar

ANS: A An evulsed tooth should be placed in a suitable medium for transplant, either cold milk or saliva (under the child or parent's tongue). Cold milk is a more suitable medium for transport than cold water, warm salt water, or a dry, clean jar.

A toddler, age 16 months, falls down a few stairs. He gets up and scolds the stairs as if they caused him to fall. What is this an example of? a. Animism b. Ritualism c. Irreversibility d. Delayed cognitive development

ANS: A Animism is the attribution of lifelike qualities to inanimate objects. By scolding the stairs, the toddler is attributing human characteristics to them. Ritualism is the need to maintain sameness and reliability. It provides a sense of comfort to toddlers. Irreversibility is the inability to reverse or undo actions initiated physically. The toddler is acting in an age-appropriate manner.

Steven, 16 months old, falls down a few stairs. He gets up and "scolds" the stairs as if they caused him to fall. This is an example of which of the following? a. Animism b. Ritualism c. Irreversibility d. Delayed cognitive development

ANS: A Animism is the attribution of lifelike qualities to inanimate objects. By scolding the stairs, the toddler is attributing human characteristics to them. Ritualism is the need to maintain the sameness and reliability. It provides a sense of comfort to the toddler. Irreversibility is the inability to reverse or undo actions initiated physically. Steven is acting in an age-appropriate manner.

A 13-year-old boy comes to the school nurse complaining of sudden and severe scrotal pain. He denies any trauma to the scrotum. What is the most appropriate nursing action? a. Refer him for immediate medical evaluation. b. Administer analgesics and recommend scrotal support. c. Apply an ice bag and observe for increasing pain. d. Reassure the adolescent that occasional pain is common with the changes of puberty.

ANS: A Any adolescent boy with redness, swelling, or pain in the scrotum is referred for immediate evaluation. These are signs of testicular torsion, which is a medical emergency. If the possibility of testicular torsion is eliminated, appropriate interventions include administering analgesics and recommending scrotal support. applying an ice bag and observing for increasing pain. and reassuring the adolescent that occasional pain is common with the changes of puberty.

The nurse is talking to the parent of a child with special needs. The parent has expressed worry about how to support the siblings at home. What suggestion is appropriate for the nurse to give to the parent? a. You should help the siblings see the similarities and differences between themselves and your child with special needs. b. You should explain that your child with special needs should be included in all activities that the siblings participate in even if they are reluctant. c. You should give the siblings many caregiving tasks for your child with special needs so the siblings feel involved. d. You should intervene when there are differences between your child with special needs and the siblings.

ANS: A Appropriate information to give to a parent who wants to support the siblings of a child with special needs includes helping the siblings see the differences and similarities between themselves and the child with special needs to promote an understanding environment. The parent should be encouraged to allow the siblings to participate in activities that do not always include the child with special needs, to limit caregiving responsibilities, and to allow the children to settle their own differences rather than step in all the time.

The parents of a 2-year-old child tell the nurse they are concerned because the toddler has started to use baby talk since the arrival of their new baby. What should the nurse recommend? a. Ignore the baby talk. b. Tell the toddler frequently, You are a big kid now. c. Explain to the toddler that baby talk is for babies. d. Encourage the toddler to practice more advanced patterns of speech.

ANS: A Baby talk is a sign of regression in the toddler. Often toddlers attempt to cope with a stressful situation by reverting to patterns of behavior that were successful in earlier stages of development. It should be ignored while the parents praise the child for developmentally appropriate behaviors. Regression is childrens way of expressing stress. The parents should not introduce new expectations and allow the child to master the developmental tasks without criticism.

A 2-year-old child is being admitted to the hospital for possible bacterial meningitis. When preparing for a lumbar puncture, what is the nurse's best action? a. Prepare child for conscious sedation during the test. b. Set up a tray with equipment the same size as for adults. c. Reassure the parents that the test is simple, painless, and risk free. d. Apply EMLA to the puncture site 15 minutes before the procedure.

ANS: A Because of the urgency of the child's condition, conscious sedation should be used for the procedure. Pediatric spinal trays have smaller needles than do adult trays. Reassuring the parents that the test is simple, painless, and risk free is incorrect information. A spinal tap does have associated risks, and analgesia will be given for the pain. EMLA (a eutectic mixture of anesthetics) should be applied approximately 60 minutes before the procedure. The emergency nature of the spinal tap precludes its use.

What is most descriptive of the spiritual development of older adolescents? a. Beliefs become more abstract. b. Rituals and practices become increasingly important. c. Strict observance of religious customs is common. d. Emphasis is placed on external manifestations, such as whether a person goes to church.

ANS: A Because of their abstract thinking abilities, adolescents are able to interpret analogies and symbols. Rituals, practices, and strict observance of religious customs become less important as adolescents question values and ideals of families. Adolescents question external manifestations when not supported by adherence to supportive behaviors

According to Piaget, magical thinking is the belief of which? a. Thoughts are all powerful. b. God is an imaginary friend. c. Events have cause and effect. d. If the skin is broken, the insides will come out.

ANS: A Because of their egocentrism and transductive reasoning, preschoolers believe that thoughts are all powerful. Believing God is an imaginary friend is an example of concrete thinking in a preschoolers spiritual development. Cause-and-effect implies logical thought, not magical thinking. Believing that if the skin is broken, the insides will come out is an example of concrete thinking in development of body image.

The nurse is teaching parents about instilling a positive body image for the preschool age. What statement made by the parents indicates the teaching is understood? a. We will make sure our child is praised about his or her looks. b. We will help our child compare his or her size with other children. c. We understand our child will have well-defined body boundaries. d. We will be sure our child understands about being little for his or her age

ANS: A Because these are formative years for both boys and girls, parents should make efforts to instill positive principles regarding body image. Children at this age are aware of the meaning of words such as pretty or ugly, and they reflect the opinions of others regarding their own appearance. Despite the advances in body image development, preschoolers have poorly defined body boundaries. By 5 years of age, children compare their size with that of their peers and can become conscious of being large or short, especially if others refer to them as so big or so little for their age. Parents should not suggest their child compare him- or herself with other children in regard to size, and parents should not focus on their childs size as being little.

The management of a child who has just been stung by a bee or wasp should include applying what? a. Cool compresses b. Antibiotic cream c. Warm compresses d. Corticosteroid cream

ANS: A Bee or wasp stings are initially treated by carefully removing the stinger, cleansing with soap and water, applying cool compresses, and using common household agents such as lemon juice or a paste made with aspirin and baking soda. Antibiotic cream is unnecessary unless a secondary infection occurs. Warm compresses are avoided. Corticosteroid cream is not part of the initial therapy. If a severe reaction occurs, systemic corticosteroids may be indicated.

Bullying can be common during the school-age years. The nurse should recognize that which applies to bullying? a. Can have a lasting effect on children b. Is not a significant threat to self-concept c. Is rarely based on anything that is concrete d. Is usually ignored by the child who is being bullied

ANS: A Bullying in this age group is common and can have a long-lasting effect. Increasing awareness of differences, especially when accompanied by unkind comments and taunts from others, may make a child feel inferior and undesirable. Physical impairments such as hearing or visual defects, ears that "stick out," or birth marks assume great importance.

The nurse is talking to the parent of a 13-month-old child. The mother states, "My child does not make noises like 'da' or 'na' like my sister's baby, who is only 9 months old." Which statement by the nurse would be most appropriate to make? a. "I am going to request a referral to a hearing specialist." b. "You should not compare your child to your sister's child." c. "I think your child is fine, but we will check again in 3 months." d. "You should ask other parents what noises their children made at this age."

ANS: A By 11 months of age a child should be making well-formed syllables such as "da" or "na" and should be referred to a specialist if not. "You should not compare your child to your sister's child," "I think your child is fine, but we will check again in 3 months," and "You should ask other parents what noises their children made at this age," are not appropriate statements to make to the parent.

Which describes a child who is abused by the parent(s)? a. Unintentionally contributes to the abusing situation b. Belongs to a low socioeconomic population c. Is healthier than the nonabused siblings d. Abuses siblings in the same way as child is abused by the parent(s)

ANS: A Child's temperament, position in the family, additional physical needs, activity level, or degree of sensitivity to parental needs unintentionally contribute to the abusing situation. Abuse occurs among all socioeconomic levels. Children who are ill or have additional physical needs are more likely to be abused. The abused child may not abuse siblings.

Samantha, age 5 years, tells the nurse that she "needs a Band-Aid" where she had an injection. Which is the best nursing action? a. Apply a Band-Aid. b. Ask her why she wants a Band-Aid. c. Explain why a Band-Aid is not needed. d. Show her that the bleeding has already stopped.

ANS: A Children at this age group still fear that their insides may leak out at the injection site. Provide the Band-Aid. No explanation should be required. The nurse should be prepared to apply a small Band-Aid after the injection.

An 8-year-old girl tells the nurse that she has cancer because God is punishing her for being bad. What should the nurse interpret this as? a. A common belief at this age b. Indicative of excessive family pressure c. Faith that forms the basis for most religions d. Suggestive of a failure to develop a conscience

ANS: A Children at this age may view illness or injury as a punishment for a real or imagined misbehavior. School-age children expect to be punished and tend to choose a punishment that they think fits the crime. This is a common belief and not related to excessive family pressure. Many faiths do not include a God that causes cancer in response for bad behavior. This statement reflects the childs belief in what is right and wrong.

An 8-year-old girl tells the nurse that she has cancer because God is punishing her for "being bad." She shares her concern that if she dies, she will go to hell. How should the nurse interpret this statement? a. A common belief at this age b. A belief that forms the basis for most religions c. Suggestive of excessive family pressure d. Suggestive of a failure to develop a conscience

ANS: A Children at this age may view illness or injury as a punishment for a real or imagined misdeed. The belief in divine punishment is common for an 8-year-old child.

Which statement is true about smoking in adolescence? a. Smoking is related to other high-risk behaviors. b. Smoking will not continue unless peer pressure continues. c. Smoking is less common when the adolescent's parent(s) smokes. d. Smoking among adolescents is becoming more prevalent.

ANS: A Cigarettes are considered a gateway drug. Teenagers who smoke are 11.4 times more likely to use an illicit drug. Teenagers begin smoking for a variety of reasons, such as imitation of adult behavior, peer pressure, imitation of behaviors portrayed in movies and advertisements, and a desire to control weight. The absence of peer pressure alone will not stop smoking. Teenagers who do not smoke usually have parents and friends who do not smoke or who oppose smoking. The percentage of young people who report current cigarette use and frequent cigarette use has declined significantly.

A school nurse is performing hearing screening on school children. The nurse recognizes that the most common type of hearing loss resulting from interference of transmission of sound to the middle ear is characteristic of which type of hearing loss? a. Conductive b. Sensorineural c. Mixed conductive-sensorineural d. Central auditory imperceptive

ANS: A Conductive or middle-ear hearing loss is the most common type. It results from interference of transmission of sound to the middle ear, most often from recurrent otitis media. Sensorineural, mixed conductive-sensorineural, and central auditory imperceptive are less common types of hearing loss.

Which is instituted for the therapeutic management of minimal change nephrotic syndrome? a. Corticosteroids b. Antihypertensive agents c. Long-term diuretics d. Increased fluids to promote diuresis

ANS: A Corticosteroids are the first line of therapy for minimal change nephrotic syndrome. Response is usually seen within 7 to 21 days. Antihypertensive agents and long-term diuretic therapy are usually not necessary. A diet that has fluid and salt restrictions may be indicated.

Which should the nurse recommend to prevent urinary tract infections in young girls? a. Wear cotton underpants. b. Limit bathing as much as possible. c. Increase fluids; decrease salt intake. d. Cleanse perineum with water after voiding.

ANS: A Cotton underpants are preferable to nylon underpants. No evidence exists that limiting bathing, increasing fluids/decreasing salt intake, or cleansing the perineum with water after voiding decrease urinary tract infections in young girls.

Four-year-old Brian appears to be upset by hospitalization. Which is an appropriate intervention? a. Let him know it is all right to cry. b. Give him time to gain control of himself. c. Show him how other children are cooperating. d. Tell him what a big boy he is to be so quiet.

ANS: A Crying is an appropriate behavior for the upset preschooler. The nurse provides support through physical presence. Giving the child time to gain control is appropriate, but the child must know that crying is acceptable. The preschooler does not engage in competitive behaviors.

The nurse is providing support to parents at the time their child is diagnosed with chronic disabilities. The nurse notices that the parents keep asking the same questions. What is the nurse's best intervention? a. Patiently continue to answer questions. b. Kindly refer them to someone else for answering their questions. c. Recognize that some parents cannot understand explanations. d. Suggest that they ask their questions when they are not upset.

ANS: A Diagnosis is one of the anticipated stress points for parents. The parents may not hear or remember all that is said to them. The nurse should continue to provide the kind of information they desire. This is a particularly stressful time for the parents; the nurse can play a key role in providing necessary information. Parents should be provided with oral and written information. The nurse needs to work with the family to ensure understanding of the information. The parents require information at the time of diagnosis. Other questions will arise as they adjust to the information.

The parents of a child born with disabilities ask the nurse for advice about discipline. The nurses response should be based on remembering that discipline is which? a. Essential for the child b. Not needed unless the childs behavior becomes problematic c. Best achieved with punishment for misbehavior d. Too difficult to implement with a special needs child

ANS: A Discipline is essential for the child. It provides boundaries on which she can test out her behavior and teaches her socially acceptable behaviors. The nurse should teach the parents ways to manage the childs behavior before it becomes problematic. Punishment is not effective in managing behavior.

The parents of a child born with disabilities ask the nurse for advice about discipline. What information about disciple should the nurse's response include? a. It is essential for the child. b. It is too difficult to implement with a special-needs child. c. It is not needed unless the child becomes problematic. d. It is best achieved with punishment for misbehavior.

ANS: A Discipline is essential for the child. It provides boundaries on which to test out their behavior and teaches them socially acceptable behaviors. The nurse should teach the parents ways to manage the child's behavior before it becomes problematic. Punishment is not effective in managing behavior.

An important distinction in understanding substance abuse is that drug misuse, abuse, and addiction are considered what? a. Voluntary behaviors based on psychosocial needs b. Problems that occur in conjunction with addiction c. Involuntary physiologic responses to the pharmacologic characteristics of drugs d. Legal use of substances for purposes other than medicinal.

ANS: A Drug misuse, abuse, and addiction are considered voluntary behaviors. Cultural norms define what is abuse and misuse. Addiction is a psychologic dependence on a substance with or without physical dependence. Physical dependence is an involuntary response to the pharmacologic characteristics of the drug such as an opiate or alcohol. Legality is not always a factor in substance abuse. Legal substances such as alcohol and tobacco can also be misused or abused and can cause addiction.

Which accurately describes the speech of the preschool child? a. Dysfluency in speech patterns is normal. b. Sentence structure and grammatic usage are limited. c. By age 5 years, child can be expected to have a vocabulary of about 1000 words. d. Rate of vocabulary acquisition keeps pace with the degree of comprehension of speech.

ANS: A Dysfluency includes stuttering and stammering, a normal characteristic of language development. Children speak in sentences of three or four words at age 3 to 4 years and eight words by age 5 years. At 5 years, children have a vocabulary of 2100 words. Children often gain vocabulary beyond degree of comprehension.

The nurse is caring for a child with suspected ingestion of some type of poison. What action should the nurse take next after initiating cardiopulmonary resuscitation (CPR)? a. Empty the mouth of pills, plants, or other material. b. Question the victim and witness. c. Place the child in a side-lying position. d. Call poison control.

ANS: A Emptying the mouth of any leftover pills, plants, or other ingested material is the next step after assessment and initiation of CPR if needed. Questioning the victim and witnesses, calling poison control, and placing the child in a side-lying position are follow-up steps.

What method is the most commonly used in completed suicides? a. Firearms b. Drug overdose c. Self-inflicted laceration d. Carbon monoxide poisoning

ANS: A Firearms are the most commonly used instruments in completed suicides among both males and females. For completed suicides in adolescent boys, firearms are followed by hanging and overdose. For adolescent girls, overdose and strangulation are the next most common means of completed suicide. The most common method of suicide attempt is overdose or ingestion of potentially toxic substances such as drugs. The second most common method of suicide attempt is self-inflicted laceration. Carbon monoxide poisoning is not one of the more frequent forms of suicide completion.

Which is the most commonly used method in completed suicides? a. Firearms b. Drug overdose c. Self-inflected laceration d. Carbon monoxide poisoning

ANS: A Firearms are the most commonly used instruments in completed suicides among both males and females. For adolescent boys, firearms are followed by hanging and overdose. For adolescent females, overdose and strangulation are the next most common means of completed suicide. The most common method of suicide attempt is overdose or ingestion of potentially toxic substances such as drugs. The second most common method of suicide attempt is self-inflicted laceration. Carbon monoxide poisoning is not one of the more frequent forms of suicide completion.

What is the most fatal type of burn in the toddler age group? a. Flame burn from playing with matches b. Scald burn from high-temperature tap water c. Hot object burn from cigarettes or irons d. Electric burn from electric outlets

ANS: A Flame burns from matches and lighters represent one of the most fatal types of burns in the toddler age group. High-temperature tap water, hot objects, and electrical outlets are all significant causes of burn injury. The child should be protected from these causes by reducing the temperature on the hot water in the home, keeping objects such as cigarettes and irons away from children, and placing protective guards over electric outlets when not in use.

The nurse is teaching parents of a preschool child strategies to implement when the child delays going to bed. What strategy should the nurse recommend? a. Use consistent bedtime rituals. b. Give in to attention-seeking behavior. c. Take the child into the parents bed for an hour. d. Allow the child to stay up past the decided bedtime.

ANS: A For children who delay going to bed, a recommended approach involves a consistent bedtime ritual and emphasizing the normalcy of this type of behavior in young children. Parents should ignore attention-seeking behavior, and the child should not be taken into the parents bed or allowed to stay up past a reasonable hour.

The nurse is teaching a mother how to perform chest physical therapy and postural drainage on her 3-year-old child, who has cystic fibrosis. How should the nurse instruct the mother? a. Cover the skin with a shirt or gown before percussing. b. Strike the chest wall with a flat-hand position. c. Percuss over the entire trunk anteriorly and posteriorly. d. Percuss before positioning for postural drainage.

ANS: A For postural drainage and percussion, the child should be dressed in a light shirt to protect the skin and placed in the appropriate postural drainage positions. The chest wall is struck with a cupped-hand, not a flat-hand position. The procedure should be done over the rib cage only. Positioning precedes the percussion.

A young adolescent boy tells the nurse he "feels gawky." How should the nurse explain why this occurs in adolescents? a. Growth of the extremities and neck precedes growth in other areas b. Growth is in the trunk and chest c. The hip and chest breadth increases d. The growth spurt occurs earlier in boys than it does in girls

ANS: A Growth in length of the extremities and neck precedes growth in other areas, and, because these parts are the first to reach adult length, the hands and feet appear larger than normal during adolescence. Increases in hip and chest breadth take place in a few months followed several months later by an increase in shoulder width. These changes are followed by increases in length of the trunk and depth of the chest. This sequence of changes is responsible for the characteristic long-legged, gawky appearance of early adolescent children. The growth spurt occurs earlier in girls than in boys.

Matthew, age 18 months, has just been admitted with croup. His parent is tearful and tells the nurse, "This is all my fault. I should have taken him to the doctor sooner so he wouldn't have to be here." Which is appropriate in the care plan for this parent who is experiencing guilt? a. Clarify misconception about the illness. b. Explain to the parent that the illness is not serious. c. Encourage the parent to maintain a sense of control. d. Assess further why the parent has excessive guilt feelings.

ANS: A Guilt is a common response of parents when a child is hospitalized. They may blame themselves for the child's illness or for not recognizing it soon enough. The nurse should clarify the nature of the problem and reassure parents that the child is being cared for. Croup is a potentially serious illness. The nurse should not minimize the parent's feelings. It would be difficult for the parent to maintain a sense of control while the child is seriously ill. No further assessment is indicated at this time; guilt is a common response for parents.

Which statement is correct about childhood obesity? a. Heredity is an important factor in the development of obesity. b. Childhood obesity in the United States is decreasing. c. Childhood obesity is the result of inactivity. d. Childhood obesity can be attributed to an underlying disease in most cases.

ANS: A Heredity is an important fact that contributes to obesity. Identical twins reared apart tend to resemble their biologic parents to a greater extent than their adoptive parents. It is difficult to distinguish between hereditary and environmental factors. The number of overweight children is increasing in the United States. Inactivity is related to childhood obesity, but it is not the only component. Underlying diseases such as hypothyroidism and hyperinsulinism account for only a small number of cases of childhood obesity.

A hospitalized child with minimal change nephrotic syndrome is receiving high doses of prednisone. Which is an appropriate nursing goal related to this? a. Prevent infection b. Stimulate appetite c. Detect evidence of edema d. Ensure compliance with prophylactic antibiotic therapy

ANS: A High-dose steroid therapy has an immunosuppressant effect. These children are particularly vulnerable to upper respiratory tract infections. A priority nursing goal is to minimize the risk of infection by protecting the child from contact with infectious individuals. Appetite is increased with prednisone therapy. The amount of edema should be monitored as part of the disease process, not necessarily related to the administration of prednisone. Antibiotics would not be used as prophylaxis.

The parents of a child on a ventilator tell the nurse that their insurance company wants the child to be discharged. They explain that they do not want the child home under any circumstances. What principle should the nurse consider when working with this family? a. Desire to have the child home is essential to effective home care. b. Parents should not be expected to care for a technology-dependent child. c. Having a technology-dependent child at home is better for both the child and the family. d. Parents are not part of the decision-making process because of the costs of hospitalization.

ANS: A Home care requires the family to manage the childs illness, including providing daily hands-on care, monitoring the childs medical condition, and educating others to care for the child. The childs home environment with the childs family is perceived as the best place for the child to be cared for. If the family does not want to or is not able to assume these responsibilities, other arrangements need to be investigated. The family is an essential part of the decision-making process. Without family involvement and support, the technology-dependent child will not be well cared for at home.

When a preschool child is hospitalized without adequate preparation, how does the child often react to the hospitalization? a. Sees it as a punishment b. A threat to child's self-image c. An opportunity for regression d. Loss of companionship with friends

ANS: A If a toddler is not prepared for hospitalization, a typical preschooler fantasy is to attribute the hospitalization to punishment for real or imagined misdeeds. Attributing the hospitalization to punishment for real or imagined misdeeds is a reaction typical of toddler and school-age children when threatened with loss of control.

A 14-year-old boy seems to be always eating, although his weight is appropriate for his height. What is the best explanation for this? a. This is normal because of increase in body mass. b. This is abnormal and suggestive of future obesity. c. His caloric intake would have to be excessive. d. He is substituting food for unfilled needs.

ANS: A In adolescence, nutritional needs are closely related to the increase in body mass. The peak requirements occur in the years of maximal growth. The caloric and protein requirements are higher than at almost any other time of life. Seemingly always eating describes the expected eating pattern for young adolescents; as long as weight and height are appropriate, obesity is not a concern.

The nurse closely monitors the temperature of a child with minimal change nephrotic syndrome. The purpose of this assessment is to detect an early sign of which possible complication? a. Infection b. Hypertension c. Encephalopathy d. Edema

ANS: A Infection is a constant source of danger to edematous children and those receiving corticosteroid therapy. An increased temperature could be an indication of an infection. Temperature is not an indication of hypertension or edema. Encephalopathy is not a complication usually associated with minimal change nephrotic syndrome. The child will most likely have neurologic signs and symptoms.

What is the leading cause of death during the toddler period? a. Injuries b. Infectious diseases c. Childhood diseases d. Congenital disorders

ANS: A Injuries are the most common cause of death in children ages 1 through 4 years. It is the highest rate of death from injuries of any childhood age group except adolescence. Congenital disorders are the second leading cause of death in this age group. Infectious and childhood diseases are less common causes of death in this age group.

Which is the leading cause of death during the toddler period? a. Injuries b. Infectious diseases c. Congenital disorders d. Childhood diseases

ANS: A Injuries are the single most common cause of death in children ages 1 through 4 years. This represents the highest rate of death from injuries of any childhood age group except adolescence. Infectious diseases and childhood diseases are less common causes of deaths in this age group. Congenital disorders are the second leading cause of death in this age group

When is isotretinoin (Accutane) indicated for the treatment of acne during adolescence? a. The acne has not responded to other treatments. b. The adolescent is or may become pregnant. c. The adolescent is unable to give up foods causing acne. d. Frequent washing with antibacterial soap has been unsuccessful.

ANS: A Isotretinoin is reserved for severe cystic acne that has not responded to other treatments. Isotretinoin has teratogenic effects and should never be used when there is a possibility of pregnancy. No correlation exists between foods and acne. Antibacterial soaps are ineffective. Frequent washing with antibacterial soap is not a recommended therapy for acne.

Katie, 4 years old, is admitted to outpatient surgery for removal of a cyst on her foot. Her mother puts the hospital gown on her, but Katie is crying because she wants to leave on her underpants. What is the most appropriate nursing action? a. Allow her to wear her underpants b. Discuss with her mother why this is important to Katie c. Ask her mother to explain to her why she cannot wear them d. Explain in a kind, matter-of-fact manner that this is hospital policy

ANS: A It is appropriate for the child to leave her underpants on. This allows her some measure of control in this procedure, foot surgery. Further discussions may make the child more upset. Katie is too young to understand what hospital policy means.

A 6-year-old child is hospitalized for intravenous (IV) antibiotic therapy. He eats little on his "regular diet" trays. He tells the nurse that all he wants to eat is pizza, tacos, and ice cream. Which is the best nursing action? a. Request these favorite foods for him. b. Identify healthier food choices that he likes. c. Explain that he needs fruits and vegetables. d. Reward him with ice cream at the end of every meal that he eats.

ANS: A Loss of appetite is a symptom common to most childhood illnesses. To encourage adequate nutrition, favorite foods should be requested for the child. These foods provide nutrition and can be supplemented with additional fruits and vegetables. Ice cream and other desserts should not be used as rewards or punishment.

A child is admitted with a suspected diagnosis of Munchausen syndrome by proxy (MSBP). What is an important consideration in the care of this child? a. Monitoring the parents whenever they are with the child b. Reassuring the parents that the cause of the disorder will be found c. Teaching the parents how to obtain necessary specimens d. Supporting the parents as they cope with diagnosis of a chronic illness

ANS: A MSBP refers to an illness that one person fabricates or induces in another. The child must be continuously observed for development of symptoms to determine the cause. MSBP is caused by an individual harming the child for the purpose of gaining attention. Nursing staff should obtain all specimens for analyzing. This minimizes the possibility of the abuser contaminating the sample. The child must be supported through the diagnosis of MSBP. The abuser must be identified and the child protected from that individual

A 3-year-old child has a fever associated with a viral illness. Her mother calls the nurse, reporting a fever of 102° F even though she had acetaminophen 2 hours ago. The nurse's action should be based on which statement? a. Fevers such as this are common with viral illnesses. b. Seizures are common in children when antipyretics are ineffective. c. Fever over 102° F indicates greater severity of illness. d. Fever over 102° F indicates a probable bacterial infection.

ANS: A Most fevers are of brief duration, with limited consequences, and are viral. Little evidence supports the use of antipyretic drugs to prevent febrile seizures. Neither the increase in temperature nor its response to antipyretics indicates the severity or etiology of infection.

Parents tell the nurse that their toddler eats little at mealtime, only sits at the table with the family briefly, and wants snacks all the time. What should the nurse recommend? a. Give her nutritious snacks. b. Offer rewards for eating at mealtimes. c. Avoid snacks so she is hungry at mealtimes. d. Explain to her in a firm manner what is expected of her.

ANS: A Most toddlers exhibit a physiologic anorexia in response to the decreased nutritional requirements associated with the slower growth rate. Parents should assist the child in developing healthy eating habits. Toddlers are often unable to sit through a meal. Frequent nutritious snacks are a good way to ensure proper nutrition. To help with developing healthy eating habits, food should be not be used as positive or negative reinforcement for behavior. The child may develop habits of overeating or eat non-nutritious foods in response. A toddler is not able to understand explanations of what is expected of her and comply with the expectations.

Parents tell the nurse that their toddler daughter eats little at mealtime, only sits at the table with the family briefly, and wants snacks "all the time." Which intervention should the nurse recommend? a. Give her nutritious snacks. b. Offer rewards for eating at mealtimes. c. Avoid snacks so she is hungry at mealtimes. d. Explain to her in a firm manner what is expected of her.

ANS: A Most toddlers exhibit a physiologic anorexia in response to the decreased nutritional requirement associated with the slower growth rate. Parents should help the child develop healthy eating habits. The toddler is often unable to sit through a meal. Frequent nutritious snacks are a good way to ensure proper nutrition. To help with developing healthy eating habits, food should be not be used as positive or negative reinforcement for behavior. The child may develop habits of overeating or eat nonnutritious foods in response.

The nurse is teaching the parents of a child recently diagnosed with ADHD who has been prescribed methylphenidate (Ritalin). Which should the nurse include in teaching about the side effects of methylphenidate? a. "Your child may experience a sense of nervousness." b. "You may see an increase in your child's appetite." c. "Your child may experience daytime sleepiness." d. "You may see a decrease in your child's blood pressure."

ANS: A Nervousness is one of the common side effects of Ritalin. Decreased appetite with subsequent weight loss, insomnia, and increased blood pressure are other common side effects.

For case management to be most effective, who should be recognized as the most appropriate case manager? a. Nurse b. Panel of experts c. Multidisciplinary team d. Insurance company

ANS: A Nursing case managers are ideally suited to provide the care coordination necessary. Care coordination is most effective if a single person works with the family to accomplish the many tasks and responsibilities that are necessary. The family retains the role as primary decision maker. Most likely the insurance company will have a case manager focusing on the financial aspects of care. This does not include coordination of care to assist the family.

Which statement characterizes moral development in the older school-age child? a. They are able to judge an act by the intentions that prompted it rather than just by the consequences. b. Rules and judgments become more absolute and authoritarian. c. They view rule violations in an isolated context. d. They know the rules but cannot understand the reasons behind them.

ANS: A Older school-age children are able to judge an act by the intentions that prompted the behavior rather than just by the consequences. Rules and judgments become less absolute and authoritarian. Rule violation is likely to be viewed in relation to the total context in which it appears. The situation and the morality of the rule itself influence reactions.

The nurse is providing support to a family who is experiencing anticipatory grief related to their child's imminent death. Which of the following is an appropriate nursing intervention? a. Be available to the family. b. Attempt to "lighten the mood." c. Suggest activities to cheer up the family. d. Discourage crying until actual time of death

ANS: A One of the most important nursing interventions of death is the availability of the nurse for the family.

A common parental reaction to a child with special needs is parental overprotection. What parental behavior is suggestive of this behavior? a. Giving inconsistent discipline b. Providing consistent, strict discipline c. Forcing child to help self, even when not capable d. Encouraging social and educational activities not appropriate to child's level of capability

ANS: A Parental overprotection is manifested by the parents' fear of letting the child achieve any new skill; they allow the child to avoid all discipline and cater to every desire to prevent frustration. Overprotective parents do not set limits and or institute discipline; prefer to remain in the role of total caregiver; do not allow the child to perform self-care; and do not encourage the child to participate in social and educational activities.

What manifestation observed by the nurse is suggestive of parental overprotection? a. Gives inconsistent discipline b. Facilitates the childs responsibility for self-care of illness c. Persuades the child to take on activities of daily living even when not able d. Encourages social and educational activities not appropriate to the childs level of capability

ANS: A Parental overprotection is manifested when the parents fear letting the child achieve any new skill, avoid all discipline, and cater to every desire to prevent frustration. Overprotective parents do not allow the child to assume responsibility for self-care of the illness. The parents prefer to remain in the role of total caregiver. The parents do not encourage the child to participate in social and educational activities.

What is descriptive of the nutritional requirements of preschool children? a. The quality of the food consumed is more important than the quantity. b. The average daily intake of preschoolers should be about 3000 calories. c. Nutritional requirements for preschoolers are very different from requirements for toddlers. d. Requirements for calories per unit of body weight increase slightly during the preschool period.

ANS: A Parents need to be reassured that the quality of food eaten is more important than the quantity. Children are able to self-regulate their intake when offered foods high in nutritional value. The average daily caloric intake should be approximately 1800 calories. Toddlers and preschoolers have similar nutritional requirements. There is an overall slight decrease in needed calories and fluids during the preschool period.

What is a priority goal in the postpartum care of an adolescent mother? a. Prevention of subsequent pregnancies b. Ensuring that the father of the baby cares for the child c. Returning the mother to a prepregnancy lifestyle d. Facilitating formula feeding to minimize interruptions

ANS: A Postpartum care of the adolescent is directed at preventing subsequent pregnancies and enhancing life outcomes for the teen parents and child. Health care programs should provide comprehensive contraceptive services at the same time the child is seen for appointments. Ensuring the father of the baby cares for the child is not part of the postpartum care of the mother. The adolescent mother cannot return to a prepregnancy lifestyle. She now has an infant to care for. Breastfeeding is recommended for the infant. The nurse and mother should explore the best nutrition for both the mothers needs and those of the infant.

A child with autism spectrum disorder (ASD) is admitted to the hospital with pneumonia. The nurse should plan which priority intervention when caring for the child? a. Maintain a structured routine and keep stimulation to a minimum. b. Place child in a room with a roommate of the same age. c. Maintain frequent touch and eye contact with the child. d. Take the child frequently to the playroom to play with other children.

ANS: A Providing a structured routine for the child to follow is a key in the management of ASD. Decreasing stimulation by using a private room, avoiding extraneous auditory and visual distractions, and encouraging the parents to bring in possessions the child is attached to may lessen the disruptiveness of hospitalization. Because physical contact often upsets these children, minimum holding and eye contact may be necessary to avoid behavioral outbursts. Children with ASD need to be introduced slowly to new situations, with visits with staff caregivers kept short whenever possible. The playroom would be too overwhelming with new people and situations and should not be a priority of care.

One of the supervisors for a home health agency asks the nurse to give a family of a child with a chronic illness a survey evaluating the nurses and other service providers. How should the nurse recognize this request? a. Appropriate to improve quality of care b. Improper because it is an invasion of privacy c. Inappropriate unless nurses and other providers agree to participate d. Not acceptable because the family lacks remembering necessary to evaluate professionals

ANS: A Quality assessment and improvement activities are essential for virtually all organizations. Family involvement in evaluating a home care plan can occur on several levels. The nurse can ask the family open-ended questions at regular intervals to assess their opinion of the effectiveness of care. Families should also be given an opportunity to evaluate the individual home care nurses, the home care agency, and other service providers periodically. Evaluation of the provision of care to the patient and family requires evaluation of the care provider, that is, the nurse. Quality-monitoring activities are required by virtually all health care agencies. During the evaluation process, the family is asked to provide their perceptions of care.

A childs parents ask the nurse many questions about their childs illness and its management. The nurse does not know enough to answer all the questions. What nursing action is most appropriate at this time? a. Tell them, I dont know, but I will find out. b. Suggest that they ask the physician these questions. c. Explain that the nurse cannot be expected to know everything. d. Answer questions vaguely so they do not lose confidence in the nurse.

ANS: A Questions from parents should be answered in a straightforward manner. Stating I dont know or Ill find out is better than pretending to know or giving excuses. Suggesting that they ask the physician these questions is not supportive of the family. The nurses role is to assist the parents in obtaining accurate information about their childs illness and its management. Although the nurse cannot be expected to know everything, it is an unprofessional attitude to state this. Nurses must provide accurate information to the extent possible. Vague answers are not helpful to the family.

A previously "potty-trained" 30-month-old child has reverted to wearing diapers while hospitalized. The nurse should reassure the parents that this is normal because of which reason? a. Regression is seen during hospitalization. b. Developmental delays occur because of the hospitalization. c. The child is experiencing urinary urgency because of hospitalization. d. The child was too young to be "potty-trained."

ANS: A Regression is expected and normal for all age groups when hospitalized. Nurses should assure the parents this is temporary and the child will return to the previously mastered developmental milestone when back home. This does not indicate a developmental delay. The child should not be experiencing urinary urgency because of hospitalization and this would not be normal. Successful "potty-training" can be started at 2 years of age if the child is ready.

An adolescent girl tells the nurse that she is very suicidal. The nurse asks her whether she has a specific plan. Asking this should be considered: a. an appropriate part of the assessment. b. not a critical part of the assessment. c. suggesting that the adolescent needs a plan. d. encouraging the adolescent to devise a plan.

ANS: A Routine health assessments of adolescents should include questions that assess the presence of suicidal ideation or intent. Questions such as, "Have you ever developed a plan to hurt yourself or kill yourself?" should be part of that assessment. Adolescents who express suicidal feelings and have a specific plan are at particular risk and require further assessment and constant monitoring. The information about having a plan is an essential part of the assessment and greatly affects the treatment plan.

Latasha, age 8 years, is being admitted to the hospital from the emergency department with an injury from falling off her bicycle. Which will help her most in her adjustment to the hospital? a. Explain hospital schedules to her, such as mealtimes. b. Use terms such as "honey" and "dear" to show a caring attitude. c. Explain when parents can visit and why siblings cannot come to see her. d. Orient her parents, because she is young, to her room and hospital facility.

ANS: A School-age children need to have control of their environment. The nurse should offer explanations or prepare the child for those experiences that are unavailable. The nurse should refer to the child by the preferred name. Explaining when parents can visit and why siblings cannot come to see her is telling the child all of the limitations, not helping her adjust to the hospital. At the age of 8 years, the child should be oriented to the environment along with the parents.

What is an important consideration in preventing injuries during middle childhood? a. Achieving social acceptance is a primary objective. b. The incidence of injuries in girls is significantly higher than it is in boys. c. Injuries from burns are the highest at this age because of fascination with fire. d. Lack of muscular coordination and control results in an increased incidence of injuries.

ANS: A School-age children often participate in dangerous activities in an attempt to prove themselves worthy of acceptance. The incidence of injury during middle childhood is significantly higher in boys compared with girls. Motor vehicle collisions are the most common cause of severe injuries in children. Children have increasing muscular coordination. Children who are risk takers may have inadequate self-regulatory behavior.

Which is a common characteristic of those who sexually abuse children? a. Pressure victim into secrecy b. Are usually unemployed and unmarried c. Are unknown to victims and victims' families d. Have many victims that are each abused once only

ANS: A Sex offenders may pressure the victim into secrecy regarding the activity as a "secret between us" that other people may take away if they find out. The offender may be anyone, including family members and persons from any level of society. Sex offenders are usually trusted acquaintances of the victims and victims' families. Many victims are abused many times over a long period.

A 9-year-old girl often comes to the school nurse complaining of stomach pains. Her teacher says she is completing her schoolwork satisfactorily but lately has been somewhat aggressive and stubborn in the classroom. How should the nurse interpret this behavior? a. A sign of stress b. A developmental delay c. A physical problem d. A lack of adjustment to school

ANS: A Signs of stress include stomach pains or headache, sleep problems, bed-wetting, changes in eating habits, aggressive or stubborn behavior, reluctance to participate, or regression to early behaviors. This child is exhibiting signs of stress

A 9-year-old girl often comes to the school nurse complaining of stomach pains. Her teacher says she is completing her school work satisfactorily but lately has been somewhat aggressive and stubborn in the classroom. The school nurse should recognize this as which? a. Signs of stress b. Developmental delay c. Lack of adjustment to school environment d. Physical problem that needs medical intervention

ANS: A Signs of stress include stomach pains or headache, sleep problems, bedwetting, changes in eating habits, aggressive or stubborn behavior, reluctance to participate, or regression to earlier behaviors. The child is completing school work satisfactorily; any developmental delay would have been diagnosed earlier. The teacher reports that this is a departure from the childs normal behavior. Adjustment issues would most likely be evident soon after a change. Medical intervention is not immediately required. Recognizing that this constellation of symptoms can indicate stress, the nurse should help the child identify sources of stress and how to use stress reduction techniques. The parents are involved in the evaluation process.

Which is the most significant factor in distinguishing those who commit suicide from those who make suicidal attempts or threats? a. Social isolation b. Level of stress c. Degree of depression d. Desire to punish others

ANS: A Social isolation is a significant factor in distinguishing adolescents who will kill themselves from those who will not. It is also more characteristic of those who complete suicide than of those who make attempts or threats. Level of stress, degree of depression, and desire to punish others are contributing factors in suicide, but they are not the most significant factor in distinguishing those who complete suicide from those who attempt suicide.

The nurse is conducting an assessment on a school-age child with urosepsis. Which assessment finding should the nurse expect? a. Fever with a positive blood culture b. Proteinuria and edema c. Oliguria and hypertension d. Anemia and thrombocytopenia

ANS: A Symptoms of urosepsis include a febrile UTI coexisting with systemic signs of bacterial illness; blood culture reveals presence of urinary pathogen. Proteinuria and edema are symptoms of minimal change nephrotic syndrome (MCNS). Oliguria and hypertension are symptoms of acute glomerulonephritis (AGN). Anemia and thrombocytopenia are symptoms of hemolytic uremic syndrome (HUS).

Parents are concerned that their child is showing aggressive behaviors. Which suggestion should the nurse make to the parents? a. Supervise television viewing. b. Ignore the behavior. c. Punish the child for the behavior. d. Accept the behavior if the child is male.

ANS: A Television is also a significant source for modeling at this impressionable age. Research indicates there is a direct correlation between media exposure, both violent and educational media, and preschoolers exhibiting physical and relational aggression (Ostrov, Gentile, and Crick, 2006). Therefore, parents should be encouraged to supervise television viewing. The behavior should not be ignored because it can escalate to hyperaggression. The child should not be punished because it may reinforce the behavior if the child is seeking attention. For example, children who are ignored by a parent until they hit a sibling or the parent learn that this act garners attention. The behavior should not be accepted from a male child; this is using a "double standard" and aggression should not be equated with masculinity.

A 10-year-old girl needs to have another intravenous (IV) line started. She keeps telling the nurse, "Wait a minute" and "I'm not ready." The nurse should recognize this as which description? a. This is normal behavior for a school-age child. b. The behavior is not seen past the preschool years. c. The child thinks the nurse is punishing her. d. The child has successfully manipulated the nurse in the past.

ANS: A The 10-year-old girl is attempting to maintain control. The nurse should provide the girl with structured choices about when the IV will be inserted. Telling the nurse "Wait a minute" and "I'm not ready" can be characteristic behavior when an individual needs to maintain some control over a situation.

How does the onset of the pubertal growth spurt compare in girls and boys? a. In girls, it occurs about 1 year before it appears in boys. b. In girls, it occurs about 3 years before it appears in boys. c. In boys. it occurs about 1 year before it appears in girls. d. It is about the same in both boys and girls.

ANS: A The average age of onset is 9 1/2 years for girls and 10 1/2 years for boys. Although pubertal growth spurts may occur in girls 3 years before it appears in boys on an individual basis, the average difference is 1 year. Usually girls begin their pubertal growth spurt earlier than boys.

The parents of a 2-year-old tell the nurse that they are concerned because the toddler has started to use "baby talk" since the arrival of their new baby. The nurse should recommend which intervention? a. Ignore the "baby talk." b. Explain to the toddler that "baby talk" is for babies. c. Tell the toddler frequently, "You are a big kid now." d. Encourage the toddler to practice more advanced patterns of speech.

ANS: A The baby talk is a sign of regression in the toddler. It should be ignored, while praising the child for developmentally appropriate behaviors. Regression is children's way of expressing stress. The parents should not introduce new expectations and allow the child to master the developmental tasks without criticism.

A nurse is caring for a hearing-impaired child who lip reads. The nurse should plan which intervention to facilitate lipreading? a. Speak at an even rate. b. Exaggerate pronunciation of words. c. Avoid using facial expressions. d. Repeat in exactly the same way if the child does not understand.

ANS: A The child should be helped to learn and understand how to read lips by speaking at an even rate. It interferes with the child's comprehension of the spoken word to exaggerate pronunciation of words, to avoid using facial expressions, and to repeat in exactly the same way if the child does not understand.

Which technique is best for dealing with the negativism of the toddler? a. Offer the child choices. b. Remain serious and intent. c. Provide few or no choices for child. d. Quietly and calmly ask the child to comply.

ANS: A The child should have few opportunities to respond in a negative manner. Questions and requests should provide choices. This allows the child to be in control and reduces opportunities for negativism. The child will continue trying to assert control. The toddler is too young for verbal explanations. The negativism is the child testing limits. These should be clearly defined by structured choices.

When should children with cognitive impairment be referred for stimulation and educational programs? a. As young as possible b. As soon as they have the ability to communicate in some way c. At age 3 years, when schools are required to provide services d. At age 5 or 6 years, when schools are required to provide services

ANS: A The child's education should begin as soon as possible. Considerable evidence exists that early intervention programs for children with disabilities are valuable for cognitively impaired children. The early intervention may facilitate the child's development of communication skills. States are encouraged to provide early intervention programs from birth under Public Law 101-476, the Individuals with Disabilities Education Act.

A sexually active female adolescent asks the nurse about the contraceptive Depo-Provera. What should the nurse explain regarding the contraceptive? a. Requires injections every 3 months b. Requires daily administration of medication by mouth c. Provides long-term continuous protection, up to 5 years d. Prevents pregnancy if given within 72 hours of unprotected sex

ANS: A The contraceptive Depo-Provera is administered by injection every 3 months. Oral contraceptives, not Depo-Provera, require daily administration of medication by mouth. Norplant, not Depo-Provera, provides long-term continuous protection for up to 5 years. Postcoital contraception, not Depo-Provera, prevents pregnancy if given within 72 hours of unprotected sex.

The nurse is discussing with a parent group the importance of fluoride for healthy teeth. What should the nurse recommend? a. Determine whether the water supply is fluoridated. b. Use fluoridated mouth rinses in children older than 1 year. c. Give fluoride supplements to infants beginning at age 2 months. d. Brush teeth with fluoridated toothpaste unless the fluoride content of water supply is adequate.

ANS: A The decision about fluoride supplementation cannot be made until it is known whether the water supply contains fluoride and the amount. It is difficult to teach toddlers to spit out mouthwash. Swallowing fluoridated mouthwashes can contribute to fluorosis. Fluoride supplementation is not recommended until after age 6 months and then only if the water is not fluoridated. Fluoridated toothpaste is still indicated if the fluoride content of the water supply is adequate, but very small amounts are used.

1. What laboratory finding should the nurse expect in a child with an excess of water? a. Decreased hematocrit b. High serum osmolality c. High urine specific gravity d. Increased blood urea nitrogen

ANS: A The excess water in the circulatory system results in hemodilution. The laboratory results show a falsely decreased hematocrit. Laboratory analysis of blood that is hemodiluted reveals decreased serum osmolality and blood urea nitrogen. The urine specific gravity is variable relative to the childs ability to correct the fluid imbalance.

When teaching a mother how to administer eye drops, where should the nurse tell her to place them? a. In the conjunctival sac that is formed when the lower lid is pulled down b. Carefully under the eye lid while it is gently pulled upward c. On the sclera while the child looks to the side d. Anywhere as long as drops contact the eye's surface

ANS: A The lower lid is pulled down, forming a small conjunctival sac. The solution or ointment is applied to this area. The medication should not be administered directly on the eyeball.

The parent of 16-month-old child asks, What is the best way to keep my child from getting into our medicines at home? What should the nurse advise? a. All medicines should be locked securely away. b. The medicines should be placed in high cabinets. c. Your child just needs to be taught not to touch medicines. d. Medicines should not be kept in the homes of small children

ANS: A The major reason for poisoning in the home is improper storage. Toddlers can climb, unlatch cabinets, and obtain access to high-security places. For medications, only a locked cabinet is safe. Toddlers can climb using furniture. High places are not a deterrent to an exploring toddler. Toddlers are not able to generalize that all the different forms of medications in the home may be dangerous. Keeping medicines out of the homes of small children is not feasible because many parents require medications for chronic or acute illnesses. Parents must be taught safe storage for their home and when they visit other homes.

The parent of a 16-month-old toddler asks, "What is the best way to keep our son from getting into our medicines at home?" The nurse's best advice is: a. "All medicines should be locked securely away." b. "The medicines should be placed in high cabinets." c. "The child just needs to be taught not to touch medicines." d. "Medicines should not be kept in the homes of small children."

ANS: A The major reason for poisoning in the home is improper storage. Toddlers can climb, unlatch cabinets, and obtain access to high-security places. For medications, only a locked cabinet is safe. Toddlers can climb by using furniture. High places are not a deterrent to an exploring toddler. Toddlers are not able to generalize as dangerous all the different forms of medications that may be available in the home. It is not feasible to not keep medicines in the homes of small children. Many parents require medications for chronic illnesses. Parents must be taught safe storage for their home and when they visit other homes.

A nurse is caring for four patients; three are toddlers and one is a preschooler. Which represents the major stressor of hospitalization for these four patients? a. Separation anxiety b. Loss of control c. Fear of bodily injury d. Fear of pain

ANS: A The major stressor for children from infancy through the preschool years is separation anxiety, also called anaclitic depression. This is a major stressor of hospitalization. Loss of control, fear of bodily injury, and fear of pain are all stressors associated with hospitalization. However, separation from family is a primary stressor in this age group.

A 14-year-old boy and his parents are concerned about bilateral breast enlargement. The nurse's discussion of this should be based on which statement? a. This is usually benign and temporary. b. This is usually caused by Klinefelter syndrome. c. Administration of estrogen effectively reduces gynecomastia. d. Administration of testosterone effectively reduces gynecomastia.

ANS: A The male breast responds to hormonal changes. Some degree of bilateral or unilateral breast enlargement occurs frequently in boys during puberty. Although individuals with Klinefelter syndrome can have gynecomastia, it is not a common cause for male breast enlargement. Estrogen is not a therapy for gynecomastia. Administration of testosterone has no benefit for gynecomastia and may aggravate the condition.

A nurse places some x-ray contrast the toddler is to drink in a small cup instead of a large cup. Which concept of a toddler's preoperational thinking is the nurse using? a. Inability to conserve b. Magical thinking c. Centration d. Irreversibility

ANS: A The nurse is using the toddler's inability to conserve. This is when the toddler is unable to understand the idea that a mass can be changed in size, shape, volume, or length without losing or adding to the original mass. Instead, toddlers judge what they see by the immediate perceptual clues given to them. A small glass means less amount of contrast. Magical thinking is believing that thoughts are all-powerful and can cause events. Centration is focusing on one aspect rather than considering all possible alternatives. Irreversibility is the inability to undo or reverse the actions initiated, such as being unable to stop doing an action when told.

The nurse is preparing a 12-year-old girl for a bone marrow aspiration. The girl tells the nurse she wants her mother with her "like before." What is the most appropriate nursing action? a. Grant her request b. Explain why this is not possible c. Identify an appropriate substitute for her mother d. Offer to provide support to her during the procedure

ANS: A The parent's preferences for assisting, observing, or waiting outside the room should be assessed, along with the child's preference for parental presence. The child's choice should be respected. If the mother and child are agreeable, then the mother is welcome to stay. An appropriate substitute for the mother is necessary only if the mother does not wish to stay. Support is offered to the child regardless of parental presence.

The nurse is caring for a child who has just died. The parents ask to be left alone so that they can rock their child one more time. What is the nurse's most appropriate response? a. Grant their request b. Assess why they feel this is necessary c. Discourage this because it will only prolong their grief d. Kindly explain that they need to say good-bye to their child now and leave

ANS: A The parents should be allowed to remain with their child after the death. The nurse can remove all the tubes and equipment and offer the parents the option of preparing the body.

A boy age 4 1/2 years has been having increasingly frequent angry outbursts in preschool. He is aggressive toward the other children and the teachers. This behavior has been a problem for approximately 8 to 10 weeks. His parent asks the nurse for advice. What is the most appropriate intervention? a. Refer the child for a professional psychosocial assessment. b. Explain that this is normal in preschoolers, especially boys. c. Encourage the parent to try more consistent and firm discipline. d. Talk to the preschool teacher to obtain validation for behavior parent reports.

ANS: A The preschool years are a time when children learn socially acceptable behavior. The difference between normal and problematic behavior is not the behavior but the severity, frequency, and duration. This childs behavior meets the definition requiring professional evaluation. Some aggressive behavior is within normal limits, but at 8 to 10 weeks, this behavior has persisted too long. There is no indication that the parent is using inconsistent discipline. A part of the evaluation is to obtain validation for behavior parent reports.

A mother calls the school nurse saying that her daughter has developed a school phobia. She has been out of school 3 days. The nurse's recommendations should include which intervention? a. Immediately return the child to school. b. Explain to the child that this is the last day she can stay home. c. Determine the cause of phobia before returning the child to school. d. Seek professional counseling before forcing the child to return to school.

ANS: A The primary goal is to return the child to school. Parents must be convinced gently, but firmly, that immediate return is essential and that it is their responsibility to insist on school attendance. The longer the child is permitted to stay out of school, the more difficult it will be for the child to reenter. Trying to find the cause of phobia will only delay the return to school and inhibit the child's ability to cope. Professional counseling is recommended if the problem persists, but the child's return to school should not wait for the counseling.

The nurse is conducting an admission assessment on a school-age child with acute renal failure. Which are the primary clinical manifestations the nurse expects to find with this condition? a. Oliguria and hypertension b. Hematuria and pallor c. Proteinuria and muscle cramps d. Bacteriuria and facial edema

ANS: A The principal feature of acute renal failure is oliguria; hypertension is a nonspecific clinical manifestation. Hematuria and pallor, proteinuria and muscle cramps, and bacteriuria and facial edema are not principal features of acute renal failure.

What nursing intervention is most appropriate in promoting normalization in a school-age child with a chronic illness? a. Give the child as much control as possible. b. Ask the childs peer to make the child feel normal. c. Convince the child that nothing is wrong with him or her. d. Explain to parents that family rules for the child do not need to be the same as for healthy siblings.

ANS: A The school-age child who is ill may be forced into a period of dependency. To foster normalcy, the child should be given as much control as possible. It is unrealistic for one individual to make the child feel normal. The child has a chronic illness, so it would be unacceptable to convince the child that nothing is wrong. The family rules should be similar for each of the children in a family. Resentment and hostility can arise if different standards are applied to each child.

Which is the most appropriate nursing intervention to promote normalization in a school-age child with a chronic illness? a. Give the child as much control as possible. b. Ask the child's peer to make the child feel normal. c. Convince the child that nothing is wrong with him or her. d. Explain to parents that family rules for the child do not need to be the same as for healthy siblings.

ANS: A The school-age child who is ill may be forced into a period of dependency. To foster normalcy, the child should be given as much control as possible. It is unrealistic to expect one individual to make the child feel normal. The child has a chronic illness. It would be unacceptable to convince the child that nothing is wrong. The family rules should be similar for each of the children in a family. Resentment and hostility can arise if different standards are applied to each child.

The nurse is teaching parents about avoiding accidental burns with their toddler. What water heater setting should the nurse recommend to the parents? a. 120 F b. 130 F c. 140 F d. 150 F

ANS: A The water heater should be set to limit household water temperatures to less than 49 C (120 F). At this temperature, it takes 10 minutes for exposure to the water to cause a full-thickness burn. Conversely, water temperatures of 54 C (130 F), the usual setting of most water heaters, expose household members to the risk of full-thickness burns within 30 seconds.

What is the earliest age at which puberty begins? a. 9 b. 10 c. 11 d. 12

ANS: A There is no universal age at which children assume the characteristics of prepubescence. The first physiologic signs appear at about 9 years of age (particularly in girls) and are usually clearly evident in 11- to 12-year-old children.

The nurse is talking with the parents of a child who died 6 months ago. They sometimes still "hear" the child's voice and have trouble sleeping. They describe feeling "empty" and depressed. How should the nurse interpret these feelings? a. These are normal grief responses b. The pain of the loss is usually less by this time c. These grief responses are more typical of the early stages of grief d. This grieving is essential until the pain is gone and the child is gradually forgotten.

ANS: A These are normal grief responses. The process of grief work is lengthy

A pregnant 15-year-old adolescent tells the nurse that she did not use any form of contraception because she was afraid her parents would find out. The nurse should recognize what? a. This is a frequent reason given by adolescents. b. This suggests a poor parentchild relationship. c. This is not a good reason to not get contraception. d. This indicates that the adolescent is unaware of her legal rights.

ANS: A This is one of the most common reasons given by teenagers for not using contraception. Although it is optimum for the parents to be involved in the health care of adolescents, some adolescents require confidential care. Privacy is important as they develop their personal identity and establish relationships. The adolescent may be concerned about parental judgment. The adolescent should discuss with the health care provider contraception that meets her needs; some of the longer acting birth control methods may be preferable. The adolescent did not tell the nurse that she was unaware that she could legally obtain contraceptive materials; she was concerned about her parents.

The potential effects of chronic illness or disability on a childs development vary at different ages. What developmental alteration is a threat to a toddlers normal development? a. Hindered mobility b. Limited opportunities for socialization c. Childs sense of guilt that he or she caused the illness or disability d. Limited opportunities for success in mastering toilet training

ANS: A Toddlers are acquiring a sense of autonomy, developing self-control, and forming symbolic representation through language acquisition. Mobility is the primary tool used by toddlers to experiment with maintaining control. Loss of mobility can create a sense of helplessness. Toddlers do not socialize. They are sensitive to changes in family routines. A sense of guilt is more likely to occur in a preschooler. Toilet training is not usually mastered until the end of the toddler period.

Parents of an 18-month-old boy tells the nurse that he says no to everything and has rapid mood swings. If he is scolded, he shows anger and then immediately wants to be held. What is the nurses best interpretation of this behavior? a. This is normal behavior for his age. b. This is unusual behavior for his age. c. He is not effectively coping with stress. d. He is showing he needs more attention.

ANS: A Toddlers use distinct behaviors in the quest for autonomy. They express their will with continued negativity and use of the word no. Children at this age also have rapid mood swings. The nurse should reassure the parents that their child is engaged in expected behavior for an 18-month-old.

A parent of an 18-month-old boy tells the nurse that he says "no" to everything and has rapid mood swings. If he is scolded, he shows anger and then immediately wants to be held. The nurse's best interpretation of this behavior is included in which statement? a. This is normal behavior for his age. b. This is unusual behavior for his age. c. He is not effectively coping with stress. d. He is showing he needs more attention.

ANS: A Toddlers use distinct behaviors in the quest for autonomy. They express their will with continued negativity and the use of the word "no." Children at this age also have rapid mood swings. The nurse should reassure the parents that their child is engaged in expected behavior for an 18-month-old. Having a rapid mood swing is an expected behavior for a toddler.

Which is a useful skill that the nurse should expect a 5-year-old child to be able to master? a. Tie shoelaces b. Use knife to cut meat c. Hammer a nail d. Make change out of a quarter

ANS: A Tying shoelaces is a fine motor task of 5-year-olds. Using a knife to cut meat is a fine motor task of a 7-year-old. Hammering a nail and making change out of a quarter are fine motor and cognitive tasks of an 8- to 9-year-old.

When a child has chronic renal failure, the progressive deterioration produces a variety of clinical and biochemical disturbances that eventually are manifested in the clinical syndrome known as: a. uremia. b. oliguria. c. proteinuria. d. pyelonephritis.

ANS: A Uremia is the retention of nitrogenous products, producing toxic symptoms. Oliguria is diminished urinary output. Proteinuria is the presence of protein, usually albumin, in the urine. Pyelonephritis is an inflammation of the kidney and renal pelvis.

The nurse notes that the parents of a critically ill child spend a large amount of time talking with the parents of another child who is also seriously ill. They talk with these parents more than with the nurses. How should the nurse interpret this situation? a. Parent-to-parent support is valuable. b. Dependence on other parents in crisis is unhealthy. c. This is occurring because the nurses are unresponsive to the parents. d. This has the potential to increase friction between the parents and nursing staff.

ANS: A Veteran parents share experiences that cannot be supplied by other support systems. They have known the stress related to diagnosis, have weathered the many transition times, and have a practical remembering of resources. The parents can be mutually supportive during times of crisis. Nursing staff cannot provide the type of support that is realized from other parents who are experiencing similar situations. Friction should not exist between the nursing staff and the family of the child who is critically ill.

What statement is correct about young children who report sexual abuse? a. They may exhibit various behavioral manifestations. b. In more than half the cases, the child has fabricated the story. c. Their stories should not be believed unless other evidence is apparent. d. They should be able to retell the story the same way to another person.

ANS: A Victims of sexual abuse have no typical profile. The child may exhibit various behavioral manifestations, none of which is diagnostic for sexual abuse. When children report potentially sexually abusive experiences, their reports need to be taken seriously. Other children in the household also need to be evaluated. In children who are sexually abused, it is often difficult to identify other evidence. In one study, approximately 96% of children who were sexually abused had normal genital and anal findings. The ability to retell the story is partly dependent on the childs cognitive level. Children who repeatedly tell identical stories may have been coached.

During a well-child visit, the nurse practitioner provides guidance about promoting healthy eating in a child who is overweight. What does the nurse advise? a. Slow down eating meals. b. Avoid between-meal snacks. c. Include low-fat foods in meals. d. Use foods that child likes as special treats

ANS: A When a child slows down the eating process, it is easier to recognize signs of fullness. If food is consumed rapidly, this feedback is lost. Regular meals and snacks are encouraged to prevent the child from becoming too hungry and overeating. Low-fat foods are usually higher in calories than the regular versions. Nutritional labels should be checked and foods high in sugar and calories avoided. Food should not be used as a special treat or reward; this encourages the child to use food as comfort measures in response to boredom and stress.

A cure is no longer possible for a young child with cancer. The nursing staff recognizes that the goal of treatment must shift from cure to palliation. Which is an important consideration at this time? a. The family is included in the decision to shift the goals of treatment. b. The decision must be made by the health professionals involved in the child's care. c. The family needs to understand that palliative care takes place in the home. d. The decision should not be communicated to the family because it will encourage a sense of hopelessness.

ANS: A When the child reaches the terminal stage, the nurse and physician should explore the family's wishes. The family should help decide what interventions will occur as they plan for their child's death.

The nurse is caring for an adolescent brought to the hospital with acute drug toxicity. Cocaine is believed to be the drug involved. Data collection should include what information? a. Mode of administration b. Drug's actual content c. Function the drug plays in the adolescent's life d. Adolescent's level of interest in rehabilitation

ANS: A When the drug is questionable or unknown, every effort must be made to determine the type, amount of drug taken, the mode and time of administration, and factors relating to the onset of presenting symptoms. The actual content of most street drugs is highly questionable. Pharmacologic agents should be administered with caution, except for the narcotic antagonists in case of suspected opioid use. The function the drug plays in the adolescent's life and the adolescent's level of interest in rehabilitation are important considerations in the long-term management during the non-acute stage.

A sexually active adolescent asks the school nurse about prevention of sexually transmitted infections (STIs). What should the nurse recommend? a. Use of condoms b. Prophylactic antibiotics c. Any type of contraception method d. Withdrawal method of contraception

ANS: A When used appropriately, condoms provide a barrier to the organisms that cause STIs. Prophylactic antibiotics are not recommended; they are effective only against bacteria, not viruses. Only condoms create a physical barrier that prevents contact with the organisms.

The nurse is providing guidance strategies to a group of parents with toddlers at a community outreach program. Which statement by a parent indicates a correct understanding of the teaching? a. I should expect my 24-month-old child to express some signs of readiness for toilet training. b. I should be firm and structured when disciplining my 18-month-old child. c. I should expect my 12-month-old child to start to develop a fear of darkness and to need a security blanket. d. I should expect my 36-month-old child to understand time and proximity of events.

ANS: A A 24-month-old toddler starts to show readiness for toilet training; it is important for the parent to be aware of this and be ready to start the process. At 18 months of age, a child needs consistent but gentle discipline because the child cannot yet understand firmness and structure with discipline. Development of fears and need for security items usually occurs at the end of the 18-to 24-month stage. A 36-month-old child does not yet understand time and proximity of events, so the parent needs to understand that the toddler cannot hurry up or we will be late

A parent asks the nurse about the characteristics of a nightmare. What response should the nurse give to the parent? (Select all that apply.) a. Nightmares are scary dreams. b. The child can describe the nightmare. c. The child is reassured by your presence. d. Nightmares occur usually 1 to 4 hours after falling asleep. e. Nightmares take place during nonrapid eye movement sleep

ANS: A, B, C Nightmares are scary dreams, the child can describe the nightmare, and the child is reassured by a parents presence. Sleep terrors occur usually 1 to 4 hours after falling asleep, but nightmares occur in the second half of sleep. Sleep terrors occur during nonrapid eye movement sleep, but nightmares occur during rapid eye movement sleep.

Parents ask the nurse, Should we be concerned our preschooler has an imaginary friend, and how should we react? Which responses should the nurse give to the parents? (Select all that apply.) a. The imaginary playmate is a sign of health. b. You can acknowledge the presence of the imaginary companion. c. It is normal for a preschool-aged child to have an imaginary friend. d. If your child wants a place setting at the table for the child, it is best to refuse. e. It is OK to allow the child to blame the imaginary playmate to avoid punishment.

ANS: A, B, C Parents should be reassured that the childs fantasy is a sign of health that helps differentiate between make-believe and reality. Parents can acknowledge the presence of the imaginary companion by calling him or her by name and even agreeing to simple requests such as setting an extra place at the table, but they should not allow the child to use the playmate to avoid punishment or responsibility.

What are the goals of organized athletics for preadolescent children? (Select all that apply.) a. Physical fitness b. Basic motor skills c. A positive self-image d. Commitment to winning

ANS: A, B, C The goals of organized athletics for preadolescent children include physical fitness, basic motor skills, and a positive self-image. The commitment is to the values of teamwork, fair play, and sportsmanship, not to winning.

The nurse is teaching coping strategies to parents of a child with a chronic illness. What coping strategies should the nurse include? (Select all that apply.) a. Listen to the child. b. Accept the childs illness. c. Establish a support system. d. Learn to care for the childs illness one day at a time. e. Do not share information with the child about the illness.

ANS: A, B, C, D Coping strategies for parents caring for a child with a chronic illness include listening to the child, accepting the childs illness, establishing a support system, and learning to care for the childs illness one day at a time. Information should be shared with the child about the illness.

What are supportive interventions that can assist a preschooler with a chronic illness to meet developmental milestones? (Select all that apply.) a. Encourage socialization. b. Encourage mastery of self-help skills. c. Provide devices that make tasks easier. d. Clarify that the cause of the childs illness is not his or her fault. e. Discuss planning for the future and how the condition can affect choices.

ANS: A, B, C, D To encourage initiative, mastery of self-help skills should be encouraged, and devices should be provided that make tasks easier. To develop peer relationships, socialization should be encouraged. To develop body image, the fact that the cause of the childs illness is not the fault of the child should be emphasized. Discussing planning for the future and how the condition can affect choices is appropriate for an adolescent.

1. What methods should the nurse use to measure compliance to a treatment plan? (Select all that apply.) a. Pill counts b. Chemical assays c. Direct observation d. Third-party reporting e. Monitoring therapeutic response

ANS: A, B, C, E Assessment of compliance must include direct measurement techniques. Pill counts, chemical assays, direct observation, and monitoring therapeutic response are direct measurement techniques. Third-party reporting would not always be available and would not be a method to measure compliance.

1. What play activities should the nurse implement to encourage fluid intake for a child? (Select all that apply.) a. Have a tea party. b. Use a crazy straw. c. Cut gelatin into fun shapes. d. Place liquid in large Styrofoam cups. e. Make ice pops using the childs favorite juice.

ANS: A, B, C, E Play activities to encourage fluid intake for a child include tea parties, crazy straws, cutting gelatin into fun shapes, and making ice pops using the childs favorite juice. Small cups, not large Styrofoam cups, should be used.

The nurse relates to parents that there are some beneficial effects of hospitalization for their child. What are beneficial effects of hospitalization? (Select all that apply.) a. Recovery from illness b. Improve coping abilities c. Opportunity to master stress d. Provide a break from school e. Provide new socialization experiences

ANS: A, B, C, E The most obvious benefit is the recovery from illness, but hospitalization also can present anopportunityforchildrentomasterstressandfeelcompetentintheircopingabilities.Thehospitalenvironment can provide children with new socialization experiences that can broaden theirinterpersonalrelationships.Havingabreakfromschoolisnotabenefitofhospitalization

The nurse is preparing an education program on hearing impairment for a group of new staff nurses. What concepts should be included? (Select all that apply.) a. A child with a slight hearing loss is usually unaware of a hearing difficulty. b. A clinical manifestation of a hearing impairment in children is avoidance of social interaction. c. A child with a severe hearing loss may hear a loud voice if nearby. d. Children with sensorineural hearing loss can benefit from the use of a hearing aid. e. A clinical manifestation of hearing impairment in an infant is lack of the startle reflex. f. Identification of a hearing loss after the first year is essential to facilitate language development in children.

ANS: A, B, C, E When discussing hearing impairment in children, the nurse should include information about differences in hearing losses, such as with a slight hearing loss, the child is usually unaware of a hearing difficulty, and with a severe loss, the child may hear a loud noise if it is nearby. An infant with a hearing loss may lack the startle response, and a hearing impaired child may avoid social interaction. Children with a sensorineural hearing loss would not benefit from a hearing aid. Identification of a hearing loss is imperative in the first 3 to 6 months to facilitate language and educational development for children.

What are characteristics of middle adolescence (1517 years) with regard to relationships with peers? (Select all that apply.) a. Behavioral standards set by peer group b. Acceptance of peers extremely important c. Seeks peer affiliations to counter instability d. Exploration of ability to attract opposite sex e. Peer group recedes in importance in favor of individual friendship

ANS: A, B, D Characteristics of middle adolescence relationships with peers include behavioral standards set by the peer group, acceptance of peers is extremely important, and exploration of the ability to attract opposite sex. Seeking peer affiliations to counter instability is a characteristic of early adolescence relationships with peers. Peer groups receding in importance in favor of individual friendships is characteristic of late adolescence relationships with peers.

The nurse is teaching a school-age child about factors that can delay wound healing. What factors should the nurse include in the teaching session? (Select all that apply.) a. Deficient vitamin C b. Deficient vitamin D c. Increased circulation d. Dry wound environment e. Increase in white blood cells

ANS: A, B, D Factors that delay wound healing are a dry wound environment (allows epithelial cells to dry), deficient vitamin C (inhibits formation of collagen fibers), and deficient vitamin D (regulates growth and differentiation of cell types). Decreased, not increased, circulation delays healing. An increase in the white blood cell count may occur but does not delay healing.

Which assessment findings indicate to the nurse a child has Down syndrome? (Select all that apply.) a. High arched narrow palate b. Protruding tongue c. Long, slender fingers d. Transverse palmar crease e. Hypertonic muscle tone

ANS: A, B, D The assessment findings of Down syndrome include high arched narrow palate, protruding tongue, and transverse palmar creases. The fingers are stubby and the muscle tone is hypotonic, not hypertonic.

What are supportive interventions that can assist an infant with a chronic illness to meet developmental milestones? (Select all that apply.) a. Encourage consistent caregivers. b. Encourage periodic respite from demands of care. c. Encourage one family member to be the primary caretaker. d. Encourage parental rooming in during hospitalization. e. Withhold age-appropriate developmental tasks until the child is older.

ANS: A, B, D To develop trust, consistent caretakers and parents rooming in should be encouraged. To develop a sense of separateness from parents, periodic respites from caregiving should be encouraged. All members of the family, not one primary caretaker, should be encouraged to participate in care. Age-appropriate developmental tasks should be encouraged, not withheld until an older age

Which toys should a nurse provide to promote imaginative play for a 3-year-old hospitalized child? (Select all that apply.) a. Plastic telephone b. Hand puppets c. Jigsaw puzzle (100 pieces) d. Farm animals and equipment e. Jump rope

ANS: A, B, D To promote imaginative play for a 3-year-old child, the nurse should provide: dress-up clothes, dolls, housekeeping toys, dollhouses, play-store toys, telephones, farm animals and equipment, village sets, trains, trucks, cars, planes, hand puppets, or medical kits. A 100-piece jigsaw puzzle and a jump rope would be appropriate for a young, school-age child but not a 3-year-old child.

1. What signs and symptoms are indicative of a urinary tract disorder in the infancy period (124 months)? (Select all that apply.) a. Pallor b. Poor feeding c. Hypothermia d. Excessive thirst e. Frequent urination

ANS: A, B, D, E Signs and symptoms of a urinary tract disorder in the infancy period are pallor, poor feeding, excessive thirst, and frequent urination. Hyperthermia is seen, not hypothermia.

An adolescent asks the nurse about the safety of getting a tattoo. The nurse explains to the adolescent that it is important to find a qualified operator using proper sterile technique because an unsterilized needle or contaminated tattoo ink can cause what? (Select all that apply.) a. Hepatitis C virus b. Hepatitis B virus c. Hepatitis E virus d. Human immunodeficiency virus (HIV) e. Mycobacterium chelonae skin infections

ANS: A, B, D, E Using the same unsterilized needle to tattoo body parts of multiple teenagers presents the same risk for human immunodeficiency virus (HIV), hepatitis C virus, and hepatitis B virus transmission as occurs with other needle-sharing activities. Contaminated tattoo ink can cause nontuberculous M. chelonae skin infections. The hepatitis E virus is transmitted via the fecaloral route, principally via contaminated water, not by contaminated needles.

1. The nurse is caring for a child with a urinary tract infection who is on intravenous gentamicin (Garamycin). What interventions should the nurse plan for this child with regard to this medication? (Select all that apply.) a. Encourage fluids. b. Monitor urinary output. c. Monitor sodium serum levels. d. Monitor potassium serum levels. e. Monitor serum peak and trough levels.

ANS: A, B, E Garamycin can cause renal toxicity and ototoxicity. Fluids should be encouraged and urinary output and serum peak and trough levels monitored. It is not necessary to monitor potassium sodium levels for patients taking this medication.

1. The nurse is caring for a child with a urinary tract infection who is on trimethoprimsulfamethoxazole (Bactrim). What side effects of this medication should the nurse teach to the parents and the child? (Select all that apply.) a. Rash b. Urticaria c. Pneumonitis d. Renal toxicity e. Photosensitivity

ANS: A, B, E Side effects of Bactrim are rash, urticaria, and photosensitivity. Pneumonitis and renal toxicity are not side effects of Bactrim.

What growth and development milestones are expected between the ages of 8 and 9 years? (Select all that apply.) a. Can help with routine household tasks b. Likes the reward system for accomplished tasks c. Uses the telephone for practical purposes d. Chooses friends more selectively e. Goes about home and community freely, alone or with friends f. Enjoys family time and is respectful of parents

ANS: A, B, E Children between the age of 8 and 9 years accomplish many growth and development milestones, including helping with routine household tasks, liking the reward system when a task is accomplished well, and going out with friends or alone more independently and freely. Using the telephone for practical reasons, choosing friends more selectively, and finding enjoyment in family with new-found respect for parents are tasks accomplished between the ages of 10 and 12 years.

Parents of an adolescent ask the school nurse, It is OK for our adolescent to get a job? The nurse should answer telling the parents the effects of adolescents who work more than 20 hours a week are what? (Select all that apply.) a. Can lead to fatigue b. Can lead to poorer grades c. Improves an interest in school d. Enhances development and identity e. Can reduce extracurricular involvement

ANS: A, B, E Detrimental effects are likely for adolescents who work more than 20 hours a week. Greater involvement in work can lead to fatigue, decreased interest in school, reduced extracurricular involvement, and poorer grades. Involvement in work may take time away from other activities that could contribute to identity development. Adolescent work as it exists today may negatively affect development.

A nurse is caring for a child in droplet precautions. Which instructions should the nurse give to the unlicensed assistive personnel caring for this child? (Select all that apply.) a. Wear gloves when entering the room. b. Wear an isolation gown when entering the room. c. Place the child in a special air handling and ventilation room. d. A mask should be worn only when holding the child. e. Wash your hands upon exiting the room.

ANS: A, B, E Droplet transmission involves contact of the conjunctivae or the mucous membranes of the nose or mouth of a susceptible person with large-particle droplets (>5 mm) containing microorganisms generated from a person who has a clinical disease or who is a carrier of the microorganism. Droplets are generated from the source person primarily during coughing, sneezing, or talking and during procedures such as suctioning and bronchoscopy. Gloves, gowns, and a mask should be worn when entering the room. Hand washing when exiting the room should be done with any patient. Because droplets do not remain suspended in the air, special air handling and ventilation are not required to prevent droplet transmission.

The nurse is preparing to administer some iron drops to a toddler. Which factor can increase iron absorption? (Select all that apply.) a. Vitamin A b. Acidity (low pH) c. Phosphates (milk) d. Malabsorptive disorders e. Ascorbic acid (Vitamin C)

ANS: A, B, E Factors that increase iron absorption are vitamin A, acidity (low pH), and ascorbic acid (vitamin C). Phosphates (milk) and malabsorptive disorders decrease absorption of iron.

What guidelines should the nurse use when interviewing adolescents? (Select all that apply.) a. Ensure privacy. b. Use open-ended questions. c. Share your thoughts and assumptions. d. Explain that all interactions will be confidential. e. Begin with less sensitive issues and proceed to more sensitive ones.

ANS: A, B, E Guidelines for interviewing adolescents include ensuring privacy, using open-ended questions, and beginning with less sensitive issues and proceeding to more sensitive ones. The nurse should not share thoughts but maintain objectivity and should avoid assumptions, judgments, and lectures. It may not be possible for all interactions to be confidential. Limits of confidentiality include a legal duty to report physical or sexual abuse and to get others involved if an adolescent is suicidal.

Parents are worried that their preschool-aged child is showing hyperaggressive behavior. What are signs of hyperaggresive behavior? (Select all that apply.) a. Disrespect b. Noncompliance c. Infrequent impulsivity d. Occasional temper tantrums e. Unprovoked physical attacks on other children

ANS: A, B, E Hyperaggressive behavior in preschoolers is characterized by unprovoked physical attacks on other children and adults, destruction of others property, frequent intense temper tantrums, extreme impulsivity, disrespect, and noncompliance.

1. What disease processes require airborne precautions? (Select all that apply.) a. Measles b. Varicella c. Pertussis d. Meningitis e. Tuberculosis

ANS: A, B, E In addition to Standard Precautions, use airborne precautions for patients known or suspected to have serious illnesses transmitted by airborne droplet nuclei. Examples of such illnesses include measles, varicella (including disseminated zoster), and tuberculosis. Pertussis and meningitis require droplet precautions.

What are symptoms of abusive head trauma (AHT) in the more severe form that may be present? (Select all that apply.) a.Seizures b. Posturing c. Tachypnea d. Tachycardia e. Altered level of consciousness

ANS: A, B, E In more severe forms, presenting symptoms of abusive head trauma may include seizures, posturing, alterations in level of consciousness, apnea, bradycardia, or death.

A nurse is teaching parents methods to reduce lead levels in their home. Which should the nurse include in the teaching? (Select all that apply.) a. Plant bushes around the outside of the house. b. Ensure your child eats frequent meals. c. Use hot water from the tap when boiling vegetables. d. Food can be stored in ceramic in the refrigerator. e. Ensure that your child's diet contains sufficient iron and calcium.

ANS: A, B, E Methods to reduce lead levels in homes include: planting bushes around the outside of the house if soil is contaminated with lead, so children cannot play there; ensuring that children eat regular meals because more lead is absorbed on an empty stomach; and ensuring that children's diets contain sufficient iron and calcium. Cold water should only be used for drinking, cooking, and reconstituting powder infant formula. Hot water dissolves lead more quickly than cold water and thus contains higher levels of lead. Do not use pottery or ceramic ware that was inadequately fired or is meant for decorative use for food storage or service

What preventive measures should the nurse teach parents of toddlers to prevent early childhood caries? (Select all that apply.) a. Avoid using a bottle as a pacifier. b. Eliminate bedtime bottles completely. c. Place juice in a bottle for the child to drink. d. Wean from the bottle by 18 months of age. e. Avoid coating pacifiers in a sweet substance.

ANS: A, B, E Prevention of dental caries involves eliminating the bedtime bottle completely, feeding the last bottle before bedtime, substituting a bottle of water for milk or juice, not using the bottle as a pacifier, and never coating pacifiers in sweet substances. Juice in bottles, especially commercially available ready-to-use bottles, is discouraged; these beverages are especially damaging because the sugar is more readily converted to acid. Juice should always be offered in a cup to avoid prolonging the bottle-feeding habit. Toddlers should be encouraged to drink from a cup at the first birthday and weaned from a bottle by 14 months of age, not 18 months.

A nurse is planning care for a 7-year-old child hospitalized with osteomyelitis. Which activities should the nurse plan to bring from the playroom for the child? (Select all that apply.) a. Paper and some paints b. Board games c. Jack-in-the-box d. Stuffed animals e. Computer games

ANS: A, B, E School-age children become fascinated with complex board, card, or computer games that they can play alone, with a best friend, or with a group. They also enjoy sewing, cooking, carpentry, gardening, and creative activities such as painting. Jack-in-the-box and stuffed animals would be appropriate for a toddler or preschool child.

1. The advantages of the ventrogluteal muscle as an injection site in young children include which considerations? (Select all that apply.) a. Less painful than vastus lateralis b. Free of important nerves and vascular structures c. Cannot be used when child reaches a weight of 20 pounds d. Increased subcutaneous fat, which increases drug absorption e. Easily identified by major landmarks

ANS: A, B, E The advantages of the ventrogluteal are being less painful, free of important nerves and vascular lateralis, and easily identified by major landmarks. The major disadvantage is lack of familiarity by health professionals and controversy over whether the site can be used before weight bearing. The use of the ventrogluteal has not been clarified. It has been used in infants, but clinical guidelines address the need for the child to be walking, thus generally being over 20 pounds. The site has less subcutaneous tissue, which facilitates intramuscular (rather than subcutaneous) deposition of the drug.

The nurse is teaching parents of a 4-year-old child about socialization developmental milestones. What milestones should the nurse include in the teaching session? (Select all that apply.) a. Very independent b. Has mood swings c. Has better manners d. Eager to do things right e. Tends to be selfish and impatient

ANS: A, B, E The socialization milestones of a 4-year-old child include being very independent, having moods swings, and tending to be selfish and impatient. Having better manners and being eager to do things right are socialization milestones seen at the age of 5 years.

What are supportive interventions that can assist a toddler with a chronic illness to meet developmental milestones? (Select all that apply.) a. Give choices. b. Provide sensory experiences. c. Avoid discipline and limit setting. d. Discourage negative and ritualistic behaviors. e. Encourage independence in as many areas as possible

ANS: A, B, E To encourage autonomy, choices should be given and independence encouraged in as many areas as possible. Sensory experiences should be encouraged to help the toddler to learn through sensorimotor experiences. Age-appropriate discipline and limit setting should be initiated. Negative and ritualistic behaviors are normal and should be allowed.

What can the nurse suggest to families to reduce blood lead levels? (Select all that apply.) a. Do not store food in open cans. b. Ensure the child eats regular meals. c. Mix formula with hot water from the tap. d. Vacuum hard-surfaced floors and window wells. e. Wash and dry the childs hands and face frequently

ANS: A, B, E To reduce blood lead levels, the family should ensure the child eats regular meals because more lead is absorbed on an empty stomach. The childs hands and face should be washed and dried frequently, especially before eating. Food should not be stored in open cans, particularly if cans are imported. Hot water dissolves lead more quickly than cold water and thus contains higher levels of lead. Hot water should not be used to mix formula. Hard-surfaced floors or window sills or wells should not be vacuumed because this spreads dust.

What are common causes of speech problems? (Select all that apply.) a. Autism b. Prematurity c. Hearing loss d. Developmental delay e. Overstimulated environment

ANS: A, C, D Common causes of speech problems are hearing loss, developmental delay, autism, lack of environmental stimulation, and physical conditions that impede normal speech production. Prematurity and an overstimulated environment are not causes of speech problems.

The nurse is teaching parents of a toddler how to handle temper tantrums. What should the nurse include in the teaching? (Select all that apply.) a. Provide realistic expectations. b. Avoid using rewards for good behavior. c. Ensure consistency among all caregivers in expectations. d. During tantrums, ignore the behavior and continue to be present. e. Use time-outs for managing temper tantrums, starting at 12 months.

ANS: A, C, D The best approach toward tapering temper tantrums requires consistency and developmentally appropriate expectations and rewards. Ensuring consistency among all caregivers in expectations, prioritizing what rules are important, and developing consequences that are reasonable for the childs level of development help manage the behavior. During tantrums, ignore the behavior, provided the behavior is not injurious to the child, such as violently banging the head on the floor. Continue to be present to provide a feeling of control and security to the child after the tantrum has subsided. Starting at 18 months, time-outs work well for managing temper tantrums, but not at 12 months.

The nurse is teaching parents of a 3-year-old child about gross motor developmental milestones. What milestones should the nurse include in the teaching session? (Select all that apply.) a. Rides a tricycle b. Catches a ball reliably c. Jumps off the bottom step d. Stands on one foot for a few seconds e. Walks downstairs using alternate footing

ANS: A, C, D The gross motor milestones of a 3-year-old child include riding a tricycle, jumping off the bottom step, and standing on one foot for a few seconds. Catching a ball reliably and walking downstairs using alternate footing are gross motor milestones seen at the age of 4 years.

The nurse is teaching parents of a 3-year-old child about language developmental milestones. What milestones should the nurse include in the teaching session? (Select all that apply.) a. Asks many questions b. Names one or more colors c. Repeats sentence of six syllables d. Uses primarily telegraphic speech e. Has a vocabulary of 1500 words or more

ANS: A, C, D The language milestones of a 3-year-old child include asking many questions, repeating a sentence of six syllables, and using primarily telegraphic speech. Naming one or more colors and having a vocabulary of 1500 words or more footing are language milestones seen at the age of 4 years.

The nurse is teaching an adolescent about the use of tretinoin (Retin-A). What should the nurse include in the teaching session? (Select all that apply.) a. Begin with a pea-sized dot of medication. b. Apply additional medication to the throat. c. Use sunscreen daily and avoid the sun when possible. d. Divide the medication into the three main areas of the face. e. Apply the medication immediately after washing the face.

ANS: A, C, D Tretinoin is available as a cream, gel, or liquid. This drug can be extremely irritating to the skin and requires careful patient education for optimal usage. The patient should be instructed to begin with a pea-sized dot of medication, which is divided into the three main areas of the face and then gently rubbed into each area. The avoidance of the sun and the daily use of sunscreen must be emphasized because sun exposure can result in severe sunburn. The medication should not be applied for at least 20 to 30 minutes after washing to decrease the burning sensation. The medication should not be applied to the throat.

A school-age child has been a victim of bullying. What characteristics does the nurse assess for in this child? (Select all that apply.) a. Anxiety b. Outgoing c. Low self-esteem d. Psychosomatic complaints e. Good academic performance

ANS: A, C, D Victims of bullying are at increased risk for low self-esteem; anxiety; depression; feelings of insecurity and loneliness; poor academic performance; and psychosomatic complaints such as feeling tense, tired, or dizzy.

Parents tell the nurse that siblings of their hospitalized child are feeling left out. What suggestions should the nurse make to the parents to assist the siblings to adjust to the hospitalization of their brother or sister? (Select all that apply.) a. Arrange for visits to the hospital. b. Limit information given to the siblings. c. Encourage phone calls to the hospitalized child. d. Make or buy inexpensive toys or trinkets for the siblings. e. Identify an extended family member to be their support system.

ANS: A, C, D, E Strategies to support siblings during hospitalization include arranging for visits, encouraging phone calls, giving inexpensive gifts, and identifying a support person. Information should be shared with the siblings not limited.

The nurse is teaching parents of preschool children consequences of inadequate sleep. What should the nurse include in the teaching session? (Select all that apply.) a. Behavior changes b. Increased appetite c. Difficulty concentrating d. Poor control of emotions e. Impaired learning ability

ANS: A, C, D, E Consequences of inadequate sleep include daytime tiredness, behavior changes, hyperactivity, difficulty concentrating, impaired learning ability, poor control of emotions and impulses, and strain on family relationships. Increased appetite is not a consequence of inadequate sleep.

1. What are signs and symptoms of a possible kidney transplant rejection in a child? (Select all that apply.) a. Fever b. Hypotension c. Diminished urinary output d. Decreased serum creatinine e. Swelling and tenderness of graft area

ANS: A, C, E The child with a kidney transplant who exhibits any of the following should be evaluated immediately for possible rejection: fever, diminished urinary output, and swelling and tenderness of graft area. Hypertension, not hypotension, and increased, not decreased, serum creatinine are signs of rejection.

A child has just been unexpectedly admitted to the intensive care unit after abdominal surgery. The nursing staff has completed the admission process, and the child's condition is beginning to stabilize. When speaking with the parents, the nurses should expect which stressors to be evident? (Select all that apply.) a. Unfamiliar environment b. Usual day-night routine c. Strange smells d. Provision of privacy e. Inadequate knowledge of condition and routine

ANS: A, C, E Intensive care units, especially when the family is unprepared for the admission, are a strange and unfamiliar place with many pieces of unfamiliar equipment. The sights and sounds are much different from those of a general hospital unit. Also, with the child's condition being more precarious, it may be difficult to keep the parents updated and knowledgeable about what is happening. Lights are usually on around the clock, seriously disrupting the diurnal rhythm. There is usually little privacy available for families in intensive care units.

What conditions are physical complications of obesity? (Select all that apply.) a. Type 2 diabetes mellitus b. QT interval prolongation c. Fatty liver disease d. Gastrointestinal dysfunction e. Abnormal growth acceleration f. Dental erosion

ANS: A, C, E Physical complications of obesity include type 2 diabetes mellitus, which is reaching epidemic proportions in children and adolescents; fatty liver disease not related to alcohol consumption; and abnormal growth acceleration in which overweight children tend to be taller and mature earlier than children who are not overweight. Prolonged QT intervals, gastrointestinal dysfunction, and dental erosion are physical complications observed in children or adolescents who have eating disorders such as anorexia nervosa or bulimia.

Parents are concerned about their child riding an all-terrain vehicle. What should the nurse tell the parents about safe use of all-terrain vehicles? (Select all that apply.) a. Restrict riding to familiar terrain. b. Limit street use to the neighborhood. c. Nighttime riding should not be allowed. d. Vehicles should not carry more than two persons. e. Vehicles should include seat belts, roll bars, and automatic headlights.

ANS: A, C, E Safe use of all-terrain vehicles includes restricting riding to familiar terrain; not allowing nighttime riding; and assuring the vehicle has seat belts, roll bars, and automatic headlights. Street use should not be allowed, and the vehicle should not carry more than one person.

A child is being discharged from an ambulatory care center after an inguinal hernia repair. Which discharge interventions should the nurse implement? (Select all that apply.) a. Discuss dietary restrictions. b. Hold any analgesic medications until the child is home. c. Send a pain scale home with the family. d. Suggest the parents fill the prescriptions on the way home. e. Discuss complications that may occur.

ANS: A, C, E The discharge interventions a nurse should implement when a child is being discharged from an ambulatory care center should include dietary restrictions, being very specific and giving examples of "clear fluids" or what is meant by a "full liquid diet." The nurse should give specific information on pain control and send a pain scale home with the family. All complications that may occur after an inguinal hernia repair should be discussed with the parents. The pain medication, as prescribed, should be given before the child leaves the building and prescriptions should be filled and given to the family before discharge.

A nurse is recommending strategies to a group of school-age children for prevention of obesity. Which should the nurse include? (Select all that apply.) a. Eat breakfast daily. b. Limit fruits and vegetables. c. Have frequent family meals with parents present. d. Eat frequently at restaurants. e. Limit television viewing to 2 hours a day.

ANS: A, C, E The nurse should counsel school-age children to eat breakfast daily, have mealtimes with family, and limit television viewing to 2 hours a day to prevent obesity. Fruits and vegetables should be consumed in the recommended quantities, and eating at restaurants should be limited.

What are supportive interventions that can assist a school-age child with a chronic illness to meet developmental milestones? (Select all that apply.) a. Encourage socialization. b. Discourage sports activities. c. Encourage school attendance. d. Provide instructions on assertiveness. e. Educate teachers and classmates about the childs condition.

ANS: A, C, E To develop a sense of accomplishment, school attendance should be encouraged, and teachers and classmates should be educated about the childs condition. To form peer relationships, socialization should be encouraged. Sports activities should be encouraged (e.g., Special Olympics), not discouraged. Providing instructions on assertiveness is appropriate for adolescence.

The nurse is teaching parents of preschool-aged children strategies to prevent sexual abuse. What should the nurse include in the teaching session? (Select all that apply.) a. Back up a childs right to say no. b. Dont take what your child says too seriously. c. Take a second look at signals of potential danger. d. Dont be too detailed about examples of sexual assault. e. Remind children that even nice people sometimes do mean things.

ANS: A, C, E To provide protection and preparation from sexual abuse, parents should back up a childs right to say no, take a second look at signals of potential danger, and remind children that even nice people sometimes do mean things. Parents should take what children say seriously and they should give specific definitions and examples of sexual assault.

The nurse is caring for an infant with a suspected urinary tract infection. Which clinical manifestations should be expected? (Select all that apply.) a. Vomiting b. Jaundice c. Failure to gain weight d. Swelling of the face e. Back pain f. Persistent diaper rash

ANS: A, C, F Vomiting, failure to gain weight, and persistent diaper rash are clinical manifestations observed in an infant with a UTI.

Which are appropriate statements the nurse should make to parents after the death of their child? (Select all that apply.) a. "We feel so sorry that we couldn't save your child." b. "Your child isn't suffering anymore." c. "I know how you feel." d. "You're feeling all the pain of losing a child." e. "You are still young enough to have another baby."

ANS: A, D By saying, "We feel so sorry that we couldn't save your child," the nurse is expressing personal feeling of loss or frustration, which is therapeutic. Stating, "You're feeling all the pain of losing a child," focuses on a feeling, which is therapeutic. The statement, "Your child isn't suffering anymore," is a judgmental statement, which is nontherapeutic. "I know how you feel" and "You're still young enough to have another baby" are statements that give artificial consolation and are nontherapeutic.

The nurse is teaching parents of a child with a cognitive impairment signs that indicate the child is developmentally ready for dressing training. What signs should the nurse include that indicate the child is developmentally ready for dressing training? (Select all that apply.) a. Can follow verbal commands b. Can sit quietly for 1 to 2 minutes c. Can master every task of dressing d. Can follow physical gestures or cues e. Can relate clothing to the appropriate body part

ANS: A, D, E Children are considered developmentally ready for dressing training if they can sit quietly for 3 to 5 minutes (not 1 to 2) while working on a task; can follow physical gestures or cues; can follow verbal commands; and can relate clothing to the appropriate body part, such as socks to feet. As with other self-help skills, the child may not be able to master every task but should be evaluated for evidence of willingness to participate at his or her level of readiness.

The clinic nurse is assessing an infant. What are early signs of cognitive impairment the nurse should discuss with the health care provider? (Select all that apply.) a. Head lag at 11 months of age b. No pincer grasp at 4 months of age c. Colicky incidents at 3 months of age d. Unable to speak two to three words at 24 months of age e. Unresponsiveness to the environment at 12 months of age

ANS: A, D, E Earlysignsofcognitiveimpairmentincludegrossmotordelay(headlagshouldbeestablishedby6 months, and head lag still present at 11 months is a delay), language delay (normal languagedevelopment is speaking two to three words by age 12 months; if unable to speak two to threewordsat24months,thatisadelay),andunresponsivenesstotheenvironmentat12months.Nopincer grasp at 4 months of age is normal (palmar grasp is the expected finding), and colickyincidentsat3monthsofageisanormalfinding

What are signs and symptoms of the stage of detachment in relation to separation anxiety in young children? (Select all that apply.) a. Appears happy b. Lacks interest in the environment c. Regresses to an earlier behavior d. Forms new but superficial relationships e. Interacts with strangers or familiar caregivers

ANS: A, D, E Manifestations of the stage of detachment seen in children during a hospitalization may include appearing happy, forming new but superficial relationships, and interacting with strangers or familiar caregivers. Lacking interest in the environment and regressing to an earlier behavior are manifestations seen in the stage of despair.

1. What signs and symptoms are indicative of a urinary tract disorder in the childhood period (2 to 14 years)? (Select all that apply.) a. Fatigue b. Dehydration c. Hypotension d. Growth failure e. Blood in the urine

ANS: A, D, E Signs and symptoms of a urinary tract disorder in the childhood period are fatigue, growth failure, and blood in the urine. Edema is noted, not dehydration, and hypertension is present, not hypotension.

The parents tell a nurse our child is having some short-term negative outcomes since the hospitalization. The nurse recognizes that what can negatively affect short-term negative outcomes? (Select all that apply.) a. Parents anxiety b. Consistent nurses c. Number of visitors d. Length of hospitalization e. Multiple invasive procedures

ANS: A, D, E The stressors of hospitalization may cause young children to experience short- and long-term negative outcomes. Adverse outcomes may be related to the length and number of admissions, multiple invasive procedures, and the parents anxiety. Consistent nurses would have a positive effect on short-term negative outcomes. The number of visitors does not have an effect on negative outcomes.

Which describes avoidance behaviors parents may exhibit when learning that their child has a chronic condition? (Select all that apply.) a. Refuses to agree to treatment b. Shares burden of disorder with others c. Verbalizes possible loss of child d. Withdraws from outside world e. Punishes self because of guilt and shame

ANS: A, D, E A parent who refuses to agree to treatment, withdraws from the outside world, and punishes self because of guilt and shame is exhibiting avoidance coping behaviors. A parent who shares the burden of disorder with others and verbalizes possible loss of child is exhibiting approach coping behaviors.

The nurse is teaching parents of a school-age child how to cleanse small wounds. What should the nurse advise the parents to avoid using to cleanse a wound? (Select all that apply.) a. Alcohol b. Normal saline c. Tepid water d. Povidoneiodine e. Hydrogen peroxide

ANS: A, D, E Caution caregivers to avoid cleansing the wound with povidoneiodine, alcohol, and hydrogen peroxide because these products disrupt wound healing. Normal saline and tepid water are safe to use when cleansing wounds.

Which side effects should the nurse monitor when a child is taking an antipsychotic medication? (Select all that apply.) a. Extrapyramidal effects b. Hypertension c. Bradycardia d. Dizziness e. Seizures

ANS: A, D, E Common side effects of antipsychotic medications include dizziness, drowsiness, tachycardia, hypotension, and extrapyramidal effects, such as abnormal movements and seizures.

A parent asks the nurse about the characteristics of a sleep terror. What response should the nurse give to the parent? (Select all that apply.) a. The child screams during the sleep terror. b. Return to sleep is delayed because of persistent fear. c. The night terror occurs during the second half of night. d. The child has no memory of the dream with a sleep terror. e. The child is not aware of anothers presence during a sleep terror.

ANS: A, D, E During sleep terrors, the child screams and has no memory of the dream. The child is not aware of anothers presence during a sleep terror. Return to sleep is usually rapid with a sleep terror, but it is delayed with a nightmare. The sleep terror occurs usually within 1 to 4 hours of sleep, but nightmares occur during the second half of night.

A school-age child is admitted to the hospital with acute glomerulonephritis and oliguria. Which dietary menu items should be allowed for this child? (Select all that apply.) a. Apples b. Bananas c. Cheese d. Carrot sticks e. Strawberries

ANS: A, D, E Moderate sodium restriction and even fluid restriction may be instituted for children with acute glomerulonephritis. Foods with substantial amounts of potassium are generally restricted during the period of oliguria. Apples, carrot sticks, and strawberries would be items low in sodium and allowed. Bananas are high in potassium, and cheese is high in sodium. Those items would be restricted.

A parent tells the nurse, My toddler tries to undo the car seat harness and climb out of the seat. What strategies should the nurse recommend to the parent to encourage the child to stay in the seat? (Select all that apply.) a. Allow your child to hold a favorite toy. b. Allow your child out of the seat occasionally. c. Avoid using rewards to encourage cooperative behavior. d. When child tries to unbuckle the seat harness, firmly say, No. e. It may be necessary to stop the car to reinforce the expected behavior.

ANS: A, D, E Strategies to encourage a child to stay in a car seat include allowing the child to hold favorite toy, firmly saying No if the child begins to undo the harness, and stopping the car to reinforce the expected behavior. Rewards, such as stars or stickers, can be used to encourage cooperative behavior. The child should stay in the car seat at all times, even for short trips.

A 13-year-old is being seen in the clinic for a routine health check. The adolescent has not been in the clinic for 3 years but was up to date on immunizations at that time. Which immunizations should the adolescent receive? (Select all that apply.) a. DTaP (tetanus, diphtheria, acellular pertussis) b. MMR (measles, mumps, rubella) c. Hepatitis B d. Influenza e. MCV4 (meningococcal)

ANS: A, D, E The DTaP (tetanus, diphtheria, acellular pertussis) vaccine is recommended for adolescents 11 to 18 years old who have not received a tetanus booster (Td) or DTaP dose and have completed the childhood DTaP/DTP series. Meningococcal vaccine (MCV4) should be given to adolescents 11 to 12 years of age with a booster dose at age 16 years. Annual influenza vaccination with either the live attenuated influenza vaccine or trivalent influenza vaccine is recommended for all children and adolescents. The adolescent, previously up to date on vaccinations, would have received the MMR and hepatitis B as a child.

A child is being admitted to the intensive care unit (ICU) and the parents are with the child. Which creates stressors for children and parents in ICUs? (Select all that apply.) a. Equipment noise b. Privacy c. Caring behavior by the nurse d. Unfamiliar smells e. Sleep deprivation

ANS: A, D, E The ICU can create physical and environmental stressors for children and their families. Equipment noise (monitors, suction equipment, telephones, computers), unfamiliar smells (alcohol, adhesive remover, body odors), and sleep deprivation all are stressors found in the ICU. Privacy as opposed to no privacy and a caring nurse as opposed to unkind or thoughtless comments from staff help reduce the stressors of the ICU.

The nurse is planning strategies to assist a slow-to-warm child to try new experiences. What strategies should the nurse plan? (Select all that apply.) a. Attend after-school activities with a friend. b. Suggest the child move quickly into a new situation. c. Avoid trying new experiences until the child is ready. d. Allow the child to adapt to the experience at his or her own pace. e. Contract for permission to withdraw after a trial of the experience.

ANS: A, D, E The nurse should encourage slow-to-warm children to try new experiences but allow them to adapt to their surroundings at their own speed. Pressure to move quickly into new situations only strengthens their tendency to withdraw. After-school activities can be a cause for reaction, but attending with a friend or contracting for permission to withdraw after a trial of a specified number of times may provide them with sufficient incentive to try

A nurse is instructing a nursing assistant on techniques to facilitate lipreading with a hearing-impaired child who lip reads. Which techniques should the nurse include? (Select all that apply.) a. Speak at eye level. b. Stand at a distance from the child. c. Speak words in a loud tone. d. Use facial expressions while speaking. e. Keep sentences short.

ANS: A, D, E To facilitate lipreading for a hearing-impaired child who can lip read, the speaker should be at eye level, facing the child directly or at a 45-degree angle. Facial expressions should be used to assist in conveying messages, and the sentences should be kept short. The speaker should stand close to the child, not at a distance, and using a loud tone while speaking will not facilitate lipreading.

1. The nurse is caring for a 12-year-old child who is on fall precautions secondary to seizures. What interventions should be included in the childs care plan? (Select all that apply.) a. Place a call light and desired items within reach. b. Keep the bed in the highest position with the two side rails up. c. Turn off the lights and television at night. d. Keep personal belongings and clutter contained in one area of the floor. e. Have the child wear an appropriate-size gown and nonskid footwear.

ANS: A, E Prevention of falls requires alterations in the environment, including keeping call light and desired items within reach and having the child wear appropriate-size gowns and nonskid footwear. The bed should be in the lowest position possible with all the side rails up; at least a dim light should be left on at night; and personal belongings and clutter should not be on the floorthey should be in a cabinet.

Autism is a complex developmental disorder. The diagnostic criteria for autism include delayed or abnormal functioning in which areas with onset before age 3 years? (Select all that apply.) a. Language as used in social communication b. Parallel play c. Gross motor development d. Growth below the 5th percentile for height and weight e. Symbolic or imaginative play f. Social interaction

ANS: A, E, F These are three of the areas in which autistic children may show delayed or abnormal functioning: language as used in social communication, symbolic or imaginative play, and social interaction. Parallel play is typical play of toddlers and is usually not affected. Gross motor development and growth below the 5th percentile for height and weight are usually not characteristic of autism.

1. What measure of fluid balance status is most useful in a child with acute glomerulonephritis? a. Proteinuria b. Daily weight c. Specific gravity d. Intake and output

ANS: B A record of daily weight is the most useful means to assess fluid balance and should be kept for children treated at home or in the hospital. Proteinuria does not provide information about fluid balance. Specific gravity does not accurately reflect fluid balance in acute glomerulonephritis. If fluid is being retained, the excess fluid will not be included. Also proteinuria and hematuria affect specific gravity. Intake and output can be useful but are not considered as accurate as daily weights. In children who are not toilet trained, measuring output is more difficult.

1. The nurse determines that a childs intravenous infusion has infiltrated. The infused solution is a vesicant. What is the most appropriate nursing action? a. Stop the infusion and apply ice. b. End the infusion and notify the practitioner. c. Slow the infusion rate and notify the practitioner. d. Discontinue the infusion and apply warm compresses.

ANS: B A vesicant causes cellular damage when even minute amounts escape into the tissue. The intravenous infusion is immediately stopped, the extremity is elevated, the practitioner is notified, and the treatment protocol is initiated. The applying of heat or ice depends on the fluid that has extravasated. The catheter is left in place until it is no longer needed.

1. What pathologic process is believed to be responsible for the development of postinfectious glomerulonephritis? a. Infarction of renal vessels b. Immune complex formation and glomerular deposition c. Bacterial endotoxin deposition on and destruction of glomeruli d. Embolization of glomeruli by bacteria and fibrin from endocardial vegetation

ANS: B After a streptococcal infection, antibodies are formed, and immune-complex reaction occurs. The immune complexes are trapped in the glomerular capillary loop. Infarction of renal vessels occurs in renal involvement in sickle cell disease. Bacterial endotoxin deposition on and destruction of glomeruli is not a mechanism for postinfectious glomerulonephritis. Embolization of glomeruli by bacteria and fibrin from endocardial vegetation is the pathology of renal involvement with bacterial endocarditis.

The nurse gives an injection in a patients room. How should the nurse dispose of the needle? a. Remove the needle from the syringe and dispose of it in a proper container. b. Dispose of the syringe and needle in a rigid, puncture-resistant container in the patients room. c. Close the safety cover on the needle and return it to the medication preparation area for proper disposal. d. Place the syringe and needle in a rigid, puncture-resistant container in an area outside of the patients room.

ANS: B All needles (uncapped and unbroken) are disposed of in a rigid, puncture-resistant, tamper-proof container located near the site of use. Consequently, these containers should be installed in the patients room. Needles and syringes are disposed of uncapped and unbroken. A used needle should not be transported to an area distant from use for disposal.

The nurse is notified that a 9-year-old boy with nephrotic syndrome is being admitted. Only semiprivate rooms are available. What roommate should be best to select? a. A 10-year-old girl with pneumonia b. An 8-year-old boy with a fractured femur c. A 10-year-old boy with a ruptured appendix d. A 9-year-old girl with congenital heart disease

ANS: B An 8-year-old boy with a fractured femur would be the best choice for a roommate. The boys are similar in age. The child with nephrotic syndrome most likely will be on immunosuppressive agents and susceptible to infection. The child with a fractured femur is not infectious. A girl should not be a good roommate for a school-age boy. In addition, the 10-year-old girl with pneumonia and the 10-year-old boy with a ruptured appendix have infections and could pose a risk for the child with nephrotic syndrome.

The nurse has just collected blood by venipuncture in the antecubital fossa. What should the nurse do next? a. Keep the childs arm extended while applying a Band-Aid to the site. b. Keep the childs arm extended and apply pressure to the site for a few minutes. c. Apply a Band-Aid to the site and keep the arm flexed for 10 minutes. d. Apply a gauze pad or cotton ball to the site and keep the arm flexed for several minutes.

ANS: B Applying pressure to the site of venipuncture stops the bleeding and aids in coagulation. Pressure should be applied before a bandage or gauze pad is applied.

27. What is an advantage of the ventrogluteal muscle as an injection site in young children? a. Easily accessible from many directions b. Free of significant nerves and vascular structures c. Can be used until child reaches a weight of 9 kg (20 lb) d. Increased subcutaneous fat, which provides sustained drug absorption

ANS: B Being free of significant nerves and vascular structure is one of the advantages of the ventrogluteal site. In addition, it is considered less painful than the vastus lateralis. The major disadvantage is lack of familiarity by health professionals and controversy over whether the site can be used before weight bearing. The vastus lateralis is a more accessible site. The ventrogluteal muscle site has safely been used from newborn through adulthood. Clinical guidelines address the need for the child to be walking. The site has less subcutaneous tissue, which facilitates intramuscular deposition of the drug rather than subcutaneous.

What intervention should be included in the nursing care of a child with autism spectrum disorder (ASD)? a. Assign multiple staff to care for the child. b. Communicate with the child at his or her developmental level. c. Provide a wide variety of foods for the child to try. d. Place the child in a semiprivate room with a roommate of a similar age.

ANS: B Children with ASD require individualized care. The nurse needs to communicate with the child at the childs developmental level. Consistent caregivers are essential for children with ASD. The same staff members should care for the child as much as possible. Children with ASD do not adapt to changing situations. The same foods should be provided to allow the child to adjust. A private room is desirable for children with ASD. Stimulation is minimized.

1. The nurse is conducting discharge teaching with the parent of a 7-year-old child with minimal change nephrotic syndrome (MCNS). What statement by the parent indicates a correct understanding of the teaching? a. My child needs to stay home from school for at least 1 more month. b. I should not add additional salt to any of my childs meals. c. My child will not be able to participate in contact sports while receiving corticosteroid therapy. d. I should measure my childs urine after each void and report the 24-hour amount to the health care provider.

ANS: B Children with MCNS can be treated at home after the initial phase with appropriate discharge instructions, including a salt restriction of no additional salt to the childs meals. The child may return to school but should avoid exposure to infected playmates. Participation in contact sports is not affected by corticosteroid therapy. The parent does not need to measure the childs urine on a daily basis but may be instructed to test for albumin.

A 13-year-old child with cystic fibrosis (CF) is a frequent patient on the pediatric unit. This admission, she is sleeping during the daytime and unable to sleep at night. What should be a beneficial strategy for this child? a. Administer prescribed sedative at night to aid in sleep. b. Negotiate a daily schedule that incorporates hospital routine, therapy, and free time. c. Have the practitioner speak with the child about the need for rest when receiving therapy for CF. d. Arrange a consult with the social worker to determine whether issues at home are interfering with her care.

ANS: B Childrens response to the disruption of routine during hospitalization is demonstrated in eating, sleeping, and other activities of daily living. The lack of structure is allowing the child to sleep during the day, rather than at night. Most likely the lack of schedule is the problem. The nurse and child can plan a schedule that incorporates all necessary activities, including medications, mealtimes, homework, and patient care procedures. The schedule can then be posted so the child has a ready reference. Sedatives are not usually used with children. The child has a chronic illness and most likely knows the importance of rest. The parents and child can be questioned about changes at home since the last hospitalization.

What description applies to fragile X syndrome? a. Chromosomal defect affecting only females b. Second most common genetic cause of cognitive impairment c. Most common cause of uninherited cognitive impairment d. Chromosomal defect that follows the pattern of X-linked recessive disorders

ANS: B Fragile X syndrome is the most common inherited cause of cognitive impairment and the second most common genetic cause of cognitive impairment after Down syndrome. Fragile X primarily affects males and follows the pattern of X-linked dominant inheritance with reduced penetrance.

29. What consideration is important for the nurse when changing dressings and applying topical medication to a childs abdomen and leg burns? a. Apply topical medication with clean hands. b. Wash hands and forearms before and after dressing change. c. If dressings have adhered to the wound, soak in hot water before removal. d. Apply dressing so that movement is limited during the healing process.

ANS: B Frequenthandandforearmwashingisthesinglemostimportantelementoftheinfection-controlprogram.Topicalmedicationsshouldbeappliedwithatonguebladeorglovedhand.Dressingsthathaveadheredtothewoundcanberemovedwithtepidwaterornormalsaline.Dressingsareapplied with sufficient tension to remain in place but not so tightly as to impair circulation orlimitmotion

A child who has cystic fibrosis is admitted to the pediatric unit with methicillin-resistant Staphylococcus aureus (MRSA) infection. The nurse recognizes that in addition to a private room, the child is placed on what precautions? a. Droplet b. Contact c. Airborne d. Standard

ANS: B MRSAisanincreasinglysignificantsourceofhospital-acquiredinfections.Thisorganismmeetsthecriteriaofbeingepidemiologicallyimportantandcanbetransmittedbydirectcontact.Gownsand gloves should be worn when exposed to potentially contagious materials, and meticuloushandwashingisrequired.S.aureusisnotanorganismthatisspreadthroughairborneordropletmechanisms.Additionalprecautions,beyondStandardPrecautions,areneededtopreventspreadofthisorganism

1. What amount of fluid loss occurs with moderate dehydration? a. <50 ml/kg b. 50 to 90 ml/kg c. <5% total body weight d. >15% total body weight

ANS: B Moderate dehydration is defined as a fluid loss of between 50 and 90 ml/kg. Mild dehydration is defined as a fluid loss of less than 50 ml/kg. Weight loss up to 5% is considered mild dehydration. Weight loss over 15% is severe dehydration.

32. To facilitate the administration of an oral medication to a preschool-age child, what action should the nurse take? a. Dilute the medication in a large amount of favorite liquid and allow the child to hold the cup. b. Set limits about the need to take medication and offer praise immediately after the task is accomplished. c. Mix the medication in a moderate amount of the childs favorite food. d. Explain the purpose of the medication and allow the child time to express resistance before giving the medication.

ANS: B Nurses who approach children with confidence and who convey the impression that they expect to be successful are less likely to encounter difficulty. It is best to approach a child as though cooperation is expected. The medication should not be placed in a favorite liquid or food. Allowingthechildtimetoexpressresistancewilldelayadministrationofthemedication.

29. Fentanyl and midazolam (Versed) are given before debridement of a childs burn wounds. What is the purpose of using these medications? a. Facilitate healing b. Provide pain relief c. Minimize risk of infection d. Decrease amount of debridement needed

ANS: B Partial-thickness burns require dbridement of devitalized tissue to promote healing. The procedure is painful and requires analgesia and sedation before the procedure. Fentanyl and midazolam provide excellent intravenous sedation and analgesia to control procedural pain in children with burns.

1. What is the narrowing of preputial opening of foreskin called? a. Chordee b. Phimosis c. Epispadias d. Hypospadias

ANS: B Phimosis is the narrowing or stenosis of the preputial opening of the foreskin. Chordee is the ventral curvature of the penis. Epispadias is the meatal opening on the dorsal surface of the penis. Hypospadias is a congenital condition in which the urethral opening is located anywhere along the ventral surface of the penis.

What is a priority intervention for an infant with a temporary colostomy for Hirschsprung disease? a. Teaching how to irrigate the colostomy b. Protecting the skin around the colostomy c. Discussing the implications of a colostomy during puberty d. Using simple, straightforward language to prepare the child

ANS: B Protection of the peristomal skin is a major priority. Well-fitting appliances and skin protectants are used. Teaching how to irrigate a colostomy is not necessary because colostomies are not irrigated in infants. The colostomy is usually reversed within 6 months to 1 year. The parents, not the infant, need to be prepared for the surgery.

The nurse is caring for an 18-month-old child with rotavirus. What clinical manifestations should the nurse expect to observe? a. Severe abdominal cramping and bloody diarrhea b. Mild fever and vomiting followed by onset of watery stools c. Colicky abdominal pain and vomiting d. High fever, diarrhea, and lethargy

ANS: B Rotavirus is one of the most common pathogens that cause gastroenteritis in children younger than the age of 2 years. Clinical manifestations include mild to moderate fever and vomiting followed by the onset of watery stools. The fever and vomiting usually abate in 1 or 2 days, but the diarrhea persists for 5 to 7 days. Severe abdominal cramping and bloody diarrhea are seen with Escherichia coli infection; colicky abdominal pain and vomiting are seen with salmonella infection; and high fever, diarrhea, and lethargy are seen with infection by Salmonella typhi.

The nurse is caring for a 10-year-old child during a long hospitalization. What intervention should the nurse include in the care plan to minimize loss of control and autonomy during the hospitalization? a. Allow the child to skip morning self-care activities to watch a favorite television program. b. Create a calendar with special events such as a visit from a friend to maintain a routine. c. Allow the child to sleep later in the morning and go to bed later at night to promote control. d. Create a restrictive environment so the child feels in control of sensory stimulation.

ANS: B School-age children may feel an overwhelming loss of control and autonomy during a longer hospitalization. One intervention to minimize this loss of control is to create a calendar with planned special events such as a visit from a friend. Maintaining the childs daily routine is another intervention to minimize the sense of loss of control; allowing the child to skip morning self-care activities, sleep later, or stay up later would work against this goal. Environments should be as nonrestrictive as possible to allow the child freedom to move about, thus allowing a sense of autonomy.

29. Hydrotherapy is required to treat a child with extensive partial-thickness burn wounds. What is the purpose of hydrotherapy? a. Provide pain relief b. Dbride the wounds c. Destroy bacteria on the skin d. Increase peripheral blood flow

ANS: B Soaking in a tub or showering once or twice a day acts to loosen and remove sloughing tissue, exudate, and topical medications. The hydrotherapy cleanses the wound and the entire body and helps maintain range of motion. Appropriate pain medications are necessary. Dressing changes are extremely painful. The total bacterial count of the skin is reduced by the hydrotherapy, but this is not the primary goal. There may be an increase in peripheral blood flow, but the primary purpose is for wound debridement.

32. A laboratory technician is performing a blood draw on a toddler. The toddler is holding still but crying loudly. The nurse should take which action? a. Have the lab technician stop the procedure until the child stops crying. b. Do nothing. Its Okay for a child to cry during a painful procedure. c. Tell the child to stop crying; its only a small prick. d. Tell the child to stop crying because the procedure is almost over.

ANS: B The child should be allowed to express feelings of anger, anxiety, fear, frustration, or any other emotion. It is natural for children to strike out in frustration or to try to avoid stress-provoking situations. The child needs to know that it is all right to cry.

What is a primary goal in caring for a child with cognitive impairment? a. Developing vocational skills b. Promoting optimum development c. Finding appropriate out-of-home care d. Helping child and family adjust to future care

ANS: B The goal for children with cognitive impairment is the promotion of optimum social, physical, cognitive, and adaptive development as individuals within a family and community. Vocational skills are only one part of that goal. The focus must also be on the family and other aspects of development. Out-of-home care is considered part of the childs development. Optimum development includes adjustment for both the family and child.

The parents of a 4-month-old infant cannot visit except on weekends. What action by the nurse indicates an understanding of the emotional needs of a young infant? a. Place her in a room away from other children. b. Assign her to the same nurse as much as possible. c. Tell the parents that frequent visiting is unnecessary. d. Assign her to different nurses so she will have varied contacts.

ANS: B The infant is developing a sense of trust. This is accomplished by the consistent, loving care of a nurturing person. If the parents are unable to visit, then the same staff nurses should be used as much as possible. Placing her in a room away from other children would isolate the child. The parents should be encouraged to visit. The nurse should describe how the staff will care for the infant in their absence.

1. Identification and treatment of cryptorchid testes should be done by age 2 years. What is an important consideration? a. Medical therapy is not effective after this age. b. Treatment is necessary to maintain the ability to be fertile when older. c. The younger child can tolerate the extensive surgery needed. d. Sexual reassignment may be necessary if treatment is not successful.

ANS: B The longer the testis is exposed to higher body heat, the greater the likelihood of damage. To preserve fertility, surgery should be done at an early age. Surgical intervention is the treatment of choice. Simple orchiopexy is usually performed as an outpatient procedure. The surgical procedure restores the testes to the scrotum. This helps the boy to have both testes in the scrotum by school age. Sexual reassignment is not indicated when the testes are not descended.

A toddler is being sent to the operating room for surgery at 9 AM. As the nurse prepares the child, what is the priority intervention? a. Administering preoperative antibiotic b. Verifying that the child and procedure are correct c. Ensuring that the toddler has been NPO since midnight d. Informing the parents where they can wait during the procedure

ANS: B The most important intervention is to ensure that the correct child is going to the operating room for the identified procedure. It is the nurses responsibility to verify identification of the child and what procedure is to be done. If an antibiotic is ordered, administering it is important, but correct identification is a priority. Clear liquids can be given up to 2 hours before surgery. If the child was NPO (taking nothing by mouth) since midnight, intravenous fluids should be administered. Parents should be encouraged to accompany the child to the preoperative area. Many institutions allow parents to be present during induction.

A bone marrow biopsy will be performed on a 7-year-old girl. She wants her mother to hold her during the procedure. How should the nurse respond? a. Holding your child is unsafe. b. Holding may help your child relax. c. Hospital policy prohibits this interaction. d. Holding your child is unnecessary given the childs age.

ANS: B The mothers preference for assisting, observing, or waiting outside the room should be assessed, as well as the childs preference for parental presence. The childs choice should be respected. This will most likely help the child through the procedure. If the mother and child agree, then the mother is welcome to stay. Her familiarity with the procedure should be assessed and potential safety risks identified (mother may sit in chair). Hospital policies should be reviewed to ensure that they incorporate family-centered care.

1. When caring for a child with an intravenous (IV) infusion, what is an appropriate nursing action? a. Change the insertion site every 24 hours. b. Check the insertion site frequently for signs of infiltration. c. Use a macrodropper to facilitate reaching the prescribed flow rate. d. Avoid restraining the child to prevent undue emotional stress.

ANS: B The nursing responsibility for IV therapy is to calculate the amount to be infused in a given length of time; set the infusion rate; and monitor the apparatus frequently, at least every 1 to 2 hours, to make certain that the desired rate is maintained, the integrity of the system remains intact, the site remains intact (free of redness, edema, infiltration, or irritation), and the infusion does not stop. Insertion sites do not need to be changed every 24 hours unless a problem is found with the site. This exposes the child to significant trauma. A minidropper (60 drops/ml) is the recommended IV tubing in pediatric patients. Intravenous sites should be protected. This may require soft restraints on the child.

23. The nurse is teaching a parent of a 10-year-old child who will be discharged with a venous access device (VAD). What statement by the parent indicates a correct understanding of the teaching? a. I should have my child wear a protective vest when my child wants to participate in contact sports. b. I should apply pressure to the entry site to the vein, not the exit site, if the VAD is accidentally removed. c. I can expect my child to have feelings of general malaise for 1 week after the VAD is inserted. d. I should give my child a sponge bath for the first 2 weeks after the VAD is inserted; then I can allow my child to take a bath.

ANS: B The parents of a child with a VAD should be taught to apply pressure to the entry site to the vein, not the exit site, if the VAD is accidentally removed. The child should not participate in contact sports, even with a protective vest, to prevent the VAD from becoming dislodged. General malaise is a sign of an infection, not an expected finding after insertion of the VAD. The child can shower or take a bath after insertion of the VAD; the child does not need a sponge bath for any length of time.

1. A child with acute glomerulonephritis is in the playroom and experiences blurred vision and a headache. What action should the nurse take? a. Check the urine to see if hematuria has increased. b. Obtain the childs blood pressure and notify the health care provider. c. Obtain serum electrolytes and send urinalysis to the laboratory. d. Reassure the child and encourage bed rest until the headache improves.

ANS: B The premonitory signs of encephalopathy are headache, dizziness, abdominal discomfort, and vomiting. If the condition progresses, there may be transient loss of vision or hemiparesis, disorientation, and generalized tonic-clonic seizures. The health care provider should be notified of these symptoms.

An 11-month-old hospitalized boy is restrained because he is receiving intravenous (IV) fluids. His grandmother has come to stay with him for the afternoon and asks the nurse if the restraints can be removed. What nurses response is best? a. Restraints need to be kept on all the time. b. That is fine as long as you are with him. c. That is fine if we have his parents consent. d. The restraints can be off only when the nursing staff is present.

ANS: B The restraints are necessary to protect the IV site. If the child has appropriate supervision, restraints are not necessary. The nurse should remove the restraints whenever possible. When parents or staff members are present, the restraints can be removed and the IV site protected. Parental permission is not needed for restraint removal.

The nurse is talking with a 10-year-old boy who wears bilateral hearing aids. The left hearing aid is making an annoying whistling sound that the child cannot hear. What intervention is the most appropriate nursing action? a. Ignore the sound. b. Suggest he reinsert the hearing aid. c. Ask him to reverse the hearing aids in his ears. d. Suggest he raise the volume of the hearing aid.

ANS: B The whistling sound is acoustic feedback. The nurse should have the child remove the hearing aid and reinsert it, making sure no hair is caught between the ear mold and the ear canal. Ignoring the sound or suggesting he raise the volume of the hearing aid would be annoying to others. The hearing aids are molded specifically for each ear.

1. The nurse notes that a child has lost 3.6 kg (8 lb) after 4 days of hospitalization for acute glomerulonephritis. What is the most likely cause of this weight loss? a. Poor appetite b. Reduction of edema c. Restriction to bed rest d. Increased potassium intake

ANS: B This amount of weight loss in this period is a result of the improvement of renal function and mobilization of edema fluid. Poor appetite and bed rest would not result in a weight loss of 8 lb in 4 days. Foods with substantial amounts of potassium are avoided until renal function is normalized.

A 10-year-old girl needs to have another intravenous (IV) line started. She keeps telling the nurse, Wait a minute, and, Im not ready. How should the nurse interpret this behavior? a. IV insertions are viewed as punishment. b. This is expected behavior for a school-age child. c. Protesting like this is usually not seen past the preschool years. d. The child has successfully manipulated the nurse in the past.

ANS: B This school-age child is attempting to maintain some control over the hospital experience. The nurse should provide the girl with structured choices about when the IV line will be inserted. Preschoolers can view procedures as punishment; this is not typical behavior of a preschool-age child.

A nurse must do a venipuncture on a 6-year-old child. What consideration is important in providing atraumatic care? a. Use an 18-gauge needle if possible. b. Show the child the equipment to be used before the procedure. c. If not successful after four attempts, have another nurse try. d. Restrain the child completely.

ANS: B To provide atraumatic care the child should be able to see the equipment to be used before the procedure begins. Use the smallest gauge needle that permits free flow of blood. A two-try-only policy is desirable, in which two operators each have only two attempts. If insertion is not successful after four punctures, alternative venous access should be considered. Restrain the child only as needed to perform the procedure safely; use therapeutic hugging.

32. At which age should a nurse keep teaching time short (5 minutes)? a. Infant b. Toddler c. Preschool d. School age

ANS: B Toddlers have limited time concept, and teaching time should be kept short(5-10minutes)

1. What is an approximate method of estimating output for a child who is not toilet trained? a. Have parents estimate output. b. Weigh diapers after each void. c. Place a urine collection device on the child. d. Have the child sit on a potty chair 30 minutes after eating.

ANS: B Weighing diapers will provide an estimate of urinary output. Each 1 g of weight is equivalent to 1 ml of urine. Having parents estimate output would be inaccurate. It is difficult to estimate how much fluid is in a diaper. The urine collection device would irritate the childs skin. It would be difficult for a toddler who is not toilet trained to sit on a potty chair 30 minutes after eating.

29. What is the nursing action related to the applying of biologic or synthetic skin coverings for a child with partial-thickness burns of both legs? a. Splint the legs to prevent movement. b. Observe wounds for signs of infection. c. Monitor closely for manifestations of shock. d. Examine dressings for indications of bleeding.

ANS: B When applied early to a superficial partial-thickness injury, biologic dressings stimulate epithelial growth and faster wound healing. If the dressing covers areas of heavy microbial contamination, infection occurs beneath the dressing. In the case of partial-thickness burns, such infection may convert the wound to a full-thickness injury. Infection is the primary concern when biologic dressings are used.

The nurse is caring for a hospitalized 4-year-old boy. His parents tell the nurse that they will be back to visit at 6 PM. When the child asks the nurse when his parents are coming, the nurse's best response is a. "They will be here soon." b. "They will come after dinner." c. "Let me show you on the clock when 6 PM is." d. "I will tell you every time I see you how much longer it will be."

ANS: B A 4-year-old child understands time in relation to events such as meals. Children perceive "soon" as a very short time. The nurse may lose the child's trust if his parents do not return in the time he perceives as "soon." Children cannot read or use a clock for practical purposes until age 7 years. I will tell you every time I see you how much longer it will be assumes the child understands the concepts of hours and minutes, which are not developed until age 5 or 6 years.

The nurse is caring for a hospitalized 4-year-old boy. His parents tell the nurse they will be back to visit at 6 PM. When he asks the nurse when his parents are coming, what would the nurses best response be? a. They will be here soon. b. They will come after dinner. c. Let me show you on the clock when 6 PM is. d. I will tell you every time I see you how much longer it will be.

ANS: B A 4-year-old child understands time in relation to events such as meals. Children perceive soon as a very short time. The nurse may lose the childs trust if his parents do not return in the time he perceives as soon. Children cannot read or use a clock for practical purposes until age 7 years. I will tell you every time I see you how much longer it will be assumes the child understands the concepts of hours and minutes, which does not occur until age 5 or 6 years.

The nurse is explaining the purpose of using a vacuum-assisted closure (VAC) device to assist in the healing of a wound. What should the nurse explain as the purpose of using a VAC device? a. The device will decrease capillary flow. b. The device applies gentle continuous suction. c. The device will allow the wound to remain open. d. The device will prevent the formation of granulation tissue.

ANS: B A VAC device uses a technique that involves placing a foam dressing into the wound, covering it with an occlusive dressing, and applying gentle continuous suction. The negative pressure of the suction is applied from the foam dressing to the wound surfaces. The mechanical force removes excess fluids from the wound, stimulates formation of granulation tissue, restores capillary flow, and fosters closure of the wound.

The parent of a 4-year-old boy tells the nurse that the child believes that monsters and boogeymen are in his bedroom at night. What is the nurse's best suggestion for coping with this problem? a. Let the child sleep with his parents. b. Keep a night-light on in the child's bedroom. c. Help the child understand that these fears are illogical. d. Tell the child frequently that monsters and boogeymen do not exist.

ANS: B A night-light shows a child that imaginary creatures do not lurk in the darkness. Letting the child sleep with parents will not get rid of the fears. A 4-year-old child is in the preconceptual age and cannot understand logical thought

At the time of a child's death, the nurse tells his mother, "We will miss him so much." What is the best interpretation of this statement? a. Pretending to be experiencing grief b. Expressing personal feelings of loss c. Denying the mother's sense of loss d. Talking when listening would be better

ANS: B A patient's death is one of the most stressful aspects of critical care or oncology nursing. Nurses experience reactions similar to those of family members because of their involvement with the child and family during the illness. Nurses often have feelings of personal loss when a patient dies.

A 15 year-old girl tells the school nurse that she has not started to menstruate yet. Onset of secondary sexual characteristics was about 2 1/2 years ago. The nurse should take which action? a. Explain that this is not unusual. b. Refer the adolescent for an evaluation. c. Make an assumption that the adolescent is pregnant. d. Suggest that the adolescent stop exercising until menarche occurs.

ANS: B A referral is indicated. Menarche should follow the onset of secondary sexual development within 2 1/2 years. A careful examination is done to reveal any physical abnormalities, signs of androgen excess, and congenital defects of the genital tract. The lack of the onset of menstruation at this age is a potential indication of a physical problem. Assuming that the adolescent is pregnant is inappropriate. The nurse does not have any indication that the adolescent is sexually active. The amount of exercise should be assessed before suggesting that the adolescent stop exercising until menarche occurs.

What statement is true concerning adolescent suicide? a. A sense of hopelessness and despair is a normal part of adolescence. b. Gay and lesbian adolescents are at a particularly high risk for suicide. c. Problem-solving skills are of limited value to the suicidal adolescent. d. Previous suicide attempts are not an indication for completed suicides.

ANS: B A significant number of teenage suicides occur among homosexual youths. Gay and lesbian adolescents who live in families or communities that do not accept homosexuality are likely to experience low self-esteem, self-loathing, depression, and hopelessness. Most adolescents do not experience this stage of life as a time of despair. Depressive symptoms, acting-out behaviors, and talk of suicide need to be taken seriously. At-risk teenagers include those who are depressed, have poor problem-solving skills, or use drugs and alcohol. A history of a previous suicide attempt is a serious indicator for possible suicide completion in the future.

A school nurse is conducting a class with adolescents on suicide. Which true statement about suicide should the nurse include in the teaching session? a. A sense of hopelessness and despair are a normal part of adolescence. b. Gay and lesbian adolescents are at a particularly high risk for suicide. c. Problem-solving skills are of limited value to the suicidal adolescent. d. Previous suicide attempts are not an indication of risk for completed suicides.

ANS: B A significant number of teenage suicides occur among homosexual youths. Gay and lesbian adolescents who live in families or communities that do not accept homosexuality are likely to suffer low self-esteem, self-loathing, depression, and hopelessness as a result of a lack of acceptance from their family or community. At-risk teenagers include those who are depressed, have poor problem-solving skills, or use drugs and alcohol. History of previous suicide attempt is a serious indicator for possible suicide completion in the future.

A 6-year-old boy with very fair skin will be joining his family during a beach vacation. What should the nurse recommend? a. Keep him off the beach during the daytime hours. b. Use sunscreen with an SPF of at least 15 and reapply it every 2 to 3 hours. c. Apply a topical sunscreen product with an SPF of 30 in the morning. d. Dress him in long pants and long-sleeved shirt and keep him under a beach umbrella.

ANS: B A sunscreen with an SPF (sun protection factor) of at least 15 is recommended. The sunscreen should be reapplied every 2 to 3 hours and after the child is in the water or sweating excessively. During a beach vacation, avoiding the beach during daytime hours is impractical. The highest risk of sun exposure is from 10 AM to 3 PM. Sunlight exposure should be limited during this time. An SPF of 30 is good, but reapplying it is necessary every 2 to 3 hours and when the child gets wet. Long pants and a shirt are impractical. The beach umbrella can be used with the sunscreen to limit exposure to the sun.

What is an appropriate nursing intervention to minimize separation anxiety in a hospitalized toddler? a. Provide for privacy b. Encourage parents to room in c. Explain procedures and routines d. Encourage contact with children the same age

ANS: B A toddler experiences separation anxiety secondary to being separated from the parents. To avoid this, the parents should be encouraged to room in as much as possible. Maintaining routines and ensuring privacy are helpful interventions, but they would not substitute for the parents. Encouraging contact with children the same age would not substitute for having the parents present.

Parents are asking about an early intervention program for their child who has special needs. The nurse relates that this program is for which age of child? a. Birth to 1 year of age b. Birth to 3 years of age c. Ages 1 to 4 d. Ages 4 and 5

ANS: B A variety of supplemental programs have been designed in the school system to accommodate special needs, both at school age and younger, through early intervention, which consists of any sustained and systematic effort to assist children from birth to age 3 years with disabilities and those who are developmentally vulnerable.

Which is descriptive of attention deficit hyperactivity disorder (ADHD)? a. Manifestations exhibited are so bizarre that the diagnosis is fairly easy. b. Manifestations affect every aspect of the child's life but are most obvious in the classroom. c. Learning disabilities associated with ADHD eventually disappear when adulthood is reached. d. Diagnosis of ADHD requires that all manifestations of the disorder be present.

ANS: B ADHD affects every aspect of the child's life, but the disruption is most obvious in the classroom. The behaviors exhibited by the child with ADHD are not unusual aspects of behavior. The difference lies in the quality of motor activity and developmentally inappropriate inattention, impulsivity, and hyperactivity that the child displays. Some children experience decreased symptoms during late adolescence and adulthood, but a significant number carry their symptoms into adulthood. Any given child will not have every symptom of the condition. The manifestations may be numerous or few, mild or severe, and will vary with the child's developmental level.

What aspects of cognition develop during adolescence? a. Ability to see things from the point of view of another b. Capability of using a future time perspective c. Capability of placing things in a sensible and logical order d. Progress from making judgments based on what they see to making judgments based on what they reason

ANS: B Adolescents are no longer restricted to the real and actual. They also are concerned with the possible; they think beyond the present. During concrete operations (between ages 7 and 11 years), children exhibit thought processes that enable them to see things from the point of view of another, place things in a sensible and logical order, and progress from making judgments based on what they see to making judgments based on what they reason.

A 14-year-old adolescent never had chickenpox as a child. What should the nurse expect the health care provider to recommend? a. One dose of the varicella vaccination b. Two doses of the varicella vaccination 4 weeks apart c. One dose of the varicella immune globulin d. No vaccinations—the child is past the age to receive it

ANS: B All adolescents should also be assessed for previous history of varicella infection or vaccination. Vaccination with the varicella vaccine is recommended for those with no previous history; for those with no previous infection or history, the varicella vaccine may be given in two doses 4 or more weeks apart to adolescents 13 years or older. The varicella immune globulin is given to immunosuppressed children exposed to chickenpox to boost immunity; it is only temporary. The varicella vaccination should be given to adolescents, no matter the age, who have not had chickenpox as a child.

An awake, alert 4-year-old child has just arrived at the emergency department after an ingestion of aspirin at home. The practitioner has ordered activated charcoal. The nurse administers charcoal in which manner? a. Giving half of the solution and then repeating the other half in 1 hour b. Mixing with a flavorful beverage in an opaque container with a straw c. Serving it in a clear plastic cup so the child can see how much has been drunk d. Administering it through a nasogastric tube because the child will not drink it because of the taste

ANS: B Although activated charcoal can be mixed with a flavorful sugar-free beverage, it will be black and resemble mud. When it is served in an opaque container, the child will not have any preconceived ideas about its being distasteful. The ability to see the charcoal solution may affect the childs desire to drink the solution. The child should be encouraged to drink the solution all at once. The nasogastric tube would be traumatic. It should be used only in children who cannot be cooperative or those without a gag reflex.

What is the major health concern of children in the United States? a. Acute illness b. Chronic illness c. Congenital disabilities d. Nervous system disorders

ANS: B An estimated 18% of children in the United States have a chronic illness or disability that warrants health care services beyond those usually required by children. Chronic illness has surpassed acute illness as the major health concern for children. Congenital disabilities exist from birth but may not be hereditary. These represent a portion of the number of children with chronic illnesses. Mental and nervous system disorders account for approximately 17% of chronic illnesses in children

A child has been admitted to the emergency department with an acetaminophen (Tylenol) poisoning. An antidote is being prescribed by the health care provider. Which antidote should the nurse prepare to administer? a. Naloxone (Narcan) b. N-acetylcysteine (Mucomyst) c. Flumazenil (Romazicon) d. Digoxin immune Fab (Digibind)

ANS: B Antidotes available to treat toxin ingestion include N-acetylcysteine for acetaminophen poisoning, naloxone for opioid overdose, flumazenil (Romazicon) for benzodiazepine (diazepam [Valium], midazolam [Versed]) overdose, and digoxin immune Fab (Digibind) for digoxin toxicity.

The nurse has just collected blood by venipuncture in the antecubital fossa. Which should the nurse do next? a. Keep the arm extended while applying a bandage to the site. b. Keep the arm extended, and apply pressure to the site for a few minutes. c. Apply a bandage to the site, and keep the arm flexed for 10 minutes. d. Apply a gauze pad or cotton ball to the site, and keep the arm flexed for several minutes.

ANS: B Applying pressure to the site of venipuncture stops the bleeding and aids in coagulation. Pressure should be applied before bandage is applied.

Which is descriptive of a toddler's cognitive development at age 20 months? a. Searches for an object only if he or she sees it being hidden b. Realizes that "out of sight" is not out of reach c. Puts objects into a container but cannot take them out d. Understands the passage of time, such as "just a minute" and "in an hour"

ANS: B At this age, the child is in the final sensorimotor stage. Children will now search for an object in several potential places, even though they saw only the original hiding place. Children have a more developed sense of objective permanence. They will search for objects even if they have not seen them hidden. When a child puts objects into a container but cannot take them out, this is indicative of tertiary circular reactions. An embryonic sense of time exists, although the children may behave appropriately to time-oriented phrases; their sense of timing is exaggerated.

Kelly, an 8-year-old girl, will soon be able to return to school after an injury that resulted in several severe, chronic disabilities. Which is the most appropriate action by the school nurse? a. Recommend that the child's parents attend school at first to prevent teasing. b. Prepare the child's classmates and teachers for changes they can expect. c. Refer the child to a school where the children have chronic disabilities similar to hers. d. Discuss with the child and her parents the fact that her classmates will not accept her as they did before.

ANS: B Attendance at school is an important part of normalization for Kelly. The school nurse should prepare teachers and classmates about her condition, abilities, and special needs. A visit by the parents can be helpful, but unless the classmates are prepared for the changes, it alone will not prevent teasing. Kelly's school experience should be normalized as much as possible. Children need the opportunity to interact with healthy peers, as well as to engage in activities with groups or clubs composed of similarly affected persons. Children with special needs are encouraged to maintain and reestablish relationships with peers and to participate according to their capabilities.

An adolescent has been diagnosed with Chlamydia infection. Which medication should the nurse expect to be prescribed for this condition? a. Ceftriaxone (Rocephin) IM b. Azithromycin (Zithromax) PO c. Acyclovir (Zovirax) PO d. Penicillin G benzathine (Bicillin) IV

ANS: B Azithromycin is used to treat Chlamydia. The patient should be rescreened in 3 to 4 months. Ceftriaxone is used to treat gonorrhea, acyclovir is used to suppress genital herpes simplex virus, and penicillin G benzathine is used to treat syphilis.

A nurse is beginning chelation therapy on a child for lead poisoning. Which intervention should the nurse implement during the time the child is receiving chelation therapy? a. Calorie counts b. Strict intake and output c. Telemetry monitoring d. Contact isolation

ANS: B Because calcium disodium edetate (EDTA) and lead are toxic to the kidneys, a nurse should keep strict records of intake and output to monitor renal functioning. Adequate hydration is essential during therapy because the chelates are excreted via the kidneys. Calorie counts, telemetry, or contact isolation would not be nursing interventions appropriate for a child undergoing chelation therapy.

Which best describes how preschoolers react to the death of a loved one? a. A preschooler is too young to have a concept of death. b. A preschooler may feel guilty and responsible for the death. c. Grief is acute but does not last long at this age. d. Grief is usually expressed in the same way in which the adults in the preschooler's life are expressing grief.

ANS: B Because of egocentricity, the preschooler may feel guilty and responsible for the death.

A male school-age student asks the school nurse, How much with my height increase in a year? The nurse should give which response? a. Your height will increase on average 1 inch a year. b. Your height will increase on average 2 inches a year. c. Your height will increase on average 3 inches a year. d. Your height will increase on average 4 inches a year.

ANS: B Between the ages of 6 and 12 years, children grow an average of 5 cm (2 inches) per year.

29. What is the most frequent cause of hypovolemic shock in children? a. Sepsis b. Blood loss c. Anaphylaxis d. Heartfailure

ANS: B Blood loss is the most frequent cause of hypovolemic shock in children. Sepsis causes septic shock, which is overwhelming sepsis and circulating bacterial toxins. Anaphylactic shock results from extreme allergy or hypersensitivity to a foreign substance. Heart failure contributes to hypervolemia, not hypovolemia.

1A child is admitted to the hospital with lesions on his abdomen that appear like cigarette burns. What should accurate documentation by the nurse include? a. Two unhealed lesions are on the childs abdomen. b. Two round 4-mm lesions are on the childs lower abdomen. c. Two round symmetrical lesions are on the childs lower abdomen. d. Two round lesions on the childs abdomen that appear to be cigarette burns.

ANS: B Burn documentation should include the location, pattern, demarcation lines, and presence of eschar or blisters. The option that includes the size of the lesions is the most accurate.

The school nurse is providing guidance to families of children who are entering elementary school. What is essential information to include? a. Meet with teachers only at scheduled conferences. b. Encourage growth of a sense of responsibility in children. c. Provide tutoring for children to ensure mastery of material. d. Homework should be done as soon as child comes home from school.

ANS: B By being responsible for school work, children learn to keep promises, meet deadlines, and succeed in their jobs as adults. Parents should meet with the teachers at the beginning of the school year, for scheduled conferences, and whenever information about the child or parental concerns needs to be shared. Tutoring should be provided only in special circumstances in elementary school, such as in response to prolonged absence. The parent should not dictate the study time but should establish guidelines to ensure that homework is done.

When a child with mild cognitive impairment reaches the end of adolescence, which characteristic should be expected? a. Achieves a mental age of 5 to 6 years b. Achieves a mental age of 8 to 12 years c. Unable to progress in functional reading or arithmetic d. Acquires practical skills and useful reading and arithmetic to an eighth-grade level

ANS: B By the end of adolescence, the child with mild cognitive impairment can acquire practical skills and useful reading and arithmetic to a third- to sixth-grade level. A mental age of 8 to 12 years is obtainable, and the child can be guided toward social conformity. The achievement of a mental age of 5 to 6 years and being unable to progress in functional reading or arithmetic are characteristics of children with moderate cognitive impairment. Acquiring practical skills and useful reading and arithmetic to an eighth-grade level is not descriptive of cognitive impairment.

Which is descriptive of central nervous system stimulants? a. They produce strong physical dependence. b. They can result in strong psychological dependence. c. Withdrawal symptoms are life threatening. d. Acute intoxication can lead to coma.

ANS: B Central nervous system stimulants such as amphetamines and cocaine produce a strong psychological dependence. This class of drugs does not produce strong physical dependence and can be withdrawn without much danger. Acute intoxication leads to violent, aggressive behavior or psychotic episodes characterized by paranoia, uncontrollable agitation, and restlessness.

What is the most common form of child maltreatment? a. Sexual abuse b. Child neglect c. Physical abuse d. Emotional abuse

ANS: B Child neglect, which is characterized by the failure to provide for the childs basic needs, is the most common form of child maltreatment. Sexual abuse, physical abuse, and emotional abuse are individually not as common as neglect.

Parents are switching their toddler, who has met the weight requirement, from a rear-facing car seat to a forward-facing seat. The nurse should recommend the parents place the seat where in the car? a. In the front passenger seat b. In the middle of the rear seat c. In the rear seat behind the driver d. In the rear seat behind the passenger

ANS: B Children 0 to 3 years of age riding properly restrained in the middle of the backseat have a 43% lower risk of injury than children riding in the outboard (window) seat during a crash.

Mark, a 9-year-old with Down syndrome, is mainstreamed into a regular third-grade class for part of the school day. His mother asks the school nurse about programs, such as Cub Scouts, that he might join. The nurse's recommendation should be based on which statement? a. Programs like Cub Scouts are inappropriate for children who are cognitively impaired. b. Children with Down syndrome have the same need for socialization as other children. c. Children with Down syndrome socialize better with children who have similar disabilities. d. Parents of children with Down syndrome encourage programs, such as scouting, because they deny that their children have disabilities.

ANS: B Children of all ages need peer relationships. Children with Down syndrome should have peer experiences similar to those of other children, such as group outings, Cub Scouts, and Special Olympics. Programs such as Cub Scouts can help children with cognitive impairment develop socialization skills. Although all children should have an opportunity to form a close relationship with someone of the same developmental level, it is appropriate for children with disabilities to develop relationships with children who do not have disabilities. The parents are acting as advocates for their child

The nurse is caring for an adolescent brought to the hospital with acute drug toxicity. Cocaine is believed to be the drug involved. Data collection by the nurse should include what information? a. Drugs actual content b. Mode of administration c. Adolescents level of interest in rehabilitation d. Function the drug plays in the adolescents life

ANS: B Cocaine is available in two forms, water soluble and nonwater soluble, and can be administered through multiple routes. For treatment purposes, it is essential to know the type of drug and route of administration. Because cocaine is a street drug, the actual content usually cannot be identified. The adolescents level of interest in rehabilitation and the function that drug plays in the adolescents life are concerns to be addressed after the initial emergency treatment is instituted.

What is a psychological effect of being obese during adolescence? a. Sexual promiscuity b. Poor body image c. Feelings of contempt for thin peers d. Accurate body image but self-deprecating attitude

ANS: B Common emotional consequences of obesity include poor body image, low self-esteem, social isolation, and feelings of depression and isolation. Sexual promiscuity, feelings of contempt for thin peers, and accurate body image but self-deprecating attitude are not usually associated with obesity.

The parents of a 7-year-old boy tell the nurse that lately he has been cruel to their family pets and actually caused physical harm. The nurses recommendation should be based on remembering what? a. This is an expected behavior at this age. b. This is a warning sign of a serious problem. c. This is harmless venting of anger and frustration. d. This is common in children who are physically abused

ANS: B Cruelty to family pets is not an expected behavior. Hurting animals can be one of the earliest symptoms of a conduct disorder. Abusing animals does not dissipate violent emotions; rather, the acts may fuel the abusive behaviors. Referral for evaluation is essential. This behavior may be seen in emotional abuse or neglect, not physical abuse

Parents of a child with Down syndrome ask the nurse about techniques for introducing solid food to their 8-month-old child's diet. The nurse should give the parents which priority instruction? a. It is too early to add solids; the parents should wait for 2 to 3 months. b. A small but long, straight-handled spoon should be used to push the food toward the back and side of the mouth. c. If the child thrusts the food out, the feeding should be stopped. d. Solids should be offered only three times a day.

ANS: B Down syndrome children have a protruding tongue which can interfere with feeding, especially of solid foods. Parents need to know that the tongue thrust is not an indication of refusal to feed but a physiologic response. Parents are advised to use a small but long, straight-handled spoon to push the food toward the back and side of the mouth. If food is thrust out, it should be re-fed. Six months is the time to introduce solid foods to a child, so waiting 2 to 3 months is inappropriate. Small frequent feedings should be initiated to prevent the child from tiring. Three times a day is too infrequent.

Which predisposes the adolescent to feel an increased need for sleep? a. An inadequate diet b. Rapid physical growth c. Decreased activity that contributes to a feeling of fatigue d. The lack of ambition typical of this age group

ANS: B During growth spurts, the need for sleep increases. Rapid physical growth, the tendency toward overexertion, and the overall increased activity of this age contribute to fatigue.

Which statement best describes fear in the school-age child? a. They are increasingly fearful for body safety. b. Most of the new fears that trouble them are related to school and family. c. They should be encouraged to hide their fears to prevent ridicule by peers. d. Those who have numerous fears need continuous protective behavior by parents to eliminate these fears.

ANS: B During the school-age years, children experience a wide variety of fears, but new fears relate predominantly to school and family. During the middle-school years, children become less fearful for body safety than they were as preschoolers. Parents and other persons involved with children should discuss children's fears with them individually or as a group activity. Sometimes school-age children hide their fears to avoid being teased. Hiding their fears does not end them and may lead to phobias

Which should the nurse expect for a toddler's language development at age 18 months? a. Vocabulary of 25 words b. Increasing level of comprehension c. Use of holophrases d. Approximately one third of speech understandable

ANS: B During the second year of life, level of comprehension and understanding of speech increases and is far greater than the child's vocabulary. This is also true for bilingual children, who are able to achieve this linguistic milestone in both languages. The 18-month-old child has a vocabulary of 10 or more words. At this age, the child does not use the one-word sentences that are characteristic of the 1-year-old child. The child has a limited vocabulary of single words that are comprehensible.

A 12-year-old girl asks the nurse about an increase in clear white odorless vaginal discharge. What response should the nurse give? a. This may mean a yeast infection. b. This is normal before menstruation starts. c. This is caused by an increase in progesterone. d. This is possibly a sign of a sexually transmitted infection.

ANS: B Early in puberty, there is often an increase in normal vaginal discharge (physiologic leukorrhea) associated with uterine development. Girls or their parents may be concerned that this vaginal discharge is a sign of infection. The nurse can reassure them that the discharge is normal and a sign that the uterus is preparing for menstruation. It is caused by an increase in estrogen, not progesterone.

Which intervention will encourage a sense of autonomy in a toddler with disabilities? a. Avoid separation from family during hospitalizations. b. Encourage independence in as many areas as possible. c. Expose child to pleasurable experiences as much as possible. d. Help parents learn special care needs of their child.

ANS: B Encouraging the toddler to be independent encourages a sense of autonomy. The child can be given choices about feeding, dressing, and diversional activities, which will provide a sense of control. Avoiding separation from family during hospitalizations, and helping parents learn special care needs of their child should be practiced as part of family-centered care. It does not necessarily foster autonomy. Exposing the child to pleasurable experiences, especially sensory ones, is a supportive intervention. It does not promote autonomy.

Families progress through various stages of reactions when a child is diagnosed with a chronic illness or disability. After the shock phase, a period of adjustment usually follows. This is often characterized by which of the following responses? a. Denial b. Guilt and anger c. Social reintegration d. Acceptance of the child's limitations

ANS: B For most families, the adjustment phase is accompanied by several responses. Guilt, self-accusation, bitterness, and anger are common reactions. The initial diagnosis of a chronic illness or disability often is met with intense emotion, characterized by shock and denial. Social reintegration and acceptance of the child's limitations are the culmination of the adjustment process.

What is the most effective way to clean a toddler's teeth? a. Child to brush regularly with a toothpaste of his or her choice b. Parent to stabilize the chin with one hand and brush with the other c. Parent to brush the mandibular occlusive surfaces, leaving the rest for the child d. Parent to brush the front labial surfaces, leaving the rest for the child

ANS: B For young children, the most effective cleaning of teeth is by the parents. Different positions can be used if the child's back is to the adult. The adult should use one hand to stabilize the chin and the other to brush the child's teeth. The child can participate in brushing, but for a thorough cleaning, adult intervention is necessary.

Which is an important consideration for the nurse when changing dressings and applying topical medication to a child's abdomen and leg burns? a. Apply topical medication with clean hands. b. Wash hands and forearms before and after dressing change. c. If dressings adhere to the wound, soak in hot water before removal. d. Apply dressing so that movement is limited during the healing process.

ANS: B Frequent hand and forearm washing is the single most important element of the infection-control program. Topical medications should be applied with a tongue blade or gloved hand. Dressings that have adhered to the wound can be removed with tepid water or normal saline. Dressings are applied with sufficient tension to remain in place but not so tightly as to impair circulation or limit motion.

Parents are concerned that their 6-year-old son continues to occasionally wet the bed. What does the nurse explain? a. This is likely because of increased stress at home. b. Enuresis usually ceases between 6 and 8 years of age. c. Drug therapy will be prescribed to treat the enuresis. d. Testing will be necessary to determine what type of kidney problem exists.

ANS: B Further data must be gathered before the diagnosis of enuresis is made. Enuresis is the inappropriate voiding of urine at least twice a week. This child does meet the age criterion, but the parents need to be questioned about and keep a diary on the frequency of events. If the bedwetting is infrequent, parents can be encouraged that the child may grow out of this behavior. Drug therapy will not be prescribed until a more complete evaluation is done. Additional assessment information must be gathered, but at this time, there is no indication of renal disease.

What is a clinical manifestation of acetaminophen poisoning? a. Hyperpyrexia b. Hepatic involvement c. Severe burning pain in stomach d. Drooling and inability to clear secretions

ANS: B Hepatic involvement is the third stage of acetaminophen poisoning. Hyperpyrexia is a severe elevation in body temperature and is not related to acetaminophen poisoning. Acetaminophen does not cause burning pain in stomach and does not pose an airway threat.

Which is a clinical manifestation of acetaminophen poisoning? a. Hyperpyrexia b. Hepatic involvement c. Severe burning pain in stomach d. Drooling and inability to clear secretions

ANS: B Hepatic involvement is the third stage of acetaminophen poisoning. Hyperpyrexia is a severe elevation in body temperature and is not related to acetaminophen poisoning. Acetaminophen does not cause burning pain in stomach or pose an airway threat.

1. What type of dehydration occurs when the electrolyte deficit exceeds the water deficit? a. Isotonic dehydration b. Hypotonic dehydration c. Hypertonic dehydration d. Hyperosmotic dehydration

ANS: B Hypotonic dehydration occurs when the electrolyte deficit exceeds the water deficit, leaving the serum hypotonic. Isotonic dehydration occurs in conditions in which electrolyte and water deficits are present in balanced proportion. Hypertonic dehydration results from water loss in excess of electrolyte loss. This is the most dangerous type of dehydration. It is caused by feeding children fluids with high amounts of solute. Hyperosmotic dehydration is another term for hypertonic dehydration.

What is descriptive of the social development of school-age children? a. Identification with peers is minimum. b. Children frequently have best friends. c. Boys and girls play equally with each other. d. Peer approval is not yet an influence for the child to conform.

ANS: B Identification with peers is a strong influence in childrens gaining independence from parents. Interaction among peers leads to the formation of close friendships with same-sex peersbest friends. Daily relationships with age mates in the school setting provide important social interactions for school-age children. During the later school years, groups are composed predominantly of children of the same sex. Conforming to the rules of the peer group provides children with a sense of security and relieves them of the responsibility of making decisions

The nurse is planning to prepare a 4-year-old child for some diagnostic procedures. Guidelines for preparing this preschooler should include which action? a. Plan for a short teaching session of about 30 minutes. b. Tell the child that procedures are never a form of punishment. c. Keep equipment out of the child's view. d. Use correct scientific and medical terminology in explanations.

ANS: B Illness and hospitalization may be viewed as punishment in preschoolers. Always state directly that procedures are never a form of punishment. Teaching sessions for this age group should be 10 to 15 minutes in length. Demonstrate the use of equipment, and allow the child to play with miniature or actual equipment. Explain procedure in simple terms and how it affects the child.

A nurse, instructing parents of a hospitalized preschool child, explains that which is descriptive of the preschooler's understanding of time? a. Has no understanding of time b. Associates time with events c. Can tell time on a clock d. Uses terms like "yesterday" appropriately

ANS: B In a preschooler's understanding, time has a relation with events such as "We'll go outside after lunch." Preschoolers develop an abstract sense of time at age 3 years. Children can tell time on a clock at age 7 years. Children do not fully understand use of time-oriented words until age 6 years.

The nurse is teaching a group of 10- to 12-year-old children about physical development during the school-age years. Which statement made by a participant, indicates the correct understanding of the teaching? a. "My body weight will be almost triple in the next few years." b. "I will grow an average of 2 inches per year from this point on." c. "There are not that many physical differences among school-age children." d. "I will have a gradual increase in fat, which may contribute to a heavier appearance."

ANS: B In middle childhood, growth in height and weight occurs at a slower pace. Between the ages of 6 and 12 years, children grow 2 inches per year. In middle childhood, children's weight will almost double; they gain 3 kg/year. At the end of middle childhood, girls grow taller and gain more weight than boys. Children take on a slimmer look with longer legs in middle childhood.

A 2-year-old child comes to the emergency department with dehydration and hypovolemic shock. Which best explains why an intraosseous infusion is started? a. It is less painful for small children. b. Rapid venous access is not possible. c. Antibiotics must be started immediately. d. Long-term central venous access is not possible.

ANS: B In situations in which rapid establishment of systemic access is vital and venous access is hampered, such as peripheral circulatory collapse and hypovolemic shock, intraosseous infusion provides a rapid, safe, lifesaving alternative. The procedure is painful, and local anesthetics and systemic analgesics are given. Antibiotics could be given when vascular access is obtained. Long-term central venous access is time-consuming, and intraosseous infusion is used in an emergency situation.

It is important that women with anogenital warts caused by the human papillomavirus (HPV) receive adequate treatment because this sexually transmitted infection increases the risk of what? a. Gonorrhea b. Cervical cancer c. Chlamydial infection d. Urinary tract infection

ANS: B Infection with HPV is associated with cervical dysplasia and cervical cancer. A vaccine has been developed and is recommended for young women.

The parent of a 4-year-old boy tells the nurse that the child believes monsters and bogeymen are in his bedroom at night. What is the nurses best suggestion for coping with this problem? a. Let the child sleep with his parents. b. Keep a night light on in the childs bedroom. c. Help the child understand that these fears are illogical. d. Tell the child that monsters and bogeymen do not exist.

ANS: B Involve the child in problem solving. A night light shows a child that imaginary creatures do not lurk in the darkness. Letting the child sleep with his parents will not get rid of the fears. A 4- year-old child is in the preconceptual stage and cannot understand logical thought.

In preparing to give "enemas until clear" to a young child, the nurse should select which solution? a. Tap water b. Normal saline c. Oil retention d. Fleet solution

ANS: B Isotonic solutions should be used in children. Saline is the solution of choice. Plain water is not used. This is a hypotonic solution and can cause rapid fluid shift, resulting in fluid overload. Oil-retention enemas will not achieve the "until clear" result. Fleet enemas are not advised for children because of the harsh action of the ingredients. The osmotic effects of the Fleet enema can result in diarrhea, which can lead to metabolic acidosis.

The nurse has been visiting an adolescent with recently acquired tetraplegia. The teens mother tells the nurse, Im sick of providing all the care while my husband does whatever he wants to, whenever he wants to do it. What reaction should be the nurses initial response? a. Refer the mother for counseling. b. Listen and reflect the mothers feelings. c. Ask the father in private why he does not help. d. Suggest ways the mother can get her husband to help.

ANS: B It is appropriate for the nurse to reflect with the mother about her feelings, exploring solutions such as an additional home health aide to help care for the child and provide respite for the mother. It is inappropriate for the nurse to agree with the mother that her husband is not helping enough. This judgment is beyond the role of the nurse and can undermine the family relationship. Counseling, if indicated, would be necessary for both parents. A support group for caregivers may be indicated. The nurse should not ask the father in private why he does not help or suggest way the mother can get her husband to help. These interventions are based on the mothers perceptions; the father may have a full-time job and other commitments. The parents may need an unbiased third person to help them through the negotiation of their new parenting responsibilities.

Parents phone the nurse and say that their child just knocked out a permanent tooth. What should the nurses instructions to the parents include? a. Place the tooth in dry container for transport. b. Hold the tooth by the crown and not by the root area. c. Transport the child and tooth to a dentist within 18 hours. d. Take the child to hospital emergency department if his or her mouth is bleeding.

ANS: B It is important to avoid touching the root area of the tooth. The tooth should be held by the crown area; rinsed in milk, saline, or running water; and reimplanted as soon as possible. The tooth is kept moist during transport to maintain viability. Cold milk is the most desirable medium for transport. The child needs to be seen by a dentist as soon as possible. Tooth evulsion causes a large amount of bleeding. The child will need to be seen by a dentist because of the loss of a tooth, not the bleeding.

Kimberly's parents have been using a rearward-facing, convertible car seat since she was born. Most car seats can be safely switched to the forward-facing position when the child reaches which age? a. 1 b. 2 c. 3 d. 4

ANS: B It is now recommended that all infants and toddlers ride in rear-facing car safety seats until they reach the age of 2 years or height recommended by the car seat manufacturer. Children 2 years old and older who have outgrown the rear-facing height or weight limit for their car safety seat should use a forward-facing car safety seat with a harness up to the maximum height or weight recommended by the manufacturer. One year is too young to switch to a forward-facing position.

In terms of cognitive development, a 5-year-old child should be expected to do which? a. Think abstractly. b. Use magical thinking. c. Understand conservation of matter. d. Understand another persons perspective.

ANS: B Magical thinking is believing that thoughts can cause events. An example is thinking of the death of a parent might cause it to happen. Abstract thought does not develop until the school-age years. The concept of conservation is the cognitive task of school-age children, ages 5 to 7 years. A 5-year-old child cannot understand another persons perspective.

An adolescent has been diagnosed with lactose maldigestion intolerance. The nurse teaches the adolescent about lactose maldigestion intolerance and notes the teen needs further teaching if which statement is made? a. "I will limit my milk consumption to one to two glasses a day." b. "I should drink the milk alone and not with other foods." c. "Hard cheese, cottage cheese, or yogurt can be substituted for milk." d. "I will take a calcium supplement daily."

ANS: B Most people are able to tolerate small amounts of lactose (≈1 cup of milk per day) even in the presence of deficient lactase activity. It is recommended that individuals with lactose maldigestion who do not experience lactose intolerance symptoms continue to consume small amounts of dairy products with meals to prevent reduced bone mass density and subsequent osteoporosis. Hard cheese, cottage cheese, and yogurt are sources of lactose that may be better tolerated. A calcium supplement should be taken daily. Milk taken at meals may be better tolerated than when taken alone.

A toddler has a deep laceration contaminated with dirt and sand. Before closing the wound, the nurse should irrigate with what solution? a. Alcohol b. Normal saline c. Povidoneiodine d. Hydrogen peroxide

ANS: B Normal saline is the only acceptable fluid for irrigation listed. The nurse should cleanse the wound with a forced stream of normal saline or water. Alcohol is not used for wound irrigation. Povidoneiodine is contraindicated for cleansing fresh, open wounds. Hydrogen peroxide can cause formation of subcutaneous gas when applied under pressure.

A school-age child with chronic renal failure is admitted to the hospital with a serum potassium level of 5.2 mEq/L. Which prescribed medication should the nurse plan to administer? a. Spironolactone (Aldactone) b. Sodium polystyrene sulfonate (Kayexalate) c. Lactulose (Cephulac) d. Calcium carbonate (Calcitab)

ANS: B Normal serum potassium levels in a school-age child are 3.5 to 5 mEq/L. Sodium polystyrene sulfonate is administered to reduce serum potassium levels. Spironolactone is a potassium-sparing diuretic and should not be used if the serum potassium is elevated. Lactulose is administered to reduce ammonia levels in patients with liver disease. Calcium carbonate may be prescribed as a calcium supplement, but it will not reduce serum potassium levels.

Which represents a common best practice in the provision of services to children with special needs? a. Care is now being focused on the child's chronologic age. b. Children with special needs are being integrated into regular classrooms. c. Children with special needs no longer have to be cared for by their families. d. Children with special needs are being separated into residential treatment facilities.

ANS: B Normalization refers to behaviors and interventions for the disabled to integrate into society by living life as persons without a disability would. For children, normalization includes attending school and being integrated into regular classrooms. This affords the child the advantages of learning with a wide group of peers. Care is necessarily focused on the child's developmental age. Home care by the family is considered best practice. The nurse can assist families by assessing social support systems, coping strategies, family cohesiveness, and family and community resources.

A new nurse is caring for a child who will require palliative care. Which statement made by the new nurse would indicate a correct understanding of palliative care? a. "Palliative care serves to hasten death and make the process easier for the family." b. "Palliative care provides pain and symptom management for the child." c. "The goal of palliative care is to place the child in a hospice setting at the end of life." d. "The goal of palliative care is to act as the liaison between the family, child, and other health care professionals."

ANS: B One of the goals of palliative care is to provide pain and symptom management.

Calcium carbonate is given with meals to a child with chronic renal disease. What is the purpose of administering calcium carbonate? a. Prevent vomiting b. Bind phosphorus c. Stimulate appetite d. Increase absorption of fat-soluble vitamins

ANS: B Oral calcium carbonate preparations combine with phosphorus to decrease gastrointestinal absorption and the serum levels of phosphate. Serum calcium levels are increased by the calcium carbonate, and vitamin D administration is necessary to increase calcium absorption. Calcium carbonate does not prevent vomiting, stimulate appetite, or increase the absorption of fat-soluble vitamins.

A nurse is planning palliative care for a child with severe pain. Which should the nurse expect to be prescribed for pain relief? a. Opioids as needed b. Opioids on a regular schedule c. Distraction and relaxation techniques d. Nonsteroidal anti-inflammatory drugs

ANS: B Pain medications, for children in palliative care, should be given on a regular schedule, and extra doses for breakthrough pain should be available to maintain comfort. Opioid drugs such as morphine should be given for severe pain, and the dose should be increased as necessary to maintain optimal pain relief. Techniques such as distraction, relaxation techniques, and guided imagery should be combined with drug therapy to provide the child and family strategies to control pain. Nonsteroidal anti-inflammatory drugs are not sufficient to manage severe pain for children in palliative care.

The nurse comes into the room of a child who was just diagnosed with a chronic disability. The child's parents begin to yell at the nurse about a variety of concerns. Which is the nurse's best response? a. "What is really wrong?" b. "Being angry is only natural." c. "Yelling at me will not change things." d. "I will come back when you settle down."

ANS: B Parental anger after the diagnosis of a child with a chronic disability is a common response. One of the most common targets for parental anger is members of the staff. The nurse should recognize the common response of anger to the diagnosis and allow the family to vent. "What is really wrong?"/"Yelling at me will not change things"/"I will come back when you settle down" will place the parents on the defensive and not facilitate communication.

Lindsey, age 5 years, will be starting kindergarten next month. She has cerebral palsy, and it has been determined that she needs to be in a special education classroom. Her parents are tearful when telling the nurse about this and state that they did not realize her disability was so severe. What is the best interpretation of this situation? a. This is a sign parents are in denial b. This is a normal anticipated time of parental stress c. The parents need to learn more about cerebral palsy d. The parents are used to having expectations that are too high

ANS: B Parenting a child with a chronic illness can be stressful for parents. There are anticipated times that parental stress increases. One of these identified times is when the child begins school. Nurses can help parents recognize and plan interventions to work through these stressful periods. The parents are not in denial; they are responding to the child's placement in school. The parents are not exhibiting signs of a knowledge deficit; this is their first interaction with the school system with this child.

A 5-year-old child will be starting kindergarten next month. She has cerebral palsy, and it has been determined that she needs to be in a special education classroom. Her parents are tearful when telling the nurse about this and state that they did not realize her disability was so severe. What is the best interpretation of this situation? a. This is a sign the parents are in denial. b. This is a normal anticipated time of parental stress. c. The parents need to learn more about cerebral palsy. d. The parents expectations are too high.

ANS: B Parenting a child with a chronic illness can be stressful. At certain anticipated times, parental stress increases. One of these identified times is when the child begins school. Nurses can help parents recognize and plan interventions to work through these stressful periods. The parents are not in denial; rather, they are responding to the childs placement in school. The parents are not exhibiting signs of a remembering deficit; this is their first interaction with the school system with this child.

A nurse is conducting parenting classes for parents of adolescents. Which parenting style should the nurse recommend? a. Laissez-faire b. Authoritative c. Disciplinarian d. Confrontational

ANS: B Parents should be guided toward an authoritative style of parenting in which authority is used to guide the adolescent while allowing developmentally appropriate levels of freedom and providing clear, consistent messages regarding expectations. The authoritative style of parenting has been shown to have both immediate and long-term protective effects toward adolescent risk reduction. The laissez-faire method would not give adolescents enough structure. The disciplinarian and confrontational styles would not allow any autonomy or independence.

The nurse is conducting a staff in-service on newborn defects of the genitourinary system. Which describes the narrowing of the preputial opening of the foreskin? a. Chordee b. Phimosis c. Epispadias d. Hypospadias

ANS: B Phimosis is the narrowing or stenosis of the preputial opening of the foreskin. Chordee is the ventral curvature of the penis. Epispadias is the meatal opening on the dorsal surface of the penis. Hypospadias is a congenital condition in which the urethral opening is located anywhere along the ventral surface of the penis.

A teen asks a nurse, "What is physical dependence in substance abuse?" Which is the correct response by the nurse? a. Problem that occurs in conjunction with addiction b. Involuntary physiologic response to drug c. Culturally defined use of drugs for purposes other than accepted medical purposes d. Voluntary behavior based on psychosocial needs

ANS: B Physical dependence is an involuntary response to the pharmacologic characteristics of drugs such as opioids or alcohol. A person can be physically dependent on a narcotic/drug without being addicted; for example, patients who use opioids to control pain need increasing doses to achieve the same effect. Dependence is a physiologic response; it is not culturally determined or subject to voluntary control.

Two toddlers are playing in a sandbox when one child suddenly grabs a toy from the other child. Which is the best interpretation of this behavior? a. This is typical behavior because toddlers are aggressive. b. This is typical behavior because toddlers are egocentric. c. Toddlers should know that sharing toys is expected of them. d. Toddlers should have the cognitive ability to know right from wrong.

ANS: B Play develops from the solitary play of infancy to the parallel play of toddlers. The toddler plays alongside other children, not with them. This typical behavior of the toddler is not intentionally aggressive. Shared play is not within their cognitive development. Toddlers do not conceptualize shared play. Because the toddler cannot view the situation from the perspective of the other child, it is okay to take the toy. Therefore, no right or wrong is associated with taking a toy.

A school nurse observes school-age children playing at recess. Which is descriptive of the play the nurse expects to observe? a. Individuality in play is better tolerated than at earlier ages. b. Knowing the rules of a game gives an important sense of belonging. c. They like to invent games, making up the rules as they go. d. Team play helps children learn the universal importance of competition and winning

ANS: B Play involves increased physical skill, intellectual ability, and fantasy. Children form groups and cliques and develop a sense of belonging to a team or club. At this age, children begin to see the need for rules. Conformity and ritual permeate their play. Their games have fixed and unvarying rules, which may be bizarre and extraordinarily rigid. With team play, children learn about competition and the importance of winning, an attribute highly valued in the United States

Which play item should the nurse bring from the playroom to a hospitalized toddler in isolation? a. Small plastic Lego b. Set of large plastic building blocks c. Brightly colored balloon d. Coloring book and crayons

ANS: B Play objects for toddlers must still be chosen with an awareness of danger from small parts. Large, sturdy toys without sharp edges or removable parts are safest. Large plastic blocks are appropriate for a toddler in isolation. Small plastic toys such as Lego can cause choking or can be aspirated. Balloons can cause significant harm if swallowed or aspirated. Coloring book and crayons would be too advanced for a toddler.

Using knowledge of child development, which is the best approach when preparing a toddler for a procedure? a. Avoid asking the child to make choices. b. Demonstrate the procedure on a doll. c. Plan for the teaching session to last about 20 minutes. d. Show necessary equipment without allowing the child to handle it.

ANS: B Prepare toddlers for procedures by using play. Demonstrate on a doll, but avoid the child's favorite doll because the toddler may think the doll is really "feeling" the procedure. In preparing a toddler for a procedure, allow the child to participate in care and help whenever possible. Teaching sessions for toddlers should be about 5 to 10 minutes. Use a small replica of the equipment, and allow the child to handle it.

The feeling of guilt that the child "caused" the disability or illness is especially critical in which child? a. Toddler b. Preschooler c. School-age child d. Adolescent

ANS: B Preschoolers are most likely to be affected by feelings of guilt that they caused the illness or disability or are being punished for wrongdoings. Toddlers are focused on establishing their autonomy. The illness will foster dependence. The school-age child will have limited opportunities for achievement and may not be able to understand limitations. Adolescents face the task of incorporating their disabilities into their changing self-concept.

A 4-year-old boy is hospitalized with a serious bacterial infection. He tells the nurse that he is sick because he was bad. What is the nurses best interpretation of this comment? a. Sign of stress b. Common at this age c. Suggestive of maladaptation d. Suggestive of excessive discipline at home

ANS: B Preschoolers cannot understand the cause and effect of illness. Their egocentrism makes them think they are directly responsible for events, making them feel guilt for things outside of their control. Children of this age react to stress by regressing developmentally or acting out. Maladaptation is unlikely. This comment does not imply excessive discipline at home.

A 4-year-old child is hospitalized with a serious bacterial infection. The child tells the nurse that he is sick because he was "bad." Which is the nurse's best interpretation of this comment? a. Sign of stress b. Common at this age c. Suggestive of maladaptation d. Suggestive of excessive discipline at home

ANS: B Preschoolers cannot understand the cause and effect of illness. Their egocentrism makes them think they are directly responsible for events, making them feel guilty for things outside their control. Children of this age show stress by regressing developmentally or acting out. Maladaptation is unlikely. Telling the nurse that he is sick because he was "bad" does not imply excessive discipline at home.

An adolescent girl is brought to the hospital emergency department by her parents after being raped. The girl is calm and controlled throughout the interview and examination. The nurse should recognize this behavior is what? a. A sign that a rape has not actually occurred b. One of a variety of behaviors normally seen in rape victims c. Indicative of a higher than usual level of maturity in the adolescent d. Suggestive that the adolescent had severe emotional problems before the rape occurred

ANS: B Rape victims display a wide range of behaviors. A controlled manner may be an attempt to maintain composure while hiding the inner turmoil. Because the observed behavior is within the range of expected behavior, there are no data to indicate that a rape has not actually occurred, that the adolescent is unusually mature, or that she had severe emotional problems before the rape occurred.

Which is an appropriate recommendation in preventing tooth decay in young children? a. Substitute raisins for candy. b. Substitute sugarless gum for regular gum. c. Use honey or molasses instead of refined sugar. d. When sweets are to be eaten, select a time not during meals.

ANS: B Regular gum has high sugar content. When the child chews gum, the sugar is in prolonged contact with the teeth. Sugarless gum is less cariogenic than regular gum. Raisins, honey, and molasses are highly cariogenic and should be avoided. Sweets should be consumed with meals so that the teeth can be cleaned afterward. This decreases the amount of time that the sugar is in contact with the teeth.

Which is an appropriate recommendation for preventing tooth decay in young children? a. Substitute raisins for candy. b. Substitute sugarless gum for regular gum. c. Use honey or molasses instead of refined sugar. d. When sweets are to be eaten, select a time not during meals.

ANS: B Regular gum has high sugar content. When the child chews gum, the sugar is in prolonged contact with the teeth. Sugarless gum is less cariogenic than regular gum. Raisins, honey, and molasses are highly cariogenic and should be avoided. Sweets should be consumed with meals so that the teeth can be cleaned afterward. This decreases the amount of time that the sugar is in contact with the teeth.

Which statement is descriptive of renal transplantation in children? a. It is an acceptable means of treatment after age 10 years. b. It is the preferred means of renal replacement therapy in children. c. Children can receive kidneys only from other children. d. The decision for transplantation is difficult because a relatively normal lifestyle is not possible.

ANS: B Renal transplant offers the opportunity for a relatively normal life and is the preferred means of renal replacement therapy in end-stage renal disease. Renal transplantation can be done in children as young as age 6 months. Both children and adults can serve as donors for renal transplant purposes. Renal transplantation affords the child a more normal lifestyle than dependence on dialysis.

The nurse has been assigned as a home health nurse for a child who is technology dependent. The nurse recognizes that the familys background differs widely from the nurses own. The nurse believes some of their lifestyle choices are less than ideal. What nursing intervention is most appropriate to institute? a. Change the family. b. Respect the differences. c. Assess why the family is different. d. Determine whether the family is dysfunctional.

ANS: B Respect for varied family structures and for racial, ethnic, cultural, and socioeconomic diversity among families is essential in home care. The nurse must assess and respect the familys background and lifestyle choices. It is not appropriate to attempt to change the family. The nurse is a guest in the home and care of the child. The family and the values held by the cultural group prevail. The nurse may assess why the family is different to help the nurse and other health professionals understand the differences. It is not appropriate to determine whether the family is dysfunctional.

An adolescent girl tells the nurse that she is very suicidal. The nurse asks her if she has a specific plan. How should asking about a specific plan be viewed? a. Not a critical part of the assessment b. An appropriate part of the assessment c. Suggesting that adolescent needs a plan d. Encouraging adolescent to devise a plan

ANS: B Routine health assessments of adolescents should include questions that assess the presence of suicidal ideation or intent. Questions such as Have you ever developed a plan to hurt yourself or kill yourself? should be part of that assessment. Adolescents who express suicidal feelings and have a specific plan are at particular risk and require further assessment and constant monitoring. The information about having a plan is an essential part of the assessment and greatly affects the treatment plan.

What is a significant secondary prevention nursing activity for lead poisoning? a. Chelation therapy b. Screening children for blood lead levels c. Removing lead-based paint from older homes d. Questioning parents about ethnic remedies containing lead

ANS: B Screening children for lead poisoning is an important secondary prevention activity. Screening does not prevent the initial exposure of the child to lead. It can lead to identification and treatment of children who are exposed. Chelation therapy is treatment, not prevention. Removing lead-based paints from older homes before children are affected is primary prevention. Questioning parents about ethnic remedies containing lead is part of the assessment to determine the potential source of lead.

A school nurse is performing hearing screening on school children. The nurse recognizes that distortion of sound and problems in discrimination are characteristic of which type of hearing loss? a. Conductive b. Sensorineural c. Mixed conductive-sensorineural d. Central auditory imperceptive

ANS: B Sensorineural hearing loss, also known as perceptive or nerve deafness, involves damage to the inner ear structures or the auditory nerve. It results in the distortion of sounds and problems in discrimination. Conductive hearing loss involves mainly interference with loudness of sound. Mixed conductive-sensorineural hearing loss manifests as a combination of both sensorineural and conductive loss. Central auditory imperceptive hearing loss includes all hearing losses that do not demonstrate defects in the conduction or sensory structures.

A child has been admitted to the hospital with a blood lead level of 72 mcg/dL. What treatment should the nurse anticipate? a. Referral to social services b. Initiation of chelation therapy c. Follow-up testing within 1 month d. Aggressive environmental intervention

ANS: B Severe lead toxicity (lead level ?5=70 mcg/dL) requires immediate inpatient chelation treatment. Referral to social service and follow-up in 1 month are prescribed for lead levels of 15 to 19 mcg/dL. Aggressive environmental intervention would be initiated after chelation treatments.

A 3-month-old infant dies shortly after arrival to the emergency department. The infant has subdural and retinal hemorrhages but no external signs of trauma. What should the nurse suspect? a. Unintentional injury b. Shaken baby syndrome c. Congenital neurologic problem d. Sudden infant death syndrome (SIDS)

ANS: B Shaken baby syndrome causes internal bleeding but may have no external signs. Unintentional injury would not cause these injuries. With unintentional injuries, external signs are usually present. Congenital neurologic problems would usually have signs of abnormal neurologic anatomy. SIDS does not usually have identifiable injuries.

A 3-month-old infant dies shortly after arrival to the emergency department. The infant has subdural and retinal hemorrhages but no external signs of trauma. What should the nurse suspect? a. Unintentional injury b. Shaken-baby syndrome c. Sudden infant death syndrome (SIDS) d. Congenital neurologic problem

ANS: B Shaken-baby syndrome causes internal bleeding but may have no external signs. Unintentional injury would not cause these injuries. SIDS and congenital neurologic problems would not appear this way.

Amy, age 6 years, needs to be hospitalized again because of a chronic illness. The clinic nurse overhears her school-age siblings tell her, "We are sick of Mom always sitting with you in the hospital and playing with you. It isn't fair that you get everything and we have to stay with the neighbors." Which is the nurse's best assessment of this situation? a. The siblings are immature and probably spoiled. b. Jealousy and resentment are common reactions to the illness or hospitalization of a sibling. c. Family has ineffective coping mechanisms to deal with chronic illness. d. The siblings need to better understand their sister's illness and needs.

ANS: B Siblings experience loneliness, fear, and worry, as well as anger, resentment, jealousy, and guilt. The siblings experience stress equal to that of the hospitalized child. There is no evidence that the family has maladaptive coping mechanisms.

A school-age child has begun to sleepwalk. What does the nurse advise the parents to perform? a. Wake the child and help determine what is wrong. b. Leave the child alone unless he or she is in danger of harming him- or herself or others. c. Arrange for psychologic evaluation to identify the cause of stress. d. Keep the child awake later in the evening to ensure sufficient tiredness for a full night of sleep.

ANS: B Sleepwalking is usually self-limiting and requires no treatment. The child usually moves about restlessly and then returns to bed. Usually the actions are repetitive and clumsy. The child should not be awakened unless in danger. If there is a need to awaken the child, it should be done by calling the childs name to gradually bring to a state of alertness. Some children, who are usually well behaved and tend to repress feelings, may sleepwalk because of strong emotions. These children usually respond to relaxation techniques before bedtime. If a child is overly fatigued, sleepwalking can increase

What is smokeless tobacco? a. Not addicting b. Proven to be carcinogenic c. Easy to stop using d. A safe alternative to cigarette smoking

ANS: B Smokeless tobacco is a popular substitute for cigarettes and poses serious health hazards to children and adolescents. Smokeless tobacco is associated with cancer of the mouth and jaw. The nicotine in the smokeless tobacco is addicting, and therefore it is very difficult to quit. Because the product is addicting and can cause cancer, it is not a safe alternative to cigarette smoking.

What is the most significant factor in distinguishing those who commit suicide from those who make suicidal attempts or threats? a. Level of stress b. Social isolation c. Degree of depression d. Desire to punish others

ANS: B Social isolation is a significant factor in distinguishing adolescents who will kill themselves from those who will not. It is also more characteristic of those who complete suicide versus those who make attempts or threats. Although the level of stress, the degree of depression, and the desire to punish others are contributing factors in suicide, they are not the most significant factor in distinguishing those who complete suicide from those who attempt suicide.

A nurse is teaching parents about language development for preschool children. Which dysfunctional speech pattern is a normal characteristic the parents might expect? a. Lisp b. Stammering c. Echolalia d. Repetition without meaning

ANS: B Stammering and stuttering are normal dysfluency patterns in preschool-age children. Lisps are not a normal characteristic of language development. Echolalia and repetition are traits of toddlers' language.

An adolescent tells the school nurse that she is pregnant. Her last menstrual period was 4 months ago. She has not received any medical care. She smokes but denies any other substance use. What is the priority nursing action? a. Notify her parents b. Refer for prenatal care c. Explain the importance of not smoking d. Discuss dietary needs for adequate fetal growth

ANS: B Teenage girls and their unborn children are at greater risk for complications during pregnancy and delivery. With improved therapies, the mortality for teenage pregnancy is decreasing, but the morbidity is high. A pregnant teenager needs careful assessment by the nurse to determine the level of social support available to her and possibly her partner. Guidance from the adults in her life would be invaluable, but confidentiality should be maintained. Although it is important to explain the importance of not smoking and to discuss dietary needs for adequate fetal growth, because of her potential for having a high-risk pregnancy, she will need a comprehensive prenatal program to minimize maternal-fetal complications.

What is a major physical risk for young adolescents during pregnancy? a. Osteoporosis frequently develops. b. Fetopelvic disproportion is a common problem. c. Delivery is usually precipitous in this age group. d. Pregnancy will adversely affect the adolescents development.

ANS: B Teenagers younger than 15 years of age have increased obstetric risks. Fetopelvic disproportion is one of the most common complications. Osteoporosis occurs later in life and is not related to adolescent pregnancy. Prolonged, not precipitous, labor is common in this age group. Teenage mothers are socially, educationally, psychologically, and economically disadvantaged. Support is necessary because the tasks of motherhood are superimposed on adolescent development tasks

The American Academy of Pediatrics (AAP) recommends that children younger than the age of 16 years be prohibited from participating in what? a. Skateboarding b. Snowmobiling c. Trampoline use d. Horseback riding

ANS: B The AAP views the use of snowmobiles and all-terrain vehicles as major health hazards for children. This group opposes the use of these vehicles by children younger than 16 years of age. The AAP recommends that children younger than the age of 10 years not use skateboards without parental supervision. Protective gear is always suggested. Trampoline use has increased along with injuries. Adults should supervise use. Horseback riding injuries are also a source of concern. Parents should determine the instructors safety record with students.

Several types of long-term central venous access devices are used. Which is considered an advantage of a Hickman-Broviac catheter? a. No need to keep exit site dry b. Easy to use for self-administered infusions c. Heparinized only monthly and after each infusion d. No limitations on regular physical activity, including swimming

ANS: B The Hickman-Broviac catheter has several benefits, including that it is easy to use for self-administered infusions. The exit site must be kept dry to decrease risk of infection. The Hickman-Broviac catheter requires daily heparin flushes. Water sports may be restricted because of risk of infection.

A child is receiving total parenteral nutrition (TPN; hyperalimentation). At the end of 8 hours, the nurse observes the solution and notes that 200 ml/8 hr is being infused rather than the ordered amount of 300 ml/8 hr. The nurse should adjust the rate so that how much will infuse during the next 8 hours? a. 200 ml b. 300 ml c. 350 ml d. 400 ml

ANS: B The TPN infusion rate should not be increased or decreased without the practitioner being informed because alterations in rate can cause hyperglycemia or hypoglycemia. Any changes from the prescribed flow rate may lead to hyperglycemia or hypoglycemia.

The nurse should suspect a hearing impairment in an infant who demonstrates which behavior? a. Absence of the Moro reflex b. Absence of babbling by age 7 months c. Lack of eye contact when being spoken to d. Lack of gesturing to indicate wants after age 15 months

ANS: B The absence of babbling or inflections in voice by age 7 months is an indication of hearing difficulties. The absence of the Moro reflex and eye contact when being spoken to does not indicate a hearing impairment. The child with hearing impairment uses gestures rather than vocalizations to express desires at this age

What are appropriate interventions to facilitate socialization of the cognitively impaired child? a. Provide age-appropriate toys and play activities. b. Provide peer experiences, such as scouting, when older. c. Avoid exposure to strangers who may not understand cognitive development. d. Emphasize mastery of physical skills because they are delayed more often than verbal skills

ANS: B The acquisition of social skills is a complex task. Children of all ages need peer relationships. Parents should enroll the child in preschool. When older, they should have peer experiences similar to those of other children such as group outings, Boy and Girl Scouts, and Special Olympics. It is important to provide age-appropriate toys and play activities, but peer interactions will facilitate social development. Parents should expose the child to strangers so that the child can practice social skills. Verbal skills are delayed more than physical skills.

What is true concerning masturbation during adolescence? a. Homosexuality is encouraged by the practice of masturbation. b. Many girls do not begin masturbation until after they have intercourse. c. Masturbation at an early age leads to sexual intercourse at an earlier age. d. Development of intimate relationships is delayed when masturbation is regularly practiced.

ANS: B The age of first masturbation for girls is variable. Some begin masturbating in early adolescence; many do not begin until after they have had intercourse. Boys typically begin masturbation in early adolescence. Masturbation provides an opportunity for self-exploration. Both heterosexual and homosexual youth use masturbation. It does not affect the development of intimacy.

Which is an important nursing consideration when performing a bladder catheterization on a young boy? a. Clean technique, not standard precautions, is needed. b. Insert 2% lidocaine lubricant into the urethra. c. Lubricate catheter with water-soluble lubricant such as K-Y Jelly. d. Delay catheterization for 20 minutes while anesthetic lubricant is absorbed.

ANS: B The anxiety, fear, and discomfort experienced during catheterization can be significantly decreased by preparation of the child and parents, by selection of the correct catheter, and by appropriate technique of insertion. Generous lubrication of the urethra before catheterization and use of lubricant containing 2% lidocaine may reduce or eliminate the burning and discomfort associated with this procedure. Catheterization is a sterile procedure, and standard precautions for body-substance protection should be followed. Water-soluble lubricants do not provide appropriate local anesthesia. Catheterization should be delayed 2 to 3 minutes only. This provides sufficient local anesthesia for the procedure.

The nurse is explaining average weight gain during the preschool years to a group of parents. Which average weight gain should the nurse suggest to the parents? a. 1 to 2 kg b. 2 to 3 kg c. 3 to 4 kg d. 4 to 5 kg

ANS: B The average weight gain remains approximately 2 to 3 kg (4.56.5 lb) per year during the preschool period

The development of sexual orientation during adolescence is what? a. Inflexible b. A developmental process c. Differs for boys and girls d. Proceeds in a defined sequence

ANS: B The development of sexual orientation as a part of sexual identity includes several developmental milestones during late childhood and throughout adolescence. The sequence and time spent in phases are different for each individual. Boys and girls pass through the same developmental milestones.

A child has had contact with some poison ivy. The school nurse understands that the fullblown reaction should be evident after how many days? a. 1 day b. 2 days c. 3 days d. 4 days

ANS: B The full-blown reaction to poison ivy is evident after about 2 days, with linear patches or streaks of erythemic, raised, fluid-filled vesicles; swelling; and persistent itching at the site of contact.

The nurse is conducting teaching for an adolescent being discharged to home after a renal transplant. The adolescent needs further teaching if which statement is made? a. "I will report any fever to my primary health care provider." b. "I am glad I only have to take the immunosuppressant medication for two weeks." c. "I will observe my incision for any redness or swelling." d. "I won't miss doing kidney dialysis every week."

ANS: B The immunosuppressant medications are taken indefinitely after a renal transplant, so they should not be discontinued after two weeks. Reporting a fever and observing an incision for redness and swelling are accurate statements. The adolescent is correct in indicating dialysis will not need to be done after the transplant.

A school-age child has been bitten on the leg by a large snake that may be poisonous. During transport to an emergency facility, what should the care include? a. Apply ice to the snakebite. b. Immobilize the leg with a splint. c. Place a loose tourniquet distal to the bite. d. Apply warm compresses to the snakebite.

ANS: B The leg should be immobilized. Ice decreases blood flow to the area, which allows the venom to work more destruction and decreases the effect of antivenin on the natural immune mechanisms. A loose tourniquet is placed proximal, not distal, to the area of the bite to delay the flow of lymph. This can delay movement of the venom into the peripheral circulation. The tourniquet should be applied so that a pulse can be felt distal to the bite. Warmth increases circulation to the area and helps the toxin into the peripheral circulation.

What is an important consideration for the school nurse who is planning a class on bicycle safety? a. Most bicycle injuries involve collision with an automobile. b. Head injuries are the major causes of bicycle-related fatalities. c. Children should wear a bicycle helmet if they ride on paved streets. d. Children should not ride double unless the bicycle has an extra large seat

ANS: B The most important aspect of bicycle safety is to encourage the rider to use a protective helmet. Head injuries are the major cause of bicycle-related fatalities. Although motor vehicle collisions do cause injuries to bicyclists, most injuries result from falls. The child should always wear a properly fitted helmet approved by the U.S. Consumer Product Safety Commission. Children should not ride double unless it is a tandem bike (built for two).

A venipuncture will be performed on a 7-year-old girl. She wants her mother to hold her during the procedure. What information should the nurse include in her response to the child? a. It is unsafe. b. It is helpful to relax the child. c. It is against hospital policy. d. It is unnecessary because of child's age.

ANS: B The mother's preference for assisting, observing, or waiting outside the room should be assessed along with the child's preference for parental presence. The child's choice should be respected. This will most likely help the child through the procedure. If the mother and child agree, then the mother is welcome to stay. Her familiarity with the procedure should be assessed and potential safety risks identified (mother may sit in chair). Hospital policies should be reviewed to ensure that they incorporate family-centered care. The child should determine whether parental support is necessary.

Guidelines for intramuscular administration of medication in school-age children include which action? a. Inject medication as rapidly as possible. b. Insert needle quickly, using a dart like motion. c. Penetrate skin immediately after cleansing site, before skin has dried. d. Have child stand, if possible, and if child is cooperative.

ANS: B The needle should be inserted quickly in a dart like motion at a 90-degree angle unless contraindicated. Inject medications slowly. Allow skin preparation to dry completely before skin is penetrated. Place child in lying or sitting position.

Guidelines for intramuscular administration of medication in school-age children include what standard? a. Inject medication as rapidly as possible. b. Insert needle quickly, using a dartlike motion. c. Have the child stand if at all possible and if the child is cooperative. d. Penetrate the skin immediately after cleansing the site while the skin is moist.

ANS: B The needle should be inserted quickly in a dartlike motion at a 90-degree angle unless contraindicated. Inject medications slowly. Allow skin preparation to dry completely before the skin is penetrated. Place the child in a lying or sitting position.

A nurse in the emergency department is assessing a 5-year-old child with symptoms of pneumonia and a fever of 102° F. Which intervention can the nurse implement to promote a sense of control for the child? a. None; this is an emergency and the child should not participate in care. b. Allow the child to hold the digital thermometer while taking the child's blood pressure. c. Ask the child if it is OK to take a temperature in the ear. d. Have parents wait in the waiting room.

ANS: B The nurse should allow the child to hold the digital thermometer while taking the child's blood pressure. Unless an emergency is life threatening, children need to participate in their care to maintain a sense of control. Because emergency departments are frequently hectic, there is a tendency to rush through procedures to save time. However, the extra few minutes needed to allow children to participate may save many more minutes of useless resistance and uncooperativeness during subsequent procedures. The child may not give permission, if asked, for a procedure that is necessary to be performed. It is better to give choices such as, "Which ear do you want me to do your temperature in?" instead of, "Can I take your temperature?" Parents should remain with their child to help with decreasing the child's anxiety.

The school nurse is caring for a child with a penetrating eye injury. Emergency treatment includes which intervention? a. Apply a regular eye patch. b. Apply a Fox shield to affected eye and any type of patch to the other eye. c. Apply ice until the physician is seen. d. Irrigate eye copiously with a sterile saline solution.

ANS: B The nurse's role in a penetrating eye injury is to prevent further injury to the eye. A Fox shield (if available) should be applied to the injured eye and a regular eye patch to the other eye to prevent bilateral movement. It may cause more damage to the eye to apply a regular eye patch, apply ice until the physician is seen, or irrigate the eye copiously with a sterile saline solution.

Which is an objective of care for a 10-year-old child with minimal change nephrotic syndrome? a. Reduce blood pressure. b. Reduce excretion of urinary protein. c. Increase excretion of urinary protein. d. Increase ability of tissues to retain fluid.

ANS: B The objectives of therapy for the child with minimal change nephrotic syndrome include reduction of the excretion of urinary protein, reduction of fluid retention, prevention of infection, and minimization of complications associated with therapy. Blood pressure is usually not elevated in minimal change nephrotic syndrome. Excretion of urinary protein and fluid retention are part of the disease process and must be reversed.

The nurse is assessing the coping behaviors of the parents of a child recently diagnosed with a chronic illness. What behavior should the nurse consider an approach behavior that results in movement toward adjustment? a. Being unable to adjust to a progression of the disease or condition b. Anticipating future problems and seeking guidance and answers c. Looking for new cures without a perspective toward possible benefit d. Failing to recognize the seriousness of the childs condition despite physical evidence

ANS: B The parents who anticipate future problems and seek guidance and answers are demonstrating approach behaviors. These are positive actions in caring for their child. Being unable to adjust, looking for new cures, and failing to recognize the seriousness of the childs condition are avoidance behaviors. The parents are moving away from adjustment or exhibiting maladaptation to the crisis of a child with chronic illness or disability

Approach behaviors are those coping mechanisms that result in a family's movement toward adjustment and resolution of the crisis of having a child with a chronic illness or disability. Which is considered an approach behavior? a. Is unable to adjust to a progression of the disease or condition b. Anticipates future problems and seeks guidance and answers c. Looks for new cures without a perspective toward possible benefit d. Fails to recognize the seriousness of the child's condition despite physical evidence

ANS: B The parents who anticipate future problems and seek guidance and answers are demonstrating approach behaviors. They are demonstrating positive actions in caring for their child. Being unable to adjust to a progression of the disease or condition, looking for new cures without a perspective toward possible benefit, and failing to recognize the seriousness of a child's condition despite physical evidence are avoidance behaviors. The parents are moving away from adjustment (and toward maladaptation) in the crisis of a child with chronic illness or disability.

What do nursing responsibilities regarding weight gain for an adolescent with anorexia nervosa include? a. Administer tube feedings until target weight is achieved. b. Restore body weight to within 10% of the adolescents ideal weight. c. Encourage continuation of strenuous exercise as long as adolescent is not losing weight. d. Facilitate as rapid a weight gain as possible with a high-calorie diet.

ANS: B The restoration of body weight to a target weight or endpoint within 10% of ideal body weight is one of the main goals of therapy. Strenuous exercise is avoided as part of the need to modify behaviors. Tube feedings are intrusive and are avoided. They should only be used when other measures have failed. Weight restoration is accomplished slowly. The goal is 1 kg/wk to avoid the risk of metabolic and cardiac problems. Slow weight gain can minimize anxiety and depression.

The nurse is teaching parents about prevention of urinary tract infections in children. Which factor predisposes the urinary tract to infection? a. Increased fluid intake b. Short urethra in young girls c. Prostatic secretions in males d. Frequent emptying of the bladder

ANS: B The short urethra in females provides a ready pathway for invasion of organisms. Increased fluid intake and frequent emptying of the bladder offer protective measures against urinary tract infections. Prostatic secretions have antibacterial properties that inhibit bacteria.

What statement is true about gonorrhea? a. It is caused by Treponema pallidum. b. Treatment of all sexual contacts is essential. c. Topical application of medication to the lesions is necessary. d. Therapeutic management includes multidose administration of penicillin.

ANS: B The treatment plan should include finding and treating all sexual partners. Gonorrhea is caused by Neisseria gonorrhoeae. Syphilis is caused by T. pallidum. Systemic therapy is necessary to treat this disease. Primary treatment is with different antibiotics because of N. gonorrhoeaes resistance to penicillin.

The nurse is teaching a class on obesity prevention to parents in the community. What is a contributing factor to childhood obesity? a. Birth weight b. Parental overweight c. Age at the onset of puberty d. Asian ethnic background

ANS: B There is a high correlation between parental adiposity and childhood adiposity. Obese children do not have higher birth weights than nonobese children. Early menarche is associated with obesity, but the age of puberty is not a contributing factor. African Americans and Hispanics have disproportionately high percentages of overweight individuals, but Asians do not.

A newborn assessment shows separated sagittal suture, oblique palpebral fissures, depressed nasal bridge, protruding tongue, and transverse palmar creases. Of what are these findings most suggestive? a. Microcephaly b. Down syndrome c. Cerebral palsy d. Fragile X syndrome

ANS: B These are characteristics associated with Down syndrome. The infant with microcephaly has a small head. Cerebral palsy is a diagnosis not usually made at birth. No characteristic physical signs are present. The infant with fragile X syndrome has increased head circumference; long, wide, and/or protruding ears; long, narrow face with prominent jaw; hypotonia; and high arched palate.

15.A child age 4 1/2 years sometimes wakes her parents up at night screaming, thrashing, sweating, and apparently frightened, yet she is not aware of her parents presence when they check on her. She lies down and sleeps without any parental intervention. This is most likely what? a. Nightmare b. Sleep terror c. Sleep apnea d. Seizure activity

ANS: B This is a description of a sleep terror. The child is observed during the episode and not disturbed unless there is a possibility of injury. A child who awakes from a nightmare is distressed. She is aware of and reassured by the parents presence. This is not the case with sleep apnea. This behavior is not indicative of seizure activity.

In terms of fine motor development, what should the 3-year-old child be expected to do? a. Tie shoelaces. b. Copy (draw) a circle. c. Use scissors or a pencil very well. d. Draw a person with seven to nine parts.

ANS: B Three-year-old children are able to accomplish the fine motor skill of copying (drawing) a circle. The ability to tie shoelaces, to use scissors or a pencil very well, and to draw a person with seven to nine parts are fine motor skills of 5-year-old children

Parents tell the nurse they found their 3-year-old daughter and a male cousin of the same age inspecting each other closely as they used the bathroom. What is the most appropriate recommendation for the nurse to make? a. Punish the children so this behavior stops. b. Neither condone nor condemn the curiosity. c. Get counseling for this unusual and dangerous behavior. d. Allow the children unrestricted permission to satisfy this curiosity.

ANS: B Three-year-old children become aware of anatomic differences and are concerned about how the other sex works. Such exploration should not be condoned or condemned. Children should not be punished for this normal exploration. This is age appropriate and not dangerous behavior. Encouraging the children to ask their parents questions and redirecting their activity is more appropriate than giving permission.

Parents tell the nurse that they found their 3-year-old daughter and a male cousin of the same age inspecting each other closely as they used the bathroom. Which is the most appropriate recommendation the nurse should make? a. Punish children so this behavior stops. b. Neither condone nor condemn the curiosity. c. Allow children unrestricted permission to satisfy this curiosity. d. Get counseling for this unusual and dangerous behavior.

ANS: B Three-year-olds become aware of anatomic differences and are concerned about how the other "works." Such exploration should not be condoned or condemned. Children should not be punished for this normal exploration. Encouraging the children to ask questions of the parents and redirecting their activity are more appropriate than giving permission. Exploration is age-appropriate and not dangerous behavior.

The nurse understands that which guideline should be followed to determine serving sizes for toddlers? a. 1/2 tbsp of solid food per year of age b. 1 tbsp of solid food per year of age c. 2 tbsp of solid food per year of age d. 2 1/2 tbsp of solid food per year of age

ANS: B To determine serving sizes for young children, the guideline to follow is 1 tbsp of solid food per year of age. One-half tbsp per year of age would not be adequate. Two or 2 1/2 tbsp per year of age would be excessive.

One of the clinical manifestations of chronic renal failure is uremic frost. Which best describes this term? a. Deposits of urea crystals in urine b. Deposits of urea crystals on skin c. Overexcretion of blood urea nitrogen d. Inability of body to tolerate cold temperatures

ANS: B Uremic frost is the deposition of urea crystals on the skin. The urea crystals are present on the skin, not in the urine. The kidneys are unable to excrete blood urea nitrogen, leading to elevated levels. There is no relation between cold temperatures and uremic frost.

A child is admitted with acute glomerulonephritis. What should the nurse expect the urinalysis to show during the acute phase? a. Bacteriuria, hematuria b. Hematuria, proteinuria c. Bacteriuria, increased specific gravity d. Proteinuria, decreased specific gravity

ANS: B Urinalysis during the acute phase characteristically shows hematuria and proteinuria. Bacteriuria and changes in specific gravity are not usually present during the acute phase.

1. A child is admitted with acute glomerulonephritis. What should the nurse expect the urinalysis during this acute phase to show? a. Bacteriuria and hematuria b. Hematuria and proteinuria c. Bacteriuria and increased specific gravity d. Proteinuria and decreased specific gravity

ANS: B Urinalysis during the acute phase characteristically shows hematuria, proteinuria, and increased specific gravity. Proteinuria generally parallels the hematuria but is not usually the massive proteinuria seen in nephrotic syndrome. Gross discoloration of urine reflects its red blood cell and hemoglobin content. Microscopic examination of the sediment shows many red blood cells, leukocytes, epithelial cells, and granular and red blood cell casts. Bacteria are not seen, and urine culture results are negative.

A father calls the clinic because he found his young daughter squirting Visine eyedrops into her mouth. What is the most appropriate nursing action? a. Reassure the father that Visine is harmless. b. Direct him to seek immediate medical treatment. c. Recommend inducing vomiting with ipecac. d. Advise him to dilute Visine by giving his daughter several glasses of water to drink.

ANS: B Visine is a sympathomimetic and if ingested may cause serious consequences. Medical treatment is necessary. Inducing vomiting is no longer recommended for ingestions. Dilution will not decrease risk.

The nurse is assessing a toddlers visual acuity. Which visual acuity is considered acceptable during the toddler years? a. 20/20 b. 20/40 c. 20/50 d. 20/60

ANS: B Visual acuity of 20/40 is considered acceptable during the toddler years.

Which statement is correct about toilet training? a. Bladder training is usually accomplished before bowel training. b. Wanting to please the parent helps motivate the child to use the toilet. c. Watching older siblings use the toilet confuses the child. d. Children must be forced to sit on the toilet when first learning.

ANS: B Voluntary control of the anal and urethral sphincters is achieved sometime after the child is walking. The child must be able to recognize the urge to let go and to hold on. The child must want to please the parent by holding on rather than pleasing him- or herself by letting go. Bowel training precedes bladder training. Watching older siblings provides role modeling and facilitates imitation for the toddler. The child should be introduced to the potty chair or toilet in a nonthreatening manner.

Which statement is correct about toilet training? a. Bladder training is usually accomplished before bowel training. b. Wanting to please the parent helps motivate the child to use the toilet. c. Watching older siblings use the toilet confuses the child. d. Children must be forced to sit on the toilet when first learning.

ANS: B Voluntary control of the anal and urethral sphincters is achieved sometime after the child is walking. The child must be able to recognize the urge to let go and to hold on. The child must want to please parent by holding on rather than pleasing self by letting go. Bowel training precedes bladder training. Watching older siblings provides role modeling and facilitates imitation for the toddler. The child should be introduced to the potty chair or toilet in a nonthreatening manner.

When teaching injury prevention during the school-age years, what should the nurse include? a. Teach children about the need to fear strangers. b. Teach basic rules of water safety. c. Avoid letting children cook in microwave ovens. d. Caution children against engaging in competitive sports.

ANS: B Water safety instruction is an important component of injury prevention at this age. The child should be taught to swim, select safe and supervised places to swim, swim with a companion, check sufficient water depth for diving, and use an approved flotation device. Teach stranger safety, not fear of strangers. This includes telling the child not to go with strangers, not to wear personalized clothing in public places, to tell parents if anyone makes child feel uncomfortable, and to say no in uncomfortable situations. Teach the child safe cooking. Caution against engaging in dangerous sports such as jumping on trampolines.

When teaching injury prevention during the school-age years, what should the nurse include? a. Teach children to fear strangers. b. Teach basic rules of water safety. c. Avoid letting child cook in microwave ovens. d. Caution child against engaging in competitive sports.

ANS: B Water safety instruction is an important source of injury prevention at this age. The child should be taught to swim, select safe and supervised places to swim, swim with a companion, check for sufficient water depth before diving, and use an approved flotation device. Teach stranger safety, not fear of strangers. This includes instructing children to not go with strangers, not wear personalized clothing in public places, tell parents if anyone makes child feel uncomfortable, and say "no" in uncomfortable situations. Teach child safe cooking. Caution against engaging in hazardous sports such as those involving trampolines.

What are core principles of patient- and family-centered care? (Select all that apply.) a. Collaboration b. Empowering families c. Providing formal and informal support d. Maintaining strict policy and procedure routines e. Withholding information that is likely to cause anxiety

ANS: B, C Core principles of patent- and family-centered care include collaboration, empowerment, andprovidingformalandinformalsupport.Thereshouldbeflexibilityinpolicyandprocedures,andcommunicationshouldbecomplete,honest,andunbiased,notwithheld.

What nursing interventions should the nurse plan for a hospitalized toddler to minimize fear of bodily injury? (Select all that apply.) a. Perform procedures slowly. b. Maintain parentchild contact. c. Use progressively smaller dressings on surgical incisions. d. Tell the child bleeding will stop after the needle is removed. e. Remove a dressing as quickly as possible from surgical incisions.

ANS: B, C Whenever procedures are performed on young children, the most supportive intervention to minimize the fear of bodily injury is to do the procedure as quickly as possible while maintaining parentchild contact. Because of toddlers and preschool childrens poorly defined body boundaries, the use of bandages may be particularly helpful. For example, telling children that the bleeding will stop after the needle is removed does little to relieve their fears, but applying a small Band-Aid usually reassures them. The size of bandages is also significant to children in this age group; the larger the bandage, the more importance is attached to the wound. Watching their surgical dressings become successively smaller is one way young children can measure healing and improvement. Prematurely removing a dressing may cause these children considerable concern for their well-being.

Which are characteristic of physical development of a 30-month-old child? (Select all that apply.) a. Birth weight has doubled. b. Primary dentition is complete. c. Sphincter control is achieved. d. Anterior fontanel is open. e. Length from birth is doubled. f. Left- or right-handedness is established.

ANS: B, C Usually by age 30 months, the primary dentition of 20 teeth is completed, and the child has sphincter control in preparation for bowel and bladder control. Birth weight doubles at approximately ages 5 to 6 months. The anterior fontanel closes at age 12 to 18 months. Birth length is doubled around age 4. Left- or right-handedness is not established until about age 5.

1. The nurse is admitting a 9-year-old child with hemolytic uremic syndrome. What clinical manifestations should the nurse expect to observe? (Select all that apply.) a. Hematuria b. Anorexia c. Hypertension d. Purpura e. Proteinuria f. Periorbital edema

ANS: B, C, D Clinical manifestations of hemolytic uremic syndrome include anorexia; hypertension; and purpura, which persists for several days to 2 weeks. Gross hematuria is seen in acute glomerulonephritis. Substantial proteinuria and periorbital edema are common manifestations in nephrotic syndrome.

1. What clinical manifestations should be observed in a 2-year-old child with hypotonic dehydration? (Select all that apply.) a. Thick, doughy feel to the skin b. Slightly moist mucous membranes c. Absent tears d. Very rapid pulse e. Hyperirritability

ANS: B, C, D Clinical manifestations of hypotonic dehydration include slightly moist mucous membranes, absent tears, and a very rapid pulse. A thick, doughy feel to the skin and hyperirritability are signs of hypertonic dehydration.

The nurse is assessing a familys use of complementary medicine practices. What practices are classified as nutrition, diet, and lifestyle or behavioral health changes? (Select all that apply.) a. Reflexology b. Macrobiotics c. Megavitamins d. Health risk reduction e. Chiropractic medicine

ANS: B, C, D Macrobiotics, megavitamins, and health risk reduction are classified as nutrition, diet, and lifestyle or behavioral health changes. Reflexology and chiropractic medicine are classified as structural manipulation and energetic therapies.

1. A child is hospitalized in acute renal failure and has a serum potassium greater than 7 mEq/L. What temporary measures that will produce a rapid but transient effect to reduce the potassium should the nurse expect to be prescribed? (Select all that apply.) a. Dialysis b. Calcium gluconate c. Sodium bicarbonate d. Glucose 50% and insulin e. Sodium polystyrene sulfonate (Kayexalate)

ANS: B, C, D Several measures are available to reduce the serum potassium concentration, and the priority of implementation is usually based on the rapidity with which the measures are effective. Temporary measures that produce a rapid but transient effect are calcium gluconate, sodium bicarbonate, and glucose 50%, and insulin. Definitive but slower-acting measures are then implemented which include administration of a cation exchange resin such as sodium polystyrene sulfonate (Kayexalate), 1 g/kg, administered orally or rectally, and/or dialysis.

A toddler is in the sensorimotor, tertiary circular reactions stage of cognitive development. What behavior should the nurse expect to assess? (Select all that apply.) a. Refers to self by pronoun b. Gestures up and down c. Able to insert round object into a hole d. Can find hidden objects but only in the first location e. Uses future-oriented words, such as tomorrow

ANS: B, C, D Children in the sensorimotor, tertiary circular reactions stage of cognitive development show the behaviors of gesturing up and down, have the ability to insert round objects into a hole, and can find hidden objects but only in the first location. The behaviors of referring to oneself by pronoun and using future-oriented words such as tomorrow are seen in the preoperational stage of cognitive development.

What are classified as hydrocarbon poisons? (Select all that apply.) a. Bleach b. Gasoline c. Turpentine d. Lighter fluid e. Oven cleaners

ANS: B, C, D Gasoline, turpentine, and lighter fluid are classified as hydrocarbon poisons. Bleach and oven cleaners are classified as corrosive poisons

The nurse is caring for a child with erythema multiforme (Stevens-Johnson syndrome). What local manifestations does the nurse expect to assess in this child? (Select all that apply.) a. Papular urticaria b. Erythematous papular rash c. Lesions absent in the scalp d. Lesions enlarge by peripheral expansion e. Firm papules that may be capped by vesicles

ANS: B, C, D Local manifestations of erythema multiforme include an erythematous popular rash, lesions involving most skin surfaces except the scalp and lesions that enlarge by peripheral expansion. Papular urticaria and firm papules capped by vesicles are characteristics of an insect bite.

A nurse plans therapeutic play time for a hospitalized child. What are the benefits of play? (Select all that apply.) a. Serves as method to assist disturbed children b. Allows the child to express feelings c. The nurse can gain insight into the child's feelings. d. The child can deal with concerns and feelings. e. Gives the child a structured play environment

ANS: B, C, D Play is an effective, nondirective modality for helping children deal with their concerns and fears, and at the same time, it often helps the nurse gain insights into children's needs and feelings. Play and other expressive activities provide one of the best opportunities for encouraging emotional expression, including the safe release of anger and hostility. Nondirective play that allows children freedom for expression can be tremendously therapeutic. Play therapy is a structured therapy that helps disturbed children. It should not be confused with therapeutic play.

A parent asks the nurse, When will I know my child is ready for toilet training? The nurse should include what in the response? (Select all that apply.) a. The child should be able to stay dry for 1 hour. b. The child should be able to sit, walk, and squat. c. The child should have regular bowel movements. d. The child should express a willingness to please.

ANS: B, C, D Signs of toilet training readiness include physical and psychological readiness. The ability to sit, walk, and squat and having regular bowel movements are physical readiness signs. Expressing a willingness to please is a sign of psychological readiness. The child should be able to stay dry for 2 hours, not 1.

A nurse teaches parents that team play is important for school-age children. Which can children develop by experiencing team play? (Select all that apply.) a. Achieve personal goals over group goals. b. Learn complex rules. c. Experience competition. d. Learn about division of labor.

ANS: B, C, D Team play helps stimulate cognitive growth because children are called on to learn many complex rules, make judgments about those rules, plan strategies, and assess the strengths and weaknesses of members of their own team and members of the opposing team. Team play can also contribute to children's social, intellectual, and skill growth. Children work hard to develop the skills needed to become team members, to improve their contribution to the group, and to anticipate the consequences of their behavior for the group. Team play teaches children to modify or exchange personal goals for goals of the group; it also teaches them that division of labor is an effective strategy for attaining a goal.

1. Parents of a child who will need hemodialysis ask the nurse, What are the advantages of a fistula over a graft or external access device for hemodialysis? What response should the nurse give? (Select all that apply.) a. It is ready to be used immediately. b. There are fewer complications with a fistula. c. There is less restriction of activity with a fistula. d. It produces dilation and thickening of the superficial vessels. e. The fistula does not require a needle insertion at each dialysis.

ANS: B, C, D The creation of a subcutaneous (internal) arteriovenous fistula by anastomosing a segment of the radial artery and brachiocephalic vein produces dilation and thickening of the superficial vessels of the forearm to provide easy access for repeated venipuncture. Fewer complications and less restriction of activity are observed with the use of a fistula. Both the graft and the fistula require needle insertion at each dialysis. The fistula cannot be used immediately.

A nurse is interviewing the parents of a toddler about use of complementary or alternative medical practices. The parents share several practices they use in their household. Which should the nurse document as complementary or alternative medical practices? (Select all that apply.) a. Use of acetaminophen (Tylenol) for fever b. Administration of chamomile tea at bedtime c. Hypnotherapy for relief of pain d. Acupressure to relieve headaches e. Cool mist vaporizer at the bedside for "stuffiness"

ANS: B, C, D When conducting an assessment, the nurse should inquire about the use of complementary or alternative medical practices. Administration of chamomile tea at bedtime, hypnotherapy for relief of pain, and acupressure to relieve headaches are complementary or alternative medical practices. Using Tylenol for fever relief and a cool mist vaporizer at the bedside to reduce "stuffiness" are not considered complementary or alternative medical practices.

The nurse is planning to use an interpreter with a nonEnglish-speaking family. What should the nurse plan with regard to the use of an interpreter? (Select all that apply.) a. Use a family member. b. The nurse should speak slowly. c. Use an interpreter familiar with the familys culture. d. The nurse should speak only a few sentences at a time. e. The nurse should speak to the interpreter during interactions.

ANS: B, C, D When parents who do not speak English are informed of their childs chronic illness, interpreters familiar with both their culture and language should be used. The nurse should speak slowly and only use a few sentences at a time. Children, family members, and friends of the family should not be used as translators because their presence may prevent parents from openly discussing the issues. The nurse should speak to the family, not the interpreter.

1. What influences a childs reaction to the stressors of hospitalization? (Select all that apply.) a. Gender b. Separation c. Support systems d. Developmental age e. Previous experience with illness

ANS: B, C, D, E Major stressors of hospitalization include separation, loss of control, bodily injury, and pain. Childrens reactions to these crises are influenced by their developmental age; previous experience with illness, separation, or hospitalization; innate and acquired coping skills; seriousness of the diagnosis; and support systems available. Gender does not have an effect on a childs reaction to stressors of hospitalization.

1. What strategies should the nurse implement to assist in feeding a sick child? (Select all that apply.) a. Serve large portions. b. Make mealtimes pleasant. c. Avoid foods that are highly seasoned. d. Provide finger foods for young children. e. Ensure a variety of foods, textures, and colors.

ANS: B, C, D, E To assist in feeding a sick child mealtimes should be pleasant; highly seasoned foods should be avoided; finger foods should be provided for young children; and a variety of foods, textures, and colors should be ensured. Small portions, not large, should be served.

1. The nurse is caring for a child with hypercalcemia. The nurse evaluates the child for which signs and symptoms of hypercalcemia? (Select all that apply.) a. Tetany b. Anorexia c. Constipation d. Laryngospasm e. Muscle hypotonicity

ANS: B, C, E Signs and symptoms of hypercalcemia are anorexia, constipation, and muscle hypotonicity. Tetany and laryngospasm are signs of hypocalcemia.

1. The nurse is caring for a child with hypernatremia. The nurse evaluates the child for which signs and symptoms of hypernatremia? (Select all that apply.) a. Apathy b. Lethargy c. Oliguria d. Intense thirst e. Dry, sticky mucos

ANS: B, C, E Signs and symptoms of hypernatremia are nausea; oliguria; and dry, sticky mucos. Apathy and lethargy are signs of hyponatremia.

The nurse is caring for a child with psoriasis. What local manifestations does the nurse expect to assess in this child? (Select all that apply.) a. Development of wheals b. First lesions appear in the scalp c. Round, thick, dry reddish patches d. Lesions appear in intergluteal folds e. Patches are covered with coarse, silvery scales

ANS: B, C, E Local manifestations of psoriasis include lesions that appear in the scalp initially and round, thick dry patches covered with coarse, silvery scales. Development of wheals is seen in urticaria. Lesions in intergluteal folds are characteristic of intertrigo.

A 6-year-old child is scheduled for an IV urography (IVP) in the morning. Which preparatory interventions should the nurse plan to implement? (Select all that apply.) a. Clear liquids in the morning before the procedure b. Cathartic in the evening before the procedure c. Soapsuds enema the morning of the procedure d. Insertion of a Foley catheter before the procedure e. Teaching with regard to insertion of an intravenous catheter before the procedure

ANS: B, C, E The IV urography is a test done to provide information about the integrity of the kidneys, ureters, and bladder. It requires an IV injection of a contrast medium with X-ray films made 5, 10, and 15 minutes after injection. Delayed films (30, 60 minutes, and so on) are also obtained. The preparation for children ages 2 to 14 years includes cathartic on the evening before examination, nothing orally after midnight, and an enema (soapsuds) on the morning of examination. Teaching about the insertion of an intravenous catheter should be part of the preoperative preparation. Insertion of a Foley catheter is not part of the preparation for an IVP.

Which screening tests should the school nurse perform for the adolescent? (Select all that apply.) a. Glucose b. Vision c. Hearing d. Cholesterol e. Scoliosis

ANS: B, C, E The school nurse should perform vision, hearing, and scoliosis screening tests according to the school district's required schedule. Glucose and cholesterol screening would be performed in the medical clinic setting.

The parent of a child with a chronic illness tells the nurse, I feel so hopeless in this situation. The nurse should take which actions to foster hopefulness for the family? (Select all that apply.) a. Avoid topics that are lighthearted. b. Convey a personal interest in the child. c. Be honest when reporting on the childs condition. d. Do not initiate any playful interaction with the child. e. Demonstrate competence and gentleness when delivering care.

ANS: B, C, E To foster hopefulness, the nurse should convey a personal interest in the child, be honest when reporting on a childs condition, and demonstrate competence and gentleness when delivering care. The nurse should introduce conversations on neutral, nondisease-related, or less sensitive topics (discuss the childs favorite sports, tell stories). The nurse should be lighthearted and initiate or respond to teasing or other playful interactions with the child.

Which should the nurse teach to parents of toddlers about accidental poison prevention? (Select all that apply.) a. Keep toxic substances in the garage. b. Discard empty poison containers. c. Know the number of the nearest poison control center. d. Remove colorful labels from containers of toxic substances. e. Caution child against eating nonedible items, such as plants.

ANS: B, C, E To prevent accidental poisoning, parents should be taught to promptly discard empty poison containers, know the number of the nearest poison control center, and to caution the child against eating nonedible items, such as plants. Parents should place all potentially toxic agents, including cosmetics, personal care items, cleaning products, pesticides, and medications in a locked cabinet, not in the garage. Parents should be taught to never remove labels from containers of toxic substances

What developmental achievements are demonstrated by a 4-year-old child? (Select all that apply.) a. Cares for self totally b. Throws a ball overhead c. Has a vocabulary of 1500 words d. Can skip and hop on alternate feet e. Tends to be selfish and impatient f. Commonly has an imaginary playmate

ANS: B, C, E, F Developmental achievements for a 4-year-old child include throwing a ball overhead, having a vocabulary of 1500 words, tending to be selfish and impatient, and perhaps having an imaginary playmate. Caring for oneself totally and skipping and hopping on alternate feet are achievements normally seen in the 5-year-old age group.

The nurse is administering activated charcoal to a preschool child with acetaminophen (Tylenol) poisoning. What potential complications from the use of activated charcoal should the nurse plan to assess for? (Select all that apply.) a. Diarrhea b. Vomiting c. Fluid retention d. Intestinal obstruction

ANS: B, D Potential complications from the use of activated charcoal include vomiting and possible aspiration, constipation, and intestinal obstruction. Diarrhea and fluid retention are not potential complications of activated charcoal administration.

1. What dietary instructions should the nurse give to parents of a child undergoing chronic hemodialysis? (Select all that apply.) a. High protein b. Fluid restriction c. High phosphorus d. Sodium restriction e. Potassium restriction

ANS: B, D, E Dietary limitations are necessary in patients undergoing chronic dialysis to avoid biochemical complications. Fluid and sodium are restricted to prevent fluid overload and its associated symptoms of hypertension, cerebral manifestations, and congestive heart failure. Potassium is restricted to prevent complications related to hyperkalemia; phosphorus restriction helps prevent parathyroid hyperactivity and its attendant risk of abnormal calcification in soft tissues. Adequate protein, not high intake, is necessary to maximize growth potential. Fluid limitations are determined by residual urinary output and the need to limit intradialytic weight gain.

1. The nurse is caring for a child with hypokalemia. The nurse evaluates the child for which signs and symptoms of hypokalemia? (Select all that apply.) a. Twitching b. Hypotension c. Hyperreflexia d. Muscle weakness e. Cardiac arrhythmias

ANS: B, D, E Signs and symptoms of hypokalemia are hypotension, muscle weakness, and cardiac arrhythmias. Twitching and hyperreflexia are signs of hyperkalemia.

A school-age child has been admitted to the hospital with an exacerbation of nephrotic syndrome. Which clinical manifestations should the nurse expect to assess? (Select all that apply.) a. Weight loss b. Facial edema c. Cloudy smoky brown-colored urine d. Fatigue e. Frothy-appearing urine

ANS: B, D, E A child with nephrotic syndrome will present with facial edema, fatigue, and frothy-appearing urine (proteinuria). Weight gain, not loss, is expected because of the fluid retention. Cloudy smoky brown-colored urine is seen with acute glomerulonephritis but not with nephrotic syndrome because there is no gross hematuria associated with nephrotic syndrome.

The school nurse teaches adolescents that the detrimental long-term effects of tanning are what? (Select all that apply.) a. Vitamin D deficiency b. Premature aging of the skin c. Exacerbates acne outbreaks d. Increased risk for skin cancer e. Possible phototoxic reactions

ANS: B, D, E Adolescents should be educated regarding the detrimental effects of sunlight on the skin. Longterm effects include premature aging of the skin; increased risk for skin cancer; and, in susceptible individuals, phototoxic reactions. Exposure to levels of sunlight cause an increase in vitamin D production. Tanning can often reduce outbreaks of acne.

The school nurse is teaching bicycle safety to a group of school-age children. What should the nurse include in the session? (Select all that apply.) a. Ride double file when possible. b. Watch for and yield to pedestrians. c. Only ride double with someone your own size. d. Ride bicycles with traffic away from parked cars. e. Keep both hands on the handlebars except when signaling.

ANS: B, D, E Bicycle safety includes watching for and yielding to pedestrians, riding bicycles with traffic away from parked cars, and keeping both hands on handlebars except when signaling. It is best to ride single file, not double file, and never to ride double on a bicycle.

4. Characteristics of bullies include what? (Select all that apply.) a. Female b. Depressed c. Good peer relationships d. Poor academic performance e. Exposed to domestic violence

ANS: B, D, E Children who are bullies are likely to be male, depressed, have poor academic performance, be exposed to domestic violence, have poor peer relationships, and have poor communication with their parents.

Which strategies should the school nurse recommend implementing in the classroom for a child with attention deficit hyperactive disorder (ADHD)? (Select all that apply.) a. Schedule heavier subjects to be taught in the afternoon. b. Accompany verbal instructions by written format. c. Limit number of breaks taken during instructional periods. d. Allow more time for testing. e. Reduce homework and classroom assignments.

ANS: B, D, E Children with ADHD need an orderly, predictable, and consistent classroom environment with clear and consistent rules. Homework and classroom assignments may need to be reduced, and more time may need to be allotted for tests to allow the child to complete the task. Verbal instructions should be accompanied by visual references such as written instructions on the blackboard. Schedules may need to be arranged so that academic subjects are taught in the morning when the child is experiencing the effects of the morning dose of medication. Regular and frequent breaks in activity are helpful because sitting in one place for an extended time may be difficult.

What are characteristics of dating relationships in early adolescence? (Select all that apply.) a. One-on-one dating b. Follow ritualized scripts c. Are psychosocially intimate d. Involve playing stereotypic roles e. Participating in mixed-gender group activities

ANS: B, D, E Early dating relationships typically follow highly ritualized scripts in which adolescents are more likely to play stereotypic roles than to really be themselves. Participating in mixed-gender group activities, such as going to parties or other events, may have a positive impact on young teenagers well-being. One-on-one dating during early adolescence, however, with a lot of time spent alone, may lead to sexual intimacy before a teen is ready. Although teenagers may begin dating during early adolescence, these early dating relationships are not usually psychosocially intimate.

The emergency department nurse is admitting a child with a temperature of 35 C (95 F). What physical effects of hypothermia should the nurse expect to observe in this child? (Select all that apply.) a. Bradycardia b. Vigorous shivering c. Decreased respiratory rate d. Decreased intestinal motility e. Task performance is impaired

ANS: B, D, E Hypothermia has varying physical effects depending on the childs core temperature. At 35 C (95 F), a child would experience vigorous shivering, decreased intestinal motility, and task performance impairment. Bradycardia and decreased respiratory rate are physical effects observed as the body temperature continues to decrease

A nurse is caring for a child who is near death. Which physical signs indicate the child is approaching death? (Select all that apply.) a. Body feels warm b. Tactile sensation decreasing c. Speech becomes rapid d. Change in respiratory pattern e. Difficulty swallowing

ANS: B, D, E Physical signs of approaching death include: tactile sensation beginning to decrease, a change in respiratory pattern, and difficulty swallowing. Even though there is a sensation of heat the body feels cool, not warm, and speech becomes slurred, not rapid.

What are risk factors of testicular cancer? (Select all that apply.) a. Hispanic b. Infertility c. Alcohol use d. Tobacco use e. Family history

ANS: B, D, E Risk factors of testicular cancer include infertility, tobacco use, and a family history. White, not Hispanic, ethnicity is a high risk, and alcohol use is not a risk.

The nurse is caring for children on an adolescent-only unit. What growth and development milestones should the nurse expect from 11- and 14-year-old adolescents? (Select all that apply.) a. Self-centered with increased narcissism b. No major conflicts with parents c. Established abstract thought process d. Have a rich, idealistic fantasy life e. Highly value conformity to group norms f. Secondary sexual characteristics appear

ANS: B, E, F Growth and development milestones in the 11- to 14-year-old age group include minimal conflicts with parents (compared with the 15- to 17-year-old age group), a high value placed on conformity to the norm, and the appearance of secondary sexual characteristics. Selfcenteredness and narcissism are seen in the 15- to 17-year-old age group along with a rich and idealistic fantasy life. Abstract thought processes are not well established until the 18- to 20- year-old age group.

The nurse is assessing a childs functional self-care level for feeding, bathing and hygiene, dressing, and grooming and toileting. The child requires assistance or supervision from another person and equipment or device. What code does the nurse assign for this child? a. I b. II c. III d. IV

ANS: C A code of III indicates the child requires assistance from another person and equipment or device. A code of I indicates use of equipment or device. A code of II indicates assistance or supervision from another person. A code of IV indicates the child is totally dependent.

The practitioner has ordered a liquid oral antibiotic for a toddler with otitis media. The prescription reads 1 1/2 tsp four times per day. What should the nurse consider in teaching the mother how to give the medicine? a. A measuring spoon should be used, and the medication must be given every 6 hours. b. The mother is not able to handle this regimen. Long-acting intramuscular antibiotics should be administered. c. A hollow-handled medication spoon is advisable, and the medication should be equally spaced while the child is awake. d. A household teaspoon should be used and the medicine given when the child wakes up, around lunch time, at dinner time, and before bed.

ANS: C A hollow-handled medication spoon allows the mother to measure the correct amount of medication. The order is written for four times a day; every 6 hours dosing is not necessary. There is no indication that the mother is not able to adhere to the medication regimen. She is asking for clarification so she can properly care for her child. Long-acting intramuscular antibiotics are not indicated. Household teaspoons vary greatly and should not be used.

1. The parents of a 2-year-old boy who had a repair of exstrophy of the bladder at birth ask when they can begin toilet training their son. The nurse replies based on what knowledge? a. Most boys in the United States can be toilet trained at age 3 years. b. Training can begin when he has sufficient bladder capacity. c. Additional surgery may be necessary to achieve continence. d. They should begin now because he will require additional time.

ANS: C After repair of the bladder exstrophy, the childs bladder is allowed to increase capacity. Several surgical procedures may be necessary to create a urethral sphincter mechanism to aid in urination and ejaculation. With the lack of a urinary sphincter, toilet training is unlikely. The child cannot hold the urine in the bladder. Bladder capacity is one component of continence. A functional sphincter is also needed.

1. Urinary tract anomalies are frequently associated with what irregularities in fetal development? a. Myelomeningocele b. Cardiovascular anomalies c. Malformed or low-set ears d. Defects in lower extremities

ANS: C Although unexplained, there is a frequent association between malformed or low-set ears and urinary tract anomalies. During the newborn examination, the nurse should have a high suspicion about urinary tract structure and function if ear anomalies are present. Children who have myelomeningocele may have impaired urinary tract function secondary to the neural defect. When other congenital defects are present, there is an increased likelihood of other issues with other body systems. Cardiac and extremity defects do not have a strong association with renal anomalies.

What condition is defined as reduced visual acuity in one eye despite appropriate optical correction? a. Myopia b. Hyperopia c. Amblyopia d. Astigmatism

ANS: C Amblyopia, or lazy eye, is reduced visual acuity in one eye. Amblyopia is usually caused by one eye not receiving sufficient stimulation. The resulting poor vision in the affected eye can be avoided with the treatment of the primary visual defect such as strabismus. Myopia, or nearsightedness, refers to the ability to see objects clearly at close range but not a distance. Hyperopia, or farsightedness, is the ability to see objects at a distance but not at close range. Astigmatism is unequal curvatures in refractive apparatus.

A 2-year-old child is being admitted to the hospital for possible bacterial meningitis. When preparing for a lumbar puncture, what should the nurse do? a. Set up a tray with equipment the same size as for adults. b. Apply EMLA to the puncture site 15 minutes before the procedure. c. Prepare the child for conscious sedation being used for the procedure. d. Reassure the parents that the test is simple, painless, and risk free.

ANS: C Because of the urgency of the childs condition, conscious sedation should be used for the procedure. Pediatric spinal trays have smaller needles than do adult trays. EMLA should be applied approximately 60 minutes before the procedure; the emergency nature of the spinal tap precludes its use. A spinal tap is not a simple procedure and does have associated risks; analgesia will be given for the pain.

1. The parent of a child hospitalized with acute glomerulonephritis asks the nurse why blood pressure readings are being taken so often. What knowledge should influence the nurses reply? a. The antibiotic therapy contributes to labile blood pressure values. b. Hypotension leading to sudden shock can develop at any time. c. Acute hypertension is a concern that requires monitoring. d. Blood pressure fluctuations indicate that the condition has become chronic.

ANS: C Blood pressure monitoring is essential to identify acute hypertension, which is treated aggressively. Antibiotic therapy is usually not indicated for glomerulonephritis. Hypertension, not hypotension, is a concern in glomerulonephritis. Blood pressure control is essential to prevent further renal damage. Blood pressure fluctuations do not provide information about the chronicity of the disease.

A 7-year-old is identified as being at risk for skin breakdown. What intervention should the nursing care plan include? a. Massaging reddened bony prominences b. Teaching the parents to turn the child every 4 hours c. Ensuring that nutritional intake meets requirements d. Minimizing use of extra linens, which can irritate the childs skin

ANS: C Children who are hospitalized and NPO (taking nothing by mouth) for several days are at risk for nutritional deficiencies and skin breakdown. If NPO status is prolonged, parenteral nutrition should be considered. Massaging bony prominences can cause deep tissue damage. This should be avoided. Although parents can participate, turning the child is the nurses responsibility. If the child is alert and can move, position shifts should be done more frequently. If the child does not move, the nurse should reposition every 2 hours. The number of linens is not an issue. The child should not be dragged across the sheet. Children should be lifted and moved to avoid friction and shearing.

Two hospitalized adolescents are playing pool in the activity room. Neither of them seems enthusiastic about the game. How should the nurse interpret this situation? a. Playing pool requires too much concentration for this age group. b. Pool is an activity better suited for younger children. c. The adolescents may be enjoying themselves but have lower energy levels than healthy children. d. The adolescents lack of enthusiasm is one of the signs of depression.

ANS: C Children who are ill and hospitalized typically have lower energy levels than healthy children. Therefore, children may not appear enthusiastic about an activity even when they are enjoying it. Pool is an appropriate activity for adolescents. They have the cognitive and psychomotor skills that are necessary. If the adolescents were significantly depressed, they would be unable to engage in the game.

A school-age child with acute diarrhea and mild dehydration is being given oral rehydration solutions (ORS). The childs mother calls the clinic nurse because he is also occasionally vomiting. The nurse should recommend which intervention? a. Bring the child to the hospital for intravenous fluids. b. Alternate giving ORS and carbonated drinks. c. Continue to give ORS frequently in small amounts. d. Keep child NPO (nothing by mouth) for 8 hours and resume ORS if vomiting has subsided.

ANS: C Children who are vomiting should be given ORS at frequent intervals and in small amounts. Intravenous fluids are not indicated for mild dehydration. Carbonated beverages are high in carbohydratesandarenotrecommendedforthetreatmentofdiarrheaandvomiting.ThechildisnotkeptNPObecausethiswouldcauseadditionalfluidlosses

1. What major complication is associated with a child with chronic renal failure? a. Hypokalemia b. Metabolic alkalosis c. Water and sodium retention d. Excessive excretion of blood urea nitrogen

ANS: C Chronic renal failure leads to water and sodium retention, which contributes to edema and vascular congestion. Hyperkalemia, metabolic acidosis, and retention of blood urea nitrogen are complications of chronic renal failure.

1. Physiologically, the child compensates for fluid volume losses by which mechanism? a. Inhibition of aldosterone secretion b. Hemoconcentration to reduce cardiac workload c. Fluid shift from interstitial space to intravascular space d. Vasodilation of peripheral arterioles to increase perfusion

ANS: C Compensatory mechanisms attempt to maintain fluid volume. Initially, interstitial fluid moves into the intravascular compartment to maintain blood volume. Aldosterone is released to promote sodium retention and conserve water in the kidneys. Hemoconcentration results from the fluid volume loss. With less circulating volume, tachycardia results. Vasoconstriction of peripheral arterioles occurs to help maintain blood pressure.

1. What diet is most appropriate for the child with chronic renal failure (CRF)? a. Low in protein b. Low in vitamin D c. Low in phosphorus d. Supplemented with vitamins A, E, and K

ANS: C Dietary phosphorus may need to be restricted by limiting protein and milk intake. Substances that bind phosphorus are given with meals to prevent its absorption, which enables a more liberal intake of phosphorus-containing protein. Protein is limited to the recommended daily allowance for the childs age. Further restriction is thought to negatively affect growth and neurodevelopment. Vitamin D therapy is administered in children with CRF to increase calcium absorption. Supplementation of vitamins A, E, and K, beyond normal dietary intake, is not advised in children with CRF. These fat-soluble vitamins can accumulate.

The parents of a child with cognitive impairment ask the nurse for guidance with discipline. What should the nurses recommendation be based on? a. Discipline is ineffective with cognitively impaired children. b. Cognitively impaired children do not require discipline. c. Behavior modification is an excellent form of discipline. d. Physical punishment is the most appropriate form of discipline.

ANS: C Discipline must begin early. Limit-setting measures must be clear, simple, consistent, and appropriate for the childs mental age. Behavior modification, especially reinforcement of desired behavior and use of time-out procedures, is an appropriate form of behavior control. Aversive strategies should be avoided in disciplining the child.

29. What sign is one of the first to indicate overwhelming sepsis in a child with burn injuries? a. Seizures b. Bradycardia c. Disorientation d. Decreased blood pressure

ANS: C Disorientation in the burn patient is one of the first signs of overwhelming sepsis and may indicate inadequate hydration. Seizures, bradycardia, and decreased blood pressure are not initial manifestations of overwhelming sepsis.

1. The nurse is teaching a child experiencing severe edema associated with minimal change nephrotic syndrome about his diet. The nurse should discuss what dietary need? a. Consuming a regular diet b. Increasing protein c. Restricting fluids d. Decreasing calories

ANS: C During the edematous stage of active nephrosis, the child has restricted fluid and sodium intake. As the edema subsides, the child is placed on a diet with increased salt and fluids. A regular diet is not indicated. There is no evidence that a diet high in protein is beneficial or has an effect on the course of the disease. Calories sufficient for growth and tissue healing are essential. With the child having little appetite and the fluid and salt restrictions, achieving adequate nutrition is difficult.

29. After the acute stage and during the healing process, what is the primary complication from burn injury? a. Shock b. Asphyxia c. Infection d. Renal shutdown

ANS: C During the healing phase, local infection or sepsis is the primary complication. Respiratoryproblems, primarily airway compromise, and shock are the primary complications during theacutestageofburninjury.Renalshutdownisnotacomplicationoftheburninjurybutmaybearesultoftheprofoundshock

1. What parents should have the most difficult time coping with their childs hospitalization? a. Parents of a child hospitalized for juvenile arthritis b. Parents of a child hospitalized with a recent diagnosis of bronchiolitis c. Parents of a child hospitalized for sepsis resulting from an untreated injury d. Parents of a child hospitalized for surgical correction of undescended testicles

ANS: C Factors that affect parents reactions to their childs illness include the seriousness of the threat to the child. The parents of a child hospitalized for sepsis resulting from an untreated injury would have more difficulty coping because of the seriousness of the illness and because the wound was not treated immediately.

1. The nurse is teaching the family of a child with a long-term central venous access device about signs and symptoms of bacteremia. What finding indicates the presence of bacteremia? a. Hypertension b. Pain at the entry site c. Fever and general malaise d. Redness and swelling at the entry site

ANS: C Fever, chills, general malaise, and an ill appearance can be signs of bacteremia and require immediate intervention. Hypotension would be indicative of sepsis and possible impending cardiovascular collapse. Pain, redness, and swelling at the entry site indicate local infection.

What behavior should most likely be manifested in an infant experiencing the protest phase of separation anxiety? a. Inactivity b. Depression and sadness c. Inconsolable and crying d. Regression to earlier behavior

ANS: C For older infants, being inconsolable and crying is seen during the protest phase of separation anxiety. Inactivity is observed during the stage of despair. The child is much less active and withdraws from others. Depression, sadness, and regression to earlier behaviors are observed during the phase of despair.

23. What organism is a parasite that causes acute diarrhea? a. Shigella organisms b. Salmonella organisms c. Giardia lamblia d. Escherichia coli

ANS: C G. lamblia is a parasite that represents 10% of nondysenteric illness in the United States. Shigella,Salmonella,andE.coliarebacterialpathogens.

1. A girl, age 5 1/2 years, has been sent to the school nurse for urinary incontinence three times in the past 2 days. The nurse should recommend to her parent that the first action is to have the child evaluated for what condition? a. School phobia b. Glomerulonephritis c. Urinary tract infection (UTI) d. Attention deficit hyperactivity disorder (ADHD)

ANS: C Girls between the ages of 2 and 6 years are considered high risk for UTIs. This child is showing signs of a UTI, including incontinence in a toilet-trained child and possible urinary frequency or urgency. A physiologic cause should be ruled out before psychosocial factors are investigated. Glomerulonephritis usually manifests with decreased urinary output and fluid retention. ADHD can contribute to urinary incontinence because the child is distracted, but the first manifestation was incontinence, not distractibility.

1. A hospitalized child with minimal change nephrotic syndrome is receiving high doses of prednisone. What nursing goal is appropriate for this child? a. Stimulate appetite. b. Detect evidence of edema. c. Minimize risk of infection. d. Promote adherence to the antibiotic regimen.

ANS: C High-dose steroid therapy has an immunosuppressant effect. These children are particularly vulnerable to upper respiratory tract infections. A priority nursing goal is to minimize the risk of infection by protecting the child from contact with infectious individuals. Appetite is increased with prednisone therapy. The amount of edema should be monitored as part of the disease process, not necessarily related to the administration of prednisone. Antibiotics would not be used as prophylaxis.

1. A child is admitted in acute renal failure (ARF). Therapeutic management to rapidly provoke a flow of urine includes the administration of what medication? a. Propranolol (Inderal) b. Calcium gluconate c. Mannitol (Osmitrol) or furosemide (Lasix) (or both) d. Sodium, chloride, and potassium

ANS: C In ARF, if hydration is adequate, mannitol or furosemide (or both) is administered to provoke a flow of urine. If glomerular function is intact, an osmotic diuresis will occur. Propranolol is a beta-blocker; it will not produce a rapid flow of urine in ARF. Calcium gluconate is administered for its protective cardiac effect when hyperkalemia exists. It does not affect diuresis. Electrolyte measurements must be done before administration of sodium, chloride, or potassium. These substances are not given unless there are other large, ongoing losses. In the absence of urine production, potassium levels may be elevated, and additional potassium can cause cardiac dysrhythmias.

A 5-year-old child returns from the pediatric intensive care unit after abdominal surgery. The orders state to monitor vital signs every 2 hours. On assessment, the nurse observes that the childs heart rate is 20 beats/min less than it was preoperatively. What should be the nurses next action? a. Follow the orders and check in 2 hours. b. Ask the parents if this is the childs usual heart rate. c. Recheck the pulse and blood pressure in 15 minutes. d. Notify the surgeon that the child is probably going into shock.

ANS: C In a 5-year-old child, this is a significant change in vital signs. The nurse should assess the child to see if his condition mirrors a drop in heart rate. The assessment and vital signs should be redone in 15 minutes to determine whether the childs condition is stable. When a disparity in vital signs or other assessment data is observed, the nurse should reassess sooner. Most parents will not know their childs heart rate. It is important to determine how the child is recovering from surgery. The nurse should collect additional information before notifying the surgeon. This includes blood pressure, respiratory rate, and pain status.

29. Prevention of burn injury is important anticipatory guidance. In the infant and toddler period, which mode is the most common cause of burn? a. Matches b. Electrical cords c. Hot liquids in the kitchen d. Microwave-heated foods

ANS: C Infants and toddlers are most commonly injured by hot liquids in the kitchen and bathroom. This often occurs as a result of inadequate supervision of this curious and energetic age group. Matches and lighters are seen as toys by young children and should be kept out of reach. Older toddlers and preschool children are at risk of chewing on electrical cords and placing objects in outlets. Microwave-heated fluids and foods can become superheated, resulting in oral burns.

A 2-week-old infant with Down syndrome is being seen in the clinic. His mother tells the nurse that he is difficult to hold, that hes like a rag doll. He doesnt cuddle up to me like my other babies did. What is the nurses best interpretation of this lack of clinging or molding? a. Sign of detachment and rejection b. Indicative of maternal deprivation c. A physical characteristic of Down syndrome d. Suggestive of autism associated with Down syndrome

ANS: C Infants with Down syndrome have hypotonicity of muscles and hyperextensibility of joints, which complicate positioning. The limp, flaccid extremities resemble the posture of a rag doll. Holding the infant is difficult and cumbersome, and parents may feel that they are inadequate. A lack of clinging or molding is characteristic of Down syndrome, not detachment. There is no evidence of maternal deprivation. Autism is not associated with Down syndrome, and it would not be evident at 2 weeks of age.

29. What is the purpose of a high-protein diet for a child with major burns? a. Promote growth b. Improve appetite c. Minimize protein breakdown d. Diminish risk of stress-induced hyperglycemia

ANS: C Initially after major burns, there is a hypometabolic phase, which lasts for 2 or 3 days. A hypermetabolic phase follows, characterized by increased body temperature, oxygen and glucose consumption, carbon dioxide production, glycogenolysis, proteolysis, and lipolysis. This response continues for up to 9 months. A diet high in protein and calories is necessary. Healing, not growth, is the primary consideration. Many children have poor appetites, and supplementation is necessary. Hypoglycemia, not hyperglycemia, can occur from the stress of burn injury because the liver glycogen stores are rapidly depleted.

1. The nurse suspects fluid overload in an infant receiving intravenous fluids. What clinical manifestation is suggestive of water intoxication? a. Oliguria b. Weight loss c. Irritability and seizures d. Muscle weakness and cardiac dysrhythmias

ANS: C Irritability, somnolence, headache, vomiting, diarrhea, and generalized seizures are manifestations of water intoxication. Urinary output is increased as the child attempts to maintain fluid balance. Weight gain is usually associated with water intoxication. Muscle weakness and cardiac dysrhythmias are not associated with water intoxication.

An appropriate method for administering oral medications that are bitter to an infant or small child should be to mix them with which? a. Bottle of formula or milk b. Any food the child is going to eat c. One teaspoon of something sweet-tasting such as jam d. Carbonated beverage, which is then poured over crushed ice

ANS: C Mix the drug with a small amount (about 1 tsp) of sweet-tasting substance. This will make the medication more palatable to the child. The medication should be mixed with only a small amount of food or liquid. If the child does not finish drinking or eating, it is difficult to determine how much medication was consumed. Medication should not be mixed with essential foods and milk. The child may associate the altered taste with the food and refuse to eat this food in the future.

1. Several types of long-term central venous access devices are used. What is a benefit of using an implanted port (e.g., Port-a-Cath)? a. You do not need to pierce the skin for access. b. It is easy to use for self-administered infusions. c. The patient does not need to limit regular physical activity, including swimming. d. The catheter cannot dislodge from the port even if the child plays with the port site.

ANS: C No limitations on physical activity are needed. The child is able to participate in all regular physical activities, including bathing, showering, and swimming. The skin over the device is pierced with a Huber needle to access. Long-term central venous access devices are difficult to use for self-administration. The port is placed under the skin. If the child manipulates the device and plays with the actual port, the catheter can be dislodged.

32. The nurse is preparing to give acetaminophen (Tylenol) to a child who has a fever. What nursing action is appropriate? a. Retake the temperature in 15 minutes after giving the Tylenol. b. Place a warm blanket on the child so chilling does not occur. c. Check to be sure the Tylenol dose does not exceed 15 mg/kg. d. Use cold compresses instead of Tylenol to control the fever.

ANS: C Nurses must have an understanding of the safe dosages of medications they administer to children, as well as the expected actions, possible side effects, and signs of toxicity. The recommended doses of acetaminophen should never be exceeded.

The parents of a 3-year-old admitted for recurrent diarrhea are upset that the practitioner has not told them what is going on with their child. What is the priority intervention for this family? a. Answer all of the parents questions about the childs illness. b. Immediately the practitioner to come to the unit to speak with the family. c. Help the family develop a written list of specific questions to ask the practitioner. d. Inform the family of the time that hospital rounds are made so that they can be present.

ANS: C Often families ask general questions of health care providers and do not receive the information they need. The nurse should determine what information the family does want and then help develop a list of questions. When the questions are written, the family can remember which questions to ask or can hand the sheet to the practitioner for answers. The nurse may have the information the parents want, but they are asking for specific information from the practitioner. Unless it is an emergency, the nurse should not place a stat MCS: for the practitioner. Being present is not necessarily the issue but rather the ability to get answers to specific questions.

Autism is a complex developmental disorder. The diagnostic criteria for autism include delayed or abnormal functioning in which area with onset before age 3 years? a. Parallel play b. Gross motor development c. Ability to maintain eye contact d. Growth below the fifth percentile

ANS: C One hallmark of autism spectrum disorders is the childs inability to maintain eye contact with another person. Parallel play is play typical of toddlers and is usually not affected. Social, not gross motor, development is affected by autism. Physical growth and development are not usually affected.

A 6-year-old child needs to drink 1 L of GoLYTELY in preparation for a computed tomography scan of the abdomen. To encourage the child to drink, what should the nurse do? a. Give him a large cup with ice so it tastes better. b. Restrict him to his room until he drinks the GoLYTELY. c. Use little cups and make a game to reward him for each cup he drinks. d. Tell him that if he does not finish drinking by a set time, the practitioner will be angry.

ANS: C One liter of GoLYTELY is difficult for many children to drink. By using small cups, the child will find the amount less overwhelming. Then a game can be made in which some type of reward (sticker, reading another MCS: of a book) is given for each cup. A large cup of ice would make it more difficult because the child would see it as too much and ice adds additional fluid to be consumed. Negative reinforcement may work if the child wishes to be out of his room. A practitioner may or may not be angry if he does not finish drinking by a set time; this is a threat that may or may not be true. If the child is having difficulty drinking, this would most likely not be effective.

The nurse is teaching feeding strategies to a parent of a 12-month-old infant with Down syndrome. What statement made by the parent indicates a need for further teaching? a. If the food is thrust out, I will reefed it. b. I will use a small, long, straight-handled spoon. c. I will place the food on the top of the tongue. d. I know the tongue thrust doesnt indicate a refusal of the food.

ANS: C Parents of a child with Down syndrome need to know that the tongue thrust does not indicate refusal to feed but is a physiologic response. Parents are advised to use a small but long, straight- handled spoon to push the food toward the back and side of the mouth. If food is thrust out, it should be refed. If the parent indicates placing the food on the tongue, further teaching is needed.

1. What statement is an advantage of peritoneal dialysis compared with hemodialysis? a. Protein loss is less extensive. b. Dietary limitations are not necessary. c. It is easy to learn and safe to perform. d. It is needed less frequently than hemodialysis.

ANS: C Peritoneal dialysis is the preferred form of dialysis for parents, infants, and children who wish to remain independent. Parents and older children can perform the treatments themselves. Protein loss is not significantly different. The dietary limitations are necessary, but they are not as stringent as those for hemodialysis. Treatments are needed more frequently but can be done at home.

Parents are being taught how to feed their infant using a newly placed gastrostomy tube (G- tube). What is essential information for the parents to receive? a. Verify placement before each feeding. b. Use a syringe with a plunger to give the infant bolus feedings. c. Position the infant on the right side during and after the feeding. d. Beefy red tissue around the G-tube site must be reported to the practitioner

ANS: C Positioning on the right side during and after feedings helps minimize the risk of aspiration. It is not necessary to verify placement before each feeing. G-tubes are inserted into the stomach and sutured in place. If the tube is through the skin, it is in the stomach. Feedings should be given by gravity flow. The plunger may be used to initiate the feeding, but then the formula should be allowed to flow. Beefy red tissue around the G-tube site is normal granulation tissue that is expected.

Using knowledge of child development, what approach is best when preparing a toddler for a procedure? a. Avoid asking the child to make choices. b. Plan for a teaching session to last about 20 minutes. c. Demonstrate on a doll how the procedure will be done. d. Show the necessary equipment without allowing child to handle it.

ANS: C Prepare toddlers for procedures by using play. Demonstrate on a doll but avoid the childs favorite doll because the toddler may think the doll is really feeling the procedure. In preparing a toddler for a procedure, the child is allowed to participate in care and help whenever possible. Teaching sessions for toddlers should be about 5 to 10 minutes. Use a small replica of the equipment and allow the child to handle it.

1. What statement is descriptive of renal transplantation in children? a. It is an acceptable means of treatment after age 10 years. b. Children can receive kidneys only from other children. c. It is the preferred means of renal replacement therapy in children. d. The decision for transplantation is difficult because a relatively normal lifestyle is not possible.

ANS: C Renal transplantation offers the opportunity for a relatively normal life and is the preferred means of renal replacement therapy in end-stage renal disease. It can be done in children as young as age 6 months. Both children and adults can serve as donors for renal transplant purposes. Renal transplantation affords the child a more normal lifestyle than dependence on dialysis.

A 5-year-old child has bilateral eye patches in place after surgery yesterday morning. Today he can be out of bed. What nursing intervention is most important at this time? a. Speak to him when entering the room. b. Allow him to assist in feeding himself. c. Orient him to his immediate surroundings. d. Reassure him and allow his parents to stay with him.

ANS: C Safety is the priority concern. Because he can now be out of bed, it is imperative that he knows about his physical surroundings. Speaking to the child is a component of nursing care that is expected with all clients unless contraindicated. Unless additional impairments are present, his meal tray should be set up, and he should be able to feed himself. Reassuring him and allowing his parents to stay with him are essential parts of nursing care for all children.

The nurse is admitting a 7-year-old child to the pediatric unit for abdominal pain. To determine what the child understands about the reason for hospitalization, what should the nurse do? a. Find out what the parents have told the child. b. Review the note from the admitting practitioner. c. Ask the child why he came to the hospital today. d. Question the parents about why they brought the child to the hospital.

ANS: C School-age children are able to answer questions. The only way for the nurse to know about the childs understanding of the reason for hospitalization is to ask the child directly. Finding out what the parents told the child and why they brought the child to the hospital or reading the admitting practitioners description of the reason for admission will not provide information about what the child has heard and retained.

The nurse is preparing a 9-year-old boy before obtaining a blood specimen by venipuncture. The child tells the nurse he does not want to lose his blood. What approach is best by the nurse? a. Explain that it will not be painful. b. Suggest to him that he not worry about losing just a little bit of blood. c. Discuss with him how his body is always in the process of making blood. d. Tell the child that he will not even need a Band-Aid afterward because it is a simple procedure.

ANS: C School-age children can understand that blood can be replaced. Explain the procedure to him using correct scientific and medical terminology. The venipuncture will be uncomfortable. It is inappropriate to tell him it will not hurt. Even though the nurse considers it a simple procedure, the boy is concerned. Telling him not to worry will not allay his fears.

1. In teaching the parent of a newly diagnosed 2-year-old child with pyelonephritis related to vesicoureteral reflux (VUR), the nurse should include which information? a. Limit fluids to reduce reflux. b. Give cranberry juice twice a day. c. Have siblings examined for VUR. d. Surgery is indicated to reverse scarring.

ANS: C Siblings are at high risk for VUR. The incidence of reflux in siblings is approximately 36%. The other children should be screened for early detection and to potentially reduce scarring. Fluids are not reduced. The efficacy of cranberry juice in reducing infection in children has not been established. Surgery may be necessary for higher grades of VUR, but the scarring is not reversible.

A 14-year-old adolescent is hospitalized with cystic fibrosis. What nursing note entry represents best documentation of his breakfast meal? a. Tolerated breakfast well b. Finished all of breakfast ordered c. One pancake, eggs, and 240 ml OJ d. No documentation is needed for this age child.

ANS: C Specific information is necessary for hospitalized children. It is essential to be able to identify caloric intake and eating patterns for assessment and intervention purposes. That he tolerated breakfast well only provides information that the child did not become ill with the meal. Even if he finished all his breakfast, an evaluation cannot be completed unless the quantity of food ordered is known. Nutritional information is essential, especially for children with chronic illnesses.

29. The nurse is teaching a group of female adolescents about toxic shock syndrome and the use of tampons. What statement by a participant indicates a need for additional teaching? a. I can alternate using a tampon and a sanitary napkin. b. I should wash my hands before inserting a tampon. c. I can use a superabsorbent tampon for more than 6 hours. d. I should call my health care provider if I suddenly develop a rash that looks like sunburn.

ANS: C Teaching female adolescents about the association between toxic shock syndrome and the use of tampons is important. The teaching should include not using superabsorbent tampons; not leaving the tampon in for longer than 4 to 6 hours; alternating the use of tampons with sanitary napkins; washing hands before inserting a tampon to decrease the chance of introducing pathogens; and informing a health care provider if a sudden high fever, vomiting, muscle pain, dizziness, or a rash that looks like a sunburn appears.

The American Association on Intellectual and Developmental Disabilities (AAIDD), formerly the American Association on Cognitive Impairment, classifies cognitive impairment based on what parameter? a. Age of onset b. Subaverage intelligence c. Adaptive skill domains d. Causative factors for cognitive impairment

ANS: C The AAIDD has categorized cognitive impairment into adaptive skill domains. The child must demonstrate functional impairment in at least two of the following adaptive skill domains: communication, self-care, home living, social skills, use of community resources, self-direction, health and safety, functional academics, leisure, and work. Age of onset before 18 years is part of the former criteria. Low intelligence quotient (IQ) alone is not the sole criterion for cognitive impairment. Etiology is not part of the classification.

What intervention is most appropriate to facilitate social development of a child with a cognitive impairment? a. Provide age-appropriate toys and play activities. b. Avoid exposure to strangers who may not understand cognitive development. c. Provide peer experiences, such as infant stimulation and preschool programs. d. Emphasize mastery of physical skills because they are delayed more often than verbal skills.

ANS: C The acquisition of social skills is a complex task. Initially, an infant stimulation program should be used. Children of all ages need peer relationships. Parents should enroll the child in preschool. When older, they should have peer experiences similar to those of other children such as group outings, Boy and Girl Scouts, and Special Olympics. Providing age-appropriate toys and play activities is important, but peer interactions facilitate social development. Parents should expose the child to individuals who do not know the child. This enables the child to practice social skills. Verbal skills are delayed more often than physical skills.

A child has a central venous access device for intravenous (IV) fluid administration. A blood sample is needed for a complete blood count, hemogram, and electrolytes. What is the appropriate procedure to implement for this blood sample? a. Perform a new venipuncture to obtain the blood sample. b. Interrupt the IV fluid and withdraw the blood sample needed. c. Withdraw a blood sample equal to the amount of fluid in the device, discard, and then withdraw the sample needed. d. Flush the line and central venous device with saline and then aspirate the required amount of blood for the sample.

ANS: C The blood specimen obtained must reflect the appropriate hemodilution of the blood and electrolyte concentration. The nurse needs to withdraw the amount of fluid that is in the device and discard it. The next sample will come from the childs circulating blood. With a central venous device, the trauma of a separate venipuncture can be avoided. The blood sample will be diluted with either the IV fluid being administered or the saline.

A father calls the emergency department nurse saying that his daughters eyes burn after getting some dishwasher detergent in them. The nurse recommends that the child be seen in the emergency department or by an ophthalmologist. The nurse also should recommend which action before the child is transported? a. Keep the eyes closed. b. Apply cold compresses. c. Irrigate the eyes copiously with tap water for 20 minutes. d. Prepare a normal saline solution (salt and water) and irrigate the eyes for 20 minutes.

ANS: C The first action is to flush the eyes with clean tap water. This will rinse the detergent from the eyes. Keeping the eyes closed and applying cold compresses may allow the detergent to do further harm to the eyes during transport. Normal saline is not necessary. The delay can allow the detergent to cause continued injury to the eyes.

1. What factor predisposes an infant to fluid imbalances? a. Decreased surface area b. Lower metabolic rate c. Immature kidney functioning d. Decreased daily exchange of extracellular fluid

ANS: C The infants kidneys are functionally immature at birth and are inefficient in excreting waste products of metabolism. Infants have a relatively high body surface area (BSA) compared with adults. This allows a higher loss of fluid to the environment. A higher metabolic rate is present as a result of the higher BSA in relation to active metabolic tissue. The higher metabolic rate increases heat production, which results in greater insensible water loss. Infants have a greater exchange of extracellular fluid, leaving them with a reduced fluid reserve in conditions of dehydration.

29. The nurse suspects shock in a child 1 day after surgery. What should be the initial nursing action? a. Place the child on a cardiac monitor. b. Obtain arterial blood gases. c. Provide supplemental oxygen. d. Put the child in the Trendelenburg position.

ANS: C The initial nursing action for a patient in shock is to establish ventilatory support. Oxygen is provided, and the nurse carefully observes for signs of respiratory failure, which indicates a need for intubation. Cardiac monitoring would be indicated to assess the childs status further, but ventilatory support comes first. Oxygen saturation monitoring should be begun. Arterial blood gases would be indicated if alternative methods of monitoring oxygen therapy were not available. The Trendelenburg position is not indicated and is detrimental to the child. The head- down position increases intracranial pressure and decreases diaphragmatic excursion and lung volume.

When teaching a mother how to administer eye drops, where should the nurse tell her to place them? a. At the lacrimal duct b. On the sclera while the child looks to the outside c. In the conjunctival sac when the lower eyelid is pulled down d. Carefully under the eyelid while it is gently pulled upward

ANS: C The lower eyelid is pulled down, forming a small conjunctival sac. The solution or ointment is applied to this area. The medication should not be administered directly on the eyeball. The lacrimal duct is not the appropriate placement for the eye medication. It will drain into the nasopharynx, and the child will taste the drug.

1. What condition is the most common cause of acute renal failure in children? a. Pyelonephritis b. Tubular destruction c. Severe dehydration d. Upper tract obstruction

ANS: C The most common cause of acute renal failure in children is dehydration or other causes of poor perfusion that may respond to restoration of fluid volume. Pyelonephritis and tubular destruction are not common causes of acute renal failure. Obstructive uropathy may cause acute renal failure, but it is not the most common cause.

A 6-year-old is being discharged home, which is 90 miles from the hospital, after an outpatient hernia repair. In addition to explicit discharge instructions, what should the nurse provide? a. An ambulance for transport home b. Verbal information about follow-up care c. Prescribed pain medication before discharge d. Driving instructions for a route with less traffic

ANS: C The nurse should anticipate that the child will begin experiencing pain on the trip home. By providing a dose of oral analgesia, the nurse can ensure the child remains comfortable during the trip. Transport by ambulance is not indicated for a hernia repair. Discharge instructions should be written. The parents will be focusing on their child and returning home, which limits their ability to retain information. The parents should know the most expedient route home.

1. What is the primary objective of care for the child with minimal change nephrotic syndrome (MCNS)? a. Reduce blood pressure. b. Lower serum protein levels. c. Minimize excretion of urinary protein. d. Increase the ability of tissue to retain fluid.

ANS: C The objectives of therapy for the child with MCNS include reducing the excretion of urinary protein, reducing fluid retention, preventing infection, and minimizing complications associated with therapy. Blood pressure is usually not elevated in minimal change nephrotic syndrome. Serum protein levels are already reduced as part of the disease process. This needs to be reversed. The tissue is already retaining fluid as part of the edema. The goal of therapy is to reduce edema.

A newborn assessment shows a separated sagittal suture, oblique palpebral fissures, a depressed nasal bridge, a protruding tongue, and transverse palmar creases. These findings are most suggestive of which condition? a. Microcephaly b. Cerebral palsy c. Down syndrome d. Fragile X syndrome

ANS: C These are characteristics associated with Down syndrome. An infant with microcephaly has a small head. Cerebral palsy is a diagnosis not usually made at birth; no characteristic physical signs are present. The infant with fragile X syndrome has increased head circumference; long, wide, or protruding ears; a long, narrow face with a prominent jaw; hypotonia; and a high-arched palate.

32. When checking the intravenous (IV) site on a child, the nurse should take which action? a. Look at the site. b. Ask the child if the site hurts. c. Look at the site while palpating the area. d. Take all the tape off, assess the site, and redress.

ANS: C To appropriately check the intravenous (IV) site, the nurse should look at the site and palpate the area. The other options would not be adequate assessments of the site.

Frequent urine tests for specific gravity are required on a 6-month-old infant. What method is the most appropriate way to collect small amounts of urine for these tests? a. Apply a urine collection bag to the perineal area. b. Tape a small medicine cup inside of the diaper. c. Aspirate urine from cotton balls inside the diaper with a syringe without a needle. d. Use a syringe without a needle to aspirate urine from a super absorbent disposable diaper.

ANS: C To obtain small amounts of urine, use a syringe without a needle to aspirate urine directly from the diaper. Diapers with super absorbent gels absorb the urine; if these are used, place a small gauze dressing or cotton balls inside the diaper to collect the urine and aspirate the urine with a syringe. For frequent urine sampling, the collection bag would be too irritating to the childs skin. It is not feasible to tape a small medicine cup to the inside of the diaper; the urine will spill from the cup.

The nurse is teaching parents of a child with cataracts about the upcoming treatment. The nurse should give the parents what information about the treatment of cataracts? a. The treatment may require more than one surgery. b. It is corrected with biconcave lenses that focus rays on the retina. c. Cataracts require surgery to remove the cloudy lens and replace it. d. Treatment is with a corrective lenses; no surgery is necessary.

ANS: C Treatment for cataracts requires surgery to remove the cloudy lens and replace it (with an intraocular lens implant, removable contact lens, or prescription glasses). Treatment for glaucoma may require more than one surgery. Anisometropia is treated with corrective lenses. Myopia is corrected with biconcave lenses that focus rays on the retina.

29. What is a strategy used to minimize scarring with burn injury in a child? a. Applying of drying agents on skin b. Use of loose-fitting garments over healing areas c. Limitation of period without pressure to areas of scarring d. Immobilization of extremities while healing is occurring

ANS: C Uniform pressure to the scar decreases the blood supply and forces the collagen into a more normal alignment. When pressure is removed, blood supply to the scar is immediately increased; therefore, periods without pressure should be brief to avoid nourishment of the hypertrophic tissue. Moisturizing agents are used with massage to help stretch tissue and prevent contractures. Compression garments, not loose-fitting garments, are indicated. Range of motion exercises are done to minimize contractures.

A critically ill child has hyperthermia. The parents ask the nurse to give an antipyretic such as acetaminophen. How should the nurse respond to the parents? a. Febrile seizures can result. b. Antipyretics may cause malignant hyperthermia. c. Antipyretics are of no value in treating hyperthermia. d. Liver damage may occur in critically ill children

ANS: C Unlike with fever, antipyretics are of no value in hyperthermia because the set point is already normal. Cooling measures are used instead. Antipyretics do not cause seizures. Malignant hyperthermia is a genetic myopathy that is triggered by anesthetic agents. Antipyretic agents do not have this effect. Acetaminophen can result in liver damage if too much is given or if the liver is already compromised. Other antipyretics are available, but they are of no value in hyperthermia.

1. The nurse is teaching a client to prevent future urinary tract infections (UTIs). What factor is most important to emphasize as the potential cause? a. Poor hygiene b. Constipation c. Urinary stasis d. Congenital anomalies

ANS: C Urinary stasis is the single most important host factor that influences the development of UTIs. Urine is usually sterile but at body temperature provides an excellent growth medium for bacteria. Poor hygiene can be a contributing cause, especially in females because their short urethras predispose them to UTIs. Urinary stasis then provides a growth medium for the bacteria. Intermittent constipation contributes to urinary stasis. A full rectum displaces the bladder and posterior urethra in the fixed and limited space of the bony pelvis, causing obstruction, incomplete micturition, and urinary stasis. Congenital anomalies can contribute to UTIs, but urinary stasis is the primary factor in many cases.

What is the best method to verify the placement of a nasogastric tube before each use? a. Radiologic confirmation b. Auscultation of injected air c. Aspiration of stomach contents d. Verification of tape placement on tube

ANS: C Visual inspection and pH check of stomach contents is a reliable method of determining placement before each use. Radiologic examination should be obtained after initial placement but would be too cumbersome to do before each use. Auscultation is an unreliable method to confirm tube placement because of the similarity of sounds produced by air in the bronchus, esophagus, or pleural space. Verification of tape placement on the tube can be inaccurate if the tube has moved within the tape or become dislodged from the stomach.

29. What explains physiologically the edema formation that occurs with burns? a. Vasoconstriction b. Reduced capillary permeability c. Increased capillary permeability d. Diminished hydrostatic pressure within capillaries

ANS: C With a major burn, capillary permeability increases, allowing plasma proteins, fluids, and electrolytes to be lost into the interstitial space, causing edema. Maximum edema in a small wound occurs about 8 to 12 hours after injury. In larger injuries, the maximum edema may not occur until 18 to 24 hours later. Vasodilation occurs, causing an increase in hydrostatic pressure.

A 17-month-old child should be expected to be in which stage, according to Piaget? a. Preoperations b. Concrete operations c. Tertiary circular reactions d. Secondary circular reactions

ANS: C A 17-month-old is in the fifth stage of the sensorimotor phase, tertiary circular reactions. The child uses active experimentation to achieve previously unattainable goals. Preoperations is the stage of cognitive development usually present in older toddlers and preschoolers. Concrete operations is the cognitive stage associated with school-age children. The secondary circular reaction stage lasts from about ages 4 to 8 months.

During a well-child visit, the father of a 4-year-old boy tells the nurse that he is not sure if his son is ready for kindergarten. The boys birthday is close to the cut-off date, and he has not attended preschool. What is the nurses best recommendation? a. Start kindergarten. b. Talk to other parents about readiness. c. Perform a developmental screening. d. Postpone kindergarten and go to preschool

ANS: C A developmental assessment with a screening tool that addresses cognitive, social, and physical milestones can help identify children who may need further assessment. A readiness assessment involves an evaluation of skill acquisition. Stating the child should start kindergarten or go to preschool and postpone kindergarten does not address the fathers concerns about readiness for school. Talking to other parents about readiness does not ascertain if the child is ready and does not address the fathers concerns.

The nurse is caring for an unconscious child. Skin care should include which action? a. Avoid use of pressure reduction on bed. b. Massage reddened bony prominences to prevent deep tissue damage. c. Use draw sheet to move child in bed to reduce friction and shearing injuries. d. Avoid rinsing skin after cleansing with mild antibacterial soap to provide a protective barrier.

ANS: C A draw sheet should be used to move the child in the bed or onto a gurney to reduce friction and shearing injuries. Do not drag the child from under the arms. Pressure-reduction devices should be used to redistribute weight. Bony prominences should not be massaged if reddened. Deep tissue damage can occur. Pressure-reduction devices should be used instead. The skin should be cleansed with mild non-alkaline soap or soap-free cleaning agents for routine bathing

The nurse case manager is planning a care conference about a young child who has complex health care needs and will soon be discharged home. Who should the nurse invite to the conference? a. Family and nursing staff b. Social worker, nursing staff, and primary care physician c. Family and key health professionals involved in the child's care d. Primary care physician and key health professionals involved in the child's care

ANS: C A multidisciplinary conference is necessary for coordination of care for children with complex health needs. The family is included, along with key health professionals who are involved in the child's care. The nursing staff can address the child's nursing care needs with the family, but other involved disciplines must be included. The family must be included in the discharge conferences, which allows them to determine what education they will require and the resources needed at home. A member of the nursing staff must be included to review the child's nursing needs.

An adolescent patient has been diagnosed with a vulvovaginal candidiasis (yeast infection). The nurse expects the health care provider to recommend which vaginal cream? a. Premarin b. Estradiol (Estrace) c. Miconazole (Monistat) d. Clindamycin phosphate (Cleocin)

ANS: C A number of antifungal preparations are available for the treatment of vulvovaginal candidiasis infections. Many of these medications (e.g., miconazole [Monistat] and clotrimazole [Gyne- Lotrimin]) are available as over-the-counter (OTC) agents. Premarin and Estrace are estrogen vaginal creams and are used to treat vaginal dryness. Cleocin is an antibacterial vaginal cream used to treat bacterial vaginal infections.

Which genetic term refers to the transfer of all or part of a chromosome to a different chromosome after chromosome breakage? a. Trisomy b. Monosomy c. Translocation d. Nondisjunction

ANS: C A translocation occurs when a part of a chromosome breaks off and attaches to another chromosome. When this occurs in the germ cells, the translocation can be transmitted to the next generation. Trisomy is the condition in which three of a specific chromosome are found rather than the usual two. Monosomy is the condition in which one of a specific chromosome is noted rather than the usual two. The term is not used for males when the normal complement of sex chromosomes (one X and one Y) is present. Nondisjunction is the failure of a chromosome to separate during cell division. Of the resultant daughter cells, one will be trisomic and one will be monosomic.

A nurse is caring for an adolescent hospitalized for cellulitis. The nurse notes that the adolescent experiences many "mood swings" throughout the day. How should the nurse interpret this behavior? a. Requires a referral to a mental health counselor b. Requires some further lab testing c. It is normal behavior d. Related to feelings of depression

ANS: C Adolescents vacillate in their emotional states between considerable maturity and childlike behavior. One minute they are exuberant and enthusiastic; the next minute they are depressed and withdrawn. Because of these mood swings, adolescents are frequently labeled as unstable, inconsistent, and unpredictable, but the behavior is normal. The behavior would not require a referral to a mental health counselor or further lab testing. The mood swings do not indicate depression.

The nurse has determined that an adolescents body mass index (BMI) is in the 90th percentile. What information should the nurse convey to the adolescent? a. The adolescent is overweight. b. The adolescent has maintained weight within the normal range. c. The adolescent is at risk for becoming overweight. d. Nutritional supplementation should occur at least three times per week

ANS: C Adolescents with BMIs between the 85th and 94th percentile for age and gender are at risk for becoming overweight. Adolescents with BMIs greater than the 95th percentile are classified as overweight. Nutritional guidance, not supplementation, is needed.

The nurse is discussing sexuality with the parents of an adolescent girl with moderate cognitive impairment. Which should the nurse consider when dealing with this issue? a. Sterilization is recommended for any adolescent with cognitive impairment. b. Sexual drive and interest are limited in individuals with cognitive impairment. c. Individuals with cognitive impairment need a well-defined, concrete code of sexual conduct. d. Sexual intercourse rarely occurs unless the individual with cognitive impairment is sexually abused

ANS: C Adolescents with moderate cognitive impairment may be easily persuaded and lack judgment. A well-defined, concrete code of conduct with specific instructions for handling certain situations should be laid out for the adolescent. Permanent contraception by sterilization presents moral and ethical issues and may have psychological effects on the adolescent. It may be prohibited in some states. The adolescent needs to have practical sexual information regarding physical development and contraception. Cognitively impaired individuals may desire to marry and have families. The adolescent needs to be protected from individuals who may make intimate advances.

The nurse is discussing sexuality with the parents of an adolescent girl who has a moderate cognitive impairment. What factor should the nurse consider when dealing with this issue? a. Sterilization is recommended for any adolescent with cognitive impairment. b. Sexual drive and interest are very limited in individuals with cognitive impairment. c. Individuals with cognitive impairment need a well-defined, concrete code of sexual conduct. d. Sexual intercourse rarely occurs unless the individual with cognitive impairment is sexually abused.

ANS: C Adolescents with moderate cognitive impairment may be easily persuaded and lack judgment. A well-defined, concrete code of conduct with specific instructions for handling certain situations should be defined for the adolescent. Permanent contraception by sterilization presents moral and ethical issues and may have psychologic effects on the adolescent. It may be prohibited in some states. The adolescent needs to have practical sexual information regarding physical development and contraception. Cognitively impaired individuals may desire to marry and have families. The adolescent needs to be protected from individuals who may make intimate advances.

A child with cyanide poisoning has been admitted to the emergency department. What antidote does the nurse anticipate being prescribed for the child? a. Atropine b. Glucagon c. Amyl nitrate d. Naloxone (Narcan)

ANS: C Amyl nitrate is the antidote for cyanide poisoning. Atropine is an antidote for organophosphate poisoning, glucagon is an antidote for a beta-blocker poisoning, and naloxone (Narcan) is an antidote for an opioid poisoning.

The nurse is talking to the parent of a 5-year-old child who refuses to go to sleep at night. What intervention should the nurse suggest in helping the parent to cope with this sleep disturbance? a. Establish a consistent punishment if the child does not go to bed when told. b. Allow the child to fall asleep in a different room and then gently move the child to his or her bed. c. Establish limited rituals that signal readiness for bedtime. d. Allow the child to watch television until almost asleep.

ANS: C An appropriate intervention for a child who resists going to bed is to establish limited rituals such as a bath or story that signal readiness for bed and consistently follow through with the ritual. Punishing the child will not alleviate the resistance problem and may only add to the frustration. Allowing the child to fall asleep in a different room and to watch television to fall asleep are not recommended approaches to sleep resistance.

The nurse is teaching parents of a child with chronic renal failure (CRF) about the use of recombinant human erythropoietin (rHuEPO) subcutaneous injections. Which statement indicates the parents have understood the teaching? a. "These injections will help with the hypertension." b. "We're glad the injections only need to be given once a month." c. "The red blood cell count should begin to improve with these injections." d. "Urine output should begin to improve with these injections."

ANS: C Anemia in children with CRF is related to decreased production of erythropoietin. Recombinant human erythropoietin (rHuEPO) is being offered to these children as thrice-weekly or weekly subcutaneous injections and is replacing the need for frequent blood transfusions. The parents understand the teaching if they say that the red blood cell count will begin to improve with these injections.

What statement is true about smoking in college students? a. The rate of smoking cigarettes is declining. b. Smokeless tobacco use is rising dramatically. c. Regular cigar use is becoming more common. d. Students in the health professions do not smoke.

ANS: C Approximately 8.5% of college students smoke cigars on a regular basis. Among college students, the rate of cigarette smoking is rising. At last report, 28.5% of this group smoked cigarettes. Use of smokeless tobacco is declining overall. Students in the health professions do smoke.

What is a major premise of family-centered care? a. The child is the focus of all interventions. b. Nurses are the authorities in the childs care. c. Parents are the experts in caring for their child. d. Decisions are made for the family to reduce stress.

ANS: C As parents become increasingly responsible for their children, they are the experts. It is essential that the health care team recognize the familys expertise. In family-centered care, consistent attention is given to the effects of the childs chronic illness on all family members, not just the child. Nurses are adjuncts in the childs care. The nurse builds alliances with parents. Family members are involved in decision making about the childs physical care.

Which play is most typical of the preschool period? a. Solitary b. Parallel c. Associative d. Team

ANS: C Associative play is group play in similar or identical activities but without rigid organization or rules. Solitary play is that of infants. Parallel play is that of toddlers. School-age children play in teams

A nurse is reviewing hormone changes that occur during adolescence. What is the hormone responsible for the growth of beard, mustache, and body hair in the male? a. Estrogen b. Pituitary c. Androgen d. Progesterone

ANS: C Beard, mustache, and body hair on the chest, upward along the linea alba, and sometimes on other areas (e.g., back and shoulders) appears in males and is androgen dependent. Estrogen and progesterone are produced by the ovaries in the female and do not contribute to body hair appearance in the male. The pituitary hormone does not have any relationship to body hair appearance in the male.

A nurse is assessing a preschool-age child and notes the child exhibits magical thinking. According to Piaget, which describes magical thinking? a. Events have cause and effect. b. God is like an imaginary friend. c. Thoughts are all-powerful. d. If the skin is broken, the child's insides will come out.

ANS: C Because of their egocentrism and transductive reasoning, preschoolers believe that thoughts are all-powerful. Cause-and-effect implies logical thought, not magical thinking. Thinking God is like an imaginary friend is an example of concrete thinking in a preschooler's spiritual development. Thinking that if the skin is broken, the child's insides will come out is an example of concrete thinking in development of body image.

The school nurse suspects a testicular torsion in a young adolescent student. What action should the nurse take? a. Place a warm moist pack on the scrotal area. b. Instruct the adolescent to lie down and elevate the legs. c. Refer the adolescent for immediate medical evaluation. d. Suggest that the adolescent wear a scrotum-protecting guard.

ANS: C Because torsion may result from trauma to the scrotum, school nurses are likely to encounter such injuries and should refer the child or adolescent for medical evaluation immediately. It would not be appropriate to apply warmth, elevate the legs, or tell the adolescent to wear a scrotum-protecting guard because these actions could delay treatment.

A female school-age child asks the school nurse, How many pounds should I expect to gain in a year? The nurse should give which response? a. You will gain about 2.4 to 4.6 lb per year b. You will gain about 3.4 to 5.6 lb per year. c. You will gain about 4.4 to 6.6 lb per year. d. You will gain about 5.5 to 7.6 lb per year.

ANS: C Between the ages of 6 and 12 years, children will almost double in weight, increasing 2 to 3 kg (4.4 to 6.6 lb) per year.

What statement accurately describes physical development during the school-age years? a. The childs weight almost triples. b. Muscles become functionally mature. c. Boys and girls double strength and physical capabilities. d. Fat gradually increases, which contributes to childrens heavier appearance.

ANS: C Boys and girls double both strength and physical capabilities. Their consistent refinement in coordination increases their poise and skill. In middle childhood, growth in height and weight occurs at a slower pace. Between the ages of 6 and 12 years, children grow 5 cm/yr and gain 3 kg/yr. Their weight will almost double. Although the strength increases, muscles are still functionally immature when compared with those of adolescents. This age group is more easily injured by overuse. Children take on a slimmer look with longer legs in middle childhood.

In the clinic waiting room, a nurse observes a parent showing an 18-month-old child how to make a tower out of blocks. What should the nurse recognize in this situation? a. Blocks at this age are used primarily for throwing b. Toddlers are too young to imitate the behavior of others c. Toddlers are capable of building a tower of blocks d. Toddlers are too young to build a tower of blocks

ANS: C Building with blocks is a good parent-child interaction. The 18-month-old child is capable of building a tower of three or four blocks. The ability to build towers of blocks usually begins at age 15 months. With ongoing development, the child is able to build taller towers. The 18-month-old child imitates others around him or her.

Developmentally, what should most children at age 12 months be able to do? a. Use a spoon adeptly b. Relinquish the bottle voluntarily c. Eat the same food as the rest of the family d. Reject all solid food in preference to the bottle

ANS: C By age 12 months, most children are eating the same food that is prepared for the rest of the family. Using a spoon usually is not mastered until age 18 months. The parents should be engaged in weaning a child from a bottle if that is the source of liquid. Toddlers should be encouraged to drink from a cup at the first birthday and be weaned from the bottle totally by 14 months. The child should be weaned from a milk- or formula-based diet to a balanced diet that includes iron-rich sources of food.

At what age do most children have an adult concept of death as being inevitable, universal, and irreversible? a. 4 to 5 years b. 6 to 8 years c. 9 to 11 years d. 12 to 16 years

ANS: C By age 9 to 11 years, children have an adult concept of death. They realize that it is inevitable, universal, and irreversible.

Which describes moral development in younger school-age children? a. The standards of behavior now come from within themselves. b. They do not yet experience a sense of guilt when they misbehave. c. They know the rules and behaviors expected of them but do not understand the reasons behind them. d. They no longer interpret accidents and misfortunes as punishment for misdeeds.

ANS: C Children who are ages 6 and 7 years know the rules and behaviors expected of them but do not understand the reasons for these rules and behaviors. Young children do not believe that standards of behavior come from within themselves, but that rules are established and set down by others. Younger school-age children learn standards for acceptable behavior, act according to these standards, and feel guilty when they violate them. Misfortunes and accidents are viewed as punishment for bad acts.

A nurse is preparing to perform a dressing change on a 6-year-old child with mild cognitive impairment (CI) who sustained a minor burn. Which strategy should the nurse use to prepare the child for this procedure? a. Verbally explain what will be done. b. Have the child watch a video on dressing changes. c. Demonstrate a dressing change on a doll. d. Explain the importance of keeping the burn area clean.

ANS: C Children with CI have a marked deficit in their ability to discriminate between two or more stimuli because of difficulty in recognizing the relevance of specific cues. However, these children can learn to discriminate if the cues are presented in an exaggerated, concrete form and if all extraneous stimuli are eliminated. Therefore, demonstration is preferable to verbal explanation, and learning should be directed toward mastering a skill rather than understanding the scientific principles underlying a procedure. Watching a video would require the use of both visual and auditory stimulation and might produce overload in the child with mild cognitive impairment. Explaining the importance of keeping the burn area clean would be too abstract for the child.

The child with Down syndrome should be evaluated for which condition before participating in some sports? a. Hyperflexibility b. Cutis marmorata c. Atlantoaxial instability d. Speckling of iris (Brushfield spots)

ANS: C Children with Down syndrome are at risk for atlantoaxial instability. Before participating in sports that put stress on the head and neck, a radiologic examination should be done. Hyperflexibility, cutis marmorata, and speckling of iris (Brushfield spots) are characteristic of Down syndrome, but they do not affect the child's ability to participate in sports.

The parents of 9-year-old twin children tell the nurse, They have filled up their bedroom with collections of rocks, shells, stamps, and bird nests. The nurse should recognize that this is which? a. Indicative of giftedness b. Indicative of typical twin behavior c. Characteristic of cognitive development at this age d. Characteristic of psychosocial development at this age

ANS: C Classification skills involve the ability to group objects according to the attributes they have in common. School-age children can place things in a sensible and logical order, group and sort, and hold a concept in their mind while they make decisions based on that concept. Individuals who are not twins engage in classification at this age. Psychosocial behavior at this age is described according to Eriksons stage of industry versus inferiority.

The nurse asks the mother of a child with a chronic illness many questions as part of the assessment. The mother answers several questions, then stops and says, I dont know why you ask me all this. Who gets to know this information? The nurse should respond in what manner? a. Determine why the mother is so suspicious. b. Determine what the mother does not want to tell. c. Explain who will have access to the information. d. Explain that everything is confidential and that no one else will know what is said.

ANS: C Communication with the family should not be invasive. The nurse needs to explain the importance of collecting the information, its applicability to the childs care, and who will have access to the information. The mother is not being suspicious and is not necessarily withholding important information. She has a right to understand how the information she provides will be used. The nurse will need to share, through both oral and written communication, clinically relevant information with other involved health professionals.

The nurses role in facilitating successful childrearing in unmarried teenage mothers includes what? a. Facilitating marriage between the mother and father of the baby b. Teaching the adolescent the long-term needs of the growing child c. Providing information and feedback about positive parenting skills d. Encouraging the infants grandmother to take responsibility for care

ANS: C Competence in a teenage mother is increased when feedback is provided about positive parenting skills and use of community resources. The nurse can identify and refer the mother to programs such as support groups for adolescent mothers, infant stimulation programs, and parenting programs. Facilitating marriage between the mother and the father of the baby may produce additional stress and detract from their ability to care for the infant. Encouraging the infants grandmother to take responsibility for care would decrease the mothers ability to develop successful childrearing behaviors. Supportive families can provide assistance to enable the teenage mother to complete school. Many adolescents do not have a future perspective for themselves. The nurse includes information on normal infant development to aid the mother in having reasonable expectations.

The school nurse recognizes that pubertal delay in boys is considered if no enlargement of the testes or scrotal changes have occurred by what age? a. 11 1/2 to 12 years b. 12 1/2 to 13 years c. 13 1/2 to 14 years d. 14 1/2 to 15 years

ANS: C Concerns about pubertal delay should be considered for boys who exhibit no enlargement of the testes or scrotal changes by ages 13 1/2 to 14 years or if genital growth is not complete 4 years after the testicles begin to enlarge

A school nurse is teaching a group of preadolescent boys about puberty. By which age should concerns about pubertal delay be considered? a. 12 years b. 13 years c. 14 years d. 15 years

ANS: C Concerns about pubertal delay should be considered for boys who exhibit no enlargement of the testes or scrotal changes from 13 1/2 to 14 years. Ages 12 to 13 1/2 years is too young for initial concern.

When caring for a newborn with Down syndrome, what should the nurse be aware is the most common congenital anomaly associated with Down syndrome? a. Hypospadias b. Pyloric stenosis c. Congenital heart disease d. Congenital hip dysplasia

ANS: C Congenital heart malformations, primarily septal defects, are the most common congenital anomaly in Down syndrome. Hypospadias, pyloric stenosis, and congenital hip dysplasia are not frequent congenital anomalies associated with Down syndrome.

During the first 4 days of hospitalization, Eric, age 18 months, cried inconsolably when his parents left him, and he refused the staff's attention. Now the nurse observes that Eric appears to be "settled in" and unconcerned about seeing his parents. The nurse should interpret this as which statement? a. He has successfully adjusted to the hospital environment. b. He has transferred his trust to the nursing staff. c. He may be experiencing detachment, which is the third stage of separation anxiety. d. Because he is "at home" in the hospital now, seeing his mother frequently will only start the cycle again.

ANS: C Detachment is a behavior manifestation of separation anxiety. Superficially it appears that the child has adjusted to the loss. Detachment is a sign of resignation, not contentment. Parents should be encouraged to be with their child. If parents restrict visits, they may begin a pattern of misunderstanding the child's cues and not meeting his needs.

Which should the nurse recommend for the diet of a child with chronic renal failure? a. High in protein b. Low in vitamin D c. Low in phosphorus d. Supplemented with vitamins A, E, and K

ANS: C Dietary phosphorus is controlled by the reduction of protein and milk intake to prevent or control the calcium-phosphorus imbalance. Protein should be limited in chronic renal failure to decrease intake of phosphorus. Vitamin D therapy is administered in chronic renal failure to increase calcium absorption. Supplementation of vitamins A, E, and K is not part of dietary management in chronic renal disease.

The nurse is caring for a child with a decubiti on the buttocks. The nurse notes that the dressing covering the decubiti is loose. What action should the nurse implement? a. Retape the dressing. b. Remove the dressing. c. Change the dressing. d. Reinforce the dressing.

ANS: C Dressings should always be changed when they are loose or soiled. They should be changed more frequently in areas where contamination is likely (e.g., sacral area, buttocks, tracheal area). The dressing should not be retaped, removed, or reinforced.

A child has been diagnosed with enuresis. TCA imipramine (Tofranil) has been prescribed for the child. The nurse understands that this medication is in which category? a. Antidepressant b. Antidiuretic c. Antispasmodic d. Analgesic

ANS: C Drug therapy is increasingly being prescribed to treat enuresis. Three types of drugs are used: tricyclic antidepressants (TCAs), antidiuretics, and antispasmodics. The selection depends on the interpretation of the cause. The drug used most frequently is the TCA imipramine (Tofranil), which exerts an anticholinergic action in the bladder to inhibit urination. Tofranil is in the antispasmodic category. Analgesics are not used to treat enuresis.

The nurse understands that medications delivered by which route are more likely to cause a drug reaction? a. Oral b. Topical c. Intravenous d. Intramuscular

ANS: C Drugs administered by the intravenous route are more likely to cause a reaction than the oral, topical, or intramuscular route.

What statement best describes fear in school-age children? a. Increasing concerns about bodily safety overwhelm them. b. They should be encouraged to hide their fears to prevent ridicule by peers. c. Most of the new fears that trouble them are related to school and family. d. Children with numerous fears need continuous protective behavior by parents to eliminate these fears.

ANS: C During the school-age years, children experience a wide variety of fears, but new fears related predominantly to school and family bother children during this time. Parents and other persons involved with children should discuss childrens fear with them individually or as a group activity. Sometimes school-age children hide their fears to avoid being teased. Hiding the fears does not end them and may lead to phobias.

What is a characteristic of a toddlers language development at age 18 months? a. Vocabulary of 25 words b. Use of holophrases c. Increasing level of understanding d. Approximately one third of speech understandable

ANS: C During the second year of life, the understanding and understanding of speech increase to a level far greater than the childs vocabulary. This is also true for bilingual children, who are able to achieve this linguistic milestone in both languages. An 18-month-old child has a vocabulary of approximately 10 words. At this age, the child does not use the one-word sentences that are characteristic of 1-year-old children. The child has a very limited vocabulary of single words that are comprehensible.

Although a 14-month-old girl received a shock from an electrical outlet recently, her parent finds her about to place a paper clip in another outlet. Which is the best interpretation of this behavior? a. Her cognitive development is delayed. b. This is typical behavior because toddlers are not very developed. c. This is typical behavior because of toddlers inability to transfer remembering to new situations. d. This is not typical behavior because toddlers should know better than to repeat an act that caused pain.

ANS: C During the tertiary circular reactions stage, children have only a rudimentary sense of the classification of objects. The appearance of an object denotes its function for these children. The slot of an outlet is for putting things into. This is typical behavior for a toddler, who is only somewhat aware of a causal relation between events. Her cognitive development is appropriate for her age

Although a 14-month-old girl received a shock from an electric outlet recently, her parent finds her about to place a paper clip in another outlet. Which is the best interpretation of this behavior? a. Her cognitive development is delayed. b. This is typical behavior because toddlers are not very developed. c. This is typical behavior because of the inability to transfer knowledge to new situations. d. This is not typical behavior because toddlers should know better than to repeat an act that caused pain.

ANS: C During the tertiary circular reactions stage, children have only a rudimentary sense of the classification of objects. The appearance of an object denotes its function for these children. The slot of an outlet is for putting things into. Her cognitive development is appropriate for her age. Trying to put things into an outlet is typical behavior for a toddler. Only some awareness exists of a causal relation between events.

In some genetically susceptible children, anesthetic agents can trigger malignant hyperthermia. The nurse should be alert in observing that, in addition to an increased temperature, what is an early sign of this disorder? a. Apnea b. Bradycardia c. Muscle rigidity d. Decreased blood pressure

ANS: C Early signs of malignant hyperthermia include tachycardia, increasing blood pressure, tachypnea, mottled skin, and muscle rigidity. Apnea is not a sign of malignant hyperthermia. Tachycardia, not bradycardia, is an early sign of malignant hyperthermia. Increased blood pressure, not decreased blood pressure, is characteristic of malignant hyperthermia.

What should injury prevention efforts emphasize during the preschool period? a. Constant vigilance and protection b. Punishment for unsafe behaviors c. Education for safety and potential hazards d. Limitation of physical activities

ANS: C Education for safety and potential hazards is appropriate for preschoolers because they can begin to understand dangers. Constant vigilance and protection is not practical at this age because preschoolers are becoming more independent. Punishment may make children scared of trying new things. Limitation of physical activities is not appropriate.

Tepid water or sponge baths are indicated for hyperthermia in children. What is the priority nursing action? a. Add isopropyl alcohol to the water. b. Direct a fan on the child in the bath. c. Stop the bath if the child begins to chill. d. Continue the bath for 5 minutes.

ANS: C Environmental measures such as sponge baths can be used to reduce temperature if tolerated by the child and if they do not induce shivering. Shivering is the body's way of maintaining the elevated set point. Compensatory shivering increases metabolic requirements above those already caused by the fever. Ice water and isopropyl alcohol are potentially dangerous solutions. Fans should not be used because of the risk of the child developing vasoconstriction, which defeats the purpose of the cooling measures. Little blood is carried to the skin surface, and the blood remains primarily in the viscera to become heated. The child is placed in a tub of tepid water for 20 to 30 minutes.

The nurse is teaching the parent about the diet of a child experiencing severe edema associated with acute glomerulonephritis. Which information should the nurse include in the teaching? a. "You will need to decrease the number of calories in your child's diet." b. "Your child's diet will need an increased amount of protein." c. "You will need to avoid adding salt to your child's food." d. "Your child's diet will consist of low-fat, low-carbohydrate foods."

ANS: C For most children, a regular diet is allowed, but it should contain no added salt. The child should be offered a regular diet with favorite foods. Severe sodium restrictions are not indicated.

A child has just been diagnosed with fragile X syndrome. What is fragile X syndrome? a. A chromosomal defect affecting females only b. A chromosomal defect that follows the pattern of X-linked recessive disorders c. The second most common genetic cause of cognitive impairment d. The most common cause of noninherited cognitive impairment

ANS: C Fragile X syndrome is the second most common cause of cognitive impairment after Down syndrome. Fragile X primarily affects males, and follows the inheritance pattern of X-linked dominant with reduced penetrance. This is in distinct contrast to the classic X-linked recessive pattern in which all carrier females are normal, all affected males have symptoms of the disorder, and no males are carriers.

A nurse is teaching parents of kindergarten children general guidelines to assist their children in school. Which statement by the parents indicates they understand the teaching? a. "We will only meet with the teacher if problems occur." b. "We will discourage hobbies so our child focuses on schoolwork." c. "We will plan a trip to the library as often as possible." d. "We will expect our child to make all As in school."

ANS: C General guidelines for parents to help their child in school include sharing an interest in reading. The library should be used frequently, and books the child is reading should be discussed. Hobbies should be encouraged. The parents should not expect all As. They should focus on growth more than grades.

Which is the most appropriate nursing diagnosis for the child with acute glomerulonephritis? a. Risk for Injury related to malignant process and treatment b. Fluid Volume Deficit related to excessive losses c. Fluid Volume Excess related to decreased plasma filtration d. Fluid Volume Excess related to fluid accumulation in tissues and third spaces

ANS: C Glomerulonephritis has a decreased filtration of plasma, which results in an excessive accumulation of water and sodium that expands plasma and interstitial fluid volumes, leading to circulatory congestion and edema. No malignant process is involved in acute glomerulonephritis. A fluid volume excess is found. The fluid accumulation is secondary to the decreased plasma filtration.

Which nursing intervention is especially helpful in assessing parental guilt when a disability or chronic illness is diagnosed? a. Ask the parents if they feel guilty. b. Discuss guilt only after the parents mention it. c. Discuss the meaning of the parents' religious and cultural background. d. Observe for signs of overprotectiveness.

ANS: C Guilt may be associated with cultural or religious beliefs. Some parents are convinced that they are being punished for some previous misdeed. Others may see the disorder as a sacrifice sent by God to test their religious beliefs. The nurse can help the parents explore their religious beliefs. The parents may not be able to identify the feelings of guilt. It would be appropriate for the nurse to explore their adjustment responses. Overprotectiveness is a parental response during the adjustment phase. The parents fear letting the child achieve any new skill and avoid all discipline.

A 14-year-old boy is of normal weight, and his parents are concerned about bilateral breast enlargement. The nurses discussion of this should be based on what? a. The presence of too much body fat b. Symptom that a hormonal imbalance is present c. Most likely part of normal pubertal development d. Indication that he is developing precocious puberty

ANS: C Gynecomastia is common during midpuberty in about one third of boys. For most, the breast enlargement disappears within 2 years. Although breast enlargement in overweight children can indicate too much body fat, in children of normal body weight, it is a normal occurrence. If the gynecomastia persists beyond 2 years, then a hormonal cause may need to be investigated. Precocious puberty is the early onset of puberty, before age 9 years in boys.

The school nurse tells adolescents in the clinic that confidentiality and privacy will be maintained unless a life-threatening situation arises. How should this practice be interpreted? a. Not appropriate in a school setting b. Never appropriate because adolescents are minors c. Important in establishing trusting relationships d. Suggestive that the nurse is meeting his or her own needs

ANS: C Health professionals who work with adolescents should consider adolescents' increasing independence and responsibility while maintaining privacy and ensuring confidentiality. However, in some circumstances, such as self-destructive behavior or maltreatment by others, they are not able to maintain confidentiality. Confidentiality and privacy are necessary to build trust with this age group. The nurse must be aware of the limits placed on confidentiality by local jurisdiction.

A mother of a 5-year-old child, with complex health care needs and cared for at home, expresses anxiety about attending a kindergarten graduation exercise of a neighbor's child. The mother says, "I wish it could be my child graduating from kindergarten." What should the nurse recognize the mother is experiencing? a. Abnormal anxiety b. Ineffective coping c. Chronic sorrow d. Denial

ANS: C Home care nurses should be aware that parents may experience chronic sorrow as a parental stressor. Chronic sorrow as a normal grief response is associated with a living loss (the loss of a healthy child) that is cyclical in nature. This is a normal response and does not indicate abnormal anxiety, ineffective coping, or denial.

What goal is most important when caring for a child with anorexia nervosa (AN)? a. Limit fluid intake. b. Prevent depression. c. Correct malnutrition. d. Encourage weight gain.

ANS: C In children diagnosed with AN or bulimia nervosa, the priority consideration is to correct the malnutrition. Severe malnutrition, electrolyte disturbances, vital sign abnormalities, and psychiatric disorders may be present. Careful monitoring is necessary to avoid complications. Often fluid intake is restricted by individuals with AN. Fluid balance must be restored. Preventing depression is important, but the correction of potentially life-threatening malnutrition takes precedence. After the initial malnutrition is corrected, then a plan is established for nutritional therapy

In girls, what is the initial indication of puberty? a. Menarche b. Growth spurt c. Breast development d. Growth of pubic hair

ANS: C In most girls, the initial indication of puberty is the appearance of breast buds, an event known as thelarche. The usual sequence of secondary sexual characteristic development in girls is breast changes, a rapid increase in height and weight, growth of pubic hair, appearance of axillary hair, menstruation (menarche), and abrupt deceleration of linear growth.

Descriptions of young people with anorexia nervosa (AN) often include which criteria? a. Impulsive b. Extroverted c. Perfectionist d. Low achieving

ANS: C Individuals with AN are described as striving for perfection, which may manifest in other compulsive disorders. They are also academically high achievers. Impulsive and extroverted personalities are more characteristic of bulimia nervosa.

What behavior is the nurse most likely to assess in an adolescent with anorexia nervosa (AN)? a. Eats in secrecy b. Uses food as a coping mechanism c. Has a marked preoccupation with food d. Lacks awareness of how eating affects weight loss

ANS: C Individuals with AN display great interest in food. They prepare meals for others, talk about food, and hoard food. During meals, food play may occur to appear as if the person is eating. Persons with AN consume a small amount of food, so they have no need to eat in secrecy. Individuals with bulimia nervosa (BN) usually binge privately. Food is not used as a coping mechanism in AN, as is common in BN. Individuals with AN know about the relationship between calorie intake and calorie expenditure. They can regulate intake and then exercise to not gain or to lose weight.

How are young people with anorexia nervosa often described? a. Independent b. Disruptive c. Conforming d. Low achieving

ANS: C Individuals with anorexia nervosa are described as perfectionist, academically high achievers, conforming, and conscientious. "Independent," disruptive," and "low achieving" are not part of the behavioral characteristics of anorexia nervosa.

After a treatment plan for acne has been initiated, which time period should the nurse explain to an adolescent before improvement will be seen? a. 2 to 4 weeks b. 4 to 6 weeks c. 6 to 8 weeks d. 8 to 10 weeks

ANS: C Inform patients that after a treatment plan for acne has been initiated, it will take 6 to 8 weeks to appreciate improvement in their skin

The school nurse is teaching an adolescent about social networking and texting on phones. What statement by the adolescent indicates a need for further teaching? a. Social networking can help me develop interpersonal skills. b. I will have an opportunity to interact with people like myself. c. My text messaging during class time in school will not cause any disruption. d. I should be cautious, as the online environment can create opportunities for cyberbullying.

ANS: C Internet chatrooms and social networking sites have created a more public arena for trying out identities and developing interpersonal skills with a wider network of people, occasionally with anonymity. This can create opportunities for young people who have a limited access to friends (because of rural location, shyness, or rare chronic conditions) to interact with people like themselves. Both the online and text environment can create opportunities for cyberbullying, in which teens engage in insults, harassment, and publicly humiliating statements online or on cell phones. Text messaging and instant messaging via cell phones has become a common activity and can sometimes be disruptive during school. If the adolescent indicates it will not be disruptive, further teaching is needed.

A preschool child needs a dressing change. To prepare the child, what strategy should the nurse implement? a. Explain the procedure using medical terminology. b. Plan a 30-minute teaching session. c. Give choices when possible but avoid delay. d. Allow time after the procedure for questions and discussion.

ANS: C Involving children helps to gain their cooperation. Permitting choices gives them some measure of control. The other options would not be appropriate for a preschool child.

Which is the most frequent source of acute childhood lead poisoning? a. Folk remedies b. Unglazed pottery c. Lead-based paint d. Cigarette butts and ashes

ANS: C Lead-based paint in houses built before 1978 is the most frequent source of lead poisoning. Some folk remedies and unglazed pottery may contain lead, but they are not the most frequent source. Cigarette butts and ashes do not contain lead.

The nurse is caring for an adolescent who had an external fixator placed after suffering a fracture of the wrist during a bicycle accident. Which statement by the adolescent should be expected about separation anxiety? a. "I wish my parents could spend the night with me while I am in the hospital." b. "I think I would like for my siblings to visit me but not my friends." c. "I hope my friends don't forget about visiting me." d. "I will be embarrassed if my friends come to the hospital to visit."

ANS: C Loss of peer-group contact may pose a severe emotional threat to an adolescent because of loss of group status, so friends visiting are an important aspect of hospitalization for an adolescent. Most adolescents do not need a parent to spend the night during hospitalization and sometimes view the hospitalization as a welcome event. Adolescents would be more concerned about friends visiting than siblings. Adolescents want visitors to keep control and maintain social status among their group of peers.

The nurse should know what about Lyme disease? a. Very difficult to prevent b. Easily treated with oral antibiotics in stages 1, 2, and 3 c. Caused by a spirochete that enters the skin through a tick bite d. Common in geographic areas where the soil contains the mycotic spores that cause the disease

ANS: C Lyme disease is caused by Borrelia burgdorferi, a spirochete spread by ticks. The early characteristic rash is erythema migrans. Tick bites should be avoided by entering tick-infested areas with caution. Light-colored clothing should be worn to identify ticks easily. Long-sleeve shirts and long pants tucked into socks should be worn. Early treatment of the erythema migrans (stage 1) can prevent the development of Lyme disease. Lyme disease is caused by a spirochete, not mycotic spores.

A school-age child, admitted for intravenous antibiotic therapy for osteomyelitis, reports difficulty in going to sleep at night. Which intervention should the nurse implement to assist the child in going to sleep at bedtime? a. Request a prescription for a sleeping pill. b. Allow the child to stay up late and sleep late in the morning. c. Create a schedule similar to the one the child follows at home. d. Plan passive activities in the morning and interactive activities right before bedtime.

ANS: C Many children obtain significantly less sleep in the hospital than at home; the primary causes are a delay in sleep onset and early termination of sleep because of hospital routines. One technique that can minimize the disruption in the child's routine is establishing a daily schedule. This approach is most suitable for non-critically ill school-age and adolescent children who have mastered the concept of time. It involves scheduling the child's day to include all those activities that are important to the child and nurse, such as treatment procedures, schoolwork, exercise, television, playroom, and hobbies. The school-age child with osteomyelitis would benefit from a schedule similar to the one followed at home. Requesting a prescription for a sleeping pill would be inappropriate and allowing the child to stay up late and sleep late would not be keeping the child in a routine followed at home. Passive activities in the morning and interactive activities at bedtime should be reversed; it would be better to keep the child active in the morning hours and plan quiet activities at bedtime.

Which snack should the nurse recommend parents offer to their slightly overweight preschool child? a. Carbonated beverage b. 10% fruit juice c. Low fat chocolate milk d. Whole milk

ANS: C Milk and dairy products are excellent sources of calcium and vitamin D (fortified). Low-fat milk may be substituted, so the quantity of milk may remain the same while limiting fat intake overall. Parents should be educated regarding non-nutritious fruit drinks, which usually contain less than 10% fruit juice yet are often advertised as healthy and nutritious; sugar content is dramatically increased and often precludes an adequate intake of milk by the child. In young children, intake of carbonated beverages that are acidic or that contain high amounts of sugar is also known to contribute to dental caries. Low fat milk should be substituted for whole milk if the child is slightly overweight.

What is an appropriate method for administering oral medications that are bitter to an infant or small child? a. Mix in a bottle of formula or milk. b. Mix with any food the child is going to eat. c. Mix with a small amount (1 teaspoon) of a sweet-tasting substance such as jam or ice cream. d. Mix with large amounts of water to dilute medication sufficiently.

ANS: C Mix the drug with a small amount (about 1 teaspoon) of sweet-tasting substance. This will make the medication more palatable to the child. The medication should be mixed with only a small amount of food or liquid. If the child does not finish drinking or eating, it is difficult to determine how much medication was consumed. Medication should not be mixed with essential foods and milk. The child may associate the altered taste with the food and refuse to eat in the future.

A young child has an intelligence quotient (IQ) of 45. The nurse should document this finding as: a. within the lower limits of the range of normal intelligence. b. mild cognitive impairment but educable. c. moderate cognitive impairment but trainable. d. severe cognitive impairment and completely dependent on others for care.

ANS: C Moderate cognitive impairment IQs range between 35 and 55. The lower limit of normal intelligence is approximately 70. Individuals with IQs of 50 to 70 are considered to have mild cognitive impairment but educable. An IQ of 20 to 40 results in severe cognitive impairment.

A 12-year-old male has short stature because of a constitutional growth delay. What should the nurse be the most concerned about? a. Proper administration of thyroid hormone b. Proper administration of human growth hormones c. Child's self-esteem and sense of competence d. Helping child understand that his height is most likely caused by chronic illness and is not his fault

ANS: C Most cases of constitutional growth delay are caused by simple constitutional delay of puberty, and the child can be assured that normal development will eventually take place. Listening to distressed adolescents and conveying interest and concern are important interventions for these children and adolescents. They should be encouraged to focus on the positive aspects of their bodies and personalities. Thyroid hormones and human growth hormones would not be beneficial in a constitutional growth delay. A constitutional growth delay is not caused by a chronic illness.

What is characteristic of dishonest behavior in children ages 8 to 10 years? a. Cheating during games is now more common. b. Stealing can occur because their sense of property rights is limited. c. Lying is used to meet expectations set by others that they have been unable to attain. d. Dishonesty results from the inability to distinguish between fact and fantasy.

ANS: C Older school-age children may lie to meet expectations set by others to which they have been unable to measure up. Cheating usually becomes less frequent as the child matures. Young children may lack a sense of property rights; older children may steal to supplement an inadequate allowance, or it may be an indication of serious problems. In this age group, children are able to distinguish between fact and fantasy.

A parent taking a preschool child to school on the first day asks the nurse, What do I do if my child wants me to stay? What is an appropriate response by the nurse? a. It is better if you do not stay. b. It is best to stay and participate in the activities. c. It is OK to stay part of the first day, but be inconspicuous. d. It would be better to have a good friend take your child to class the first day

ANS: C On the first day of preschool, in some instances, it is helpful for parents to remain for at least part of the first day until the child is comfortable. If parents stay, they should be available to the child but inconspicuous. It would not be appropriate not to stay, to have someone else take the child to school, or to stay and participate in activities

A nurse planning care for a school-age child should take into account that which thought process is seen at this age? a. Animism b. Magical thinking c. Ability to conserve d. Thoughts are all-powerful

ANS: C One cognitive task of school-age children is mastering the concept of conservation. At an early age (5 to 7 years), children grasp the concept of reversibility of numbers as a basis for simple mathematics problems (e.g., 2 + 4 = 6 and 6 - 4 = 2). They learn that simply altering their arrangement in space does not change certain properties of the environment, and they are able to resist perceptual cues that suggest alterations in the physical state of an object. Animism, magical thinking, and believing that thoughts are all powerful are thought processes seen in preschool children

A group of boys ages 9 and 10 years have formed a "boys-only" club that is open to neighborhood and school friends who have skateboards. How should this be interpreted? a. Behavior that encourages bullying and sexism b. Behavior that reinforces poor peer relationships c. Characteristic of social development at this age d. Characteristic of children who later are at risk for membership in gangs

ANS: C One of the outstanding characteristics of middle childhood is the creation of formalized groups or clubs. Peer-group identification and association are essential to a child's socialization. Poor relationships with peers and a lack of group identification can contribute to bullying. A boys-only club does not have a direct correlation with later gang activity.

Why are imaginary playmates beneficial to the preschool child? a. Take the place of social interactions b. Take the place of pets and other toys c. Become friends in times of loneliness d. Accomplish what the child has already successfully accomplished

ANS: C One purpose of an imaginary friend is to be a friend in time of loneliness. Imaginary friends do not take the place of social interaction, but may encourage conversation. Imaginary friends do not take the place of pets or toys. Imaginary friends accomplish what the child is still attempting.

Which of the following terms refers to opacity of the crystalline lens that prevents light rays from entering the eye and refracting on the retina? a. Myopia b. Amblyopia c. Cataract d. Glaucoma

ANS: C Opacity of the crystalline lens that prevents light rays from entering the eye and refracting on the retina is the definition of a cataract. Myopia, or nearsightedness, refers to the ability to see objects clearly at close range but not at a distance. Amblyopia, or lazy eye, is reduced visual acuity in one eye. Glaucoma is a group of eye diseases characterized by increased intraocular pressure.

Which is an important consideration when the nurse is discussing enuresis with the parents of a young child? a. Enuresis is more common in girls than in boys. b. Enuresis is neither inherited nor has a familial tendency. c. Organic causes that may be related to enuresis should be considered first. d. Psychogenic factors that cause enuresis persist into adulthood.

ANS: C Organic causes that may be related to enuresis should be ruled out before psychogenic factors are considered. Enuresis is more common in boys than in girls and has a strong familial tendency. Psychogenic factors may influence enuresis, but it is doubtful that they are causative.

What is a nursing consideration related to the administration of oxygen in an infant? a. Humidify oxygen if the infant can tolerate it. b. Assess the infant to determine how much oxygen should be given. c. Ensure uninterrupted delivery of the appropriate oxygen concentration. d. Direct oxygen flow so that it blows directly into the infant's face in a hood.

ANS: C Oxygen is a prescribed medication. It is the nurse's responsibility to ensure that the ordered concentration is delivered and the effects of therapy are monitored. Oxygen is drying to the tissues. Oxygen should always be humidified when delivered to a patient. A child receiving oxygen therapy should have the oxygen saturation monitored at least as frequently as vital signs. Oxygen is a medication, and it is the responsibility of the practitioner to modify dosage as indicated. Humidified oxygen should not be blown directly into an infant's face.

What is true about pelvic inflammatory disease (PID)? a. It can be prevented by proper personal hygiene. b. It is easily prevented by compliance with any form of contraception. c. It may have devastating effects on the reproductive tract of affected adolescents. d. It can potentially cause life-threatening and serious defects in the future children of affected adolescents.

ANS: C PID is a major concern because of its devastating effects on the reproductive tract. Short-term complications include abscess formation in the fallopian tubes, and long-term complications include ectopic pregnancy, infertility, and dyspareunia. PID is an infection of the upper female genital tract, most commonly caused by sexually transmitted infections. Personal hygiene, oral contraceptives, and many other forms of contraception do not prevent transmission of the disease. There is a possibility of ectopic pregnancy but not birth defects in children.

A nurse is conducting a class for adolescent girls about pelvic inflammatory disease (PID). Why should the nurse emphasize the importance of preventing pelvic inflammatory disease (PID)? a. PID can be sexually transmitted. b. PID cannot be treated. c. PID can have devastating effects on the reproductive tract. d. PID can cause serious defects in future children of affected adolescents.

ANS: C PID is a major concern because of its devastating effects on the reproductive tract. Short-term complications include abscess formation in the fallopian tubes, whereas long-term complications include ectopic pregnancy, infertility, and dyspareunia. PID is an infection of the upper female genital tract, most commonly caused by sexually transmitted infections but it is not sexually transmitted to another person. PID can be treated by treating the underlying cause. There is a possibility of ectopic pregnancy but not birth defects in children.

Parents need further teaching about the use of car safety seats if they make which statement? a. "Even if our toddler helps buckle the straps, we will double-check the fastenings." b. "We won't start the car until everyone is properly restrained." c. "We won't need to use the car seat on short trips to the store." d. "We will anchor the car seat to the car's anchoring system."

ANS: C Parents need to be taught to always use the restraint even for short trips. Further teaching is needed if they make this statement. Parents have understood the teaching if they encourage the child to help attach buckles, straps, and shields but always double-check fastenings; do not start the car until everyone is properly restrained; and anchor the car safety seat securely to the car's anchoring system and apply the harness snugly to the child.

Kimberly, age 3 years, is being admitted for about 1 week of hospitalization. Her parents tell the nurse that they are going to buy her "a lot of new toys, because she will be in the hospital." The nurse's reply should be based on an understanding of which concept? a. New toys make hospitalization easier. b. New toys are usually better than older ones for children of this age. c. At this age, children often need the comfort and reassurance of familiar toys from home. d. Buying new toys for a hospitalized child is a maladaptive way to cope with parental guilt.

ANS: C Parents should bring favorite items from home to be with the child. Young children associate inanimate objects with people who are significant in their lives. The favorite items will comfort and reassure the child. Because the parents left the objects, the preschooler knows the parents will return. New toys will not serve the purpose of familiar toys and objects from home. The parents may experience some guilt as a response to the hospitalization, but there is no evidence that it is maladaptive.

What intervention is most appropriate for fostering the development of a school-age child with disabilities associated with cerebral palsy? a. Provide sensory experiences. b. Help develop abstract thinking. c. Encourage socialization with peers. d. Give choices to allow for feeling of control.

ANS: C Peer interaction is especially important in relation to cognitive development, social development, and maturation. Cognitive development is facilitated by interaction with peers, parents, and teachers. The identification with those outside the family helps the child fulfill the striving for independence. Sensory experiences are beneficial, especially for younger children. School-age children are too young for abstract thinking. Giving school-age children choices is always an important intervention. Providing structured choices allows for a feeling of control.

A parent asks about whether a 7-year-old child is able to care for a dog. Based on the childs age, what does the nurse suggest? a. Caring for an animal requires more maturity than the average 7-year-old possesses. b. This will help the parent identify the childs weaknesses. c. A dog can help the child develop confidence and emotional health. d. Cats are better pets for school-age children

ANS: C Pets have been observed to influence a childs self-esteem. They can have a positive effect on physical and emotional health and can teach children the importance of nurturing and nonverbal communication. Most 7-year-old children are capable of caring for a pet with supervision. Caring for a pet should be a positive experience. It should not be used to identify weaknesses. The pet chosen does not matter as much as the childs being responsible for a pet.

Parents have a concern that their child is depressed. The nurse relates that which characteristic best describes children with depression? a. Increased range of affective response b. Preoccupation with need to perform well in school c. Change in appetite, resulting in weight loss or gain d. Tendency to prefer play instead of schoolwork

ANS: C Physiologic characteristics of children with depression include change in appetite resulting in weight loss or gain, nonspecific complaints of not feeling well, alterations in sleeping pattern, insomnia or hypersomnia, and constipation. Children who are depressed have sad facial expressions with absence or diminished range of affective response. These children withdraw from previously enjoyed activities and engage in solitary play or work with a lack of interest in play. A lack of interest is seen in doing homework or achieving in school, resulting in lower grades in children who are depressed.

What is a characteristic of children with depression? a. Increased range of affective response b. Tendency to prefer play instead of schoolwork c. Change in appetite resulting in weight loss or gain d. Preoccupation with need to perform well in school

ANS: C Physiologic characteristics of children with depression include changes in appetite resulting in weight loss or gain, nonspecific complaints of not feeling well, alterations in sleeping patterns, insomnia or hypersomnia, and constipation. Children who are depressed have sad facial expressions with absent or diminished range of affective response. These children withdraw from previously enjoyed activities and engage in solitary play or work with a lack of interest in play. They are uninterested in doing homework or achieving in school, resulting in lower grades.

What is descriptive of the play of school-age children? a. They like to invent games, making up the rules as they go. b. Individuality in play is better tolerated than at earlier ages. c. Knowing the rules of a game gives an important sense of belonging. d. Team play helps children learn the universal importance of competition and winning.

ANS: C Play involves increased physical skill, intellectual ability, and fantasy. Children form groups and cliques and develop a sense of belonging to a team or club. At this age, children begin to see the need for rules. Conformity and ritual permeate their play. Their games have fixed and unvarying rules, which may be bizarre and extraordinarily rigid. With team play, children learn about competition and the importance of winning, an attribute highly valued in the United States but not in all cultures

A 4-year-old child tells the nurse that she doesnt want another blood sample drawn because I need all of my insides and I dont want anyone taking them out. What is the nurses best interpretation of this? a. The child is being overly dramatic. b. The child has a disturbed body image. c. Preschoolers have poorly defined body boundaries. d. Preschoolers normally have a good understanding of their bodies.

ANS: C Preschoolers have little understanding of body boundaries, which leads to fears of mutilation. The child is not capable of being dramatic at this age. She truly has fear. Body image is just developing in school-age children. Preschoolers do not have good understanding of their bodies.

A 4-year-old child tells the nurse that she does not want another blood sample drawn because "I need all my insides, and I don't want anyone taking them out." Which is the nurse's best interpretation of this? a. Child is being overly dramatic. b. Child has a disturbed body image. c. Preschoolers have poorly defined body boundaries. d. Preschoolers normally have a good understanding of their bodies.

ANS: C Preschoolers have little understanding of body boundaries, which leads to fears of mutilation. The child is not capable of being dramatic at 4 years of age. She truly has fear. Body image is just developing in the school-age child. Preschoolers do not have good understanding of their bodies.

What behavior seen in children should be addressed by the nurse who is providing care to a child with a chronic illness? a. An infant who is uncooperative b. A toddler who expresses loneliness c. A preschooler who refuses to participate in self-care d. An adolescent who is showing independence

ANS: C Preschoolers thrive on being independent and are in the phase of gaining autonomy, so they want to perform as many self-care tasks as possible. If a preschooler is refusing to participate in selfcare activities, then the home health nurse should address this. Infants are uncooperative by nature, and toddlers do not understand the concept of loneliness, so these are not observations that would need to be addressed. Adolescents are always striving for independence, so this is a normal observation; if the adolescent were becoming more dependent on family, it might require intervention.

A nurse must do a venipuncture on a 6-year-old child. What is an important consideration in providing atraumatic care? a. Use an 18-gauge needle if possible. b. If not successful after four attempts, have another nurse try. c. Restrain the child only as needed to perform venipuncture safely. d. Show the child equipment to be used before the procedure.

ANS: C Restrain the child only as needed to perform the procedure safely; use therapeutic hugging. Use the smallest-gauge needle that permits free flow of blood. A two-try-only policy is desirable, in which two operators each have only two attempts. If insertion is not successful after four punctures, alternative venous access should be considered. Keep all equipment out of sight until used.

A 16-year-old with a chronic illness has recently become rebellious and is taking risks such as missing doses of his medication. What is the best explanation for this behavior? a. Needs more discipline b. Needs more socialization with peers c. This is part of normal adolescence d. This is how he is asking for more parental control

ANS: C Risk taking, rebelliousness, and lack of cooperation are normal parts of adolescence.

What does the nurse understand about caloric needs for school-age children? a. The caloric needs for the school-age children are the same as for other age groups. b. The caloric needs for school-age children are more than they were in the preschool years. c. The caloric needs for school-age children are lower than they were in the preschool years. d. The caloric needs for school-age children are greater than they will be in the adolescent years.

ANS: C School-age children do not need to be fed as carefully, as promptly, or as frequently as before. Caloric needs are lower than they were in the preschool years and lower than they will be during the coming adolescent growth spurt.

What statement best describes the relationship school-age children have with their families? a. Ready to reject parental controls b. Desire to spend equal time with family and peers c. Need and want restrictions placed on their behavior by the family d. Peer group replaces the family as the primary influence in setting standards of behavior and rules

ANS: C School-age children need and want restrictions placed on their behavior, and they are not prepared to cope with all the problems of their expanding environment. Although increased independence is the goal of middle childhood, they feel more secure knowing that an authority figure can implement controls and restriction. In the middle school years, children prefer peer group activities to family activities and want to spend more time in the company of peers. Family values usually take precedence over peer value systems.

The nurse is teaching a class on nutrition to a group of parents of 10- and 11-year-old children. What statement by one of the parents indicates a correct understanding of the teaching? a. My child does not need to eat a variety of foods, just his favorite food groups. b. My child can add salt and sugar to foods to make them taste better. c. I will serve foods that are low in saturated fat and cholesterol. d. I will continue to serve red meat three times per week for extra iron.

ANS: C School-age children should be eating foods that are low in saturated fat and cholesterol to prevent long-term consequences. The childs diet should include a variety of foods, include moderate amounts of extra salt and sugar, emphasize consumption of lean protein (chicken and pork), and limit red meat.

A child with corrosive poisoning is being admitted to the emergency department. What clinical manifestation does the nurse expect to assess on this child? a. Nausea and vomiting b. Alterations in sensorium, such as lethargy c. Severe burning pain in the mouth, throat, and stomach d. Respiratory symptoms of acute pulmonary involvement

ANS: C Severe burning pain in the mouth, throat, and stomach is a clinical manifestation of corrosive poisoning. Nausea and vomiting; alterations in sensorium, such as lethargy; and respiratory symptoms of acute pulmonary involvement are clinical manifestations of hydrocarbon poisoning.

The nurse must suction a child with a tracheostomy. What is the appropriate technique? a. Encourage the child to cough to raise the secretions before suctioning. b. Select a catheter with diameter three-fourths as large as the diameter of the tracheostomy tube. c. Ensure each pass of the suction catheter should take no longer than 5 seconds. d. Allow the child to rest after every five times the suction catheter is passed.

ANS: C Suctioning should require no longer than 5 seconds per pass. Otherwise, the airway may be occluded for too long. If the child is able to cough up secretions, suctioning may not be indicated. The catheter should have a diameter one-half the size of the tracheostomy tube. If it is too large, it might block the child's airway. The child is allowed to rest for 30 to 60 seconds after each aspiration to allow oxygen tension to return to normal. Then the process is repeated until the trachea is clear.

Which statement regarding chlamydia infection is correct? a. Treatment of choice is oral penicillin. b. Treatment of choice is nystatin or miconazole. c. Clinical manifestations include dysuria and urethral itching in males. d. Clinical manifestations include small, painful vesicles on genital areas.

ANS: C Symptoms of chlamydia infection in males include meatal erythema, tenderness, itching, dysuria, and urethral discharge. Some infected males have no symptoms. Oral penicillin and nystatin or miconazole are not the antibiotics of choice. Small, painful vesicles on genital areas are clinical manifestations true of chlamydia infection but may also indicate herpetic lesions.

In boys, what is the initial indication of puberty? a. Voice changes b. Growth of pubic hair c. Testicular enlargement d. Increased size of penis

ANS: C Testicular enlargement is the first change that signals puberty in boys; it usually occurs between the ages of 9 1/2 and 14 years during Tanner stage 2. Voice change occurs between Tanner stages 3 and 4. Fine pubic hair may occur at the base of the penis; darker hair occurs during Tanner stage 3. The penis enlarges during Tanner stage 3.

The Allen test is performed as a precautionary measure before which procedure? a. Heel stick b. Venipuncture c. Arterial puncture d. Lumbar puncture

ANS: C The Allen test assesses the circulation of the radial, ulnar, or brachial arteries before arterial puncture. The Allen test is used before arterial punctures, not heel sticks, venipunctures, or lumbar punctures.

A child with diazepam (Valium) poisoning has been admitted to the emergency department. What antidote does the nurse anticipate being prescribed? a. Succimer (Chemet) b. EDTA (Versenate) c. Flumazenil (Romazicon) d. Octreotide acetate (Sandostatin)

ANS: C The antidote for diazepam (Valium) poisoning is flumazenil (Romazicon). Succimer (Chemet) and EDTA (Versenate) are antidotes for heavy metal poisoning. Octreotide acetate (Sandostatin) is an antidote for sulfonylurea poisoning.

The school nurse is teaching female school-age children about the average age of puberty. What is the average age of puberty for girls? a. 10 years b. 11 years c. 12 years d. 13 years

ANS: C The average age of puberty is 12 years in girls.

A school-age child is diagnosed with a life-threatening illness. The parents want to protect their child from knowing the seriousness of the illness. What should the nurse tell the parents? a. This will help the child cope effectively by denial. b. This attitude is helpful to give parents time to cope. c. Terminally ill children know when they are seriously ill. d. Terminally ill children usually choose not to discuss the seriousness of their illness.

ANS: C The child needs honest and accurate information about the illness, treatments, and prognosis. Children, even at a young age, realize that something is seriously wrong and that it involves them. The nurse should help the parents understand the importance of honesty

The emergency department nurse is cleaning multiple facial abrasions on a 9-year-old child whose mother is present. The child is crying and screaming loudly. What is the best nursing action? a. Ask the child to be quieter b. Have the child's mother give instructions about relaxation c. Tell the child it is okay to cry and scream d. Remove the mother from the room

ANS: C The child should be allowed to express feelings of anger, anxiety, fear, frustration, or any other emotion. The child needs to know it is all right to cry. There is no reason for the child to be quieter and feelings need to be able to be expressed. The mother should stay in the room to provide comfort to the child.

The nurse is counseling a pregnant 35-year-old woman about estimated risk of Down syndrome. What is the estimated risk for a woman who is 35 years of age? a. One in 1200 b. One in 900 c. One in 350 D. One in 100

ANS: C The estimated risk of Down syndrome for a 35-year-old woman is one in 350. One in 1200 is the risk for a 25-year-old woman, one in 900 is the risk for a 30-yearold woman, and one in 100 is the risk for a 40-year-old woman.

An adolescent with long-term, complex health care needs will soon be discharged from the hospital. The nurse case manager has been assigned to the teen and family. The adolescents care involves physical therapy, occupational therapy, and speech therapy in addition to medical and nursing care. Who should be the decision maker in the adolescents care? a. Adolescent b. Nurse case manager c. Adolescent and family d. Multidisciplinary health care team

ANS: C The extent to which children are involved in their own care and decision making depends on many factors, including the childs developmental age, level of interest, physical ability, and parental support. If the adolescent is developmentally age appropriate, then decision making should be the responsibility of child and family. Family needs to be involved because they will be caring for the adolescent in the home. Health care providers have necessary input into the care of the child, but ultimate decision making rests with the adolescent and family.

A 9-year-old boy has several physical disabilities. His father explains to the nurse that his son concentrates on what he can, rather than cannot, do and is as independent as possible. What is the nurse's best interpretation of this statement? a. The father is experiencing denial b. The father is expressing his own views c. The child is using an adaptive coping style d. The child is using a maladaptive coping style

ANS: C The father is describing a well-adapted child who has learned to accept physical limitations. These children function well at home, at school, and with peers. They have an understanding of their disorder that allows them to accept their limitations, assume responsibility for care, and assist in treatment and rehabilitation. The father is describing his child's behavior. He is not denying the child's limitations. The father is exhibiting an adaptive coping style.

A father calls the emergency department nurse saying that his daughter's eyes burn after getting some dishwasher detergent in them. The nurse recommends that the child be seen in the emergency department or by an ophthalmologist. The nurse also should recommend which action before the child is transported? a. Keep eyes closed. b. Apply cold compresses. c. Irrigate eyes copiously with tap water for 20 minutes. d. Prepare a normal saline solution (salt and water) and irrigate eyes for 20 minutes.

ANS: C The first action is to flush the eyes with clean tap water. This will rinse the detergent from the eyes. Keeping eyes closed and applying cold compresses may allow the detergent to do further harm to the eyes during transport. Normal saline is not necessary. The delay can allow the detergent to cause continued injury to the eyes.

The school nurse is seeing a child who collected some poison ivy leaves during recess. He says only his hands touched it. What is the most appropriate nursing action? a. Soak his hands in warm water. b. Apply Burows solution compresses. c. Rinse his hands in cold running water. d. Scrub his hands thoroughly with antibacterial soap.

ANS: C The first recommended action is to rinse his hands in cold running water within 15 minutes of exposure. This will neutralize the urushiol not yet bonded to the skin. Soaking his hands in warm water is effective for soothing the skin lesions after the dermatitis has begun. Antibacterial soap removes protective skin oils and dilutes the urushiol, allowing it to spread.

The middle school nurse is planning a behavior modification program for overweight children. What is the most important goal for participants of the program? a. Learn how to cook low-fat meals. b. Improve relationships with peers. c. Identify and eliminate inappropriate eating habits. d. Achieve normal weight during the program.

ANS: C The goal of behavior modification in weight control is to help the participant identify abnormal eating processes. After the abnormal patterns are identified, then techniques, including problem solving, are taught to eliminate inappropriate eating. Learning how to cook low-fat meals can be a component of the program, but the focus of behavior modification is identifying target behaviors that need to be changed. Improving relationships is not the focus of weight management behavior management programs. Achieving normal weight during the program is an inappropriate goal. As the child incorporates the techniques, weight gain will slow. In childhood obesity, the goal is to stop the increase of weight gain

What statement regarding chlamydial infections is correct? a. The treatment of choice is oral penicillin. b. The treatment of choice is nystatin or miconazole. c. Both men and women may have asymptomatic infections. d. Clinical manifestations include small, painful vesicles on the genital areas.

ANS: C The incidence of asymptomatic chlamydial infections is as high as 50% of men and 75% of women. Symptoms of chlamydial infection in men include meatal erythema, tenderness, itching, dysuria, and urethral discharge. Oral penicillin, nystatin, and miconazole are not the antibiotics of choice. Small, painful vesicles on genital areas are clinical manifestations of herpetic infections

Which should the nurse consider when having consent forms signed for surgery and procedures on children? a. Only a parent or legal guardian can give consent. b. The person giving consent must be at least 18 years old. c. The risks and benefits of a procedure are part of the consent process. d. A mental age of 7 years or older is required for a consent to be considered "informed."

ANS: C The informed consent must include the nature of the procedure, benefits and risks, and alternatives to the procedure. In special circumstances, such as emancipated minors, the consent can be given by someone younger than 18 years without the parent or legal guardian. A mental age of 7 years is too young for consent to be informed.

What is an important nursing consideration when a child is hospitalized for chelation therapy to treat lead poisoning? a. Maintain bed rest. b. Maintain isolation precautions. c. Keep an accurate record of intake and output. d. Institute measures to prevent skeletal fracture.

ANS: C The iron chelates are excreted though the kidneys. Adequate hydration is essential. Periodic measurement of renal function is done. Bed rest is not necessary. Often the chelation therapy is done on an outpatient basis. Chelation therapy is not infectious or dangerous. Isolation is not indicated. Skeletal weakness does not result from high levels of lead

Tretinoin (Retin-A) is a commonly used topical agent for the treatment of acne. What do nursing considerations with this drug include? a. Sun exposure increases effectiveness. b. Cosmetics with lanolin and petrolatum are preferred in acne. c. Applying of the medication occurs at least 20 to 30 minutes after washing. d. Erythema and peeling are indications of toxicity and need to be reported.

ANS: C The medication should not be applied for at least 20 to 30 minutes after washing to decrease the burning sensation. The avoidance of sun and the use of sunscreen agents must be emphasized because sun exposure can result in severe sunburn. Cosmetics with lanolin, petrolatum, vegetable oil, lauryl alcohol, butyl stearate, and oleic acid can increase comedone production. Erythema and peeling are common local manifestations.

Tretinoin (Retin-A) is a topical agent commonly used to treat acne. Nursing considerations with this drug should include: a. teaching to avoid use of sunscreen agents. b. applying generously to the skin. c. explaining that medication should not be applied until at least 20 to 30 minutes after washing. d. explaining that erythema and peeling are indications of toxicity.

ANS: C The medication should not be applied for at least 20 to 30 minutes after washing to decrease the burning sensation. The avoidance of sun and the use of sunscreen agents must be emphasized because sun exposure can result in severe sunburn. The agent should be applied sparingly to the skin. Erythema and peeling are common local manifestations.

The nurse is admitting a school-age child in acute renal failure with reduced glomerular filtration rate. Which urine test is the most useful clinical indication of glomerular filtration rate? a. pH b. Osmolality c. Creatinine d. Protein level

ANS: C The most useful clinical indication of glomerular filtration is the clearance of creatinine. It is a substance that is freely filtered by the glomerulus and secreted by the renal tubule cells. The pH and osmolality are not estimates of glomerular filtration. Although protein in the urine demonstrates abnormal glomerular permeability, it is not a measure of filtration rate.

The nurse observes that a seriously ill child passively accepts all painful procedures. The nurse should recognize that this is most likely an indication that the child is experiencing what emotional response? a. Hopefulness b. Chronic sorrow c. Belief that procedures are a deserved punishment d. Understanding that procedures indicate impending death

ANS: C The nurse should be particularly alert to a child who withdraws and passively accepts all painful procedures. This child may believe that such acts are inflicted as deserved punishment for being less worthy. A child who is hopeful is mobilized into goal-directed actions. This child would actively participate in care. Chronic sorrow is the feeling of sorrow and loss that recurs in waves over time. It is usually evident in the parents, not in the child. The seriously ill child would actively participate in care. Nursing interventions should be used to minimize the pain.

The nurse observes that a seriously ill child passively accepts all painful procedures. What should the nurse recognize this child is most likely experiencing? a. A sense of hopefulness b. A sense of chronic sorrow c. A belief that procedures are a deserved punishment d. A belief that procedures are an important part of care

ANS: C The nurse should be particularly alert to the child who passively accepts all painful procedures. This child may believe that such acts are inflicted as deserved punishment. The child who is hopeful is mobilized into goal-directed actions. This child would actively participate in care. Chronic sorrow is the feeling of sorrow and loss that recurs in waves over time. It is usually evident in the parents, not in the child. A child who believes that procedures are an important part of care would actively participate in care. Nursing interventions should be used to minimize the pain.

A nurse is preparing to complete an admission assessment on a 2-year-old child. The child is sitting on the parent's lap. Which technique should the nurse implement to complete the physical exam? a. Ask the parent to place the child in the hospital crib. b. Take the child and parent to the exam room. c. Perform the exam while the child is on the parent's lap. d. Ask the child to stand by the parent while completing the exam.

ANS: C The nurse should complete the exam while the child is on the parent's lap. For young children, particularly infants and toddlers, preserving parent-child contact is the best means of decreasing the need for or stress of restraint. The entire physical examination can be done in a parent's lap with the parent hugging the child for procedures such as an otoscopic examination. Placing the child in the crib, taking the child to the exam room, or asking the child to stand by the parent would separate the child from the parent and cause anxiety.

A toddler's parent asks the nurse for suggestions on dealing with temper tantrums. Which is the most appropriate recommendation? a. Punish the child. b. Leave the child alone until the tantrum is over. c. Remain close by the child but without eye contact. d. Explain to child that this is wrong.

ANS: C The parent should be told that the best way to deal with temper tantrums is to ignore the behaviors, provided that the actions are not dangerous to the child. Tantrums are common in toddlers as the child becomes more independent and overwhelmed by increasingly complex tasks. The parents and caregivers need to have consistent and developmentally appropriate expectations. Punishment and explanations will not be beneficial. The parent's presence is necessary both for safety and to provide a feeling of control and security to the child when the tantrum is over.

Parents are concerned about the number of hours their teenage daughter spends with peers. The nurse explains that peer relationships are important during adolescence for which reason? a. Adolescents dislike their parents. b. Adolescents no longer need parental control. c. They provide adolescents with a feeling of belonging. d. They promote a sense of individuality in adolescents.

ANS: C The peer group serves as a strong support to teenagers, providing them with a sense of belonging and a sense of strength and power. During adolescence, the parent-child relationship changes from one of protection-dependency to one of mutual affection and quality. Parents continue to play an important role in the personal and health-related decisions. The peer group forms the transitional world between dependence and autonomy.

The home health nurse is caring for a child who requires complex care. The family expresses frustration related to obtaining accurate information about their child's illness and its management. Which is the best action for the nurse? a. Determine why the family is easily frustrated. b. Refer the family to the child's primary care practitioner. c. Clarify the family's request, and provide the information they want. d. Answer only questions that the family needs to know about.

ANS: C The philosophic basis for family-centered practice is the recognition that the family is the constant in the child's life. It is essential and appropriate that the family have complete and accurate information about their child's illness and management. The nurse may first have to clarify what information the family believes has not been communicated. The family's frustration arises from their perception that they are not receiving information pertinent to their child's care. Referring the family to the child's primary care practitioner does not help the family. The home health nurse should have access to the necessary information. Questions about what they need and want to know concerning their child's care should be addressed.

What is an important consideration for the school nurse planning a class on injury prevention for adolescents? a. Adolescents generally are not risk takers. b. Adolescents can anticipate the long-term consequences of serious injuries. c. Adolescents need to discharge energy, often at the expense of logical thinking. d. During adolescence, participation in sports should be limited to prevent permanent injuries.

ANS: C The physical, sensory, and psychomotor development of adolescents provides a sense of strength and confidence. There is also an increase in energy coupled with risk taking that puts them at risk. Adolescents are risk takers because their feelings of indestructibility interfere with understanding of consequences. Sports can be a useful way for adolescents to discharge energy. Care must be taken to avoid overuse injuries.

A child is playing in the playroom. The nurse needs to obtain a child's blood pressure. Which is the appropriate procedure for obtaining the blood pressure? a. Take the blood pressure in the playroom. b. Ask the child to come to the exam room to obtain the blood pressure. c. Ask the child to return to his or her room for the blood pressure, then escort the child back to the playroom. d. Document that the blood pressure was not obtained because the child was in the playroom.

ANS: C The play room is a safe haven for children, free from medical or nursing procedures. The child can be returned to his or her room for the blood pressure and then escorted back to the playroom. The exam room is reserved for painful procedures that should not be performed in the child's hospital bed. Documenting that the blood pressure was not obtained because the child was in the playroom is inappropriate.

What signals the resolution of the Oedipus or Electra complex? a. Learns sex differences b. Learns sexually appropriate behavior c. Identifies with the same-sex parent d. Has guilt over feelings toward the father or mother

ANS: C The resolution of the Oedipus or Electra complex is identification with the same-sex parent. Learning sex differences and sexually appropriate behavior is a goal in further differentiation of oneself but does not signal the resolution of the Oedipus or Electra complex. Guilt over feelings toward the father or mother is seen as a stage in the complex, not the resolution.

A parent calls the clinic nurse because his 7-year-old child was bitten by a black widow spider. What action should the nurse advise the parent to take? a. Apply warm compresses. b. Carefully scrape off the stinger. c. Take the child to the emergency department. d. Apply a thin layer of corticosteroid cream.

ANS: C The venom of the black widow spider has a neurotoxic effect. The parent should take the child to the emergency department for treatment with antivenin and muscle relaxants as needed. Warm compresses increase the circulation to the area and facilitate the spread of the venom. The black widow spider does not have a stinger. Corticosteroid cream has no effect on the venom.

The nurse is talking with a 10-year-old boy who wears bilateral hearing aids. The left hearing aid is making an annoying whistling sound that the child cannot hear. Which is the most appropriate nursing action? a. Ignore the sound. b. Ask him to reverse the hearing aids in his ears. c. Suggest he reinsert the hearing aid. d. Suggest he raise the volume of the hearing aid.

ANS: C The whistling sound is acoustic feedback. The nurse should have the child remove the hearing aid and reinsert it, making certain no hair is caught between the ear mold and the ear canal. It would be annoying to others to ignore the sound or to suggest he raise the volume of the hearing aid. The hearing aids are molded specifically for each ear.

The nurse is explaining about the developmental sequence in childrens capacity to conserve matter to a group of parents. What type of matter is last in the sequence for a child to develop? a. Mass b. Length c. Volume d. Numbers

ANS: C There is a developmental sequence in childrens capacity to conserve matter. Children usually grasp conservation of numbers (ages 5 to 6 years) before conservation of substance. Conservation of liquids, mass, and length usually is accomplished at about ages 6 to 7 years, conservation of weight sometime later (ages 9 to 10 years), and conservation of volume or displacement last (ages 9 to 12 years).

The nurse notes that a child has lost 8 pounds after 4 days of hospitalization for acute glomerulonephritis. What is most likely the cause of the weight loss? a. Poor appetite b. Increased potassium intake c. Reduction of edema d. Restriction to bed rest

ANS: C This amount of weight loss in this period is a result of the improvement of renal function and mobilization of edema fluid. Poor appetite and bed rest would not result in a weight loss of 8 pounds in 4 days. Foods with substantial amounts of potassium are avoided until renal function is normalized.

Frequent urine testing for specific gravity and glucose are required on a 6-month-old infant. Which is the most appropriate way to collect small amounts of urine for these tests? a. Apply a urine-collection bag to the perineal area. b. Tape a small medicine cup to the inside of the diaper. c. Aspirate urine from cotton balls inside the diaper with a syringe. d. Aspirate urine from a superabsorbent disposable diaper with a syringe.

ANS: C To obtain small amounts of urine, use a syringe without a needle to aspirate urine directly from the diaper. If diapers with absorbent material are used, place a small gauze dressing or cotton balls inside the diaper to collect the urine, and aspirate the urine with a syringe. For frequent urine sampling, the collection bag would be too irritating to the child's skin. It is not feasible to tape a small medicine cup to the inside of the diaper; the urine will spill from the cup. Diapers with superabsorbent gels absorb the urine, so there is nothing to aspirate.

A critically ill child has hyperthermia. The parents ask the nurse to give an antipyretic such as acetaminophen (Tylenol). What should the nurse explain about antipyretics? a. They may cause malignant hyperthermia b. They may cause febrile seizures c. They are of no value in treating hyperthermia d. They are of limited value in treating hyperthermia

ANS: C Unlike with fever, antipyretics are of no value in hyperthermia because the set point is already normal. Cooling measures are used instead. Malignant hyperthermia is a genetic myopathy that is triggered by anesthetic agents. Antipyretic agents do not have this effect. Antipyretics do not cause seizures and are of no value in hyperthermia.

What developmental characteristic does not occur until a child reaches age 2 1/2 years? a. Birth weight has doubled. b. Anterior fontanel is still open. c. Primary dentition is complete. d. Binocularity may be established.

ANS: C Usually by age 30 months, the primary dentition of 20 teeth is complete. Birth weight doubles at approximately ages 5 to 6 months. The anterior fontanel closes at ages 12 to 18 months. Binocularity is established by age 15 months.

The nurse is facilitating a conference between the teachers and parents of a 7-year-old child newly diagnosed with attention deficit hyperactivity disorder (ADHD). What does the nurse stress? a. Academic subjects should be taught in the afternoon. b. Low-interest activities in the classroom should be minimized. c. Visual references should accompany verbal instruction. d. The childs environment should be visually stimulating.

ANS: C Verbal instructions should always be accompanied by visual or written instructions. This provides the child with reinforcement and a reference to expectations. Academic subjects should be taught in the morning when the child is experiencing the effects of the morning dose of medication. Low-interest activities should be mixed with high-interest activities to maintain the childs attention. Environmental stimulation should be minimized to help eliminate distractions that can overexcite the child.

A 14-year-old boy is being admitted to the hospital for an appendectomy. Which roommate should the nurse assign with this patient? a. A 4-year-old boy post-appendectomy surgery b. A 6-year-old boy with pneumonia c. A 15-year-old boy admitted with a vasoocclusive sickle cell crisis d. A 12-year-old boy with cellulitis

ANS: C When a child is admitted, nurses follow several fairly universal admission procedures. The minimum considerations for room assignment are age, sex, and nature of the illness. Age grouping is especially important for adolescents. The 14-year-old boy being admitted to the unit after appendectomy surgery should be placed with a noninfectious child of the same sex and age. The 15-year-old child with sickle cell is the best choice. The 4-year-old post-appendectomy is too young, and the child with pneumonia is too young and possibly has an infectious process. The 12-year-old boy with cellulitis is the right age, but he has an infection (cellulitis).

The school nurse is presenting sexual information to a group of school-age girls. What approach should the nurse take when presenting the information? a. Put off answering questions. b. Give technical terms when giving the presentation. c. Treat sex as a normal part of growth and development. d. Plan to give the presentation with boys and girls together.

ANS: C When nurses present sexual information to children, they should treat sex as a normal part of growth and development. Nurses should answer questions honestly, matter-of-factly, and at the childrens level of understanding. School-age children may be more comfortable when boys and girls are segregated for discussions.

Parents ask the nurse, How should we deal with our toddlers regression since our new baby has come home? The nurse should give the parents which response? a. Introduce new areas of learning. b. Use time-out as punishment when regression occurs. c. Ignore the behavior and praise appropriate behavior. d. Explain to the toddler that the behavior is not acceptable

ANS: C When regression does occur, the best approach is to ignore it while praising existing patterns of appropriate behavior. It is advisable not to introduce new areas of learning when an additional crisis is present or expected, such as beginning toilet training shortly before a sibling is born or during a brief hospitalization. Time-out should not be used as a punishment, and the toddler does not have the cognitive ability to understand an explanation that the behavior is not acceptable.

The school nurse has been asked to begin teaching sex education in the fifth grade. What should the nurse recognize? a. Questions need to be discouraged in this setting. b. Most children in the fifth grade are too young for sex education. c. Sexuality is presented as a normal part of growth and development. d. Correct terminology should be reserved for children who are older.

ANS: C When sexual information is presented to school-age children, sex should be treated as a normal part of growth and development. They should be encouraged to ask questions. At 10 to 11 years old, fifth graders are not too young to speak about physiologic changes in their bodies. Preadolescents need precise and concrete information.

The nurse is explaining to an adolescent the rationale for administering a Tdap (tetanus, diphtheria, acellular pertussis) vaccine 3 years after the last Td (tetanus) booster. What should the nurse tell the adolescent? a. It is time for a booster vaccine. b. It is past the time for a booster vaccine. c. This vaccine will provide pertussis immunity. d. This vaccine will be the last booster you will need.

ANS: C When the Tdap is used as a booster dose, it may be administered earlier than the previous 5-year interval to provide adequate pertussis immunity (regardless of interval from the last Td dose). It is not time or past time for a booster because they are required every 5 years. Another booster will be needed in 5 years, so it is not the last dose.

Which of the following is the most common clinical manifestation of retinoblastoma? a. Glaucoma b. Amblyopia c. Cat's eye reflex d. Sunken eye socket

ANS: C When the eye is examined, the light will reflect off the tumor, giving the eye a whitish appearance. This is called a cat's eye reflex. A late sign of retinoblastoma is a red, painful eye with glaucoma. Amblyopia, or lazy eye, is reduced visual acuity in one eye. The eye socket is not sunken.

1. The nurse is preparing to obtain a nasal washing from a child. What equipment should the nurse gather for the procedure? (Select all that apply.) a. Sterile water b. A sterile swab c. Syringe with tubing d. Sterile normal saline e. Tracheal suction catheter

ANS: C, D Nasal washings may be obtained to identify viral pathogens and guide therapy in some respiratory conditions. The child is placed supine, and 1 to 3 ml of sterile normal saline is instilled with a sterile syringe (without a needle) into one nostril. The contents are aspirated with a syringe with 5 cm (2 inches) of 18- to 20-gauge tubing. The saline is quickly instilled and then aspirated to recover the nasal specimen. A tracheal suction catheter would not trap the mucus. Normal saline is used, not sterile water. A sterile swab is used for a throat culture, not for nasal washings.

Which are adaptive coping patterns used by children with special needs? (Select all that apply.) a. Feels different and withdraws b. Is irritable, moody, and acts out c. Seeks support d. Develops optimism

ANS: C, D Adaptive coping patterns used by children with special needs include seeking support and developing optimism. Maladaptive behaviors are seeing themselves as different and withdrawing and becoming irritable, moody, and beginning to act out.

The nurse is assessing a child with Down syndrome. The nurse recognizes that which are possible comorbidities that can occur with Down syndrome? (Select all that apply.) a. Diabetes mellitus b. Hodgkins disease c. Congenital heart defects d. Respiratory tract infections e. Acute megakaryoblastic leukemia

ANS: C, D, E Children with Down syndrome often have multiple comorbidities, contributing to numerous other conditions. Respiratory tract infections are prevalent; when combined with cardiac anomalies, they are the chief cause of death, particularly during the first year. The incidence of leukemia is several times more frequent than expected in the general population, and in about half of the cases, the type is acute megakaryoblastic leukemia.

1. What signs and symptoms are indicative of a urinary tract disorder in the neonatal period (birth to 1 month)? (Select all that apply.) a. Edema b. Bradypnea c. Frequent urination d. Poor urinary stream e. Failure to gain weight

ANS: C, D, E Signs and symptoms of a urinary tract disorder in the neonatal period are frequent urination, poor urinary stream, and failure to gain weight. The respirations would be rapid, not slow, and dehydration, not edema, occurs.

What factors influence the effects of a childs hospitalization on siblings? (Select all that apply.) a. Older siblings b. Experiencing minimal changes c. Receiving little information about their ill brother or sister d. Being cared for outside the home by care providers who are not relatives e. Perceiving that their parents treat them differently compared with before their siblings hospitalization

ANS: C, D, E Various factors have been identified that influence the effects of a childs hospitalization on siblings. Factors that are related specifically to the hospital experience and increase the effects on the sibling are being cared for outside the home by care providers who are not relatives, receiving little information about their ill brother or sister, and perceiving that their parents treat them differently compared with before their siblings hospitalization. Being younger, not older, and experiencing many changes, not minimal changes, are factors that influence the effects of a childs hospitalization on siblings.

The nurse is teaching a parent of an 18-month-old about developmental milestones associated with feeding. What should the nurse include in the teaching? (Select all that apply.) a. The child will begin to use a fork. b. The child will be able use a straw and cup. c. The child will be able to hold a cup with both hands. d. The child will be able to drink from a cup with a lid. e. The child will begin to use a spoon but may turn it before reaching the mouth.

ANS: C, D, E An 18-month-old child can hold a cup with both hands, is able to drink from a cup with a lid, and begins to use a spoon but may turn it before reaching the mouth. Using a fork is a developmental milestone of a 36-month-old child. Using a straw and cup is a milestone seen at 24 months.

What are characteristics of early adolescence (1114 years) with regard to identity? (Select all that apply.) a. Mature sexual identity b. Increase in self-esteem c. Trying out of various roles d. Conformity to group norms e. Preoccupied with rapid body changes

ANS: C, D, E Characteristics of early adolescence identity include trying out of various roles, conformity to group norms, and preoccupation with rapid body changes. Mature sexual identity and increase in self-esteem are characteristics of late adolescent identity.

The nurse understands that traits of gifted children include what? (Select all that apply.) a. Fair memory skills b. Limited sense of humor c. Perfectionism as a focus d. Inquisitive; always asking questions e. Displays intense feelings and emotion

ANS: C, D, E Characteristics of gifted children include perfectionism as a focus; inquisitive, always asking questions; and displaying intense feelings and emotion. Memory skills are pronounced, and humor is exceptional.

What are characteristics of late adolescence (1820 years) with regard to sexuality? (Select all that apply.) a. Exploration of self-appeal b. Limited dating, usually group c. Intimacy involves commitment d. Growing capacity for mutuality and reciprocity e. May publicly identify as gay, lesbian, or bisexual

ANS: C, D, E Characteristics of late adolescence sexuality include intimacy involving commitment; growing capacity for mutuality and reciprocity; and publicly identifying as gay, lesbian, or bisexual. Exploration of self-appeal is a characteristic of middle adolescence sexuality. Limited dating, usually group, is a characteristic of early adolescence sexuality.

The nurse is caring for a child who has a temperature of 30 C (86 F). What physical effects of hypothermia should the nurse expect to observe in this child? (Select all that apply.) a. Reduced urinary output b. Injury to peripheral tissue c. Increased blood pressure d. Tachycardia e. Irritability with loss of consciousness f. Rigid extremities

ANS: C, D, E Hypothermia has varying physical effects depending on the childs core temperature. At 30 C (86 F), a child would experience an increase in blood pressure, tachycardia, and irritability followed by a loss of consciousness. Reduced urinary output from a decrease of blood flow to the kidneys, injury to peripheral tissue, and rigid extremities are physical effects observed as the body temperature continues to decrease.

What child behavior indicates to the nurse that temper tantrums have become a problem? (Select all that apply.) a. The child is 2 to 3 years old b. Tantrums occur at bedtime c. Tantrums occur past 5 years of age d. Tantrums last longer than 15 minutes e. Tantrums occur more than five times a day

ANS: C, D, E Temper tantrums are common during the toddler years and essentially represent normal developmental behaviors. However, temper tantrums can be signs of serious problems. Temper tantrums that occur past 5 years of age, last longer than 15 minutes, or occur more than five times a day are considered abnormal and may indicate a serious problem. A popular time for a tantrum is before bedtime.

The nurse is teaching parents of a 4-year-old child about fine motor developmental milestones. What milestones should the nurse include in the teaching session? (Select all that apply.) a. Can lace shoes b. Uses scissors successfully c. Builds a tower of nine or 10 cubes d. Builds a bridge with three cubes e. Adeptly places small pellets in a narrow-necked bottle

ANS: C, D, E The fine motor milestones of a 4-year-old child include building a tower of nine or 10 cubes, building a bridge with three cubes, and adeptly placing small pellets in a narrow-necked bottle. Lacing shoes and using scissors successfully are fine motor milestones seen at the age of 5 years.

The nurse is teaching parents about safety for their latchkey children. What should the nurse include in the teaching session? (Select all that apply.) a. Teach the child first-aid procedures. b. Keep the key in an easy place to find. c. Teach the child weather-related safety. d. Teach the child to open the door for delivery people. e. Emphasize fire safety rules and conduct practice fire drills.

ANS: C, E Safety for latchkey children includes teaching the child first-aid procedures, teaching the child weather-related safety, and emphasizing fire safety rules and conducting practice fire drills. Teach the child not to display keys and to always lock doors. The child should be taught to not open the door to anyone, even delivery people.

1. The presence of which pair of factors is a good predictor of a fluid deficit of at least 5% in an infant? a. Weight loss and decreased heart rate b. Capillary refill of less than 2 seconds and no tears c. Increased skin elasticity and sunken anterior fontanel d. Dry mucous membranes and generally ill appearance

ANS: D A good predictor of a fluid deficit of at least 5% is any two four factors: capillary refill of more than 2 seconds, absent tears, dry mucous membranes, and ill general appearance. Weight loss is associated with fluid deficit, but the degree needs to be quantified. Heart rate is usually elevated. Skin elasticity is decreased, not increased. The anterior fontanel is depressed.

The nurse needs to take the blood pressure of a preschool boy for the first time. What action would be best in gaining his cooperation? a. Tell him that this procedure will help him get well faster. b. Take his blood pressure when a parent is there to comfort him. c. Explain to him how the blood flows through the arm and why the blood pressure is important. d. Permit him to handle the equipment and see the cuff inflate and deflate before putting the cuff in place.

ANS: D A preschooler is at the stage of preoperational thought. The nurse needs to explain the procedure in simple terms and allow the child to see how the equipment works. This will help allay fears of bodily harm. Blood pressure measurement is used for assessment, not therapy, and will not help him get well faster. Although the parent will be able to support the child, he may still be uncooperative. Also, the assessment of blood pressure may be needed before the parent is available. Explaining to a preschooler how the blood flows through the artery and why the blood pressure is important is too complex.

The nurse is caring for a 3-year-old child during a long hospitalization. The parent is concerned about how to support the childs siblings during the hospitalization. What statement is appropriate for the nurse to make? a. You should choose one parent to spend every night in the hospital while the other parent stays at home with the other children. b. You could leave your hospitalized child for periods at night to be at home with the other children. C. You should discourage the siblings from visiting because this could upset everyone in the family. D. You could encourage a nightly phone call between the siblings as part of the bedtime routine.

ANS: D A supportive measure for siblings of a hospitalized child is to have a routine of a phone call at some point during the day or evening so the parent at the hospital can stay in touch and the children at home are involved and can hear that their sibling is doing well. Parents should alternate who stays at the hospital overnight to prevent burnout and to allow each parent time at home with the siblings. Encourage siblings to visit if appropriate to keep the family unit intact. Leaving the hospitalized child alone at night will not support the siblings at home and may cause problems with the hospitalized child.

1. What substance is released from the posterior pituitary gland and promotes water retention in the renal system? a. Renin b. Aldosterone c. Angiotensin d. Antidiuretic hormone (ADH)

ANS: D ADH is released in response to increased osmolality and decreased volume of intravascular fluid; it promotes water retention in the renal system by increasing the permeability of renal tubules to water. Renin release is stimulated by diminished blood flow to the kidneys. Aldosterone is secreted by the adrenal cortex. It enhances sodium reabsorption in renal tubules, promoting osmotic reabsorption of water. Renin reacts with a plasma globulin to generate angiotensin, which is a powerful vasoconstrictor. Angiotensin also stimulates the release of aldosterone.

Cognitive development influences response to pain. What age group is most concerned with the fear of losing control during a painful experience? a. Toddlers b. Preschoolers c. School-age children d. Adolescents

ANS: D Adolescents view illness as physiologic (an organ malfunction) and psychophysiologic (psychologic factors that affect health). Adolescents usually approach pain with self-control. They are concerned with remaining composed and feel embarrassed and ashamed of losing control. Toddlers and preschoolers react to pain primarily as a physical, concrete experience. Preschoolers may try to escape a procedure with verbal statements such as go away. Young school-agechildrenmayviewpainaspunishmentforwrongdoing.Thisagegroupfearsbodilyhar

29. What type of shock is characterized by a hypersensitivity reaction causing massive vasodilation and capillary leaks, which may occur with drug or latex allergy? a. Neurogenic shock b. Cardiogenic shock c. Hypovolemic shock d. Anaphylactic shock

ANS: D Anaphylactic shock results from extreme allergy or hypersensitivity to a foreign substance. Neurogenic shock results from loss of neuronal control, such as the interruption of neuronal transmission after a spinal cord injury. Cardiogenic shock is decreased cardiac output. Hypovolemic shock is a reduction in the size of the vascular compartment, decreasing blood pressure, and low central venous pressure.

What choice of words or phrases would be inappropriate to use with a child? a. Rolling bed for stretcher b. Special medicine for dye c. Make sleepy for deaden d. Catheter for intravenous

ANS: D Children can grasp information only if it is presented on or close to their level of cognitive development. This necessitates an awareness of the words used to describe events or processes, and exploring family traditions or approaches to information sharing and creating patient specific language or context. Therefore, to prevent or alleviate fears, nurses must be aware of the medical terminology and vocabulary that they use every day and be sensitive to the use of slang or confusing terminology. Catheter is a medical term and would be confusing.

1. A child is admitted for minimal change nephrotic syndrome (MCNS). The nurse recognizes that the childs prognosis is related to what factor? a. Admission blood pressure b. Creatinine clearance c. Amount of protein in urine d. Response to steroid therapy

ANS: D Corticosteroids are the drugs of choice for MCNS. If the child has not responded to therapy within 28 days of daily steroid administration, the likelihood of subsequent response decreases. Blood pressure is normal or low in MCNS. It is not correlated with prognosis. Creatinine clearance is not correlated with prognosis. The presence of significant proteinuria is used for diagnosis. It is not predictive of prognosis.

23. What type of diarrhea is associated with an inflammation of the mucosa and submucosa in the ileum and colon caused by infectious agents? a. Osmotic b. Secretory c. Cytotoxic d. Dysenteric

ANS: D Dysenteric diarrhea is associated with an inflammation of the mucosa and submucosa in the ileum and colon caused by infectious agents such as Campylobacter, Salmonella, or Shigella organisms. Edema, mucosal bleeding, and leukocyte infiltration occur. Osmotic diarrhea occurs when the intestine cannot absorb nutrients or electrolytes. It is commonly seen in malabsorption syndromes such as lactose intolerance. Secretory diarrhea is usually a result of bacterial enterotoxins that stimulate fluid and electrolyte secretion from the mucosal crypt cells, the principal secretory cells of the small intestine. Cytotoxic diarrhea is characterized by the viral destruction of the villi of the small intestine. This results in a smaller intestinal surface area, with a decreased capacity for fluid and electrolyte absorption.

23. A school-age child with diarrhea has been rehydrated. The nurse is discussing the childs diet with the family. What food or beverage should be tolerated best? a. Clear fluids b. Carbonated drinks c. Applesauce and milk d. Easily digested foods

ANS: D Easily digested foods such as cereals, cooked vegetables, and meats should be provided for the child. Early reintroduction of nutrients is desirable. Continued feeding or reintroduction of a regular diet has no adverse effects and actually lessens the severity and duration of the illness. Clear fluids (e.g., fruit juices and gelatin) and carbonated drinks have high carbohydrate content and few electrolytes. Caffeinated beverages should be avoided because caffeine is a mild diuretic. In some children, lactose intolerance will develop with diarrhea, and cows milk should be avoided in the recovery stage.

29. A child is admitted with extensive burns. The nurse notes burns on the childs lips and singed nasal hairs. The nurse should suspect what condition in the child? a. A chemical burn b. A hot-water scald c. An electrical burn d. An inhalation injury

ANS: D Evidence of an inhalation injury includes burns of the face and lips, singed nasal hairs, and laryngeal edema. Clinical manifestations may be delayed for up to 24 hours. Chemical burns, electrical burns, and burns associated with hot-water scalds would not produce singed nasal hair.

One of the techniques that has been especially useful for learners having cognitive impairment is called fading. What description best explains this technique? a. Positive reinforcement when tasks or behaviors are mastered b. Repeated verbal explanations until tasks are faded into the childs development c. Negative reinforcement for specific tasks or behaviors that need to be faded out d. Gradually reduces the assistance given to the child so the child becomes more independent

ANS: D Fading is physically taking the child through each sequence of the desired activity and gradually fading out the physical assistance so the child becomes more independent. Positive reinforcement when tasks or behaviors are mastered is part of behavior modification. An essential component is ignoring undesirable behaviors. Verbal explanations are not as effective as demonstration and physical guidance. Consistent negative reinforcement is helpful, but positive reinforcement that focuses on skill attainment should be incorporated.

The nurse needs to assess a 15-month-old child who is sitting quietly on his fathers lap. What initial action by the nurse would be most appropriate? a. Ask the father to place the child on the exam table. b. Undress the child while he is still sitting on his fathers lap. c. Talk softly to the child while taking him from his father. d. Begin the assessment while the child is in his fathers lap.

ANS: D For young children, particularly infants and toddlers, preserving parentchild contact is a good way of decreasing stress or the need for physical restraint during an assessment. For example, much of a patients physical examination can be done with the patient in a parents lap with the parent providing reassuring and comforting contact. The initial action would be to begin the assessment while the child is in his fathers lap.

1. Parents of a newborn with ambiguous genitalia want to know how long they will have to wait to know whether they have a boy or a girl. The nurse answers the parents based on what knowledge? a. Chromosome analysis will be complete in 7 days. b. A physical examination will be able to provide a definitive answer. c. Additional laboratory testing is necessary to assign the correct gender. d. Gender assignment involves collaboration between the parents and a multidisciplinary team.

ANS: D Gender assignment is a complex decision-making process. Endocrine, genetic, social, psychologic, and ethical elements of sex assignment have been integrated into the process. Parent participation is included. The goal is to enable the affected child to grow into a well- adjusted, psychosocially stable person. Chromosome analysis usually takes 2 or 3 days. A physical examination reveals ambiguous genitalia, but additional testing is necessary. A correct gender may not be identifiable.

1. A 12-year-old child is injured in a bicycle accident. When considering the possibility of renal trauma, the nurse should consider what factor? a. Flank pain rarely occurs in children with renal injuries. b. Few nonpenetrating injuries cause renal trauma in children. c. Kidneys are immobile, well protected, and rarely injured in children. d. The amount of hematuria is not a reliable indicator of the seriousness of renal injury.

ANS: D Hematuria is consistently present with renal trauma. It does not provide a reliable indicator of the seriousness of the renal injury. Flank pain results from bleeding around the kidney. Most injuries that cause renal trauma in children are of the nonpenetrating or blunt type and usually involve falls, athletic injuries, and motor vehicle accidents. In children, the kidneys are more mobile, and the outer borders are less protected than in adults.

A parent needs to leave a hospitalized toddler for a short period of time. What action should the nurse suggest to the parent to ease the separation for the toddler? a. Bring a new toy when returning. b. Leave when the child is distracted. c. Tell the child when they will return. d. Leave a favorite article from home with the child.

ANS: D If the parents cannot stay with the child, they should leave favorite articles from home with the child, such as a blanket, toy, bottle, feeding utensil, or article of clothing. Because young children associate such inanimate objects with significant people, they gain comfort and reassurance from these possessions. They make the association that if the parents left this, the parents will surely return. Bringing a new toy would not help with the separation. The parent should not leave when the child is distracted, and toddlers would not understand when the parent should return because time is not a concept they understand.

29. A toddler sustains a minor burn on the hand from hot coffee. What is the first action in treating this burn? a. Apply burn ointment. b. Put ice on the burned area. c. Cover the hand with gauze dressing. D. Hold the hand under cool running water

ANS: D In minor burns, the best method to stop the burning process is to hold the burned area under cool running water. Ointments are not applied to a new burn; the ointment will contribute to the burning. Ice is not recommended. Gauze dressings do not stop the burning process.

1. What do the clinical manifestations of minimal change nephrotic syndrome include? a. Hematuria, bacteriuria, and weight gain b. Gross hematuria, albuminuria, and fever c. Hypertension, weight loss, and proteinuria d. Massive proteinuria, hypoalbuminemia, and edema

ANS: D Massive proteinuria, hypoalbuminemia, and edema are clinical manifestations of minimal change nephrotic syndrome. Hematuria and bacteriuria are not seen, and there is usually weight loss, not gain. The blood pressure is normal or hypotensive.

1. What clinical manifestation(s) is associated with calcium depletion (hypocalcemia)? a. Nausea, vomiting b. Weakness, fatigue c. Muscle hypotonicity d. Neuromuscular irritability

ANS: D Neuromuscular irritability is a clinical manifestation of hypocalcemia. Nausea and vomiting occur with hypercalcemia and hypernatremia. Weakness, fatigue, and muscle hypotonicity are clinical manifestations of hypercalcemia.

1. The nurse is caring for an adolescent who has just started dialysis. The child always seems angry, hostile, or depressed. The nurse should recognize that this is most likely related to what underlying cause? a. Physiologic manifestations of renal disease b. The fact that adolescents have few coping mechanisms c. Neurologic manifestations that occur with dialysis d. Resentment of the control and enforced dependence imposed b ydialysis

ANS: D Older children and adolescents need to feel in control. Dialysis forces the adolescent into a dependent relationship, which results in these behaviors. Being angry, hostile, or depressed are functions of the age of the child, not neurologic or physiologic manifestations of the dialysis.

1. Congenital defects of the genitourinary tract, such as hypospadias, are usually repaired as early as possible to accomplish what? a. Minimize separation anxiety. b. Prevent urinary complications. c. Increase acceptance of hospitalization. d. Promote development of normal body image.

ANS: D Promoting development of normal body image is extremely important. Surgery involving sexual organs can be upsetting to children, especially preschoolers, who fear mutilation and castration. Proper preprocedure preparation can facilitate coping with these issues. Preventing urinary complications is important for defects that affect function, but for all external defects, repair should be done as soon as possible.

29. What finding is the most reliable guide to the adequacy of fluid replacement for a small child with burns? a. Absence of thirst b. Falling hematocrit c. Increased seepage from burn wound d. Urinary output of 1 to 2 ml/kg of body weight/hr

ANS: D Replacement fluid therapy is delivered to provide a urinary output of 30 ml/hr in older children or 1 to 2 ml/kg of body weight/hr for children weighing less than 30 kg (66 lb). Thirst is the result of a complex set of interactions and is not a reliable indicator of hydration. Thirst occurs late in dehydration. A falling hematocrit would be indicative of hemodilution. This may reflect fluid shifts and may not accurately represent fluid replacement therapy. Increased seepage from a burn wound would be indicative of increased output, not adequate hydration.

Secondary prevention for cognitive impairment includes what activity? a. Genetic counseling b. Avoidance of prenatal rubella infection c. Preschool education and counseling services d. Newborn screening for treatable inborn errors of metabolism

ANS: D Secondary prevention involves activities that are designed to identify the condition early and initiate treatment to avert cerebral damage. Inborn errors of metabolism such as hypothyroidism, phenylketonuria, and galactosemia can cause cognitive impairment. Genetic counseling and avoidance of prenatal rubella infections are examples of primary prevention strategies to preclude the occurrence of disorders that can cause cognitive impairment. Preschool education and counseling services are examples of tertiary prevention. These are designed to include early identification of conditions and provision of appropriate therapies and rehabilitation services.

1. What nursing consideration is most important when caring for a child with end-stage renal disease (ESRD)? a. Children with ESRD usually adapt well to minor inconveniences of treatment. b. Children with ESRD require extensive support until they outgrow the condition. c. Multiple stresses are placed on children with ESRD and their families until the illness is cured. d. Multiple stresses are placed on children with ESRD and their families because childrens lives are maintained by drugs and artificial means.

ANS: D Stressors on the family are often overwhelming because of the progressive deterioration. The child progresses from renal insufficiency to uremia to dialysis and transplantation, each of which requires intensive therapy and supportive care. The treatment of ESRD is intense and requires multiple examinations, dietary restrictions, and medications. Adherence to the regimen is often difficult for children and families because of the progressive nature of the renal failure. ESRD has an unrelenting course that has no known cure. Children do not outgrow the renal failure.

1. For minimal change nephrotic syndrome (MCNS), prednisone is effective when what occurs? a. Appetite increases and blood pressure is normal b. Urinary tract infection is gone and edema subsides c. Generalized edema subsides and blood pressure is normal d. Diuresis occurs as urinary protein excretion diminishes

ANS: D Studies suggest that the duration of steroid treatment for the initial episode should be at least 3 months. In most patients, diuresis occurs as the urinary protein excretion diminishes within 7 to 21 days after the initiation of steroid therapy. The blood pressure is normal with MCNS, so remaining so is not an improvement. There is no urinary tract infection with MCNS.

1. An infant has been diagnosed with bladder obstruction. What do symptoms of this disorder include? a. Renal colic b. Strong urinary stream c. Urinary tract infections d. Posturination dribbling

ANS: D Symptoms of bladder obstruction include poor force of urinary stream, intermittency of voided stream, feelings of incomplete bladder emptying, and posturination dribbling. They may also include urinary frequency, nocturia, nocturnal enuresis, and urgency. Renal colic is a symptom of upper urinary tract obstruction. Children with bladder obstruction have a weak urinary stream. Urinary tract infections are not associated with bladder obstruction.

A 6-year-old child is admitted to the pediatric unit and requires bed rest. Having art supplies available meets which purpose? a. Allows the child to create gifts for parents b. Provides developmentally appropriate activities c. Is essential for play therapy so the child can work on past problems d. Lets the child express thoughts and feelings through pictures rather than words

ANS: D The art supplies allow the child to draw images that come into the mind. This can help the child develop symbols and then verbalize reactions to illness and hospitalization. The child can make gifts and drawings for parents, but the goal is to allow expression of feelings. Although art is developmentally and situationally appropriate, the child benefits by being able to express feelings nonverbally. The art supplies are not therapeutic play but a mechanism for expressive play. The child will not work on past problems.

1. A nurse is evaluating the effectiveness of teaching regarding care of a child with minimal change nephrotic syndrome (MCNS) that is in remission after administration of prednisone. The nurse realizes further teaching is required if the parents state what? a. We will keep our child away from anyone who is ill. b. We will be sure to administer the prednisone as ordered. c. We will encourage our child to eat a balanced diet, but we will watch his salt intake. d. We understand our child will not be able to attend school, so we will arrange for home schooling.

ANS: D The child with MCNS in remission can attend school. The child needs socialization and will be socially isolated if home schooled. The other statements are accurate for home care for a child with MCNS.

Many of the clinical features of Down syndrome present challenges to caregivers. Based on these features, what intervention should be included in the childs care? a. Delay feeding solid foods until the tongue thrust has stopped. b. Modify the diet as necessary to minimize the diarrhea that often occurs. c. Provide calories appropriate to the childs mental age. d. Use a cool-mist vaporizer to keep the mucous membranes moist and secretions liquefied.

ANS: D The constant stuffy nose forces the child to breathe by mouth, drying the mucous membranes and increasing the susceptibility to upper respiratory tract infections. A cool-mist vaporizer will keep the mucous membranes moist and liquefy secretions. Respiratory tract infections combined with cardiac anomalies are the primary cause of death in the first years. The child has a protruding tongue, which makes feeding difficult. The parents must persist with feeding while the child continues the physiologic response of the tongue thrust. The child is predisposed to constipation. Calories should be appropriate to the childs weight and growth needs, not mental age.

1. A parent asks the nurse what would be the first indication that acute glomerulonephritis was improving. What would be the nurses best response? a. Blood pressure will stabilize. b. Your child will have more energy. c. Urine will be free of protein. d. Urine output will increase.

ANS: D The first sign of improvement in acute glomerulonephritis is an increase in urinary output with a corresponding decrease in body weight. With diuresis, the child begins to feel better, the appetite improves, and the blood pressure decreases to normal with the reduction of edema. Gross hematuria diminishes, in part because of dilution of the red blood cells in the more dilute urine. Renal function and hypocomplementemia usually normalize by 8 weeks.

29. What is a systemic response to severe burns in a child? a. Metabolic alkalosis b. Decreased metabolic rate c. Increased renal plasma flow d. Abrupt drop in cardiac output

ANS: D The initial physiologic response to a burn injury is a dramatic change in circulation. A precipitous drop in cardiac output precedes any change in circulating blood or plasma volumes. A circulating myocardial depressant factor associated with severe burn injury is thought to be the cause. Metabolic acidosis usually occurs secondary to the disruption of the bodys buffering action resulting from fluid shifting to extravascular space. There is a greatly accelerated metabolic rate in burn patients, supported by protein and lipid breakdown. With the loss of circulating volume, there is decreased renal blood flow and depressed glomerular filtration.

The nurse is instructing student nurses about the stress of hospitalization for children from middle infancy throughout the preschool years. What major stress should the nurse relate to the students? a. Pain b. Bodily injury c. Loss of control d. Separation anxiety

ANS: D The major stress from middle infancy throughout the preschool years, especially for children ages 6 to 30 months, is separation anxiety.

1. What physiologic state(s) produces the clinical manifestations of nervous system stimulation and excitement, such as overexcitability, nervousness, and tetany? a. Metabolic acidosis b. Respiratory alkalosis c. Metabolic and respiratory acidosis d. Metabolic and respiratory alkalosis

ANS: D The major symptoms and signs of alkalosis include nervous system stimulation and excitement, including overexcitability, nervousness, tingling sensations, and tetany that may progress to seizures. Acidosis (both metabolic and respiratory) has clinical signs of depression of the central nervous system, such as lethargy, diminished mental capacity, delirium, stupor, and coma. Respiratory alkalosis has the same symptoms and signs as metabolic alkalosis.

The school nurse is caring for a child with a penetrating eye injury. Emergency treatment includes what intervention? a. Place a cool compress on eye during transport to the emergency department. b. Irrigate the eye copiously with a sterile saline solution. c. Remove the object with a lightly moistened gauze pad. d. Apply a Fox shield to the affected eye and any type of patch to the other eye.

ANS: D The nurses role in a penetrating eye injury is to prevent further injury to the eye. A Fox shield (if available) should be applied to the injured eye and a regular eye patch to the other eye to prevent bilateral movement. Placing cool compress on the eye during transport to emergency department, irrigating eye copiously with a sterile saline solution, or removing object with a lightly moistened gauze pad may cause more damage to the eye.

A spinal tap must be done on a 9-year-old boy. While he is waiting in the treatment room, the nurse observes that he seems composed. When the nurse asks him if he wants his mother to stay with him, he says, I am fine. How should the nurse interpret this situation? a. This child is unusually brave. b. He has learned that support does not help. c. Nine-year-old boys do not usually want a parent present during the procedure. D. Children in this age group often do not request support even though they need and want it

ANS: D The school-age childs visible composure, calmness, and acceptance often mask an inner longing for support. Children of this age have a more passive approach to pain and an indirect request for support. It is especially important to be aware of nonverbal cues such as facial expression, silence, and lack of activity. Usually when someone identifies the unspoken messages, the child will readily accept support.

The nurse is doing a prehospitalization orientation for a girl, age 7 years, who is scheduled for cardiac surgery. As part of the preparation, the nurse explains that after the surgery, the child will be in the intensive care unit. How might the explanation by the nurse be viewed? a. Unnecessary b. The surgeons responsibility c. Too stressful for a young child d. An appropriate part of the childs preparation

ANS: D Theexplanationisanecessarypartofpreoperativepreparationandwillhelpreducetheanxietyassociatedwithsurgery.Ifthechildwakesintheintensivecareunitandisnotpreparedfortheenvironment,shewillbeevenmoreanxious.Thisisajointresponsibilityofnursing,medicalstaff,andchildlifepersonnel

32. The nurse on a pediatric unit is writing guidelines for age-specific preparation of children for procedures based on developmental characteristics. What guideline is accurate? a. Inform toddlers about an upcoming procedure 2 hours before the procedure is to be performed. b. Inform school-age children about an upcoming procedure immediately before the procedure is scheduled to occur. c. Discourage parent presence during procedures on infants and toddlers. d. Use simple diagrams of anatomy and physiology to explain a procedure to a school-age child.

ANS: D To assist the school-age child in meeting Ericksons developmental stage of industry, using simple diagrams of anatomy and physiology to explain a procedure is the accurate guideline. Toddlers should be told about a procedure right before the procedure. School-age children should know about the procedure in advance, not right before, and parents should be present for procedures for infants and toddlers.

1. What laboratory finding, in conjunction with the presenting symptoms, indicates minimal change nephrotic syndrome? a. Low specific gravity b. Decreased hemoglobin c. Normal platelet count D. Reduced serum albumin

ANS: D Total serum protein concentrations are reduced, with the albumin fractions significantly reduced. Specific gravity is high and proportionate to the amount of protein in the urine. Hemoglobin and hematocrit are usually normal or elevated. The platelet count is elevated as a result of hemoconcentration.

The camp nurse is choosing a toy for a child with cognitive impairment to play with during swimming time. What toy should the nurse choose to encourage improvement of developmental skills? a. Dive rings b. An inner tube c. Floating ducks d. A large beach ball

ANS: D Toys are selected for their recreational and educational value. For example, a large inflatable beach ball is a good water toy; encourages interactive play; and can be used to learn motor skills such as balance, rocking, kicking, and throwing. Dive rings, an inner tube, and floating ducks are not interactive toys.

Because of their striving for independence and productivity, which age group of children is particularly vulnerable to events that may lessen their feeling of control and power? a. Infants b. Toddlers c. Preschoolers d. School-age children

ANS: D When a child is hospitalized, the altered family role, physical disability, loss of peer acceptance, lack of productivity, and inability to cope with stress usurp individual power and identity. This is especially detrimental to school-age children, who are striving for independence and productivity and are now experiencing events that decrease their control and power. Infants, toddlers, and preschoolers, although affected by loss of power, are not as significantly affected as school-age children.

The nurse is developing a teaching pamphlet for parents of school-age children. What anticipatory guidelines should the nurse include in the pamphlet? a. At age 6 years, parents should be certain that the child is reading independently with books provided by school. b. At age 8 years, parents should expect a decrease in involvement with peers and outside activities. c. At age 10 years, parents should expect a decrease in admiration of the parents with little interest in parentchild activities. d. At age 12 years, parents should be certain that the childs sex education is adequate with accurate information.

ANS: D A 12-year-old child should have been introduced to sex education, and parents should be certain that the information is adequate and accurate and that the child is not embarrassed to talk about sexual feelings or other aspects of sex education. At age 6 years, a child does not need to be reading independently and usually still needs help with reading and enjoys being read to. At 8 years of age, parents should expect their child to show increased involvement with peers and outside activities and should encourage this behavior. A 10-year-old child exhibits increased feelings of admiration of parents, especially fathers, and parentchild activities should be encouraged.

The nurse is conducting an assessment of fine motor development in a 3-year-old child. Which is the expected drawing skill for this age? a. Can draw a complete stick figure b. Holds the instrument with the fist c. Can copy a triangle and diamond d. Can copy a circle and imitate a cross

ANS: D A 3-year-old child copies a circle and imitates a cross and vertical and horizontal lines. He or she holds the writing instrument with the fingers rather than the fist. A 3-year-old is not able to draw a complete stick figure but draws a circle, later adds facial features, and by age 5 or 6 years can draw several parts (head, arms, legs, body, and facial features). Copying a triangle and diamond are mastered sometime between ages 5 and 6 years.

At a seminar for parents with preschool-age children, the nurse has discussed anticipatory tasks during the preschool years. Which statement by a parent should indicate a correct understanding of the teaching? a. I should be worried if my 4-year-old child has an increase in sexual curiosity because this is a sign of sexual abuse. b. I should expect my 5-year-old to change from a tranquil child to an aggressive child when school starts. c. I should be concerned if my 4-year-old child starts telling exaggerated stories and has an imaginary playmate, since these could be signs of stress. d. I should expect my 3-year-old child to have a more stable appetite and an increase in food selections.

ANS: D A 3-year-old child exhibits a more stable appetite than during the toddler years and is more willing to try different foods. A 4-year-old child is imaginative and indulges in telling tall tales and may have an imaginary playmate; these are normal findings, not signs of stress. Also a 4- year-old child has an increasing curiosity in sexuality, which is not a sign of child abuse. A 5- year-old child is usually tranquil, not aggressive like a 4-year-old child.

The clinic nurse is evaluating an adolescent with menses that have stopped occurring. The nurse understands that which minimum amount of time should the menses be absent after a period of menstruation to be diagnosed as secondary amenorrhea? a. 3 months b. 4 months c. 5 months d. 6 months

ANS: D A 6-month or more cessation of menses after a period of menstruation is secondary amenorrhea.

The nurse is presenting an educational program to a group of parents about differences between anorexia nervosa (AN) and bulimia nervosa (BN) at a community outreach program. What statement by a parent would indicate a need for additional teaching? a. A child with AN will turn away from food to cope, but a child with BN turns to food to cope. b. A child with AN maintains rigid control and is introverted, but a child with BN is an extrovert and frequently loses control. c. A child with AN denies the illness, but a child with BN recognizes the illness. d. A child with AN is usually sexually active and seeks intimacy, but a child with BN avoids intimacy and is usually not sexually active.

ANS: D A child with AN is usually the one who avoids intimacy and is not sexually active, but a child BN often seeks intimacy and is sexually active. A child with AN turns away from food to cope with life, maintains rigid control, is introverted, and denies the illness. A child with BN turns to food to cope, is an extrovert who loses control, and recognizes that he or she has an illness.

During a well-child visit, the nurse plots the childs BMI on the health record. What is the purpose of the BMI? a. To determine medication dosages b. To predict adult height and weight c. To identify coping strategies used by the child d. To provide a consistent measure of obesity

ANS: D A consistent measure of the degree of obesity is important to determine whether modification of the body fat component is indicated. Body surface area (BSA), not BMI, is used for medication dosage calculation. The BMI is not a predictor of adult height. A child with a high BMI may use food as a coping mechanism, but the BMI is not correlated with coping strategy use.

What is the result of acute salicylate (ASA, aspirin) poisoning? a. Chemical pneumonitis b. Hepatic damage c. Retractions and grunting d. Disorientation and loss of consciousness

ANS: D ASA poisoning causes disorientation and loss of consciousness. Chemical pneumonitis is caused by hydrocarbon ingestion. Hepatic damage is caused by acetaminophen overdose. ASA does not cause airway obstruction.

1. What recommendation should the nurse make to prevent urinary tract infections (UTIs) in young girls? a. Avoid public toilet facilities. b. Limit long baths as much as possible. c. Cleanse the perineum with water after voiding. d. Ensure clear liquid intake of 2 L/day.

ANS: D Adequate fluid intake minimizes urinary stasis. The recommended fluid intake is 50 ml/kg or 100 ml/lb per day. The average 5- to 6-year-old weighs approximately 18 kg (40 lb), so she should drink 2 L/day of fluid. There is no evidence that using public toilet facilities increases UTIs. Long baths are not associated with increased UTIs. Proper hand washing and perineal cleansing are important, but no evidence exists that these decrease UTIs in young girls.

Many adolescents use alcohol for self-medication. How does an adolescent view the benefit of alcohol? a. Believes it has a stimulant effect b. Believes it increases alertness c. Provides a sense of euphoria d. Provides a defense against depression

ANS: D Adolescents who abuse alcohol often rely on it as a defense against depression, anxiety, fear, and anger. Alcohol is a depressant and has a sedative effect. Alcohol does not provide a sense of euphoria. It does reduce inhibitions against aggressive behaviors.

What factor is most likely to increase the likelihood that an adolescent will misuse alcohol? a. Female gender b. Regular school attendance c. Rural environment d. Unconventional behavior

ANS: D Adolescents who are connected and engage in conventional behavior are less likely to misuse alcohol. Those who are disconnected from school, family, and other social supports have fewer assets and are more likely to abuse alcohol. School attendance is a sign of connectedness. Girls and boys report a similar onset and course of experimentation with alcohol. Urban youths have a higher likelihood of alcohol abuse than rural adolescents.

At what developmental period do children have the most difficulty coping with death, particularly if it is their own? a. Toddlerhood b. Preschool c. School-age d. Adolescence

ANS: D Adolescents, because of their mature understanding of death, remnants of guilt and shame, and issues with deviations from normal, have the most difficulty coping with death.

A young child has just arrived at the emergency department after ingestion of aspirin at home. The practitioner has ordered activated charcoal. The nurse administers charcoal in which way? a. Administer through a nasogastric tube because the child will not drink it because of the taste. b. Serve in a clear plastic cup so the child can see how much has been drunk. c. Give half of the solution, and then give the other half in 1 hour. d. Serve in an opaque container with a straw.

ANS: D Although the activated charcoal can be mixed with a flavorful beverage, it will be black and resemble mud. When it is served in an opaque container, the child does not have any preconceived ideas about its being distasteful. The nasogastric tube should be used only in children without a gag reflex. The ability to see the charcoal solution may affect the child's desire to drink it. The child should be encouraged to drink the solution all at once.

The nurse is preparing a pamphlet for parents of adolescents about guidance during the adolescent years. What suggestion should the nurse include in the pamphlet? a. Provide criticism when mistakes are made or when views are different. b. Use comparisons with older siblings or extended family to promote good outcomes. c. Begin to disengage from school functions to allow the adolescent to gain independence. d. Provide clear, reasonable limits and define consequences when rules are broken.

ANS: D An anticipatory guideline to include when teaching parents of adolescents is to provide clear, reasonable limits and have clear consequences when rules are broken. Parents should avoid criticism when mistakes are made and should allow opportunities for the teen to voice different views and opinions. Parents should try to avoid comparing the teen with a sibling or extended family member. Parents should try to be more engaged in the teens school functions to show support and unconditional love

A child with acetylsalicylic acid (aspirin) poisoning is being admitted to the emergency department. What early clinical manifestation does the nurse expect to assess on this child? a. Hematemesis b. Hematochezia c. Hyperglycemia d. Hyperventilation

ANS: D An early clinical manifestation of acetylsalicylic acid (aspirin) poisoning is hyperventilation. Hematemesis, hematochezia, and hyperglycemia are clinical manifestations of iron poisoning.

A mother asks the nurse what would be the first indication that acute glomerulonephritis is improving. What is the nurse's best response? a. Blood pressure will stabilize. b. The child will have more energy. c. Urine will be free of protein. d. Urinary output will increase.

ANS: D An increase in urinary output may signal resolution of the acute glomerulonephritis. If blood pressure is elevated, stabilization usually occurs with the improvement in renal function. The child having more energy and the urine being free of protein are related to the improvement in urinary output.

The nurse is teaching an adolescent about acne care. What statement by the adolescent indicates a need for further teaching? a. I will cleanse my face twice a day. b. I will frequently shampoo my hair. c. I will brush my hair away from my forehead. d. I will use my antibacterial soap to cleanse my face.

ANS: D Antibacterial soaps are ineffective and may be drying when used in combination with topical acne medications. Further teaching is needed if the adolescent indicates using antibacterial soap. Gentle cleansing with a mild cleanser once or twice daily is usually sufficient. For some adolescents, hygiene of the hair and scalp appears to be related to the clinical activity of acne. Acne on the forehead may improve with brushing the hair away from the forehead and more frequent shampooing.

Which type of play is most typical of the preschool period? a. Team b. Parallel c. Solitary d. Associative

ANS: D Associative play is group play in similar or identical activities but without rigid organization or rules. School-age children play in teams. Parallel play is that of toddlers. Solitary play is that of infants.

At what age should the nurse expect a child to give both first and last names when asked? a. 15 months b. 18 months c. 24 months d. 30 months

ANS: D At 30 months, the child is able to give both first and last names and refer to self with an appropriate pronoun. At 15 and 18 months, the child is too young to give his or her own name. At 24 months, the child is able to give first name and refer to self by that name.

By which age should the nurse expect that most children could obey prepositional phrases such as "under," "on top of," "beside," and "behind"? a. 18 months b. 24 months c. 3 years d. 4 years

ANS: D At 4 years, children can understand directional phrases. Children at 18 months, 24 months, and 3 years are too young.

3.The developmental task with which the child of 15 to 30 months is likely to be struggling is a sense of which? a. Trust b. Initiative c. Intimacy d. Autonomy

ANS: D Autonomy versus shame and doubt is the developmental task of toddlers. Trust versus mistrust is the developmental stage of infancy. Initiative versus guilt is the developmental stage of early childhood. Intimacy and solidarity versus isolation is the developmental stage of early adulthood.

What characteristic best describes the language skills of a 3-year-old child? a. Asks meanings of words b. Follows directional commands c. Can describe an object according to its composition d. Talks incessantly regardless of whether anyone is listening

ANS: D Because of the dramatic vocabulary increase at this age, 3-year-old children are known to talk incessantly regardless of whether anyone is listening. A 4- to 5-year-old child asks lots of questions and can follow simple directional commands. A 6-year-old child can describe an object according to its composition

Which characteristic best describes the language of a 3-year-old child? a. Asks meanings of words b. Follows directional commands c. Describes an object according to its composition d. Talks incessantly regardless of whether anyone is listening

ANS: D Because of the dramatic vocabulary increase at this age, 3-year-olds are known to talk incessantly regardless of whether anyone is listening. A 4- to 5-year-old asks lots of questions and can follow simple directional commands. A 6-year-old can describe an object according to its composition.

The nurse must do a heel stick on an ill neonate to obtain a blood sample. What action is recommended to facilitate blood flow? a. Apply cool, moist compresses. b. Apply a tourniquet to the ankle. c. Elevate the foot for 5 minutes. d. Wrap the foot in a warm washcloth.

ANS: D Before the blood sample is taken, the heel is heated with warm moist compresses for 5 to 10 minutes to dilate the blood vessels in the area. Cooling causes vasoconstriction, making blood collection more difficult. A tourniquet is used to constrict superficial veins. It will have an insignificant effect on capillaries. Elevating the foot will decrease the blood in the foot available for collection.

At what blood level is chelation therapy for lead poisoning initiated in a child? a. 10 to 14 g/dl b. 15 to 19 g/dl c. 20 to 44 g/dl d. ≥45 g/dl

ANS: D Chelation therapy is initiated if the child's blood level is greater than or equal to 45 g/dl. At 10 to 14 g/dl, the family should have lead-poisoning education and follow-up level. At 15 to 19 g/dl, the family should have lead-poisoning education and follow-up level but if it persists, initiate environmental investigation. At 20 to 44 g/dl environmental investigation and lead hazard control are necessary

The nurse is assisting the family of a child with a history of encopresis. What should be included in the nurses discussion with the family? a. Instruct the parents to sit the child on the toilet at twice-daily routine intervals. b. Instruct the parents that the child will probably need to have daily enemas. c. Suggest the use of stimulant cathartics weekly. d. Reassure the family that most problems are resolved successfully, with some relapses during periods of stress.

ANS: D Children may be unaware of a prior sensation and be unable to control the urge after it begins. They may be so accustomed to bowel accidents that they may be unable to smell or feel them. Family counseling is directed toward reassurance that most problems resolve successfully, although relapses during periods of stress are possible. Sitting the child on the toilet is not recommended because it may intensify the parentchild conflict. Enemas may be needed for impactions, but long-term use prevents the child from assuming responsibility for defecation. Stimulant cathartics may cause cramping that can frighten children.

The nurse is assisting the family of a child with a history of encopresis. Which should be included in the nurse's discussion with this family? a. Instruct the parents to sit the child on the toilet at twice-daily routine intervals. b. Instruct the parents that the child will probably need to have daily enemas. c. Suggest the use of stimulant cathartics weekly. d. Reassure the family that most problems are resolved successfully, with some relapses during periods of stress.

ANS: D Children may be unaware of a prior sensation and unable to control the urge once it begins. They may be so accustomed to bowel accidents that they are unable to smell or feel it. Family counseling is directed toward reassurance that most problems resolve successfully, although relapses during periods of stress are possible. Sitting the child on the toilet is not recommended because it may intensify the parent-child conflict. Enemas may be needed for impactions, but long-term use prevents the child from assuming responsibility for defecation. Stimulant cathartics may cause cramping that can frighten the child.

The school nurse is conducting a class on bicycle safety. Which statement made by a participant indicates a need for further teaching? a. "Most bicycle injuries occur from a fall off the bicycle." b. "Head injuries are the major causes of bicycle-related fatalities." c. "I should replace my helmet every 5 years." d. "I can ride double with a friend if the bicycle has an extra-large seat."

ANS: D Children should not ride double. Most injuries result from falls. The most important aspect of bicycle safety is to encourage the rider to use a protective helmet. Head injuries are the major cause of bicycle-related fatalities. The child should always wear a properly fitted helmet approved by the US Consumer Product Safety Commission and should replace the helmet at least every 5 years.

Which clinical manifestation would be seen in a child with chronic renal failure? a. Hypotension b. Massive hematuria c. Hypokalemia d. Unpleasant "uremic" breath odor

ANS: D Children with chronic renal failure have a characteristic breath odor resulting from the retention of waste products. Hypertension may be a complication of chronic renal failure. With chronic renal failure, little or no urinary output occurs. Hyperkalemia is a concern in chronic renal failure.

A nurse is preparing a teaching session for parents on prevention of childhood hearing loss. What is the most common cause of hearing impairment in children? a. Auditory nerve damage b. Congenital ear defects c. Congenital rubella d. Chronic otitis media

ANS: D Chronic otitis media is the most common cause of hearing impairment in children. It is essential that appropriate measures be instituted to treat existing infections and prevent recurrences. Auditory nerve damage, congenital ear defects, and congenital rubella are rarer causes of hearing impairment.

What finding by the nurse is most characteristic of chronic sorrow? a. Lack of acceptance of childs limitation b. Lack of available support to prevent sorrow c. Periods of intensified sorrow when experiencing anger and guilt d. Periods of intensified sorrow at certain landmarks of the childs development

ANS: D Chronic sorrow is manifested by feelings of sorrow and loss that recur in waves over time. The sorrow is a response to the recognition of the childs limitations. The family should be assessed in an ongoing manner to provide appropriate support as their needs change. The sorrow is not preventable. The chronic sorrow occurs during the reintegration and acknowledgment stage.

Most parents of children with special needs tend to experience chronic sorrow. What characterizes chronic sorrow? a. Lack of acceptance of the child's limitation b. Lack of available support to prevent sorrow c. Periods of intensified sorrow when experiencing anger and guilt d. Periods of intensified sorrow and loss that occur in waves over time

ANS: D Chronic sorrow is manifested by feelings of sorrow and loss that recur in waves over time. The sorrow is in response to the recognition of the child's limitations. The family should be assessed in an ongoing manner to provide appropriate support as the needs of the family change. The sorrow is not preventable. The chronic sorrow occurs during the reintegration and acknowledgment stage.

According to Piaget, adolescents tend to be in what stage of cognitive development? a. Concrete operations b. Conventional thought c. Postconventional thought d. Formal operational thought

ANS: D Cognitive thinking culminates in the capacity for abstract thinking. This stage, the period of formal operations, is Piagets fourth and last stage. Concrete operations usually occur between ages 7 and 11 years. Conventional and postconventional thought refers to Kohlbergs stages of moral development

Which is probably the most important criterion on which to base the decision to report suspected child abuse? a. Inappropriate parental concern for the degree of injury b. Absence of parents for questioning about child's injuries c. Inappropriate response of child d. Incompatibility between the history and injury observed

ANS: D Conflicting stories about the "accident" are the most indicative red flags of abuse. Inappropriate response of caregiver or child may be present, but is subjective. Parents should be questioned at some point during the investigation.

What is probably the most important criterion on which to base the decision to report suspected child abuse? a. Inappropriate response of child b. Inappropriate parental concern for the degree of injury c. Absence of parents for questioning about childs injuries d. Incompatibility between the history and injury observed

ANS: D Conflicting stories about the accident are the most indicative red flags of abuse. The child or caregiver may have an inappropriate response, but this is subjective. Parents should be questioned at some point during the investigation.

What is true concerning the development of autonomy during adolescence? a. Development of autonomy typically involves rebellion. b. Development of autonomy typically involves parentchild conflicts. c. Parent and peer influences are opposing forces in the development of autonomy. d. Conformity to both parents and peers gradually declines toward the end of adolescence.

ANS: D During middle and late adolescence, the conformity to parents and peers declines. Subjective feelings of self-reliance increase steadily over the adolescent years. Adolescents have genuine behavioral autonomy. Rebellion is not typically part of adolescence. It can occur in response to excessively controlling circumstances or to growing up in the absence of clear standards. Parent and peer relationships can play complementary roles in the development of a healthy degree of individual independence.

Trisomy 13, trisomy 18, and trisomy 21 have which in common? a. Viability is rare. b. They are considered deletion syndromes. c. Diagnosis is difficult, time-consuming, and expensive. d. Diagnosis can be made early, based on physical characteristics.

ANS: D Each of these disorders, trisomy 13, 18, and 21, has unique physical characteristics. A presumptive diagnosis can often be made soon after birth and later confirmed by chromosomal analysis. Children with trisomy 13 and 18 usually have short life expectancies. Trisomy 21 has a variable life expectancy, with 80% of individuals living to age 30 years. Trisomy 13, trisomy 18, and trisomy 21 are not deletion syndromes.

The nurse suspects that a child has ingested some type of poison. Which clinical manifestation would be most suggestive that the poison was a corrosive product? a. Tinnitus b. Disorientation c. Stupor, lethargy, coma d. Edema of lips, tongue, pharynx

ANS: D Edema of lips, tongue, and pharynx indicates a corrosive ingestion. Tinnitus is indicative of aspirin ingestion. Corrosives do not act on the central nervous system (CNS).

The nurse suspects that a child has ingested some type of poison. What clinical manifestation would be most suggestive that the poison was a corrosive product? a. Tinnitus b. Disorientation c. Stupor, lethargy, and coma d. Edema of the lips, tongue, and pharynx

ANS: D Edema of the lips, tongue, and pharynx indicates a corrosive ingestion. Tinnitus is indicative of aspirin ingestion. Corrosives do not act on the central nervous system.

During the preschool period, the emphasis of injury prevention should be placed on what? a. Limitation of physical activities b. Punishment for unsafe behaviors c. Constant vigilance and protection d. Teaching about safety and potential hazards

ANS: D Education about safety and potential hazards is appropriate for preschoolers because they can begin to understand dangers. Limitation of physical activities is not appropriate. Punishment may make children scared of trying new things. Constant vigilance and protection are not practical at this age because preschoolers are becoming more independent.

The nurse is doing a pre-hospitalization orientation for a 7-year-old child who is scheduled for cardiac surgery. As part of the preparation, the nurse explains that she will not be able to talk because of an endotracheal tube but that she will be able to talk when it is removed. This explanation is: a. unnecessary. b. the surgeon's responsibility. c. too stressful for a young child. d. an appropriate part of the child's preparation.

ANS: D Explanation is a necessary part of preoperative preparation. If the child wakes and is not prepared for the inability to speak, she will be even more anxious. This is a necessary component for preparation for surgery that will help reduce the anxiety associated with surgery. It is a joint responsibility of nursing, medical staff, and child life personnel.

An adolescent girl asks the school nurse for advice because she has dysmenorrhea. She says that a friend recommended she try an over-the-counter nonsteroidal anti-inflammatory drug (NSAID). The nurses response should be based on what? a. Hormone therapy is necessary for the treatment of dysmenorrhea. b. Acetaminophen is the drug of choice for the treatment of dysmenorrhea. c. Over-the-counter NSAIDs are rarely strong enough to provide adequate pain relief. d. NSAIDs are effective because they inhibit prostaglandins, leading to reduction in uterine activity.

ANS: D First-line therapy for adolescents with dysmenorrhea is NSAIDs. NSAIDs are potent antiinflammatory agents that block the formation of prostaglandins, resulting in decreased uterine activity. Hormone therapy may be indicated if there is no physical abnormality and NSAIDs are ineffective. Acetaminophen does not have an antiprostaglandin action. It can help with pain control but will not be as effective as NSAIDs.

An adolescent girl asks the school nurse for advice because she has dysmenorrhea. She says that a friend recommended she try an over-the-counter nonsteroidal anti-inflammatory drug (NSAID). The nurse's response should be based on which statement? a. Aspirin is the drug of choice for the treatment of dysmenorrhea. b. Over-the-counter NSAIDs are rarely strong enough to provide adequate pain relief. c. NSAIDs are effective because of their analgesic effect. d. NSAIDs are effective because they inhibit prostaglandins, leading to reduction in uterine activity.

ANS: D First-line therapy for adolescents with dysmenorrhea is NSAIDs. This group of drugs blocks the formation of prostaglandins. NSAIDs, not aspirin, are the drugs of choice in dysmenorrhea. NSAIDs are potent anti-inflammatory agents that inhibit prostaglandin. Although NSAIDs have analgesic effects, the mechanism of action in dysmenorrhea is most likely the antiprostaglandin effect.

What is the most common cause of death in the adolescent age group? a. Drownings b. Firearms c. Drug overdoses d. Motor vehicles

ANS: D Forty percent of all adolescent deaths in the United States are the result of motor vehicle accidents. Drownings, firearms, and drug overdoses are major concerns in adolescence but are not the most common cause of death.

The school nurse recognizes that pubertal delay in girls is considered if breast development has not occurred by which age? a. 10 years b. 11 years c. 12 years d. 13 years

ANS: D Girls may be considered to have pubertal delay if breast development has not occurred by age 13 years or if menarche has not occurred within 2 to 2 1/2 years of the onset of breast development

What nursing intervention is especially helpful in assessing feelings of parental guilt when a disability or chronic illness is diagnosed? a. Ask the parents if they feel guilty. b. Observe for signs of overprotectiveness. c. Talk about guilt only after the parents mention it. d. Discuss the meaning of the parents religious and cultural background.

ANS: D Guilt may be associated with cultural or religious beliefs. Some parents are convinced that they are being punished for some previous misdeed. Others may see the disorder as a trial sent by God to test their religious beliefs. The nurse can help the parents explore their religious beliefs. On direct questioning, the parents may not be able to identify the feelings of guilt. It would be appropriate for the nurse to explore their adjustment responses. Overprotectiveness is a parental response during the adjustment phase. The parents fear letting the child achieve any new skill and avoid all discipline.

The nurse is admitting a newborn with hypospadias to the nursery. The nurse expects which finding in this newborn? a. Absence of a urethral opening is noted. b. Penis appears shorter than usual for age. c. The urethral opening is along the dorsal surface of the penis. d. The urethral opening is along the ventral surface of the penis.

ANS: D Hypospadias is a congenital condition in which the urethral opening is located anywhere along the ventral surface of the penis. The urethral opening is present, but not at the glans. Hypospadias refers to the urethral opening, not to the size of the penis. A urethral opening along the ventral surface of the penis is known as epispadias

What is one of the major physical characteristics of the child with Down syndrome? a. Excessive height b. Spots on the palms c. Inflexibility of the joints d. Hypotonic musculature

ANS: D Hypotonic musculature is one of the major characteristics. Children with Down syndrome have short stature and a transverse palmar crease. Hyperflexibility is a characteristic of Down syndrome

A 9-year-old boy has an unplanned admission to the intensive care unit (ICU) after abdominal surgery. The nursing staff has completed the admission process, and his condition is beginning to stabilize. When speaking with the parents, the nurse should expect what additional stressor to be evident? a. Usual daynight routine b. Calming influence of staff c. Adequate privacy and support d. Insufficient remembering of his condition and routine

ANS: D ICUs, especially when the family is unprepared for the admission, are strange and unfamiliar. There are many pieces of unfamiliar equipment, and the sights and sounds are much different from those of a general hospital unit. Also, with the childs condition being more precarious, it may be difficult to keep the parents updated on what is happening. Lights are usually on around the clock, seriously disrupting the diurnal rhythm. In most ICUs, the staff works with a sense of urgency. It is difficult for parents to ask questions about their child when staff is with other patients. Usually little privacy is available for families in ICUs

What do the psychosocial developmental tasks of toddlerhood include? a. Development of a conscience b. Recognition of sex differences c. Ability to get along with age mates d. Ability to delay gratification

ANS: D If the need for basic trust has been satisfied, then toddlers can give up dependence for control, independence, and autonomy. One of the tasks that toddlers are concerned with is the ability to delay gratification. Development of a conscience and recognition of sex differences occur during the preschool years. The ability to get along with age mates develops during the preschool and school-age years.

The psychosocial developmental tasks of toddlerhood include which characteristic? a. Development of a conscience b. Recognition of sex differences c. Ability to get along with age-mates d. Ability to delay gratification

ANS: D If the need for basic trust has been satisfied, then toddlers can give up dependence for control, independence, and autonomy. One of the tasks that the toddler is concerned with is the ability to delay gratification. Development of a conscience occurs during the preschool years. The recognition of sex differences occurs during the preschool years. The ability to get along with age-mates develops during the preschool and school-age years.

20. What do inflicted immersion burns often appear as? a. Partial-thickness, asymmetrical burns b. Splash pattern burns on hands or feet c. Any splash burn with dry linear marks d. Sharply demarcated, symmetrical burns

ANS: D Immersion burns are sharply demarcated symmetrical burns. Asymmetrical burns and splash burns are often accidental.

Which describes the cognitive abilities of school-age children? a. Have developed the ability to reason abstractly b. Are capable of scientific reasoning and formal logic c. Progress from making judgments based on what they reason to making judgments based on what they see d. Are able to classify, to group and sort, and to hold a concept in their minds while making decisions based on that concept

ANS: D In Piaget's stage of concrete operations, children have the ability to group and sort and make conceptual decisions. Children cannot reason abstractly and logically until late adolescence. Making judgments based on what they reason to making judgments based on what they see is not a developmental skill.

The nurse is assessing the Tanner stage in an adolescent male. The nurse recognizes that the stages are based on what? a. Hair growth on the face and chest b. Changes in the voice to a deeper timbre c. Muscle growth in the arms, legs, and shoulders d. Size and shape of the penis and scrotum and distribution of pubic hair

ANS: D In males, the Tanner stages describe pubertal development based on the size and shape of the penis and scrotum and the shape and distribution of pubic hair. During puberty, hair begins to grow on the face and chest; the voice becomes deeper; and muscles grow in the arms, legs, and shoulders, but these are not used for the Tanner stages.

Adolescents often do not use reasoned decision making when issues such as substance abuse and sexual behavior are involved. What is this because of? a. They tend to be immature. b. They do not need to use reasoned decision making. c. They lack cognitive skills to use reasoned decision making. d. They are dealing with issues that are stressful and emotionally laden.

ANS: D In the face of time pressures, personal stress, or overwhelming peer pressure, young people are more likely to abandon rational thought processes. Many of the health-related decisions adolescents confront are emotionally laden or new. Under such conditions, many people do not use their capacity for formal decision making. The majority of adolescents have cognitive skills and are capable of reasoned decision making. Stress affects their ability to process information. Reasoned decision making should be used in issues that are crucial such as substance abuse and sexual behavior.

The nurse outlines short- and long-term goals for a 10-year-old child with many complex health problems. Who should agree on these goals? a. Family and nurse b. Child, family, and nurse c. All professionals involved d. Child, family, and all professionals involved

ANS: D In the home, the family is a partner in each step of the nursing process. The family priorities should guide the planning process. Both short- and long-term goals should be outlined and agreed on by the child, family, and professionals involved. Elimination of any one of these groups can potentially create a care plan that does not meet the needs of the child and family.

Which is descriptive of bulimia during adolescence? a. Strong sense of control over eating behavior b. Feelings of elation after the binge-purge cycle c. Profound lack of awareness that the eating pattern is abnormal d. Weight that can be normal, slightly above normal, or below normal

ANS: D Individuals with bulimia are of normal or more commonly slightly above normal weight. Those who also restrict their intake can become severely underweight. The adolescent has a lack of control over eating during the episode. Patients with bulimia commonly have self-deprecating thoughts and a depressed mood after binge-purge cycles; they are also aware that the eating pattern is abnormal but are unable to stop.

What should the nurse determine to be the priority intervention for a family with an infant who has a disability? a. Focus on the childs disabilities to understand care needs. b. Institute age-appropriate discipline and limit setting. c. Enforce visiting hours to allow parents to have respite care. d. Foster feelings of competency by helping parents learn the special care needs of the infant.

ANS: D It is important that the parents learn how to care for their infant so they feel competent. The nurse facilitates this by teaching special holding techniques, supporting breastfeeding, and encouraging frequent visiting and rooming in. The focus should be on the infants capabilities and positive features. Infants do not usually require discipline. As the child gets older, this is necessary, but it is not a priority intervention at this time. The nursing staff negotiates with the family about the need for respite care.

It is important to make certain that sensory connectors and oximeters are compatible. What can incompatible wiring cause? a. Hyperthermia b. Electrocution c. Pressure necrosis d. Burns under sensors

ANS: D It is important to make certain that sensor connectors and oximeters are compatible. Wiring that is incompatible can generate considerable heat at the tip of the sensor, causing second- and third-degree burns under the sensor. Incompatibility would cause a local irritation or burn. A low voltage is used, which should not present risk of electrocution. Pressure necrosis can occur from the sensor being attached too tightly, but this is not a problem of incompatibility.

A school-age child falls on the playground and has a small laceration on the forearm. The school nurse should do which to cleanse the wound? a. Slowly pour hydrogen peroxide over wound. b. Soak arm in warm water and soap for at least 30 minutes. c. Gently cleanse with sterile pad and a non-stinging povidone-iodine solution. d. Wash wound gently with mild soap and water for several minutes.

ANS: D Lacerations should be washed gently with mild soap and water or normal saline. A sterile pad is not necessary, and hydrogen peroxide and povidone-iodine should not be used because they have a cytotoxic effect on healthy cells and minimal effect on controlling infection. Soaking the arm will not effectively clean the wound.

A young boy is found squirting lighter fluid into his mouth. His father calls the emergency department. The nurse taking the call should know that the primary danger is which result? a. Hepatic dysfunction b. Dehydration secondary to vomiting c. Esophageal stricture and shock d. Bronchitis and chemical pneumonia

ANS: D Lighter fluid is a hydrocarbon. The immediate danger is aspiration. Acetaminophen overdose, not hydrocarbons, causes hepatic dysfunction. Dehydration is not the primary danger. Esophageal stricture is a late or chronic issue of hydrocarbon ingestion.

The nurse is caring for an adolescent who has just started dialysis. The child seems always angry, hostile, or depressed. To what are these behaviors most likely related? a. Neurologic manifestations that occur with dialysis b. Physiologic manifestations of renal disease c. Adolescents having few coping mechanisms d. Adolescents often resenting the control and enforced dependence imposed by dialysis

ANS: D Older children and adolescents need control. The necessity of dialysis forces the adolescent into a dependent relationship, which results in these behaviors. These are a function of the child's age, not neurologic or physiologic manifestations of the dialysis. Feelings of anger, hostility, and depression are functions of the child's age, not neurologic or physiologic manifestations of the dialysis. Adolescents do have coping mechanisms, but they need to have some control over their disease management.

What statement characterizes moral development in the older school-age child? a. Rule violations are viewed in an isolated context. b. Judgments and rules become more absolute and authoritarian. c. The child remembers the rules but cannot understand the reasons behind them. d. The child is able to judge an act by the intentions that prompted it rather than just by the consequences.

ANS: D Older school-age children are able to judge an act by the intentions that prompted the behavior rather than just by the consequences. Rule violation is likely to be viewed in relation to the total context in which it appears. Rules and judgments become less absolute and authoritarian. The situation and the morality of the rule itself influence reactions.

The nurse is assessing a 20-month-old toddler during a well-child visit and notices tooth decay. The nurse should understand that early childhood caries are caused by what? a. Allowing the child to eat citrus foods at bedtime b. A hereditary factor that cannot be prevented c. Poor fluoride supply in the drinking water d. Giving the child a bottle of juice or milk at naptime

ANS: D One cause of early childhood caries is allowing the child to go to sleep with a bottle of milk or juice; as the sweet liquid pools in the mouth, the teeth are bathed for several hours in this cariogenic environment. Eating citrus fruit at bedtime and poor fluoride supply in drinking water do not cause early childhood caries. The problem is not hereditary and can be prevented with proper education.

A nurse is observing children playing in the playroom. What describes parallel play? a. A child playing a video game b. Two children playing a card game c. Two children watching a movie on a television d. A child playing with blocks next to a child playing with trucks

ANS: D Parallel play is when a toddler plays alongside, not with, other children. A child playing with blocks next to a child playing with trucks is descriptive of parallel play. The child playing a video game is descriptive of solitary play. Two children playing cards is descriptive of cooperative play. Two children watching a television is descriptive of associative play.

Parents ask the nurse for strategies to help their toddler adjust to a new baby. What should the nurse suggest? a. Start talking about the baby very early in the pregnancy. b. Move the toddler to a new bed after the baby comes home. c. Tell the toddler that a new playmate will be coming home soon. d. Alert visitors to the new baby to include the toddler in the visit.

ANS: D Parents can minimize sibling rivalry by alerting visitors to the toddlers needs, having small presents on hand for the toddler, and including the child in the visits as much as possible. Time is a vague concept for toddlers. A good time to start talking about the new baby is when the toddler becomes aware of the pregnancy and the changes occurring in the home in anticipation of the new member. To avoid additional stresses when the newborn arrives, parents should perform anticipated changes, such as moving the toddler to a different room or bed, well in advance of the birth. Telling the toddler that a new playmate will come home soon sets up unrealistic expectations.

The nurse is teaching parents of toddlers about animal safety. Which should be included in the teaching session? a. Petting dogs in the neighborhood should be encouraged to prevent fear of dogs. b. The toddler is safe to approach an animal if the animal is chained. c. It is permissible for your toddler to feed treats to a dog. d. Teach your toddler not to disturb an animal that is eating.

ANS: D Parents should be taught that toddlers should not disturb an animal that is eating, sleeping, or caring for young puppies or kittens. The child should avoid all strange animals and not be encouraged to pet dogs in the neighborhood. The child should never approach a strange dog that is confined or restrained. The inexperienced child should not feed a dog (if the child pulls back when the animal moves to take the food, this can frighten and startle the animal).

What is the role of the peer group in the life of school-age children? a. Decreases their need to learn appropriate sex roles b. Gives them an opportunity to learn dominance and hostility c. Allows them to remain dependent on their parents for a longer time d. Provides them with security as they gain independence from their parents

ANS: D Peer group identification is an important factor in gaining independence from parents. Through peer relationships, children learn ways to deal with dominance and hostility. They also learn how to relate to people in positions of leadership and authority and how to explore ideas and the physical environment. A childs concept of appropriate sex roles is influenced by relationships with peers.

Parents ask the nurse whether it is common for their school-age child to spend a lot of time with peers. The nurse should respond, explaining that the role of the peer group in the life of school-age children provides: a. opportunity to become defiant. b. time to remain dependent on their parents for a longer time. c. time to establish a one-on-one relationship with the opposite sex. d. security as they gain independence from their parents.

ANS: D Peer-group identification is an important factor in gaining independence from parents. Children learn how to relate to people in positions of leadership and authority and how to explore ideas and the physical environment. Becoming defiant in a peer-group relationship may lead to bullying. Peer-group identification helps in gaining independence rather than remaining dependent. One-on-one opposite sex relationships do not occur until adolescence. School-age children form peer groups of the same sex.

What strategy is considered one of the best for preventing smoking in teenagers? a. Large-scale printed information campaigns b. Emphasis on the long-term effects of smoking on health c. Threatening the social norms of groups most likely to smoke d. Peer-led programs emphasizing the social consequences of smoking

ANS: D Peer-led programs emphasizing the social consequences of smoking have proved most successful. Short-term effects such as an unpleasant odor and stains on the teeth and hands are stressed. If a significant number of peers convince their classmates that smoking is not popular, others will follow. Large-scale printed information campaigns are not effective. A specified curriculum and teaching can increase benefit. Long-term effects do not dissuade adolescents because they do not have a future perspective. Threatening the norms of the social group is one of the least effective means of prevention.

What is an advantage of peritoneal dialysis? a. Treatments are done in hospitals. b. Protein loss is less extensive. c. Dietary limitations are not necessary. d. Parents and older children can perform treatments.

ANS: D Peritoneal dialysis is the preferred form of dialysis for parents, infants, and children who wish to remain independent. Parents and older children can perform the treatments themselves. Treatments can be done at home. Protein loss is not significantly different. The dietary limitations are necessary, but they are not as stringent as those for hemodialysis.

How should the nurse administer a gavage feeding to a school-age child? a. Lubricate the tip of the feeding tube with Vaseline to facilitate passage. b. Check the placement of the tube by inserting 20 ml of sterile water. c. Administer feedings over 5 to 10 minutes. d. Position the patient on the right side after administering feeding.

ANS: D Position the child with the head elevated about 30 degrees and on the right side or abdomen for at least 1 hour. This is in the same manner as after any infant feeding to minimize the possibility of regurgitation and aspiration. Insert a tube that has been lubricated with sterile water or water-soluble lubricant. With a syringe, inject a small amount of air into the tube, while simultaneously listening with a stethoscope over the stomach area. Feedings should be administered via gravity flow and take from 15 to 30 minutes to complete.

Parents of a preschool child tell the nurse, Our child seems to have many imaginary fears. What suggestion should the nurse give to the parents to help their child resolve the fears? a. Ignore the fears; they will go away. b. Explain to your child the fears are not real. c. Give your child some new toys to allay the fears. d. Help your child to resolve the fears through play activities.

ANS: D Preschoolers are able to work through many of their unresolved fears, fantasies, and anxieties through play, especially if guided with appropriate play objects (e.g., dolls or puppets) that represent family members, health professionals, and other children. The fears should not be ignored because they may escalate. Preschoolers are not cognitively prepared for explanations about the fears. They gain security and comfort from familiar objects such as toys, dolls, or photographs of family members, so new toys should not be introduced.

What is a common initial reaction of parents to illness or injury and hospitalization in their child? a. Anger b. Fear c. Depression d. Helplessness

ANS: D Recent research has identified common themes among parents whose children were hospitalized, including feeling an overall sense of helplessness, questioning the skills of staff, accepting the reality of hospitalization, needing to have information explained in simple language, dealing with fear, coping with uncertainty, and seeking reassurance from caregivers. Anger or guilt is usually the second reaction stage. Fear, anxiety, and frustrations also are common feelings. Parents may finally react with some form of depression related to the physical and emotional exhaustion associated with a hospitalized child.

What is the most common type of burn in the toddler age group? a. Electric burn from electrical outlets b. Flame burn from playing with matches c. Hot object burn from cigarettes or irons d. Scald burn from high-temperature tap water

ANS: D Scald burns are the most common type of thermal injury in children, especially 1- and 2-year-old children. Temperature should be reduced on the hot water in the house and hot liquids placed out of the childs reach. Electric burns from electrical outlets and hot object burns from cigarettes or irons are both significant causes of burn injury. The child should be protected by reducing the temperature on the hot water heater in the home, keeping objects such as cigarettes and irons away from children, and placing protective guards over electrical outlets when not in use. Flame burns from matches and lighters represent one of the most fatal types of burns in the toddler age group but not one of the most common types of burn.

An adolescent girl calls the nurse at the clinic because she had unprotected sex the night before and does not want to be pregnant. What should the nurse explain? a. It is too late to prevent an unwanted pregnancy. b. An abortion may be the best option if she is pregnant. c. The risk of pregnancy is minimal, so no action is necessary. d. Postcoital contraception is available to prevent implantation and therefore pregnancy.

ANS: D Several emergency methods of contraception (ECP) are available and appropriate for use after unprotected sexual intercourse. A progestin-only ECP (levonorgestrel [Plan B]) is approved by the U.S. Food and Drug Administration and has high effectiveness and low rates of side effects. Plan B is effective if given within 72 hours of unprotected intercourse. An abortion is not indicated. Although the risk of pregnancy depends on the time during her menstrual cycle, a low risk of pregnancy exists. ECP is indicated.

An adolescent girl calls the nurse at the clinic because she had unprotected sex the night before and does not want to be pregnant. What should the nurse explain to the girl? a. It is too late to prevent an unwanted pregnancy b. An abortion may be the best option if she is pregnant c. Norplant can be administered to prevent pregnancy for up to 5 years d. Postcoital contraception is available to prevent implantation

ANS: D Several emergency methods of contraception are available. Postcoital contraception options do exist. It is nontherapeutic to tell her it is too late or that an abortion is the best option. Norplant is not a postcoital contraceptive.

Parents ask for help for their other children to cope with the changes in the family resulting from the special needs of their sibling. What strategy does the nurse recommend? a. Explain to the siblings that embarrassment is unhealthy. b. Encourage the parents not to expect siblings to help them care for the child with special needs. c. Provide information to the siblings about the childs condition only as requested. d. Invite the siblings to attend meetings to develop plans for the child with special needs.

ANS: D Siblings should be invited to attend meeting to be part of the care team for the child. They can learn about an individualized education plan and help design strategies that will work at home. Embarrassment may be associated with having a sibling with a chronic illness or disability. Parents must be able to respond in an appropriate manner without punishing the sibling. The parents may need assistance with the care of the child. Most siblings are positive about the extra responsibilities. Parents need to inform the siblings about the childs condition before a nonfamily member does so. The parents do not want the siblings to fantasize about what is wrong with the child.

Which is a common side effect of short-term corticosteroid therapy? a. Fever b. Hypertension c. Weight loss d. Increased appetite

ANS: D Side effects of short-term corticosteroid therapy include an increased appetite. Fever is not a side effect of therapy. It may be an indication of infection. Hypertension is not usually associated with initial corticosteroid therapy. Weight gain, not weight loss, is associated with corticosteroid therapy.

What is an appropriate intervention to encourage food and fluid intake in a hospitalized child? a. Force the child to eat and drink to combat caloric losses. b. Discourage participation in non-eating activities until caloric intake is sufficient. c. Administer large quantities of flavored fluids at frequent intervals and during meals. d. Give high-quality foods and snacks whenever the child expresses hunger.

ANS: D Small, frequent meals and nutritious snacks should be provided for the child. Favorite foods such as peanut butter and jelly sandwiches, fruit yogurt, cheese, pizza, and macaroni and cheese should be available. Forcing a child to eat only meets with rebellion and reinforces the behavior as a control mechanism. Large quantities of fluid may decrease the child's hunger and further inhibit food intake.

During the physical examination of an adolescent with significant weight loss, what finding may indicate an eating disorder? a. Diarrhea b. Amenorrhea c. Appetite suppression d. Erosion of tooth enamel

ANS: D Some of the signs of bulimia include erosion of tooth enamel and increased dental caries. Check the back of the hands for abrasions caused by rubbing against the maxillary incisors during selfinduced vomiting. Diarrhea is not a result of vomiting. Rather, it may occur in patients with inflammatory bowel disease and other gastrointestinal diseases. Amenorrhea can occur with anorexia nervosa, but it can also be a result of the weight loss from other causes. It can also indicate pregnancy in adolescent females. Appetite suppression can occur from central nervous system lesions or from oncologic and metabolic disorders.

An adolescent teen has bulimia. Which assessment finding should the nurse expect to assess? a. Diarrhea b. Amenorrhea c. Cold intolerance d. Erosion of tooth enamel

ANS: D Some of the signs of bulimia include erosion of tooth enamel, increased dental caries from vomited gastric acid, throat complaints, fluid and electrolyte disturbances, and abdominal complaints from laxative abuse. Diarrhea is not a result of the vomiting. It may occur in patients with bulimia who also abuse laxatives. Amenorrhea and cold intolerance are characteristics of anorexia nervosa, which some bulimics have. These symptoms are related to the extreme low weight.

Which symptoms should the nurse expect to observe during the physical assessment of an adolescent girl with severe weight loss and disrupted metabolism associated with anorexia nervosa? a. Dysmenorrhea and oliguria b. Tachycardia and tachypnea c. Heat intolerance and increased blood pressure d. Lowered body temperature and brittle nails

ANS: D Symptoms of anorexia nervosa include lower body temperature, severe weight loss, decreased blood pressure, dry skin, brittle nails, altered metabolic activity, and presence of lanugo hair. Amenorrhea, rather than dysmenorrhea, and cold intolerance are manifestations of anorexia nervosa. Bradycardia, rather than tachycardia, may be present.

A 14-year-old male mentions that he now has to use deodorant but never had to before. The nurse's response should be based on knowledge that which occurs during puberty? a. Eccrine sweat glands in the axillae become fully functional during puberty. b. Sebaceous glands become extremely active during puberty. c. New deposits of fatty tissue insulate the body and cause increased sweat production. d. Apocrine sweat glands reach secretory capacity during puberty.

ANS: D The apocrine sweat glands, nonfunctional in children, reach secretory capacity during puberty. They secrete a thick substance as a result of emotional stimulation that, when acted on by surface bacteria, becomes highly odoriferous. They are limited in distribution and grow in conjunction with hair follicles, in the axilla, genital, anal, and other areas. Eccrine sweat glands are present almost everywhere on the skin and become fully functional and respond to emotional and thermal stimulation. Sebaceous glands become extremely active at this time, especially those on the genitalia and the "flush" areas of the body such as face, neck, shoulders, upper back, and chest. This increased activity is important in the development of acne. New deposits of fatty tissue is not the etiology of apocrine sweat gland activity.

The parents of a newborn say that their toddler "hates the baby; he suggested that we put him in the trash can so the trash truck could take him away." Which is the nurse's best reply? a. "Let's see if we can figure out why he hates the new baby." b. "That's a strong statement to come from such a small boy." c. "Let's refer him to counseling to work this hatred out. It's not a normal response." d. "That is a normal response to the birth of a sibling. Let's look at ways to deal with this."

ANS: D The arrival of a new infant represents a crisis for even the best-prepared toddler. Toddlers have their entire schedule and routines disrupted because of the new family member. The nurse should work with parents on ways to involve the toddler in the newborn's care and to help focus attention on the toddler. The toddler does not hate the infant. This is an expected response to the changes in routines and attention that affect the toddler. The toddler can be provided with a doll to tend to the doll's needs at the same time the parent is performing similar care for the newborn.

A toddlers parent asks the nurse for suggestions on dealing with temper tantrums. What is the most appropriate recommendation? a. Punish the child. b. Explain to child that this is wrong. c. Leave the child alone until the tantrum is over. d. Remain close by the child but without eye contact.

ANS: D The best way to deal with temper tantrums is to ignore the behaviors, provided that the actions are not dangerous to the child. Tantrums are common during this age group as the child becomes more independent and overwhelmed by increasingly complex tasks. The parents and caregivers need to have consistent and developmentally appropriate expectations. Punishment and explanations will not be beneficial. The presence of the parent is necessary both for safety and to provide a feeling of control and security to the child when the tantrum is over.

1A father tells the nurse that his daughter wants the same plate and cup used at every meal, even if they go to a restaurant. The nurse should explain that this is what? a. A sign the child is spoiled b. An attempt to exert unhealthy control c. Regression, which is common at this age d. Ritualism, an expected behavior at this age

ANS: D The child is exhibiting the ritualism, which is characteristic at this age. Ritualism is the need to maintain sameness and reliability. It provides a sense of structure and comfort to the toddler. It will dictate certain principles in feeding practices, including rejecting a favorite food because it is served in a different container. This does not indicate the child has unreasonable expectations but rather is part of normal development. Ritualism is not regression, which is a retreat from a present pattern of functioning.

The nurse approaches a group of school-age patients to administer medication to Sam Hart. What should the nurse do to identify the correct child? a. Ask the group, "Who is Sam Hart?" b. Call out to the group, "Sam Hart?" c. Ask each child, "What's your name?" d. Check the patient's identification name band

ANS: D The child must be correctly identified before the administration of any medication. Children are not totally reliable in giving correct names on request; the identification bracelet should always be checked. Asking children or the group for names is not an acceptable way to identify a child. Older children may exchange places, give an erroneous name, or choose not to respond to their name as a joke.

When only one child is abused in a family, the abuse is usually a result of what? a. The child is the firstborn. b. The child is the same gender as the abusing parent. c. The parent abuses the child to avoid showing favoritism. d. The parent is unable to deal with the childs behavioral style.

ANS: D The child unintentionally contributes to the abuse. The fit or compatibility between the childs temperament and the parents ability to deal with that behavior style is an important predictor. Birth order and gender can contribute to abuse, but there is not a specific birth order or gender relationship that is indicative of abuse. Being the firstborn or the same gender as the abuser is not linked to child abuse. Avoidance of favoritism is not usually a cause of abuse.

Many of the physical characteristics of Down syndrome present nursing problems. Care of the child should include which intervention? a. Delay feeding solid foods until the tongue thrust has stopped. b. Modify diet as necessary to minimize the diarrhea that often occurs. c. Provide calories appropriate to child's age. d. Use a cool-mist vaporizer to keep mucous membranes moist.

ANS: D The constant stuffy nose forces the child to breathe by mouth, drying the mucous membranes and increasing the susceptibility to upper respiratory tract infections. A cool-mist vaporizer will keep the mucous membranes moist and liquefy secretions. The child has a protruding tongue, which makes feeding difficult. The parents must persist with feeding while the child continues the physiologic response of the tongue thrust. The child is predisposed to constipation. Calories should be appropriate to the child's weight and growth needs, not age.

What is an important consideration in the diagnosis of attention deficit hyperactivity disorder (ADHD)? a. Learning disabilities are apparent at an early age. b. The child will always be distracted by external stimuli. c. Parental observations of the childs behavior are most relevant. d. It must be determined whether the childs behavior is age appropriate or problematic.

ANS: D The diagnosis of ADHD is complex. A multidisciplinary evaluation should be done to determine whether the childs behavior is appropriate for the developmental age or whether it is problematic. Learning disabilities are usually not evident until the child enters school. Each child with ADHD responds differently to stimuli. Some children are distracted by internal stimuli and others by external stimuli. Parents can only provide one viewpoint of the childs behavior. Many observers should be asked to provide input with structured tools to facilitate the diagnosis.

What is the best description of anorexia nervosa? a. Occurs most frequently in adolescent males b. Occurs most frequently in adolescents from lower socioeconomic groups c. Results from a posterior pituitary disorder d. Results in severe weight loss in the absence of obvious physical causes

ANS: D The etiology of anorexia remains unclear, but a distinct psychological component is present. The diagnosis is based primarily on psychological and behavioral criteria. Females account for 90% to 95% of the cases. No relation has been identified between socioeconomic groups and anorexia. Posterior pituitary disorders are not associated with anorexia nervosa.

An adolescent asks the nurse what causes primary dysmenorrhea. The nurse's response should be based on which statement? a. It is an inherited problem. b. Excessive estrogen production causes uterine pain. c. There is no physiologic cause; it is a psychological reaction. d. There is a relation between prostaglandins and uterine contractility.

ANS: D The exact etiology of primary dysmenorrhea is debated. Overproduction of uterine prostaglandins has been implicated, as has overproduction of vasopressin. Dysmenorrhea is not known to be inherited. Excessive estrogen has not been implicated in the etiology. It has a physiologic cause. Women with dysmenorrhea have higher prostaglandin levels.

An older school-age child asks the nurse, What is the reason for this topical corticosteroid cream? What rationale should the nurse give? a. The cream is used for an antifungal effect. b. The cream is used for an analgesic effect. c. The cream is used for an antibacterial effect. d. The cream is used for an anti-inflammatory effect.

ANS: D The glucocorticoids are the therapeutic agents used most widely for skin disorders. Their local anti-inflammatory effects are merely palliative, so the medication must be applied until the disease state undergoes a remission or the causative agent is eliminated. It does not have an antifungal, analgesic, or antibacterial effect.

The school nurse is reviewing the process of wound healing. What is the initial response at the site of injury? a. Contraction b. Maturation c. Fibroplasia d. Inflammation

ANS: D The initial response at the site of injury is inflammation, a vascular and cellular response that prepares the tissues for the subsequent repair process. Fibroplasia (granulation or proliferation), the second phase of healing, lasts from 5 days to 4 weeks. During contraction and maturation, the third and fourth phases of wound healing, collagen continues to be deposited and organized into layers, compressing the new blood vessels and gradually stopping blood flow across the wound.

A mother calls the outpatient clinic requesting information on appropriate dosing for over-the-counter medications for her 13-month-old who has symptoms of an upper respiratory tract infection and fever. The box of acetaminophen says to give 120 mg q4h when needed. At his 12-month visit, the nurse practitioner prescribed 150 mg. What is the nurse's best response? a. "The doses are close enough; it doesn't really matter which one is given." b. "It is not appropriate to use dosages based on age because children have a wide range of weights at different ages." c. "From your description, medications are not necessary. They should be avoided in children at this age." d. "The nurse practitioner ordered the drug based on weight, which is a more accurate way of determining a therapeutic dose."

ANS: D The method most often used to determine children's dosage is based on a specific dose per kilogram of body weight. The mother should be given correct information. For a therapeutic effect, the dosage should be based on weight, not age. Acetaminophen can be used to relieve discomfort in children at this age group.

A 16-year-old adolescent boy tells the school nurse that he is gay. The nurses response should be based on what? a. He is too young to have had enough sexual activity to determine this. b. The nurse should feel open to discussing his or her own beliefs about homosexuality. c. Homosexual adolescents do not have concerns that differ from those of heterosexual adolescents. d. It is important to provide a nonthreatening environment in which he can discuss this.

ANS: D The nurse needs to be open and nonjudgmental in interactions with adolescents. This will provide a safe environment in which to provide appropriate health care. Adolescence is when sexual identity develops. The nurses own beliefs should not bias the interaction with this student. Homosexual adolescents face very different challenges as they grow up because of societys response to homosexuality.

An 8-month-old infant is restrained to prevent interference with the IV infusion. How should the nurse appropriately care for this child? a. Remove the restraints once a day to allow movement. b. Keep the restraints on constantly. c. Keep the restraints secure so the infant remains supine. d. Remove restraints whenever possible.

ANS: D The nurse should remove the restraints whenever possible. When parents or staff are present, the restraints can be removed and the IV site protected. Restraints must be checked and documented every 1 to 2 hours. They should be removed for range of motion on a periodic basis. The child should not be securely restrained in the supine position because of risks of aspiration.

A parent asks the nurse about negativism in toddlers. Which is the most appropriate recommendation? a. Punish the child. b. Provide more attention. c. Ask child not always to say "no." d. Reduce the opportunities for a "no" answer.

ANS: D The nurse should suggest to the parent that questions be phrased with realistic choices rather than yes or no answers. This provides the toddler with a sense of control and reduces the opportunity for negativism. Negativism is not an indication of stubbornness or insolence and should not be punished. The negativism is not a function of attention; the child is testing limits to gain an understanding of the world. The toddler is too young to be asked to not always say "no."

A parent asks the nurse about negativism in toddlers. What is the most appropriate recommendation? a. Punish the child. b. Provide more attention. c. Ask child not to always say no. d. Reduce the opportunities for a no answer.

ANS: D The nurse should suggest to the parent that questions should be phrased with realistic choices rather than yes or no answers. This provides a sense of control for the toddler and reduces the opportunity for negativism. Negativism is not an indication of stubbornness or insolence and should not be punished. The negativism is not a function of attention; the child is testing limits to gain an understanding of the world. The toddler is too young to comply with requests not to say no.

What should the nurse do when caring for a child with an intravenous infusion? a. Use a macrodropper to facilitate reaching the prescribed flow rate. b. Avoid restraining the child to prevent undue emotional stress. c. Change the insertion site every 24 hours. d. Observe the insertion site frequently for signs of infiltration.

ANS: D The nursing responsibility for IV therapy is to calculate the amount to be infused in a given length of time; set the infusion rate; and monitor the apparatus frequently (at least every 1 to 2 hours) to make certain that the desired rate is maintained, the integrity of the system remains intact, the site remains intact (free of redness, edema, infiltration, or irritation), and the infusion does not stop. A minidropper (60 drops/ml) is the recommended IV tubing in pediatrics. The IV site should be protected. This may require soft restraints on the child. Insertion sites do not need to be changed every 24 hours unless a problem is found with the site. This exposes the child to significant trauma

An adolescent gets hit in the eye during a fight. The school nurse, using a flashlight, notes the presence of gross hyphema (hemorrhage into anterior chamber). What is the priority nursing action? a. Apply a Fox shield b. Instruct the adolescent to apply ice for 24 hours c. Have the adolescent rest with eye closed and ice applied d. Notify parents that the adolescent needs to see an ophthalmologist

ANS: D The parents should be notified that the adolescent needs to see an ophthalmologist as soon as possible. Applying a Fox shield, instructing the adolescent to apply ice for 24 hours, and having the adolescent rest with eye closed and ice applied may cause further damage. Referral to an ophthalmologist is indicated.

What do nursing interventions to promote health during middle childhood include? a. Stress the need for increased calorie intake to meet increased demands. b. Instruct parents to defer questions about sex until the child reaches adolescence. c. Advise parents that the child will need increasing amounts of rest toward the end of this period. d. Educate parents about the need for good dental hygiene because these are the years in which permanent teeth erupt.

ANS: D The permanent teeth erupt during the school-age years. Good dental hygiene and regular attention to dental caries are vital parts of health supervision during this period. Caloric needs are decreased in relation to body size for this age group. Balanced nutrition is essential to promote growth. Questions about sex should be addressed honestly as the child asks questions. The child usually no longer needs a nap, but most require approximately 11 hours of sleep each night at age 5 years and 9 hours at age 12 years.

A preschooler is found digging up a pet bird that was recently buried after it died. What is the best explanation for this behavior? a. Has a morbid preoccupation with death b. Is looking to see whether a ghost took it away c. The loss is not yet resolved, and professional counseling is needed d. Reassurance is needed that the pet has not gone somewhere else

ANS: D The preschooler can recognize that the pet has died but has difficulties with the permanence. Digging up the bird gives reassurance that the bird is still present.

Which nursing intervention should the nurse include to help the siblings of a child with special needs cope? a. Explain to the siblings that embarrassment is unhealthy b. Encourage the parents not to expect siblings to help them care for the child with special needs c. Provide information to the siblings about the child's condition only as they request it d. Suggest to the parents ways of showing gratitude to the siblings who help care for the child with special needs

ANS: D The presence of a child with special needs in a family will change the family dynamic. Siblings may be asked to take on additional responsibilities to help the parents to care for the child. The parents should show gratitude, such as an increase in allowance, special privileges, and verbal praise. Embarrassment may be associated with having a sibling with a chronic illness or disability. Parents must be able to respond in an appropriate manner without punishing the sibling. The parents may need assistance with the care of the child. Most siblings are positive about the extra responsibilities. The siblings need to be informed about the child's condition before a nonfamily member does so. The parents do not want the siblings to fantasize about what is wrong with the child.

What is characteristic of children with posttraumatic stress disorder (PTSD)? a. Denial as a defense mechanism is unusual. b. Traumatic effects cannot remain indefinitely. c. Previous coping strategies and defense mechanisms are not useful. d. Children often play out the situation over and over again.

ANS: D The third phase of adjustment to PTSD involves the children playing out the situation over and over to come to terms with their fears. Denial is frequently used as a defense mechanism during the second phase. For some children, traumatic effects can remain indefinitely. Coping is a learned response. During the third stage, the children can be helped to use their coping strategies to deal with their fears.

Which statement is true about gonorrhea? a. It is caused by Treponema pallidum. b. Treatment is by multidose administration of penicillin. c. Treatment is by topical applications to lesions. d. Treatment of all sexual contacts is an essential part of treatment.

ANS: D The treatment plan should include finding and treating all sexual partners. Gonorrhea is caused by Neisseria gonorrhoeae. Syphilis is caused by T. pallidum. Primary treatment is with different antibiotics because of N. gonorrhoeae resistance to penicillin. Systemic therapy is necessary to treat this disease.

An adolescent boy tells the nurse that he has recently had homosexual feelings. What knowledge should the nurse's response be based on? a. This indicates the adolescent is homosexual. b. This indicates the adolescent will become homosexual as an adult. c. The adolescent should be referred for psychotherapy. d. The adolescent should be encouraged to share his feelings and experiences.

ANS: D These adolescents are at increased risk for health-damaging behaviors, not because of the sexual behavior itself, but because of society's reaction to the behavior. The nurse's first priority is to give the young man permission to discuss his feelings about this topic, knowing that the nurse will maintain confidentiality, appreciate his feelings, and remain sensitive to his need to talk about the topic. In recent studies among self-identified gay, lesbian, and bisexual adolescents, many of the adolescents report changing self-labels one or more times during their adolescence. An assessment must be made about any risks to himself or others. If these do not exist, the adolescent needs a supportive person to talk with.

The nurse is talking with the parent of a child newly diagnosed with a chronic illness. The parent is upset and tearful. The nurse asks, "Whom do you talk to when something is worrying you?" How should the nurse's statement be interpreted? a. Inappropriate, because the parent is so upset b. A diversion of the present crisis to similar situations with which the parent has dealt c. An intervention to find someone to help the parent d. Part of assessing the parent's available support system

ANS: D These are important data for the nurse to obtain. This question will provide information about the marital relationship (whether the parent speaks to the spouse), alternate support systems, and ability to communicate. By assessing these areas, the nurse can facilitate the identification and use of community resources as needed. It is an important part of assessment information to determine the parent's support network. The nurse is obtaining information to help support the parent through the diagnosis. The parent is not in need of additional parenting help at this time.

The father of a 12-year-old child tells the nurse that he is concerned about his son getting "fat." His son is at the 50th percentile for height and the 75th percentile for weight on the growth chart. What is the most appropriate nursing action? a. Reassure the father that his child is not fat b. Reassure the father that his child is just growing c. Suggest a low-calorie, low-fat diet d. Explain that this is typical of the growth pattern of boys at this age

ANS: D This is a characteristic pattern of growth in preadolescent boys, where the growth in height has slowed in preparation for the pubertal growth spurt, but weight is still gained. The nurse should review this with both the father and the child and develop a plan to maintain physical exercise and a balanced diet. It is false reassurance to tell the father that his son is not fat. His weight is high for his height. The child needs to maintain his physical activity. The father is concerned, so an explanation is required. A nutritional diet with physical activity should be sufficient to maintain his balance.

A child with a serious chronic illness will soon go home. The case manager requests that the family provide total care for the child for a couple of days while the child is still hospitalized. How should the request be viewed? a. Improper because of legal issues b. Supportive because families are usually eager to get involved c. Unacceptable because the family will have to assume the care soon enough d. Important because it can be beneficial to the transition from hospital to home

ANS: D This type of groundwork is essential for the family. Adequate family training and preparation will assist in the childs transition home. The nursing staff in the hospital is responsible for the childs care. The family will provide the care with assistance as needed. Although parents are eager to be involved, the purpose of this intervention is the development of family competency and confidence that they are capable. Arrangements for respite care are important for the family both during hospitalizations and while the child is at home.

In terms of fine motor development, which should the 3-year-old child be expected to do? a. Lace shoes and tie shoelaces with a bow. b. Use scissors to cut pictures, and print a few numbers. c. Draw a person with seven parts and correctly identify the parts. d. Draw a circle and name what has been drawn.

ANS: D Three-year-olds are able to accomplish this fine motor skill. Being able to lace shoes and tie shoelaces with a bow, use scissors to cut pictures, and print a few numbers, or draw a person with seven parts and correctly identify the parts are fine motor skills of 4- or 5-year-olds.

Which factor is most important in predisposing toddlers to frequent infections? a. Respirations are abdominal. b. Pulse and respiratory rates are slower than those in infancy. c. Defense mechanisms are less efficient than those during infancy. d. Toddlers have a short, straight internal ear canal and large lymph tissue.

ANS: D Toddlers continue to have the short, straight internal ear canal of infants. The lymphoid tissue of the tonsils and adenoids continues to be relatively large. These two anatomic conditions combine to predispose the toddler to frequent infections. The abdominal respirations and lowered pulse and respiratory rate of toddlers do not affect their susceptibility to infection. The defense mechanisms are more efficient compared with those of infancy.

What factor is most important in predisposing toddlers to frequent infections? a. Respirations are abdominal. b. Pulse and respiratory rates in toddlers are slower than those in infants. c. Defense mechanisms are less efficient than those during infancy. d. Toddlers have short, straight internal ear canals and large lymph tissue.

ANS: D Toddlers continue to have the short, straight internal ear canals of infants. The lymphoid tissue of the tonsils and adenoids continues to be relatively large. These two anatomic conditions combine to predispose toddlers to frequent infections. The abdominal respirations and lowered pulse and respiratory rate of toddlers do not affect their susceptibility to infection. The defense mechanisms are more efficient compared with those of infancy.

What should the nurse suggest to parents of preschoolers about sensitive questions regarding sex? a. Distract your child from the topic. b. Offer complete factual information. c. Dismiss the topic until the child is older. d. Find out what your child knows or thinks.

ANS: D Two rules govern answering sensitive questions about topics such as sex. The first is to find out what children know and think. By investigating the theories children have produced as a reasonable explanation, parents can not only give correct information but also help children understand why their explanation is inaccurate. Another reason for ascertaining what the child thinks before offering any information is to avoid giving an unasked for answer. The child should not be distracted from the topic. If parents offer too much information, the child will simply become bored or end the conversation with an irrelevant question. What matters is that parents are approachable and do not dismiss their childs inquiries.

The school nurse is discussing after-school sports participation with parents of children age 10 years. The nurses presentation includes which important consideration? a. Teams should be gender specific. b. Organized sports are not appropriate at this age. c. Competition is detrimental to the establishment of a positive self-image. d. Sports participation is encouraged if the type of sport is appropriate to the childs abilities.

ANS: D Virtually every child is suited for some type of sport. The child should be matched to the type of sport appropriate to his or her abilities and physical and emotional makeup. At this age, girls and boys have the same basic structure and similar responses to exercise and training. After puberty, teams should be gender specific because of the increased muscle mass in boys. Organized sports help children learn teamwork and skill acquisition. The emphasis should be on playing and learning. Children do enjoy appropriate levels of competition.

A preschool child is being admitted to the hospital with dehydration and a urinary tract infection (UTI). Which urinalysis result should the nurse expect with these conditions? a. WBC <1; specific gravity 1.008 b. WBC <2; specific gravity 1.025 c. WBC >2; specific gravity 1.016 d. WBC >2; specific gravity 1.030

ANS: D WBC count in a routine urinalysis should be <1 or 2. Over that amount indicates a urinary tract inflammatory process. The urinalysis specific gravity for children with normal fluid intake is 1.016 to 1.022. When the specific gravity is high, dehydration is indicated. A low specific gravity is seen with excessive fluid intake, distal tubular dysfunction, or insufficient antidiuretic hormone secretion.

What usually triggers the weight loss of anorexia nervosa? a. Sexual abuse b. School failure c. Independence from family d. Traumatic interpersonal conflict

ANS: D Weight loss may be triggered by a typical adolescent crisis such as the onset of menstruation or a traumatic interpersonal incident; situations of severe family stress, such as parental separation or divorce; or circumstances in which the young person lacks personal control, such as being teased, changing schools, or entering college. "Sexual abuse," "school failure," and "independence from family" are not part of the behavioral characteristics of anorexia nervosa.

Which age group should the pediatric nurse recognize as being vulnerable to events that lessen their feeling of control and power? a. Infants b. Toddlers c. Preschoolers d. School-age children

ANS: D When a child is hospitalized, the altered family role, physical disability, loss of peer acceptance, lack of productivity, and inability to cope with stress usurp individual power and identity. This is especially detrimental to school-age children, who are striving for independence and productivity and are now experiencing events that lessen their control and power. Infants, toddlers, and preschoolers, although affected to different extents by loss of power, are not as significantly affected as are school-age children.

The school nurse has been asked to begin teaching sex education in the fifth grade. What should the nurse recognize about this age group? a. Children in fifth grade are too young for sex education. b. Children should be discouraged from asking too many questions. c. Correct terminology should be reserved for children who are older. d. Sex can be presented as a normal part of growth and development

ANS: D When sexual information is presented to school-age children, sex should be treated as a normal part of growth and development. Fifth-graders are usually 10 or 11 years old. This age is not too young to speak about physiologic changes in their bodies. They should be encouraged to ask questions. Preadolescents need precise and concrete information

1. What dietary instructions should the nurse give to parents of a child with minimal change nephrotic syndrome with massive edema? (Select all that apply.) a. Soft diet b. High protein c. Fluid restricted d. No salt added at the table e. Restriction of foods high in sodium

ANS: D, E The child with minimal change nephrotic syndrome maintains a regular diet, not soft. However, salt is restricted during periods of massive edema and while the patient is on corticosteroid therapy; no salt is added at the table, and foods with very high salt content are excluded. Although a low-sodium diet will not remove edema, its rate of increase may be reduced. Water is seldom restricted. A diet generous in protein is logical, but there is no evidence that it is beneficial or alters the outcome of the disease.

Hearing is expressed in decibels (dB), or units of loudness. Which is the softest sound a normal ear can hear? a. 0 dB b. 10 dB c. 40 to 50 dB d. 100 dB

ANS: A By definition, 0 dB is the softest sound the normal ear can hear. Ten decibels is the sound of the heartbeat or the rustling of leaves. 40 to 50 dB is in the range of normal conversation. The noise of a train is approximately 100 dB.

Prevention of hearing impairment in children is a major goal for the nurse. This can be achieved through which intervention? a. Being involved in immunization clinics for children b. Assessing a newborn for hearing loss c. Answering parents' questions about hearing aids d. Participating in hearing screening in the community

ANS: A Childhood immunizations can eliminate the possibility of acquired sensorineural hearing loss from rubella, mumps, or measles encephalitis. Assessing a newborn for hearing loss, answering parents' questions about hearing aids, and participating in hearing screening in the community are interventions to screen for the presence of hearing loss or deal with an identified loss, not prevention.

Which is an effective strategy to reduce the stress of burn dressing procedures? a. Give the child as many choices as possible. b. Reassure the child that dressing changes are not painful. c. Explain to the child why analgesics cannot be used. d. Encourage the child to master stress with controlled passivity.

ANS: A Children who understand the procedure and have some perceived control demonstrate less maladaptive behavior. They respond well to participating in decisions and should be given as many choices as possible. The dressing change procedure is painful and stressful. Misinformation should not be given to the child. Analgesia and sedation can and should be used. Encouraging the child to master stress with controlled passivity is not a positive coping strategy.

A nurse is planning a teaching session for a group of adolescents. The nurse understands that by adolescence the individual is in which stage of cognitive development? a. Formal operations b. Concrete operations c. Conventional thought d. Postconventional thought

ANS: A Cognitive thinking culminates with capacity for abstract thinking. This stage, the period of formal operations, is Piaget's fourth and last stage. Concrete operations usually occur between ages 7 and 11 years. Conventional and postconventional thought refers to Kohlberg's stages of moral development.

The nurse has just given a subcutaneous injection to a preschool child, and the child asks for a Band-Aid over the site. Which action should the nurse implement? a. Place a Band-Aid over the site. b. Massage the injection site with an alcohol swab. c. Show the child there is no bleeding from the site. d. Explain that a Band-Aid is not needed after a subcutaneous injection.

ANS: A Despite the advances in body image development, preschoolers have poorly defined body boundaries and little knowledge of their internal anatomy. Intrusive experiences are frightening, especially those that disrupt the integrity of the skin (e.g., injections and surgery). They fear that all their blood and insides can leak out if the skin is broken. Therefore, preschoolers may believe it is critical to use bandages after an injury. The nurse should place a Band-Aid over the site

A nurse should suspect possible visual impairment in a child who displays which characteristic? a. Excessive rubbing of the eyes b. Rapid lateral movement of the eyes c. Delay in speech development d. Lack of interest in casual conversation with peers

ANS: A Excessive rubbing of the eyes is a clinical manifestation of visual impairment. Rapid lateral movement of the eyes, delay in speech development, and lack of interest in casual conversation with peers are not associated with visual impairment.

Which term refers to the ability to see objects clearly at close range but not at a distance? a. Myopia b. Amblyopia c. Cataract d. Glaucoma

ANS: A Myopia, or nearsightedness, refers to the ability to see objects clearly at close range but not at distance. Amblyopia, or lazy eye, is reduced visual acuity in one eye. A cataract is opacity of the lens of the eye. Glaucoma is a group of eye diseases characterized by increased intraocular pressure.

The nurse is planning care for a hospitalized toddler. What is the rationale for planning to continue the toddlers rituals while hospitalized? a. To provide security b. To prevent regression c. To prevent dependency d. To decrease negativism

ANS: A Ritualism, the need to maintain sameness and reliability, provides a sense of security and comfort. It will not prevent regression or dependency or decrease negativism.

Which should be the major consideration when selecting toys for a child who is cognitively impaired? a. Safety b. Age appropriateness c. Ability to provide exercise d. Ability to teach useful skills

ANS: A Safety is the primary concern in selecting recreational and exercise activities for all children. This is especially true for children who are cognitively impaired. Age appropriateness, the ability to provide exercise, and the ability to teach useful skills should all be considered in the selection of toys, but safety is of paramount importance

Which is most descriptive of a school-age child's reaction to death? a. Is very interested in funerals and burials b. Has little understanding of words such as forever c. Imagines the deceased person to be still alive d. Has an idealistic view of the world and criticizes funerals as barbaric

ANS: A The school-age child is interested in post-death services and may be inquisitive about what happens to the body.

Which is an appropriate nursing intervention when providing comfort and support for a child when death is imminent? a. Limit care to essentials. b. Avoid playing music near the child. c. Explain to the child the need for constant measurement of vital signs. d. Whisper to the child instead of using a normal voice.

ANS: A When death is imminent, care should be limited to interventions for palliative care.

The nurse wore gloves during a dressing change. What should the nurse do after the gloves are removed? a. Wash hands thoroughly b. Check the gloves for leaks c. Rinse gloves in disinfectant solution d. Apply new gloves before touching the next patient

ANS: A When gloves are worn, the hands are washed thoroughly after removing the gloves because both latex and vinyl gloves fail to provide complete protection. Gloves should be disposed of after use. Hands should be thoroughly washed before new gloves are applied.

A child has a slight (2640 dB) degree of hearing loss. The nurse recognizes this amount of hearing loss can have what effect? (Select all that apply.) a. No speech defects b. Difficulty hearing faint speech c. Usually is unaware of the hearing difficulty d. Can distinguish vowels but not consonants e. Unable to understand conversational speech

ANS: A, B, C A child with a slight degree of hearing loss has no speech defects, may have difficulty hearing faint speech, and is usually unaware of the hearing difficulty. The ability to distinguish vowels but not consonants is an effect of severe hearing loss and being unable to understand conversational speech is an effect of moderately severe hearing loss.

1. The clinic nurse is teaching parents about when to call the office immediately for a child with a fever. What should the nurse include in the teaching session? (Select all that apply.) a. The child has a stiff neck. b. The fever is over 40.6 C (105 F). c. The child is younger than 2 months. d. The fever has lasted for more than 3 days. e. The fever went away for more than 24 hours and then returned.

ANS: A, B, C Parents should call the office immediately if a child has a fever over 40.6 C (105 F), the child is younger than 2 months, or the child has a stiff neck. Parents are to call within 24 hours if the fever went away for more than 24 hours and then returned or the fever has lasted for more than 3 days.

The nurse understands that which gestational disorders can cause a cognitive impairment in the newborn? (Select all that apply.) a. Prematurity b. Postmaturity c. Low birth weight d. Physiological jaundice e. Large for gestational age

ANS: A, B, C Prematurity, postmaturity, and low birth weight can be causes of cognitive impairment in newborns. Physiological jaundice and large for gestational age are not associated causes of cognitive impairment in newborns.

1. What are the advantages of an implanted port (Port-a-Cath)? (Select all that apply.) a. Reduced risk of infection b. Reduced cost for the family c. Placed completely under the skin d. Easy to use for self-administered infusions e. Removal does not require a surgical procedure

ANS: A, B, C The advantages of an implanted port include reduced risk of infection, reduced cost for the family, and placed completely under the skin. Because it is implanted and must be accessed, it is not easy to use for self-administered infusions, and removal does require a surgical procedure.

1. In what condition should the nurse be alert for altered fluid requirements in children? (Select all that apply.) a. Oliguric renal failure b. Increased intracranial pressure c. Mechanical ventilation d. Compensated hypotension e. Tetralogy of Fallot f. Type 1 diabetes mellitus

ANS: A, B, C The nurse should recognize that conditions such as oliguric renal failure, increased intracranial pressure, and mechanical ventilation can cause an increase or a decrease in fluid requirements. Conditions such as hypotension, tetralogy of Fallot, and diabetes mellitus (type 1) do not cause an alteration in fluid requirements.

What is the reason pedestrian motor vehicle injuries increase in the preschool age? (Select all that apply.) a. Riding tricycles b. Running after balls c. Playing in the street d. Crossing streets at the crosswalk e. Crossing streets with an adult

ANS: A, B, C Pedestrian motor vehicle injuries increase because of activities such as playing in the street, riding tricycles, running after balls, and forgetting safety regulations when crossing streets. Crossing streets at the crosswalk or with an adult are safety measures.

1. The nurse is preparing a presentation on compensated, decompensated, and irreversible shock in children. What clinical manifestations related to decompensated shock should the nurse include? (Select all that apply.) a. Tachypnea b. Oliguria c. Confusion d. Pale extremities e. Hypotension f. Thready pulse

ANS: A, B, C, D As shock progresses, perfusion in the microcirculation becomes marginal despite compensatory adjustments, and the signs are more obvious. Signs include tachypnea, oliguria, confusion, and pale extremities, as well as decreased skin turgor and poor capillary filling. Hypotension and a thready pulse are clinical manifestations of irreversible shock.

The nurse is planning strategies to assist difficult or easily distracted children when they participate in activities. What strategies should the nurse plan? (Select all that apply.) a. Role-play before the activity. b. Handle behavior with firmness. c. Acquaint them with what to expect. d. Be patient with inappropriate behavior. e. Dont give them much information about the activity.

ANS: A, B, C, D Difficult or easily distracted children may benefit from practice sessions in which they are prepared for a given event by role-playing, visiting the site, reading or listening to stories, or using other methods to acquaint them with what to expect. Nurses need to handle children with difficult temperaments with exceptional patience, firmness, and understanding so they can learn appropriate behavior in their interactions with others.

The nurse is caring for a child with neurofibromatosis. What local manifestations does the nurse expect to assess in this child? (Select all that apply.) a. Pigmented nevi b. Axillary freckling c. Caf-au-lait spots d. Slowly growing cutaneous neurofibromas e. Wheals that spread irregularly and fade within a few hours

ANS: A, B, C, D Local manifestations of neurofibromatosis include pigmented nevi, axillary freckling, caf-au-lait spots, and slowly growing cutaneous neurofibromas. Wheals that spread irregularly and fade within a few hours are characteristic of urticaria.

The nurse is preparing a staff education program about growth and development of an 18- month-old toddler. Which characteristics should the nurse include in the staff education program? (Select all that apply.) a. Eats well with a spoon and cup b. Runs clumsily and can walk up stairs c. Points to common objects d. Builds a tower of three or four blocks e. Has a vocabulary of 300 words f. Dresses self in simple clothes

ANS: A, B, C, D Tasks accomplished by an 18-month-old toddler include eating well with a spoon and cup, running clumsily, walking up stairs, pointing to common objects such as shoes, and building a tower with three or four blocks. An 18-month-old toddler has a vocabulary of only 10 words, not 300. Toddlers cannot dress themselves in simple clothing until 24 months of age.

The nurse is assessing coping behaviors of a family with a child with a chronic illness. What indicates avoidance coping behaviors? (Select all that apply.) a. Refuses to agree to treatment b. Avoids staff, family members, or child c. Is unable to discuss possible loss of the child d. Recognizes own growth through a passage of time e. Makes no change in lifestyle to meet the needs of other family members

ANS: A, B, C, E Avoidance coping behaviors include refusing to agree to treatment; avoiding staff, family members, or child; unable to discuss possible loss of the child; and making no change in lifestyle to meet the needs of other family members. Recognizing ones own growth through a passage of time is an approach behavior.

The school nurse recognizes that students who are targeted for repeated harassment and bullying may exhibit what? (Select all that apply.) a. Skip school b. Attempt suicide c. Bring weapons to school d. Attend extracurricular activities e. Report symptoms of depression

ANS: A, B, C, E Students targeted for repeated teasing and harassment are more likely to skip school, to report symptoms of depression, and to attempt suicide. Equally troubling, teens who are regularly harassed or bullied are also more likely to bring weapons to school to feel safe. Students who are bullied do not want to attend extracurricular activities.

1. The nurse is caring for a child with acute renal failure. What laboratory findings should the nurse expect to find? (Select all that apply.) a. Hyponatremia b. Hyperkalemia c. Metabolic alkalosis d. Elevated blood urea nitrogen level e. Decreased plasma creatinine level

ANS: A, B, D A child with acute renal failure would have hyponatremia, hyperkalemia, and elevated blood urea nitrogen levels. The child would have metabolic acidosis, not alkalosis, and the plasma creatinine levels would be increased, not decreased.

The nurse is assessing coping behaviors of a family with a child with a chronic illness. What indicates approach coping behaviors? (Select all that apply.) a. Plans realistically for the future b. Verbalizes possible loss of the child c. Uses magical thinking and fantasy d. Realistically perceives the childs condition e. Does not share the burden of the disorder with others

ANS: A, B, D Approach coping behaviors include planning realistically for the future, verbalizing possible loss of a child, and realistically perceiving the childs behavior. Using magical thinking and fantasy is an avoidance behavior. The family should share the burden of the disorder with others as an approach behavior.

The clinic nurse is assessing an adolescent on a topical antibacterial agent. The nurse should assess for which side effects that can be seen with topical antibacterial agents? (Select all that apply.) a. Burning b. Dryness c. Dry eyes d. Erythema e. Nasal irritation

ANS: A, B, D Side effects of topical antibacterial medications include erythema, dryness, and burning; using the medications every other day will decrease the adverse effects. Dry eyes and nasal irritation are seen with use of isotretinoin, 13-cis-retinoic acid (Accutane).

1. What disease processes require contact isolation? (Select all that apply.) a. Rotavirus b. Hepatitis A c. Streptococcal pharyngitis d. Mycoplasmal pneumonia e. Respiratory syncytial virus

ANS: A, B, E In addition to Standard Precautions, use contact precautions for patients known or suspected to have serious illnesses easily transmitted by direct patient contact or by contact with items in the patients environment. Examples of such illnesses include rotavirus, hepatitis A, and respiratory syncytial virus. Streptococcal pharyngitis and mycoplasmal pneumonia require droplet precautions.

A child dependent on medical technology is preparing to be discharged from the hospital to home. Which predischarge assessments should the nurse ensure? (Select all that apply.) a. Emergency care and transport plan b. Reliance on private duty nurses to teach the family infection control practices c. Financial arrangements d. Individualized home plan to be completed within the first month of the child's discharge

ANS: A, C The predischarge plan for a child dependent on medical technology going home should include emergency care and transport plan and financial arrangements. The infection control practices and individualized home plan should be completed before discharge, not after the child goes home.

The nurse is assessing a familys use of complementary medicine practices. What practices are classified as mindbody control therapies? (Select all that apply.) a. Relaxation b. Acupuncture c. Prayer therapy d. Guided imagery e. Herbal medicine

ANS: A, C, D Relaxation, prayer therapy, and guided imagery are classified as mindbody control therapies. Acupuncture and herbal medicine are classified as traditional and ethnomedicine therapies.

Which gross motor milestones should the nurse assess in an 18-month-old child? (Select all that apply.) a. Jumps in place with both feet b. Takes a few steps on tiptoe c. Throws ball overhand without falling d. Pulls and pushes toys e. Stands on one foot momentarily

ANS: A, C, D An 18-month-old child can jump in place with both feet, throw a ball overhand without falling, and pull and push toys. Taking a few steps on tiptoe and standing on one foot momentarily is not acquired until 30 months of age.

The nurse should teach the adolescent that the long-term effects of tanning can cause which conditions? (Select all that apply.) a. Phototoxic reactions b. Increased number of moles c. Premature aging d. Striae e. Increased risk of skin cancer

ANS: A, C, E Long-term effects of tanning include premature aging of the skin, increased risk of skin cancer, and, in susceptible individuals, phototoxic reactions. There has been no correlation to an increase in moles or striae (streaks or stripes on the skin, usually on the abdomen) development.

32. The nurse needs to start an intravenous (IV) line on an 8-year-old child to begin administering intravenous antibiotics. The child starts to cry and tells the nurse, Do it later, okay? What action should the nurse take? a. Postpone starting the IV until the next shift. b. Start the IV line and then allow for expression of feelings. c. Change the route of the antibiotics to PO. d. Postpone starting the IV line until the child is ready.

ANS: B A school-age child may try to delay the procedure, but it is best to complete the procedure and allow time for the child to express his or her feelings. The nurse should not postpone administering the antibiotic, change it to PO, or wait to start the IV line until the child is ready.

32. The nurse is preparing to administer a liquid medication by a nasogastric feeding tube. What is the first thing the nurse should do? a. Check placement of the tube. b. Check the pH of the gastric aspirate. c. Flush the tube with a small amount of water. d. Give the medication and then flush with a small amount of water.

ANS: B The most accurate way to check the position of the nasogastric tube is by checking the pH. Auscultation as a verification tool is reliable only 60% to 80% of the time and should not be used without additional methods. The tube should not be flushed or the medication administered until placement of the tube is checked.

1. The nurse is preparing to admit a child to the hospital with a diagnosis of acute poststreptococcal glomerulonephritis. The nurse understands that the peak age at onset for this disease is what? a. 2 to 4 years b. 5 to 7 years c. 8 to 10 years d. 11 to 13 years

ANS: B Thepeakageatonsetforacutepoststreptococcalglomerulonephritisis5to7yearsofage.

Which best describes acute glomerulonephritis? a. Occurs after a urinary tract infection b. Occurs after a streptococcal infection c. Associated with renal vascular disorders d. Associated with structural anomalies of genitourinary tract

ANS: B Acute glomerulonephritis is an immune-complex disease that occurs after a streptococcal infection with certain strains of the group A β-hemolytic streptococcus. Acute glomerulonephritis usually follows streptococcal pharyngitis and is not associated with renal vascular disorders or genitourinary tract structural anomalies.

Which is an implanted ear prosthesis for children with sensorineural hearing loss? a. Hearing aid b. Cochlear implant c. Auditory implant d. Amplification device

ANS: B Cochlear implants are surgically implanted, and they provide a sensation of hearing for individuals who have severe or profound hearing loss of sensorineural origin. Hearing aids and amplification devices are external devices for enhancing hearing. Auditory implants do not exist.

Families progress through various stages of reactions when a child is diagnosed with a chronic illness or disability. After the shock phase, a period of adjustment usually follows. This is often characterized by what response? a. Denial b. Guilt and anger c. Social reintegration d. Acceptance of the childs limitations

ANS: B For most families, the adjustment phase is accompanied by several responses, including guilt, self-accusation, bitterness, and anger. The initial diagnosis of a chronic illness or disability often is met with intense emotion and characterized by shock and denial. Social reintegration and acceptance of the childs limitations are the culmination of the adjustment process.

The clinic nurse is evaluating a patient with a vaginal infection. The nurse knows that the normal vaginal pH is in which range? a. 3.0 to 4.0 b. 4.0 to 5.0 c. 5.0 to 6.0 d. 6.0 to 7.0

ANS: B Normal vaginal secretions are acidic, with a pH range of 4.0 to 5.0.

A 16-year-old boy with a chronic illness has recently become rebellious and is taking risks such as missing doses of his medication. What should the nurse explain to his parents? a. That he needs more discipline b. That this is a normal part of adolescence c. That he needs more socialization with peers d. That this is how he is asking for more parental control

ANS: B Risk taking, rebelliousness, and lack of cooperation are normal parts of adolescence, during which young adults are establishing independence. If the parents increase the amount of discipline, he will most likely be more rebellious. More socialization with peers does not address the problem of risk-taking behavior.

Parents of a 12-year-old child ask the clinic nurse, "How many hours of sleep should our child get?" The nurse should respond that 12-year-old children need how many hours of sleep at night? a. 8 b. 9 c. 10 d. 11

ANS: B School-age children usually do not require naps, but they do need to sleep approximately 11 hours at age 5 years and 9 hours at age 12 years each night.

A school nurse is teaching a group of preadolescent girls about puberty. Which is the mean age of menarche for girls in the United States? a. 11 1/2 years b. 12 3/4 years c. 13 1/2 years d. 14 years

ANS: B The average age of menarche is 12 years 9.5 months in North American girls, with a normal range of 10 1/2 to 15 years. Ages 11 1/2, 13 1/2, and 14 are within the normal range for menarche, but these are not the average ages.

What is the usual presenting symptom for testicular cancer? a. Solid, painful mass b. Hard, painless mass c. Scrotal swelling and pain d. Epididymis easily palpated

ANS: B The usual presenting symptom for testicular cancer is a heavy, hard, painless mass that is either smooth or nodular and palpated on the testes. Pain is not usually associated with a testicular tumor. Scrotal swelling needs to be evaluated. The epididymis is easily palpated in a normal scrotum.

To avoid a fall from a crib, the nurse recommends to parents that their toddler should sleep in a bed rather than a crib when reaching what height? a. 30 in b. 35 in c. 40 in d. 45 in

ANS: B When children reach a height of 89 cm (35 in), they should sleep in a bed rather than a crib.

The school nurse recognizes that children respond to stress by using which tactics? (Select all that apply.) a. Passivity b. Delinquency c. Daydreaming d. Delaying tactics e. Becoming outgoing

ANS: B, C, D Children respond to stress by using coping mechanisms that include internalizing symptoms such as withdrawal, delaying tactics, and daydreaming, along with externalizing symptoms such as aggression and delinquency.

What are sources of stress in preschoolers? (Select all that apply.) a. Shares possessions b. Damages or destroys objects c. May fear dogs or other animals d. Seems to be in perpetual motion e. May stutter or stumble over words

ANS: B, C, D, E Sources of stress in preschoolers include damaging or destroying objects, fearing dogs or other animals, in perpetual motion, and may stutter or stumble over words. Guarding possessions, not sharing, is a source of stress

In terms of language and cognitive development, a 4-year-old child would be expected to have which traits? (Select all that apply.) a. Think in abstract terms. b. Follow directional commands. c. Understand conservation of matter. d. Use sentences of eight words. e. Tell exaggerated stories. f. Comprehend another person's perspective.

ANS: B, E Children ages 3 to 4 years can give and follow simple commands and tell exaggerated stories. Children cannot think abstractly at age 4 years. Conservation of matter is a developmental task of the school-age child. Five-year-old children use sentences with eight words with all parts of speech. A 4-year-old child cannot comprehend another's perspective.

1. What is an appropriate nursing intervention for a child with minimal change nephrotic syndrome (MCNS) who has scrotal edema? a. Place an ice pack on the scrotal area. b. Place the child in an upright sitting position. c. Elevate the scrotum with a rolled washcloth. d. Place a warm moist pack to the scrotal area.

ANS: C In children hospitalized with MCNS, elevating edematous parts may be helpful to shift fluid to more comfortable distributions. Areas that are particularly edematous, such as the scrotum, abdomen, and legs, may require support. The scrotum can be elevated with a rolled washcloth. Ice or heat should not be used. Sitting the child in an upright position will not decrease the scrotal edema.

What action should the school nurse take for a child who has a hematoma (black eye) with no hemorrhage into the anterior chamber? a. Apply a warm moist pack. b. Have the child keep the eyes open. c. Apply ice for the first 24 hours. D. Refer to an ophthalmologist immediately

ANS: C The care for a hematoma eye injury with no hemorrhage into the anterior chamber is to apply ice for the first 24 hours. A warm moist pack should not be applied, and the child should keep the eyes closed. Referral to an ophthalmologist is recommended if hyphema (hemorrhage into the anterior chamber) is present.

When assessing the eyes of a neonate, the nurse observes opacity of the lens. This represents which impairment? a. Blindness b. Glaucoma c. Cataracts d. Retinoblastoma

ANS: C A cataract is opacity of the lens of the eye. The child may have visual impairment secondary to the cataract, but the opacity is a cataract. Glaucoma is increased intraocular pressure. Retinoblastoma is a tumor of the eye.

The child of 15 to 30 months is likely to be struggling with which developmental task? a. Trust b. Initiative c. Autonomy d. Intimacy

ANS: C Autonomy vs shame and doubt is the developmental task of toddlers. Trust vs mistrust is the developmental stage of infancy. Initiative vs guilt is the developmental stage of early childhood. Intimacy and solidarity vs isolation is the developmental stage of early adulthood.

Which is a major complication in a child with chronic renal failure? a. Hypokalemia b. Metabolic alkalosis c. Water and sodium retention d. Excessive excretion of blood urea nitrogen

ANS: C Chronic renal failure leads to water and sodium retention, which contributes to edema and vascular congestion. Hyperkalemia, metabolic acidosis, and retention of blood urea nitrogen are complications of chronic renal failure.

The nurse is assessing the Tanner stage in an adolescent female. The nurse recognizes that the stages are based on which? a. The stages of vaginal changes b. The progression of menstrual cycles to regularity c. Breast size and the shape and distribution of pubic hair d. The development of fat deposits around the hips and buttocks

ANS: C In females, the Tanner stages describe pubertal development based on breast size and the shape and distribution of pubic hair. The stages of vaginal changes, progression of menstrual cycles to regularity, and the development of fat deposits occur during puberty but are not used for the Tanner stages.

What dysfunctional speech pattern is a normal characteristic of the language development of a preschool child? a. Lisp b. Echolalia c. Stammering d. Repetition without meaning

ANS: C Stammering and stuttering are normal dysfluency in preschool-age children. Lisps are not a normal characteristic of language development. Echolalia and repetition are traits of toddlers language.

When communicating with other professionals about a child with a chronic illness, what is important for nurses to do? a. Ask others what they want to know. b. Share everything known about the family. c. Restrict communication to clinically relevant information. d. Recognize that confidentiality is not possible in home care.

ANS: C The nurse needs to share, through both oral and written communication, clinically relevant information with other involved health professionals. Asking others what they want to know and sharing everything known about the family are inappropriate measures. Patients have a right to confidentiality. Confidentiality permits the disclosure of information to other health professionals on a need-to-know basis.

29. What clinical manifestation(s) should the nurse expect to see as shock progresses in a child and becomes decompensated shock? a. Thirst b. Irritability c. Apprehension d. Confusion and somnolence

ANS: D Confusion and somnolence are beginning signs of decompensated shock. Thirst, irritability, and apprehension are signs of compensated shock.

1. Clinical manifestations of sodium excess (hypernatremia) include which signs or symptoms? a. Hyperreflexia b. Abdominal cramps c. Cardiac dysrhythmias d. Dry, sticky mucous membranes

ANS: D Dry, sticky mucous membranes are associated with hypernatremia. Hyperreflexia is associated with hyperkalemia. Abdominal cramps, weakness, dizziness, nausea, and apprehension are associated hyponatremia. Cardiac dysrhythmias are associated with hypokalemia.

What suggestion by the nurse for parents regarding stuttering in children is most helpful? a. Offer rewards for proper speech. b. Encourage the child to take it easy and go slow when stuttering. c. Help the child by supplying words when he or she is experiencing a block. d. Give the child plenty of time and the impression that you are not in a hurry.

ANS: D Hesitancy and dysfluency should be considered a normal part of speech development. Animportantapproachistoallowthechildplentyoftimetospeak.Promisingrewardsforproperspeechplacesadditionalpressureonthechild.Encouragingthechildtotakeiteasyandgoslowwhen stuttering draws attention to the dysfluency. The child needs to complete a sentence and thought without being interrupted

Which characteristic best describes the gross motor skills of a 24-month-old child? a. Skips b. Broad jumps c. Rides tricycle d. Walks up and down stairs

ANS: D A 24-month-old child can go up and down stairs alone with two feet on each step. Skipping and broad jumping are skills acquired at age 3 years. Tricycle riding is achieved at age 4 years.

The nurse is guiding parents in selecting a daycare facility for their child. Which is especially important to consider when making the selection? a. Structured learning environment b. Socioeconomic status of children c. Cultural similarities of children d. Teachers knowledgeable about development

ANS: D A teacher knowledgeable about development will structure activities for learning. A structured learning environment is not necessary at this age. Socioeconomic status is not the most important factor in selecting a preschool. Preschool is about expanding experiences with others, so cultural similarities are not necessary.

What best describes central nervous system (CNS) stimulants? a. Acute intoxication can lead to coma. b. They produce strong physical dependence. c. Withdrawal symptoms are life threatening. d. They can result in strong psychologic dependence

ANS: D CNS stimulants such as amphetamines and cocaine produce a strong psychologic dependence. Acute intoxication leads to violent aggressive behavior or psychotic episodes characterized by paranoia, uncontrollable agitation, and restlessness. This class of drugs does not produce strong physical dependence and can be withdrawn without much danger.

The nurse recommends to parents that peanuts are not a good snack food for toddlers. What is the nurse's rationale for this action? a. Low in nutritive value b. High in sodium c. Cannot be entirely digested d. Can be easily aspirated

ANS: D Foreign-body aspiration is common during the second year of life. Although they chew well, this age child may have difficulty with large pieces of food, such as meat and whole hot dogs, and with hard foods, such as nuts or dried beans. Peanuts have many beneficial nutrients, but should be avoided because of the risk of aspiration in this age group. The sodium level may be a concern, but the risk of aspiration is more important. Many foods pass through the gastrointestinal tract incompletely undigested. This is not necessarily detrimental to the child.

The nurse is caring for a child with acute renal failure. Which clinical manifestation should the nurse recognize as a sign of hyperkalemia? a. Dyspnea b. Seizure c. Oliguria d. Cardiac arrhythmia

ANS: D Hyperkalemia is the most common threat to the life of the child. Signs of hyperkalemia include electrocardiograph anomalies such as prolonged QRS complex, depressed ST segments, peaked T waves, bradycardia, or heart block. Dyspnea, seizure, and oliguria are not manifestations of hyperkalemia.

What is the initial indication of puberty in girls? a. Menarche b. Growth spurt c. Growth of pubic hair d. Breast development

ANS: D In most girls, the initial indication of puberty is the appearance of breast buds, an event known as thelarche. The usual sequence of secondary sex characteristic development in girls is breast changes, rapid increase in height and weight, growth of pubic hair, appearance of axillary hair, menstruation, and abrupt deceleration of linear growth.

Which is characteristic of dishonest behavior in children ages 8 to 10 years? a. Cheating during games is now more common. b. Lying results from the inability to distinguish between fact and fantasy. c. They may steal because their sense of property rights is limited. d. They may lie to meet expectations set by others that they have been unable to attain.

ANS: D Older school-age children may lie to meet expectations set by others to which they have been unable to measure up. Cheating usually becomes less frequent as the child matures. In this age group, children are able to distinguish between fact and fantasy. Young children may lack a sense of property rights; older children may steal to supplement an inadequate allowance, or it may be an indication of serious problems

The parents of a 5-year-old child ask the nurse, How many hours of sleep a night does our child need? The nurse should give which response? a. A 5-year-old child requires 8 hours of sleep. b. A 5-year-old child requires 9.5 hours of sleep. c. A 5-year-old child requires 10 hours of sleep. d. A 5-year-old child requires 11.5 hours of sleep.

ANS: D Sleep requirements decrease during school-age years; 5-year-old children generally require 11.5 hours of sleep.

A school nurse is teaching dental health practices to a group of sixth-grade children. How often should the nurse recommend the children brush their teeth? a. Twice a day b. Three times a day c. After meals d. After meals, after snacks, and at bedtime

ANS: D Teeth should be brushed after meals, after snacks, and at bedtime. Children who brush their teeth frequently and become accustomed to the feel of a clean mouth at an early age usually maintain the habit throughout life. Twice a day, three times a day, or after meals would not be often enough.

A nurse is planning care for a 17-month-old child. According to Piaget, which stage should the nurse expect the child to be in cognitively? a. Trust b. Preoperational c. Secondary circular reaction d. Tertiary circular reaction

ANS: D The 17-month-old child is in the fifth stage of the sensorimotor phase, tertiary circular reactions. The child uses active experimentation to achieve previously unattainable goals. Trust is Erikson's first stage. Preoperational is the stage of cognitive development usually present in older toddlers and preschoolers. Secondary circular reactions last from about ages 4 to 8 months.

Which characteristic best describes the gross motor skills of a 24-month-old child? a. Skips and can hop in place on one foot b. Rides tricycle and broad jumps c. Jumps with both feet and stands on one foot momentarily d. Walks up and down stairs and runs with a wide stance

ANS: D The 24-month-old child can go up and down stairs alone with two feet on each step and runs with a wide stance. Skipping and hopping on one foot are achieved by 4-year-old children. Jumping with both feet and standing on one foot momentarily are achieved by 30-month-old children. Tricycle riding and broad jumping are achieved at age 3.

A child with acetaminophen (Tylenol) poisoning has been admitted to the emergency department. What antidote does the nurse anticipate being prescribed? a. Carnitine (Carnitor) b. Fomepizole (Antizol) c. Deferoxamine (Desferal) d. N-acetylcysteine (Mucomyst)

ANS: D The antidote for acetaminophen (Tylenol) poisoning is N-acetylcysteine (Mucomyst). Carnitine (Carnitor) is an antidote for valproic acid (Depakote), fomepizole (Antizol) is the antidote for methanol poisoning, and deferoxamine (Desferal) is the antidote for iron poisoning.

The parents of a newborn say that their toddler hates the baby. . . . He suggested that we put him in the trash can so the trash truck could take him away. What is the nurse's best reply? a. Let's see if we can figure out why he hates the new baby. b. That's a strong statement to come from such a small boy. c. Let's refer him to counseling to work this hatred out. It's not a normal response. d. That is a normal response to the birth of a sibling. Let's look at ways to deal with this.

ANS: D The arrival of a new infant represents a crisis for even the best prepared toddler. Toddlers have their entire schedules and routines disrupted because of the new family member. The nurse should work with the parents on ways to involve the toddler in the newborns care and to help focus attention on the toddler. The toddler does not hate the infant. This is an expected, normal response to the changes in routines and attention that affect the toddler. The toddler can be provided with a doll to imitate parents behaviors. The child can care for the dolls needs at the same time the parent is performing similar care for the newborn.

A father tells the nurse that his daughter wants the same plate and cup used at every meal, even if they go to a restaurant. What should the nurse explain to the father? a. A sign the child is spoiled b. A way to exert unhealthy control c. Regression, common at this age d. Ritualism, common at this age

ANS: D The child is exhibiting the ritualism that is characteristic at this age. Ritualism is the need to maintain sameness and reliability. It provides a sense of comfort to the toddler. It will dictate certain principles in feeding practices, including rejecting a favorite food because it is served in a different container. Ritualism is not indicative of a child who has unreasonable expectations, but rather normal development. Toddlers use ritualistic behaviors to maintain necessary structure in their lives. This is not regression, which is a retreat from a present pattern of functioning.

The school nurse is teaching a class on injury prevention. What should be included when discussing firearms? a. Adolescents are too young to use guns properly for hunting. b. Gun carrying among adolescents is on the rise, primarily among inner-city youth. c. Nonpowder guns (air rifles, BB guns) are a relatively safe alternative to powder guns. d. Adolescence is the peak age for being a victim or offender in the case of injury involving a firearm.

ANS: D The increase in gun availability in the general population is linked to increased gun deaths among children, especially adolescents. Gun carrying among adolescents is on the rise and not limited to the stereotypic inner-city youth. Adolescents can be taught to safely use guns for hunting, but they must be stored properly and used only with supervision. Nonpowder guns (air rifles, BB guns) cause almost as many injuries as powder guns.

The nurse is planning to bring a preschool child a toy from the playroom. What toy is appropriate for this age group? a. Building blocks b. A 500-piece puzzle c. Paint by number picture d. Farm animals and equipment

ANS: D The most characteristic and pervasive preschooler activity is imitative, imaginative, and dramatic play. Farm animals and equipment would provide hours of self-expression. Building blocks are appropriate for older infants and toddlers. A 500-piece puzzle or a paint by number picture would be appropriate for a school-age child.

Which is the most common cause of acute renal failure in children? a. Pyelonephritis b. Tubular destruction c. Urinary tract obstruction d. Inadequate perfusion

ANS: D The most common cause of acute renal failure in children is poor perfusion that may respond to restoration of fluid volume. Pyelonephritis and tubular destruction are not common causes of acute renal failure. Obstructive uropathy may cause acute renal failure, but it is not the most common cause.

The nurse is teaching parents about toilet training. What should the nurse include in the teaching session? a. Bladder training is accomplished before bowel training. b. The mastery of skills required for toilet training is present at 18 months. c. By 12 months, the child is able to retain urine for up to 2 hours or longer. d. The physiologic ability to control the sphincters occurs between 18 and 24 months.

ANS: D The physiologic ability to control the sphincters occurs somewhere between ages 18 and 24 months. Bowel training is usually accomplished before bladder training because of its greater regularity and predictability. The mastery of skills required for training are not present before 24 months of age. By 14 to 18 months of age, the child is able to retain urine for up to 2 hours or longer.

1. The psychosexual conflicts of preschool children make them extremely vulnerable to which threat? a. Loss of control b. Loss of identity c. Separation anxiety d. Bodily injury and pain

ANS: D The psychosexual conflicts of children in this age group make them vulnerable to threats of bodily injury. Intrusive procedures, whether painful or painless, are threatening to preschoolers, whose concept of body integrity is still poorly developed. Loss of control, loss of identity, and separation anxiety are not related to psychosexual conflicts.

Which should the nurse expect of a healthy 3-year-old child? a. Jump rope b. Ride a two-wheel bicycle c. Skip on alternate feet d. Balance on one foot for a few seconds

ANS: D Three-year-olds are able to accomplish this gross motor skill. Jumping rope, riding a two-wheel bicycle, and skipping on alternate feet are gross motor skills of 5-year-olds.

1. The nurse is performing a physical assessment on a 3-year-old child. The parents state that the child excessively rubs the eyes and often tilts the head to one side. What visual impairment should the nurse suspect? a. Strabismus b. Astigmatism c. Hyperopia, or farsightedness Myopia, or nearsightedness

a. ANS: D Clinical manifestations of myopia include excessive eye rubbing, head tilting, difficulty reading, headaches, and dizziness. Strabismus, astigmatism, and hyperopia have other clinical manifestations.

The estimated average requirement of calcium for an adolescent is _____ milligrams.

1100

The recommendation for calcium for children 1 to 3 years of age is _____ milligrams.

500

The nurse is performing a pH dipstick test on a urine specimen. Which is the average pH expected for this test? (Record your answer in a whole number.)

6

The school nurse is assessing a childs severely scraped knee for infection. What are signs of a wound infection? (Select all that apply.) a. Odor b. Edema c. Dry scab d. Purulent exudate e. Decreased temperature

ANS: A, B, D Signs of wound infection are odor, edema, and purulent exudate. Increased, not decreased, temperature indicates infection. A dry scab over the wound is part of the healing process.

Place in order the sequence of maturational changes for girls. Begin with the first change seen, sequencing to the last change. Provide answer in using lowercase letters, separated by commas (e.g., a, b, c, d, e). a. Growth of pubic hair b. Rapid increase in height and weight c. Breast changes d. Menstruation e. Appearance of axillary hair

ANS: c, b, a, e, d

32. A 2-year-old child has to receive Rocephin IM injections every 12 hours. What nursing intervention should be implemented for the child? a. Hold the child while rocking in a chair after each injection. b. Prepare the child several hours before the injection is given. c. Allow the child to watch a younger child receive an injection. d. Encourage the child to draw a picture of the pain experienced when an injection is given.

ANS: A After the procedure, the child continues to need reassurance that he or she performed well and is accepted and loved. The other options are not appropriate for a toddler.

A feeling of guilt that the child caused the disability or illness is especially common in which age group? a. Toddler b. Preschooler c. School-age child d. Adolescent

ANS: B Preschoolers are most likely to be affected by feelings of guilt that they caused the illness or disability or are being punished for wrongdoings. Toddlers are focused on establishing their autonomy. The illness fosters dependency. School-age children have limited opportunities for achievement and may not be able to understand limitations. Adolescents face the task of incorporating their disabilities into their changing self-concept.

1. What dietary instructions should the nurse give to parents of a child in the oliguria phase of acute glomerulonephritis with edema and hypertension? (Select all that apply.) a. High fat b. Low protein c. Encouragement of fluids d. Moderate sodium restriction e. Limit foods high in potassium

ANS: D, E Dietary restrictions depend on the stage and severity of acute glomerulonephritis, especially the extent of edema. A regular diet is permitted in uncomplicated cases, but sodium intake is usually limited (no salt is added to foods). Moderate sodium restriction is usually instituted for children with hypertension or edema. Foods with substantial amounts of potassium are generally restricted during the period of oliguria. Protein restriction is reserved only for children with severe azotemia resulting from prolonged oliguria. A low-protein, high-fat diet with encouragement of fluids would not be recommended.

1. What name is given to inflammation of the bladder? a. Cystitis b. Urethritis c. Urosepsis d. Bacteriuria

ANS: A Cystitis is an inflammation of the bladder. Urethritis is an inflammation of the urethra. Urosepsis is a febrile urinary tract infection with systemic signs of bacterial infection. Bacteriuria is the presence of bacteria in the urine.

The nurse should expect a toddler to cope with the stress of a short period of separation from parents by displaying what? a. Regression b. Happiness c. Detachment d. Indifference

ANS: A Children in the toddler stage demonstrate goal-directed behaviors when separated from parents for short periods. They may demonstrate displeasure on the parents return or departure by having temper tantrums; refusing to comply with the usual routines of mealtime, bedtime, or toileting; or regressing to more primitive levels of development. Detachment would be seen with a prolonged absence of parents, not a short one. Toddlers would not be indifferent or happy when experiencing short separations from parents.

A child, age 4 years, tells the nurse that she needs a Band-Aid where she had an injection. What nursing action should the nurse implement? a. Apply a Band-Aid. b. Ask her why she wants a Band-Aid. c. Explain why a Band-Aid is not needed. d. Show her that the bleeding has already stopped.

ANS: A Children in this age group still fear that their insides may leak out at the injection site. The nurse should be prepared to apply a small Band-Aid after the injection. No explanation should be required.

A 16-year-old girl comes to the pediatric clinic for information on birth control. The nurse knows that before this young woman can be examined, consent must be obtained from which source? a. Herself b. Her mother c. Court order d. Legal guardian

ANS: A Contraceptiveadviceisoneoftheconditionsthatisconsideredmedicallyemancipated.Theadolescentisabletoprovideherowninformedconsent

According to Erikson, the psychosocial task of adolescence is developing what? a. Identity b. Intimacy c. Initiative d. Independence

ANS: A Traditional psychosocial theory holds that the developmental crises of adolescence lead to the formation of a sense of identity. Intimacy is the developmental stage for early adulthood. Independence is not one of Eriksons developmental stages.

What is the single most prevalent cause of disability in children and responsible for the recent increase in childhood disability? a. Cancer b. Asthma c. Seizures d. Heart disease

ANS: B Asthma is the single most prevalent cause of disability in children and has been largely responsible for much of the recent increase in childhood disability.

Which is included in the diet of a child with minimal change nephrotic syndrome? a. High protein b. Salt restriction c. Low fat d. High carbohydrate

ANS: B Salt is usually restricted (but not eliminated) during the edema phase. The child has little appetite during the acute phase. Favorite foods are provided (with the exception of high-salt ones) in an attempt to provide nutritionally complete meals.

According to Erikson, which psychosocial task is developing in adolescence? a. Intimacy b. Identity c. Initiative d. Independence

ANS: B Traditional psychosocial theory holds that the developmental crises of adolescence lead to the formation of a sense of identity. Intimacy is the developmental stage for early adulthood. Independence is not one of Erikson's developmental stages.

1. What is the required number of milliliters of fluid needed per day for a 14-kg child? a. 800 b. 1000 c. 1200 d. 1400

ANS: C For the first 10 kg of body weight, a child requires 100 ml/kg. For each additional kilogram of body weight, an extra 50 ml is needed.

The school nurse is teaching male school-age children about the average age of puberty. What is the average age of puberty for boys? a. 12 years b. 13 years c. 14 years d. 15 years

ANS: C The average age of puberty is 14 years in boys. Boys experience little sexual maturation during preadolescence

An occlusive dressing is applied to a large abrasion. This is advantageous because the dressing will accomplish what? a. Deliver vitamin C to the wound. b. Provide an antiseptic for the wound. c. Maintain a moist environment for healing. d. Promote mechanical friction for healing.

ANS: C Occlusive dressings, such as Acuderm, are not adherent to the wound site. They provide a moist wound surface and insulate the wound. The dressing does not have vitamin C or antiseptic capabilities. Acuderm protects against friction.

1. An infant is brought to the emergency department with the following clinical manifestations: poor skin turgor, weight loss, lethargy, tachycardia, and tachypnea. This is suggestive of which situation? a. Water excess b. Sodium excess c. Water depletion d. Potassium excess

ANS: C These clinical manifestations indicate water depletion or dehydration. Edema and weight gain occur with water excess or over hydration. Sodium or potassium excess would not cause these symptoms.

The school nurse recognizes that adolescents should get how many hours of sleep each night? a. 6 hours b. 7 hours c. 8 hours d. 9 hours

ANS: D Adolescents should generally get around 9 hours of sleep each night.

A young boy is found squirting lighter fluid into his mouth. His father calls the emergency department. The nurse taking the call should know that the primary danger is what? a. Hepatic dysfunction b. Dehydration secondary to vomiting c. Esophageal stricture and shock d. Bronchitis and chemical pneumonia

ANS: D Lighter fluid is a hydrocarbon. The immediate danger is aspiration. Acetaminophen overdose, not hydrocarbons, causes hepatic dysfunction. Dehydration is not the primary danger. Esophageal stricture is a late or chronic consequence of hydrocarbon ingestion.

Which is the preferred site for intramuscular injections in infants? a. Deltoid b. Dorsogluteal c. Rectus femoris d. Vastus lateralis

ANS: D The preferred site for infants is the vastus lateralis. The deltoid and dorsogluteal sites are used for older children and adults. The rectus femoris is not a recommended site.

Which term best describes a multidisciplinary approach to the management of a terminal illness that focuses on symptom control and support? a. Dying care b. Curative care c. Restorative care d. Palliative care

ANS: D This is one of the definitions of palliative care. The goal of palliative care is the achievement of the highest possible quality of life for patients and their families.

The nurse is explaining the preconventional stage of moral development to a group of nursing students. What characterizes this stage? a. Children in this stage focus on following the rules. b. Children in this stage live up to social expectations and roles. c. Children in this stage have a concrete sense of justice and fairness. d. Children in this stage have little, if any, concern for why something is wrong.

ANS: D Young childrens development of moral judgment is at the most basic level in the preconventional stage. They have little, if any, concern for why something is wrong. Following the rules, living up to social expectations, and having a concrete sense of justice and fairness are characteristics in the conventional stage.

1. What interventions should the nurse implement to prevent a pressure ulcer in a critically ill child? (Select all that apply.) a. Nutrition consults b. Using skin moisturizers c. Turning the child every 2 hours d. Using plastic disposable under pads e. Using draw sheets to minimize shear

a. ANS: A, B, C, E Interventions found to prevent pressure ulcers in critically ill children include nutrition consults, using skin moisturizers, turning the child every 2 hours, and using draw sheets to minimize shear. Dry weave under pads, not under pads with plastic, should be used to reduce moisture.

What are signs and symptoms of the stage of despair in relation to separation anxiety in young children? (Select all that apply.) a. Withdrawn from others b. Uncommunicative c. Clings to parents d. Physically attacks strangers e. Forms new but superficial relationships F. Regresses to early behaviors

a. ANS: A, B, F Manifestations of the stage of despair seen in children during a hospitalization may include withdrawing from others, being uncommunicative, and regressing to earlier behaviors. Clinging to parents and physically attacking a stranger should be seen during the stage of protest, and forming new but superficial relationships is seen during the stage of detachment.

What factors can negatively affect parents reactions to their childs illness? (Select all that apply.) a. Additional stresses b. Previous coping abilities c. Lack of support systems d. Seriousness of the threat to the child E. Previous experience with hospitalization

a. ANS: A, C, D The factors that can negatively affect parents reactions to their childs illness are additional stresses, lack of support systems, and the seriousness of the threat to the child. Previous coping abilities and previous experience with hospitalization would have a positive effect on coping.

Place in order the correct sequence for emergency treatment of poisoning in a child. Provide answer using lowercase letters separated by commas (e.g., a, b, c, d). a. Locate the poison. b. Assess the child. c. Prevent absorption of poison. d. Terminate exposure to the toxic substance

b, d, a, c


Related study sets

Chapter 11 Homework: Cardiovascular System

View Set

ch. 17-2: Psychoanalysis and Humanistic Therapy

View Set

ANTH 344 Altruism & Kin Selection

View Set

Chapter 8: Sarbanes-Oxley, Internal Control, & Cash

View Set

Physics: Energy and momentum study guide 2

View Set